Download as pdf or txt
Download as pdf or txt
You are on page 1of 401

MATHEMATICS B.Tech.

Computer Science and Engineering


Semester-lV
.K.G.P.T.U., JALANDHAR

Scanned by
Scanned by TapScanner
TapScanner
CONTENTS
MODULE-1

1. Sets, Relation and Function 1-176

MODULE-2

1. Permutations and Combinations 179 —222 ©

2. Pigeonhole Principle 223 —250

3. The Inclusion—Exclusion Principle 238 — 264 —

MODULE-3

_ Logic and Propositional Calculus 267 —316


f om |
7 gt #

_ be

7
LE
‘i

i:
ee
MODULE-4
: ry ,

Algebraic Structures and Morphism 319=> 4079)


es }
A.pec raic Structures with Two Binary Operations, 408 —444 —
2 i igs, IIntegral Domain and Fields ,

joolean Algebra 445— 478


ee MODULE-5

Scanned by TapScanner
Scanned by TapScanner
SYLLABUS
BTCS401-18
DISCRETE MATHEMATICS |
3L:1T:0P — 4Credits—
|
Detailed contents :
= —
Module 1:

Sets, Relation and Function : Operations and Law of Sets, Cartesian | |


Binary Relation, Partial Ordering Relation, Equivalence Relation, Image of a Set, Sum .
and Product of Functions, Bijective functions, Inverse and Composite Function, Size of a _ |
Set, Finite and infinite Sets, Countable and uncountable Sets, Cantor’s diagonal argument | i

ef.
and The Power Set theorem, Schroeder-Bernstein theorem. 3
r

i,
SN
he
Principles of Mathematical Induction : The Well-Ordering Principle, Recursive: |

gate
definition, The Division algorithm :Prime Numbers, The Greatest Common Divisor : if

a
Euclidean Algorithm, The Fundamental Theorem of Arithmetic.

Module 2:

Basic counting techniques-inclusion and exclusion, pigeon-hole


permutation and combination.

Ss a
a i 0 ie ; F s a ‘= - .
7 —_—- ae eae re ee te 7_* —- verEwee
Aw : es

eR IL Vk > i a ™
. 7 : =
7 ij .
VY i "4
en cE oe TI r) e L 4
ie {
AW ¥¥
ro A J 4 ve
if 7 i oA
ia 5 | 7 ! Pi rE | ‘ we + ne
a = Fle T= ae ~ cee a oe am Pa eo - - Spa 7 us
e “

a Zs om _ Lav
~~ ait Sa) Me Sl 27) ee aoe EN a _
re C7 5 2 ie 4, : |a cai a Bes 7 =.
a aac!“a a - i .
a f jy. ts _ p= . < g ie —— - g eri
ae —_ 1sa E
—s . =<
| 4I i 7 i . sg - a a
is f " my yd 5 : 5 4 7 o —
7

nee, he use
et pe
Sea ea
She
Vi.
:
.eS ~~
va
» 0 : f£
es
= a
et
er
;
on) Tepaeilaetete mae
Pa | i | i i | | oe | | ii i 1 '
“TR Ty i = 1 Fl
Ge ‘. iw 1 fl ES . i . >»
: || By a Qt 19) yt acs
=

aia ella alee


- — a’
eet
ae \ oA r ne Tae ae aon seat + i %
1 ~ fF te@ mo ac" wa ] ye
= rg ee Nd al 1 oa ad ve — ate i L i _ IL
IL e
ae
ee :
a eA
, “

Ld | =
f F ae

Scanned by
Scanned by TapScanner
TapScanner
Scanned by
Scanned by TapScanner
TapoScanner
®.
— = a
=

i £ as = ‘a: as a

Scanned byby TapS


4 7 r = fi 7 . ane ek < jz 5
io rks fe eA a : =, , 7 _ = # oe ae :
Pe tee er aes Ww f

Scanned TapScanner
canner
1.1. Introduction
Set is a basic and
unifying idea of m
terms of sets. In almos t whole of the busin ess a t hematics. In fact all mathematictaal
l 3
ideas can be expres sed in
mathematics, the set theory is applied in one form or the other.
1.2. Def. of a set

By the words ‘well defined’ we mean that we are given a rule with the help of which we can say
whether a particular object belongs to the set or not. The word ‘different’ implies that repetition of objects
is not allowed.
The words ‘family’, ‘class’, ‘collection’ are also used as synonyms for the word set when the elements
are themselves sets.
Element of a Set
Each object of the set is called an element of the set.
Examples of sets
(i) The set of days of a week. (ii) The set of integers from | to 100000.
(iii) The set of even integers. (iv) The set of all states of India.

(v) The set of all solutions of equation x° = 1.


Set Notations
Sets are generally denoted by capital letters A, B, C.,......
The elements of sets are denoted mostly by small letters a, 6, c...
Some Standard Sets
N = Set of all natural numbers 1, 2, 3, 4,..---.
W =Set of all whole numbers 0, 1, 2, 3, 4,------
I or Z = Set of all integers
Q = Set of all rational numbers
R = Set of all real numbers

Scanned by
Scanned by TapScanner
TapScanner
2

Methods of Designating a Set ——ZECTRUM Discae


“'set can be specified in two ways :
(1) Tabular, > Ros Noster or Enumeration Method: amples # bie he Ree (ae a
aa mar.
Bee
5 ieee bene eee
ot
1
+ RaeWheae
f re
n we re
Se pr
resen esaen
settby y listi
listi ng all its
i elements
Ex A= {1, 2, Sua siehh bgt i A PEE = Pre 2
with;
1s called the tabular, roaster or enumeration method, _ ig Curly brac B={& . x is an odd integer }
. a
© Asof
se vowe
tls : A = {a, @, i, 0, 1},
ran su | C= {x: yx is a multiple of 6 }
i= «hy = hms
rps Hey
(i) A set of positive even integers upto ae Lae or re o-Vifit
10 : B = {2,4, 6 "7 - ele men t is
fe
c a a
l
al oe le
sin
l gle
e ton
d
=
tai nin g onl y single
(ii) A set of odd natural numbers : C = fia Sa
s
i
t
Asetc con
: < oe
(2) Selector, Set-builder or Rule Method : example :

A= { x: x is a vowel in English alphabets }


B= {x: x is a positive even integer up to 10 } | ' oti c itive integer sa - ele
A=1x:x is a pos
- po s "2
Examples : A f
C= {x/x is an odd positive integer }
B= { x: x isa fractiaone satisfying
| Here (:) or (/) means such that.
___ Note 1. The order of elements in a set is immaterial, _ low? ing r-Set
Sub-Set, Supe
eles aa
__ Thus {2, 5,9, 11}, {2,9, 5, 11}, (11,5, 2,9} representthesamese, 9 Ifevery element of a
setA is a
su
2. Repetition of elements is not allowed in a set. Boi wai le => x €B, then A isa
i” nit x. orif xeA
ership ofa Set
If an object x isa member of the set A, we write x & A, whican
ch be read as ‘x hel
toler
é ie |
as AACBisi ancontaindB>A‘in.
e wrAiteCthBesemeans
WThus
fains x’. Similarly we write x @ A to show that x is not a member of th wath can }

Scanned by
Scanned by TapScanner
TapoScanner
4 a
Equality of Sets
sa id to be eq ua l if bot h have the same elem,
Two sets A and B are
eve ry el em en t of A is an el em en t of B and every :
B are equal when 2
B.
ie, fA C Band B CA, then A=
% 8, 9, 10}
Example. A= {1, a as 4, a 6,
d < 19 }
1 =x
B = {xx:isa natural number an
AD arene nin.
Here A=B. 7
~) ee SS
+!

Prope r Sub-set PlE2 yD yay sei iek eg:*


A non-empty set A is said to be a proper subset of B if ACB
Note 1. ¢ and A are called improper subsets of A. 3
Power set ° tril ial

) Example. Take A = {1, 2, 3} viizions Hae a


POA)= {0, {02}
= {¢, {1}, (2}
1,2) {1,3} 23), (12.3)
{3), {1, 2), {1, 3}, {2,3}. 41. rs

Universal Set ‘ees i

If all the sets under consideration


are sub-sets
of a fi ed se
Example. In Plane geometry, the universal setc onsis iad
Comparable and Non-comparable
Sets
Two sets are said to be comparable if one of thetwo set

Scanned by
Scanned by TapScanner
TapoScanner
e SSR Sy nese ene eee ce
co (a) Let = 1 & | [s3}= € ER: 3x53)UTD ceeeeae Proms M
ssol.
; = R: x = ea a Whe mg * “we A) i Ee
ss, RELATI ONTION
AND FUNC
i ee am
mg oe
ra abe tet ae oa ean
Re PS i
em ae

a se oo eg = (X= serail 28)


ZB =2 a 0 xg ae a ‘since 7 <7 > n2<0!

wer > mis“eo


and Q.
the poinPts
by the points on the line from P to Q including
1, 2, 3}
8 atesJ
{x Ee Z, : | x | <= 3}= {- ae: 13 0,
(6) LetA=

PQ.
The setA is represented by the points marked by dot onthe line
esAue2 ES 3
aas
-1 0 1 2 3

(c) Let A= {x EN, |x| $3} = {1 2, 3}


ater aE

x+y= ody Ss +
(d) LetA={@,¥); ee
The setA is represented by all the points which lie on thecele whose
a» ibd
radius as shown below :

=e seen or x=2nw

Scanned by
Scanned by TapScanner
TapoScanner
‘Example7. Find the cardinal number ofeach set:

ii) A= {x:xEN,x7=5} - | ay
Sol. (i) A= {x:x"=25,3x=6) ll
Since x°=25 > x=+5 and 3x=6 +x=2 eats
ahh Fed x igs
ee A=9

6. Scag
Card(A)=0 ie, #(A)=0
, a ee
(ii) Here B= {1, 4, 5,9}
“ P(B)= (@ {1}, {4}, (5), (9), 1,4}, (1,5), 11.9), (4.5),
pee (59
_ Cardinal number of (P- a 16
i Asx’ =5

> Pads ow

” a x a Pi 4%, es ¥) (he SS cniant a ‘i ~~

ae 2 A= (a,5,2,3)
eet a aes, aeniet 2, 3)
Vi eae te e asete

Scanned by
Scanned by TapScanner
TapoScanner
EXERCISE 1.0
he 7
men ts in eac h of the fo ll ow in g sets using braces ea alin
1. List the ele {x:xisane at
(a) {xix is a natural nu mber divisible by 5} | (b)

fe) tx7s Bone prime number} er (a) ix 2x Mis an int


by 2 and by 5}
(e) {xix is an integer divisible a
Roster form :
Write the following sets in ae
ish alphabet} 2
(i) {x:xisa vowel before g in the Engl
‘chen
tween 6 and 30}
(ii) {x € N: x is a prime number be
(iii) {x EN:3x+5<3]}

(iv) {xix +5x+6=0}


:
3. Enumerate the element, in the following sets:
(a) {xERx—3x+2=0} (b) {x ERX +1=0}
in setbuilderform:
Describe the following sets .
(ii) B= (2,4, 6,8, 10}. fe
(i) 8,,9, 10, 11}
A={5,6,7
ee se 7
(iit) C= {18, 27, 36, 45, 54, 63, 72, 81,90} . “levees
s usigng nc
setin
Describe the follow
aro 77,79) ae |
(a)

(c) The even integers


tae 6. Which of the following
sets are null sets?
() A={x:xEN,x<1} (i) B= (xix.
What
is the set {x :x © R, x? ==9,2x=4}7 * aaa
ieee” wee
8 ie

Scanned by
Scanned by TapScanner
TapoScanner
is the set
then their union
ts
her with all the elemen

ot * «gt ei |

sosncinaal Us a
An samy con
Note : If Aj, Az Msi
es we ie
ae cs
ein
Sets
1.6. Intersection of Two
: a5 one

ty k CRAIN re
se ts A an d B, ca gh es
The intersection of two ott ast ca
ar e co mm on to A an d B.
is the seoftall elem ents, which
In symbols,
AN € Aa=
{xixB ndx € B}
Example. Let A = {2, 4, 6, 8, 10, 12}. vidi as
(Be 3,5,710 a neem
ot a

Scanned by
Scanned by TapScanner
TapoScanner
PAU
AN(BNO)=
B U C ) = ( A N B ) U (ANC)
AN(
B N O = ( A U B )NAUO)
AU
(AUB) =A°NB
(ANB) =A UB
AU(ANB)=A
AN(AUB)=A
AUA =U
ANA =¢ ae
he
=

ie mt
en
Statement. If A is any set, th
(J AN A= A f e a
(QAUA= A_ — t iie wa
Proof. () LHS. =AUA
= {xix €AUA}={x:x€ AorxGA
RS) 1. 0.) 6
ae ian
(i) LHS. =ANA

‘ad al —

Scanned by
Scanned by TapScanner
TapoScanner
s : (¢ € A o r x € B) and,
= fixe Aor@ Bane rEcp=t
(AUB) N(AL
E A U OF = {x:x€
fx€ A UBands he ats
C)}
= {(AUB)N(AU
=RHS. | -Sea ee
se dish
AUBNE=AUBINAYOSowing that cena 2
result by sh
C)and (A UB) (AU C) CAUE
ve ab ov e
Note. We can also pro B) N (AU
AUBNCCAY | |
= (ij LHS.=ANBUO
ED and x € (B UC)}
={x:xEAN (BU C)} = {x
ee
. 7
vo

A and x € B) ace
Coal

} = {x :@ E
*

={x:xEAand@e Bo rx EC
col e tea!
=: a

==({xAN12B)€(U(ANANB)C)orx EAN} = fix EANB)U(ANG2)


ee
se
oe
Hig
Aa
PS a
Tope hes ¥ shea
a

=RHS.
dae

ee
+

cae . AN(BUC)=(ANB)U(ANC)
x
VI. De Morgan’s Laws
a

a ene ae
b-sets of U, then
Statement. If A and B are two su ~ 2 i,

() (AUB) =A°NB
OR :
n sets is equal to the intersec
of two
Complement of unio comon
of ti pleme:
Sa t018 B oe
(i) (ANBY=A° UB
OR. 2 a iqasie plem ah an
Complement of intersection of two sets is equal to the union of com
tea
Proof, (i) L.H.S.=(A UB)
~ {e: x € (AU BY } = {xi x ¢ (A UB) } = {x22 @ A a n c
= fr:x€A°andx EB }={e:rE(ANB)} bn )Sam
Ae Aa Be teak 0-8) ~ 1 ee 2»
Rt

2 (AUB) =A°N BS | ie PLR PER

CDE (A OB ck ash
dose tun:
SRiodso BY)
ae eee

Scanned by
Scanned by TapScanner
TapoScanner
18
SPECTRUM DiscR i ;
(7) A+B = {xix GAand = {0}
x ¢ B} TE MATHEM Tic ; ; a soe ideale

(g) B-A x€ A}={}or ¢


={x:x€Band be any element of AT) A co re
eA and xe Aes 7a bg & oes ME ie a
(A) AS= {xix € Uand x € A} = {1,4,5,6,7,8 9}
(i) C= {x:x © U and x € A} = {0, 2, 3, 4, 6, 7, 8} ANAC? : he :
aire CAN A‘ always
Example 2. Let A= {x: x is an even integer} and let
nA =: '
B= {x:x is an in
by 2 or 3}, and let D = fx. ‘ ate sets A and B, prove that A U B=so) U ew J(A NB).
#8 toge divisible by oad ap Deiahiins aad oe aha ay
determine whether it is proper 8 relations hold. If Containment ai | som ys, <(A\B) UIA) (ANB)
ACD
(@) ACB ()BCC)CCB@DCB (DCC wCcH, *) sot R.HS. _ (AB) U[(B-A) U(ANB)) 3
A = {x : x is an integer}= {......-3,~2,—1, 0, 1, 2,3, ..0..3
Sol. Here out B)
U [BOA
=(AN U (AN BY} 5
ea -(ANB)U[B NADUBNA | pas
B = {x: x is an integer divisible by 6} = {......- 12,6,0, 6, 12,...)
C = {x: x is an integer divisible by 2 or 3} “eg =(ANB)U[BN (ATU A)] Age ‘i Fi ay e.
0, 2, 3, 2, 4,6,......}
= {......-6,-4,-3,—- | =(ANB)UBNX) ee
-(ANB)UB | | ae
D = {x: x is an integer divisible by 2 and 3}
={......—12,—-6, 0, 6, 12,......} | = (A UB) N (BY UB) Bg. a
| x =(A UB) NX : ae ie i
does not holdas -3 € A but -3 €B
(a) ACB
(b) BCC holds as any integer which is divisible by 6 is also divisible by 2 or 3. =AUB ’ a Se '
| =LHS. ia he Yi ae
(c) CCB does not hold as 2 € C but 2 €B.
GE A and B be two sets. Prove that
example 5. Let =
(d) DC Bholds since D=B oa :
B) U(B— A).
does D
(e:) AC |€D
not hold as | € A but AAB=(A —
a et
A. and B. Tris defin
is divisible by 2 and3 i also divisible by 2or3. Sol. A A B is symmetric difference of sets ia 1 tue e e tapeaaile
(f) DCC holds as any integer which B but not to both .
| orset
€ D.
C CD does not hold as 2 € C but2
(g)
| AAB={1:@GEA ands €B)
3. Proveethat AU A°=U and AN A‘=¢
_ Exampl
_ Sol. eT AUAS FIR cA <x
> EA or xe Ae natin: pS Accor Gohimaties isu
. Hence proved.
‘Example 6. For sets ee oa C usil

7) a U 1B)

Scanned by
Scanned by TapScanner
TapoScanner
tc _, poco
(ii) A-(BNC =ANBAC
nie
=(ANB)U(ANC')
=(A-B) U (A-C)

(iit) (A UB)-C =(AUB)N
a
=(ANC’)UBNC’)
=(A-C) U (B-C). pee

53 (wv) A-B=A-(ANB) MAS Het i} £34

Let x€ A-B EL 5 rnOC |


fa 4) Shue
ff xEA and x¢B
nae a
iff xEA and x€ANB
| 4 a |
iff xEA — (ANB)

A e a oan
eras
A UB=(A-B)UB ,
xGAUB | fines
Let
rEASES oF Ba Oe Bae
and x€B eee adil
xE€A

; f LS AcP GAD si am
iffx
as EA-BUB ee ish

Scanned by
Scanned by TapScanner
TapoScanner
apa
|

12 . Le t X= Cl, 2, 3, 4
~ Example
E X A y E X A (x — y )
is an in te gral non-zero multi
If R={% pix
lt
S= {<x pl|xe XAyE
X j (ey) isan integral non-zero mu

Find RU SandR 1S.


Sol. X= {1, 2, 3,4} gral nc
inte) )
RE («is—an»
ae
aa ={<1,3>, <2,4>}
eX tm Meet
mBE ittn rr we xr | 7 ee Bie mr
a.
1,4>} > pea eee
Sector
ee sy

ample 13.For A= 20 3}, 9}, d

Scanned by
Scanned by TapScanner
TapoScanner
=
Also g-AUB
AUB=¢
A=?¢. B=¢ > AUB=¢
:
From(1) and (2), we have |
. “4 a4 a Wer.
AUB=¢@ > A=¢, B=¢.

>, @}, Pn L SE OR A BN = AcB = ADE ti Sidi ag ae


18. Let A = {1, 2, 4}, B=,
Example - ieee
sol. Let xEA-B x@B
CAM © x GAand
oa Sais
reBs ;
_ a
x |
ott
og () eeAUB a ={I,2,4} U {4,5, 6} = {1,2,4, 5, 6} AcBGAGBSs
| ‘ e
ae :
ANB ={1,2, 4} 1M 4, 5, 6} = {4}
(ii)
ag weer
(i) A-B={1,2,4}— (4,5, 6} ={1,2}. |
a
sets of A and B? _» x@€Aandx EBS > xGAand x¢
Example 19. Is it true that power set of A U B is equal to union of power
AN Pape Bo
ik

Sol. Let A= {a,b}, B= {c} | a


thenA U B= {a, 5, c} Aaa
iS a3
P (A)= {@, {a}, {b}, {a, 5}}
acide
P (B) = {P,{C}} ae
i
P(A) UP (B)= {@, {a}, {8}, fet, ta, bf}
nests: Ingubaaale
where as ;
PORE
Pe RET ee

P (AUB) = {9, {a},{b}, {c}, {a, 5}, {a, ch, {B, c}, {a, b, c}}
showing that P(AUB) # P (A) U P (B)
Example 20. Prove that P (A M B) = P(A) M P (B)
Sol. Let X € P(ANB)
then X C (AN B) ee

= CB
XC AandalsoX vom
= X€P(A)andalsoX €P(B) > XEP(A)NP(B)
eg ER I pe es

Scanned by
Scanned by TapScanner
TapoScanner
SPECTRUM D SCREN a
26 =. “gall
3. Prove that each ofthe following reJations
holds
multiple se me (x: ‘is an integer m ;
(a) AC Bwhere A= t: x isan integer ege r} and B = {x: x* is an even integer)
is even int ~~
A=B, where A = (x:
(b)
eee : 1 False (4) False oT
Let U={0,1, 2, 3, 4,5, 6, 7, 8: 935A = {x € U: x mult
ofle
ip 3}, 4
proper (6) Notp oper
ais ‘
B={xEU: x -52 9}. Td {0, 3, 4, 5, 6, 7, 8, 3
‘wigs ;
petermine (a) AUB (6) ANB (c)B | (h, bf bo B |
ined as follows : eee {n, i,j k, Bb
LetA and B be subsets of natural numbers def
A= {x: ifp is prime and if x is divisible by p, then x is ble A: 4 {7,8 hese
eger y such that x=y}. eo | r (a,b, 4eh 8}
and B= {x: thise anrinte
proper. Be {a, a G a oh a4
Prove that B C A. Show that the containment is
, ey
‘Let U be the set of letters of the alphabet. Let A = {a, b, c,......,3} B= thi
and C = {0, p, q,..--».2}. Find the elements in each of the following s set: ee “it
(@ ANB (6) AUC (e) AN (BUC) | @ (ANB) hae .
aes

(g) A\B (h) B\A (i) AYB\C) § (j) SPP awe tea
F
(f) (AN BY
LetU be the set of integers and leAt= Sees: divisible by 3},le |
-
on vy ihe
a

1 7 B = {x: x is divisible by 2}. LeCt= ie x is civ ee


following set:
< #
(a) ANB (b) AUC io} Aree {. ol

() (ANBY (AB () BA
Ans ee
truw or r
false
eo eee UBY

Scanned by
Scanned by TapScanner
TapoScanner
Srectinia
a |
a
of th re e se ts A, B and C such that
ple
Example 2. G ve an exam |
A C H P A N C HP D U A NBNC=
Bee ANB#¢, B
t A= ( 1 , 2 3 ) B= 3 ,451.07 45,2)
Sol. Le
. B N C = (4 ,5 ) #9, CNA {2} HH
A N B = ( 3 } HP
ButANBNC=¢
orsetas lA,lB.
Example 3. Prove that AC BBC AY f
C B
Sol. (i) Assume that A

We are to prove that B® C A®

Letx be an element of B°
ys
i ee Be
ec

Scanned by
Scanned by TapScanner
TapoScanner
SPECT DISCRETE .
3
80 .( B- _ C ) = (ANB)-(ANC )
‘Examp le 6. sh ow ta t
N O ) = - ( A N B ) N A N O F = ( A NB)NAT CS
RHS. =(AMB)-(A ]= [ANA )NBIUAN G
g
=[(ANB)MA SJUL(ANB)NC
Sol.

-C)]
=(@NB)Y[ANB-O)}= =~ U[AN(B
=AN(B-C)
4,12.
= LHS. bed a as B be two given i non-emp

3 dae cn wo
EXEKCISE 1.3
oan
1. Verify the following identities : ARG
(ii) ANBUC=ANB)U
(i) AU(BNO)=(AUB)N(AUC)
* peau
where, A, B, C are three sets defined by
A={1,2,4,5 },
3, 5,6 }, B= {2
C={4,5 ,}
, 6,7
2, If X and Y are two sets, then find X M (XU Y)’.
3, Showthat ()ACAUB (i/)ANBCA.
‘ : i :
Prove the following :
(ii) BCA®ANB=B
f° 4.
a () BCAUB (jij) ANBCB
(v) ACCandBCD * AUBCCUD (vy) BCC*ANB
ee SI

hae

ie (vi) A=BeACBandBCA.
'r,

Bi | _ §. Forany two sets A and B, prove thatANB=~ => ACB‘,


La 6. Prove that
at () AM(A'UB)=ANB (i) A-(A--B)=ANB
fy. 7. Prove that A°\ B= B\A. | * ae im |
Prove the following:

as sm A-B=A-(AND
chee.
(B.

Scanned by
Scanned by TapScanner
TapoScanner
a ae |
of all num ber s, the n A con sis ts of all points ina line. A x A Will ¢
iis 1. IFA is the set
8} andH =(5:4,2), nd Gx
= Pe an ‘ ;

e co- ord ina tes are (a, 6).


ro
1
"
OE REL

os
:

Note 2. The ordered pair (a, 6) rep


an a pair
res ent s a poi nt wh ¥ 0. 3},H= (5, 4,2}
pee”
y 2
er of elements of A.
Note 3. Let 7 (A) denote the numb a ; iGen xe a x CG, 42 2} = {(7, 5),
Thenn (A X B)=n (A) x 7 (B). it ,

245; Prove that

f A xX B = {( a, ») (y), 6, x),
() AX(BUC)=(AXB)U(AXC) (i) AX(BAC)= Ax 3). cae exampl>e .
7 ¥), (6, x); Oy
Proof. (i) LHS. =AX (BUC) e A* BE= {(4, x), (a,
(
asst fist lements=
| = {(x,y):x@Aand y E(B UC) = {@, y):x © A and oil 2

= {(x,y): (x © Aand y © B) or (x © A and yEC)} : opel


= {(x, y): (yy) € (A X B) or (x,y) E (A X C)}

={(@ y):(@ y) €(A X B)U(AX ©} =(AXB)U(AxG) i ,


=RHS. ——
| 3 (A
xX B)U(A
* AX(BUC)= XC)
Ss Gj) LHS.=Ax(@BNC) eT
is 2 © A and
= {(x,y):x © A andy E(B NO} = {x y):x ‘

E rf = ee y): (« € Aandy © B) pride vi i_ a. e. e : | ned r

fea ={ &y): ‘erewamru)am ;


, =RHS.
o + AX (BNC)=(A x B)N(A x C)

Scanned by
Scanned by TapScanner
TapoScanner
a}, B= {2, 3, 5}, C={ -1, — 2}, then verify that
— {F.

*.
Ax(BUC)=(A*B)U(A©)
B=@3.8)0> 61-7)
Sol. Here A = {2,2},

2
3, 5,
(2,-
=, , -2)
-1}
=
.
BUC={2,3,5} U (-1
ig

ax- {5.2 2,3.9={(2.2}(5. a} (3. }12.2,2,3,@, -

mecltapercnelfgafe-nacaf oy
1 I l ,
Wy

‘39
* ‘
<

LHS.= Ax (BUC)= . U2 3512}


os , . EC = cEB
i {5 | 2| (=.
! 3}(3-5
I } (=.l - 1} & : je. 2),
(2, 3), (2, 5) (2,—1),(2 a
my
C Cc B
Wia a
i

RH.S. = (A x B) U (A x C) re

GeIeeHejancnag)s ie ase
ie
A
;
r

-(33}2}09}-9-2Janane
= '
3
i
nay

HII" ey 4 a:
} Tah
wollen
ae te hag ey
ie

LHS.=RHS. ai 4g ig
-

Example7. Let A= {1, 2, 3}, B= {2,3,4} and iar ee ; yt ie


get 2 cagbaseal B= @, 3,se ore spe a ohh be

Scanned by
Scanned by TapScanner
TapoScanner
| SPECTRUM DISCRETE MATHEM | _qevaTion any Buncear
L.H.S. == Ax (B-C) {3
> 3, ‘ s¥0 -41—=,TS aye -, sit-, cre a yep = tl mae xP
Ax | =i,
} x
(2, (2 }G a} (4 Arana) to #
L 4
i
4 is the set of all real nunumbe rs. | :
|
R.H.S. = (A x B)~(A x C) — 5. ? (ce. $9,
Migs.)
See a} Ppa Te
- {2 .2}(3 3} (4$5}.2.23¢ @3.a5} --{(¢ -1}(2. -2}.2,-0, B= } (7.ana (Po),(met a)
x {t2
2"
dB= 3,4),

3} GJeraras} |? somes
: 1. ae following 4
I l ah i i state whether each
of the 1Si0¥ ae
2 2 ~
a f IfP = {m, n} and Q= {n, m)}, tt
L.H.S.= R.HLS.
a if A and B are non-empty setsth,
Example 11. If A and B be non-empty subsets, then show that A
x B = B X A iff A=B “a a (ii) that x € A and y SB.
Sol. (i) Assume that A x B=xBA 33
| sol) i) WA= (1,23,B= (3,4), then A>
| os) Sam Le A and B be vo sets such that
NowxEA > (x y)EAXBVYEB (A
Ppa 2)s (on 3) (a, 2), (Qe 3). (as, 2)
we naA [off andB.
> +rEB Fd
P Let Band 8 bo fo oot
ia
“. xEA>xXxEB ee
12 We “find A and B, where x, y, 2
~ ACB e
Let z be any element of B 1, Let A= {1,2}, B= fl, 2.3,ae
Nowz€B > («,27ECAXBVIEA
; @ Ax (BNC)= “(A>pa
7 Me (ii) A x Cisa subset of B x
7 SRS sh i
> rEA a | .
12, LetA={1,2, 3},
Et B= {4}
ae. an
. zEB >= zEA
GA

6 wulatn Pate a gh 45 gee Re


> Na te ia ae fo chk oe

Scanned by
Scanned by TapScanner
TapoScanner
SPECTRUM Discp)
ih
I aa
7. SubofsAeXtBasre . %
3), 1, 3;
g, (3) (0. 4} {2 DH (2, 4}, (C1, I - i
(2, 3} {C1 3), 2, 493, {C1 4), (2, 3)}, {C1, 4), (2, 49},
{(1, 3),
49}, apie: ners ne noe
4)} 5 {(1, 3) CL, 4), 2, 3}, {(1, 3), (1, 4), 2,
; Let =

{( 3).2(2,, A 4 5 ’ 4

4)},AXB
{(1, 4), (2, 3), (2, 4}, {A 3), 2, 3). @ pet R= {(1,2) 2.2.6
A x B has 16 subsets.
r e f o r e cert tet
8. (i) False , t h e
12)eE 1) ERso ——e List
R
Here P = {m, n}, Q= {n,m} on
}. ate oe fs,
P Xx Q= {(m, n), (m, m), (n,n), (n, m)} er Domain of R= (1, , 2, 3
rs a
. 5 3}.
is the correct statement. ee
a l
(ii) True (iii) True +n 2: For any 4 i. =
9. A={ ah, a, a3, a4, as }, B= {2, 3}
10. A= {x,y,z}, B= {1,2}
13. {(1, 1), (1, 2), (1, 3), (1, 4), 2, 2), 2, 4), (3, 3), (4, 4}
1.16. Relation ~ 4)
A relation R from a non-empty set A to a non-empty set B is a subset of the czCa
The subset is derived by describing a relationship between the first element and t
ba ordered
pairs in A X B. The second element is called the image of the first eleme
fi? Let R bea relation from A into B. If (a, 6) R, then we write itas aR Pa
oe bo to b. ee

| 4 Ifa, 6) € R, then we wrt itana R band read it, ais not related to
a ae
is | Let A=
Example: {1,2,3},B= {4,5} | b's
At “ AXB= {l, 2,3} x & 5}= (0,4) (1, 9,0, 0,53 .

=; »-~jka(e ar

Scanned by
Scanned by TapScanner
TapoScanner
SPECTRUM D SCRE} .
40
on R can be re pr es en te d in tabular form as follows :
Above relati
4 5 ’ era a
R

0 0 }
1 Fe Kg :
:
l l
2

3 0 1
de a8
e | and if (a, b) € R, we
In tabular form, if (a, 6) € R, then we writ
e
con tai nin g | and the co lu mn containing4. Also (2, 4) eR
write0 in the row ‘ihe
ntaining 4.
containing 2 and the column co
| |
ons
Note: Total Number of Relati
eee es ss "I
1 eh te tivo noneemety feeaetntaioe/ gene
elements. Therefore numofbsub ersets ofA x B is OMB.

1.19. Particular Types of Relations


As we know that every subset af 83cH in Section ee aan
A into B. These relations also include gp andA XB. a6 g tb
1. Empty Relation + ari
i | IfR =¢, then Ris called the emptyrelation, = 6.04 sata

2. Universal Relation i Stem Wie ts


If xR is caled the unin
B, then=R A

tae TE

Scanned by
Scanned by TapScanner
TapoScanner
SPECTR
42
B = {7, 11}.
A = {3. 5} and
Example3. Let
Let R= {(a, 6): a€A.beB, a-b is odd}. Shona R is an empty re
{3,5},b):aB={7,
Sol. Here A=gait
11}
EA, DEB, a—bis odd } 4

Now 3-7=-4,3-11= -8,5-7=-2,5-I1 =~6 are not odd numbers,


.. Ris an empty relation.
oa

Example4. LeAt= {1, 2, 3. 4, 6}. Let R be the relation on. A defined by sas
_
aye
{(a,b):a,0 GA, bis exactly divisible by a}
() Wr Ri t
in ros tereform
(i) thendomdain of R
Fi
(iii) Find range of R. AY Fie
Sol. Here A={1,2,3,4,6} “a, ages
R= {(a,b):a,bEA, Bi ext aia os 4th
_ (j Inroaster form, : herr > (dsbbeib a

eos R= {(1, 1), (1, 2), (1, 3), CL, 4), a, 6),2,2),2,4),2,6) 3),
if) DomaofinR= {1, 2, 3, 4, 6} “ites
eee (ii) Ran Re
ofg = {1, 2, 3, 4, 6} os ge ae
Set 2,3: ‘dacs doe
(1,2), G, x), 4, »)}. eprstea in he ee ata ty AAR)
Robie A {l, 2a ee & » 23 se oR

Scanned by
Scanned by TapScanner
TapoScanner
; SPECTRUM DISCRETE yy

i) * Do ma in o fR = {1. 2. 3:4,5};
(i
Rangeof R= {2,3,4,
given
5,6}
below shows
o Oem
a relation R between the sets A and
B. Write this.
| _ ?.
Example The figure ‘OF Lae( 2

(i) Set builder form (ii) Roster form.


4
oo

Scanned by
Scanned by TapScanner
TapoScanner
pel
Lk Domain= {1,3,4},Range=(x, 9,2) | 4
2. Doma = Z , Rang
in e=Z : 7 Re
cee: 3. @ No; (4,2) Ristrue for g=1
3 A (ii) No; (4,2) ER but(2,4)ER
=. Ah (iii) No; (16, 4) ER, (4,2) E R but (16,2)
ER isk ach ‘
4. () R={(a,a):a €Z}U ((a,—a):a EZ}
oie A isi
(ii) Domain = Z (iii) Range=Z r e l a t i o n Ronaset
) A
4 5. @ R={@,1),©,2),G,3)} (ii) Domain = {9, 6, 3}
: (iii) Range= {1,
2, 3} example.
a _ 6 Domain
= {1, 2, 3, 4}, Codoma=in
{1, 2, 3, ....,. 14}, Range= {3, 6, 9, 12}
eee ey R= {(1, 6), (2, 7), (3, 8)}, Domain = {1, 2,
‘te
Ai sreaueeee {6, 7, 8}
8 R= (1, 4), (1, 6)2,,9), G, 4), 3, 6), (5, 4), (5, 6) 9 bin
rij
ee
i.

10% (0,D)42.23,{0,2}, (2D), (Dh (22),(D,0.93,


ee {(1,1), 2, 1}, {2, 2),1, 2}, {2, 2), 2 D}, {A, 2), (2, D}, a) ae
{0 ), 2, 2), 1, 2}, (0D, 2, 2), 2 D} {A,D, (1,2), (2, 1},
{(2,2), (1, 2), (2, D}, Ax A

>, 3), (6, 2 ©.


‘ 7,5), Domain = 45, 6, a ash

be 2 PLY: ate ame

flexive relation if (x, x) © Rh alll x ties


ial amor 8‘em gs

Scanned by
Scanned by TapScanner
TapoScanner
48

Asymmetric Relations : a
v R
e r4, thenb R a,
ym
asn me tr ic if w h e n e
A rel aat
R on A Iso
seti
e se t of re al nu mb er s an d let Rbe the relation‘<’,
Example. Let A = R. th bide
b ¢ a (b is not le ss th an a) , so ‘<° is asymmetric.
Ifa<b, then ric, a af ree i .
(2, 1), (2, 3)} - Is R sy mm et
lteA.= {1. 2, 3} and leR= t {(1. 2),
ExampLe
Sol. Here A= {1, 2,3}, R= {(1, 2.2, 1,2 3)}-
tr
eher
Symmetry : R is not symmeit (2, 3) © R butG, 2) ER
cec
sini
Asymmetry: R is not asymmetric since both (1, 2) and (2,1) ER
c 2) and’, DER
tri(1,
Antisymmetry: R is not antisymeesince
Anti-Symmetric Relation
A relation R ona set Ais called an anti-symmetrc relation ifaR b and b
ie, if(a, b)€ Rand(b, a) ER>a=b. gis ‘
OR * is a) 28 ite

hos ay
Arelation R on a set A is called anti-symmetric if a,b € A (orn
and (4,b)ER = (b, a) ER. sat) ne
ca a sie
Sea
Example. LatiA be the set of all ines in a plnne: Let Wi
l Lz. Since in any plane there
L; || Ly ie, if L; is paralleto ex
ee # a ie, Li RL, and LR Li butL, # Ly, the

Scanned by
Scanned by TapScanner
TapoScanner
r
we

SPECTRUM DISCRE)
a TeI é
iz

ae

Ee

SN
j

an d A) © X su ch tha t A; R Ap. Then A and A, are¢


R is Symmetric. Let A,
a

pl w
Bese ARAL Aer i e
he MAE T
ee

A Gos achdiae A ar ad 8 Rs
ee e
Bs eee (iii) R is Transitive. a 42,
tive. Let Ay, A. This alse ve and As tee
x @
cose congruent triangles and so are A:
. Hy’, ie he
Hence A, R As. af '
| 1
So, R is reflexive, symmetric and transitive. t Eta
o
EB iat ‘Therefore, R is an equivalence relation ] on X. = : i Leth iy '

Partial Order Relation


ye"

pare oan follo


ar i6!S) Sy Gigi
Re: BS | () xRx, for every x € X (reflexivity) ty Lenten
ho beaut
(ii) x Ry and yRx > x== y (anti-symmetry) inal ¥ cae i ;
6 he J

xRz (transitivity), x y,2EX


rt MmOGF

(iii) xRyand yRz >


on i “a por 4 =

Ge e
eq ui va le nc e re la ti on on a set X which is nlag. a parti
Remark: The only d by ER Y iff x=y. Ae ;
th e re la ti on de fi ne
relation Ix, that is, bee? ¢ exist b in B such th
grat t Se : ae aye
eee
*
, = ch

Scanned by
Scanned by TapScanner
TapoScanner
(ii) Let b E R(X,
U X2)
U X;
“there exist some a € X,
ik joe
st. (a, b) © R(X, U Xz) )
:
21}=)[4or][a]
in ta n (41-0 Va bE Xie, am
Now a@€X,UX; > aE&X, or aEX, jit) on X.
j) Since R is an equivalence relation 7 a

If acG&X, = bE R(X) Sia


proof ( R is reflexive. stoee eid

Similarly
if a€ X; > 6 © R(X)
WR ee
BER(X)) UR(X2)
ae
so. jp vera, be Xsuch
thats € [6] a

R(X; UX) ©RO%G) U R(X) . aRb


|
=
Ra
relation
we know X,; © X;UX2 and X>€ X,U X
Also > bRa@, s i n R
c eis eq ui va le nc e
we show that[a]=[4]. Lode e.3 silot 7 re Ae
By part (i) & R(X; U Xz)
R(X)) Now
72 Pers it nolaly ge Mf,
Let pela].
R(X2)
& R(X; U X32) ante
> R(X) U RO&) S RO%G U Xz) he ss pRa
nc e R isan g >quivalen:
BGs From, (1) an d ( 3p)R,4, si
a a From (1) and (2) i otc
“A pele}.
: = R(X) U R(X).
R(X, U X2) 7 pé[b}. Theref
ore [ a] ¢ ia “|
sopEla] oe
(iii) Let b © ROX, N X2) Now let g € [ 4]. ; 7a
& a
) ).
“. there existsome a © X, M X, R a (From (2
qR6. Also 6 a
| “i ce g € [a].
st. & R(X, MN X2)
(a,b) qRaand hen
a bee a ‘Now a@@X,;NX, + aEX, and EX, a2 )o8 De Aa (6]E[4@]
hone bERM) and bER(X2) > SPE GORROD). ph 3 Ti Hence [a]=
[6]

aan peel Ha) & BOG) ROX) | > (To? ua aie


Co nv er se ly
a
le

t [ a
a]
] .
= [b ] fo r so na
Fr o m (/ ), e
inc e [a)=[4].
o — a€[b},s

Sas ae :
a i). Leven

ie om aa a fi

ha iJ 0m

‘ahi—d ae%

Scanned by
Scanned by TapScanner
TapoScanner
NCTION ie
54
SPECTRUM DISCRETE MATE , peraTionAD® Be itt ‘that [a] and [5]
We prove that X= U_ [a]. . ae SS yin against UF SUPPOS |
aexX a ut is ;
Let a eX. (a) 9 [4] = ?- as
Then a€[a] X is union of distinct caval aes classes
a eA ae [a]
ihe set of distinct equivalence classes of R for
es XC yy [a] Bi he: of X, hee he ieee) =
Since [a]={bEX:bRa}, therefore | a ess Ayjuen be a partition of X. Therefore we have jig
[a] &XWVaeEx ae proof: Let A So) hg gelaR anaes
a . [fa] oX a (i) x= Dehiad a6) od 7 ce
ae 0) » Ay Ag = @ 184 8 where zane ai ——
From (2) and (3), we get X = pe [a]. pe eles te sepeieneoen Finan ; 1
| or
union of distinct equivalence ate o ‘ Then for any 4; b, ¢, EA, we have a:
be Now we prove that any two distinct equivalence classes are disjoint. iN 4 (p Ris reflexive
ee Let [a ] and [ 6 ] be any two distinct equivalence classes where a, b € X a Let a € X. 7 2 a
a We want to prove that [a] N[b]=¢. 1G a ~ by@ae A Amba trad \
If possible, let [a] N [5] ¥ ¢. ae i sibe antag
“. Hx ©X such thatx E[a]N[d]. ae Enercon
D. selo lends e[) ). 4 atiee
Leta
<< “ xRaandxRb.
| > @RxandbRx. | ee a and b belong to Ay for somea GA.
> b and a belong to some Ag. => BD
Now we prove that [ a ] = [6]. ceil. Gh | ‘ion: ; am
a) Leta RBand BR — - a mae

Scanned by
Scanned by TapScanner
TapoScanner
3 Sea peat SPECTRUM DISCRETE MATE . a :
Conversely we prove that each A, is equal to some equivalence of X. = i Ee oo saenty Sha . te —
Consider any A, a ;
Take any a € A, |
Such an a exist, since A, is non-empty. : ‘ oe (Bp Assesa re
} ia partion of A, So ry
We prove that A; =[ a] | : aset A, of the partition
fend fs a =
Let 5 € A,. Then a and b belong to same A, theres such that a
reflexive. =
“.F € [a]
bRaandhenceb Risis te 9 eee
t <= oe Say
(ii) Let
Le @, 4 € Asuch tha aRé. s i
peo bEA, »> x€A, > A, [a]

there is a subset A, of the partition such


Now, let x E[a]

“. | xRaand hence x, a belong to same A, ie, both b and a belong to some A, = bRa,
ButaGA, .. xEA, .. Ris symmetric. prs
~ x€[a] = xGA, > [a]CA, (iii) Leta,6, ¢ © A such that aR band bR o of
Hence A, =[a] | there are subsetsA; and A , of the partitionsu
ae “. {Aj};ea is the set of all equivalence classes under the relationR. 3 We claim that A, =A; . ah. Se— 4
(1.29. Quotient of A byR | If possible, suppose that A, #A ..yo Ther PA
a Let R be an equival
relation
ence on A. Then the collection of equivalence classes of the el at wala é +
is called Quotient of A by R and is denoted by A | R. nS enea disjoint. aea e Rd 4
SAR ial we EA). Butbis in both A, and A\,. HencA,
e n
oi
hn an we arrive at a sontraiction . ai
c a }

A, =) , showing that a, 6 an

thataRcandsoRistransitive.

sett 2 eo ti atLee , ‘ag


3 a. 1 = Ee iT

ines Of Lrolad 0 hanes =a as 9

Scanned by
Scanned by TapScanner
TapoScanner
4

Sol. (i) Let A= {2, 3,4}:


, 4) . (4 , 2) , ( 4 , 3 4 , 4 ) }
C 2 2 3 2 4 4 8 . 2 2© 3), (3
then XA= { (3, 4) }
ct a
, 3 4 , 4 ) 2 9 ) (4 , 3),
tet R= {22 on A:
A X A, th er ef or e R is a relation
Since RE
fl ex iv e si nc e (a, 2) € R VaGA.
Ris re
sy mm et ri c si nc e (2 , 3) ER but (3, 2) ER.
R is not .
si ti ve si nc e (2, 3) an d (3, 4) € R but (2, 4) ER
a is not tran
ym me tr ic sin ce (3, 4) an d (4, 3) € R but3 # 4.
FurtheRr is not anti -s
hs Es

(i) Let A= {1,2}.


Mg
@,2)}- “Naa
Then AX A={(1, 1), (1,2) 2 1, i dow Saale
Let R= {(1, 2),2, D}-
ation on the set A.
Then RCA X Aand hence R is a rel
t(a
since
Ris symme i)cER
r,b > (6, a)ER.
mi, 2d
paar S| Risnotreflexive sinceLE Abut(I,I)@A0
t(1, re )
% Ris not tr ansisinticeve(1, 2) € R, (2, 1) ER bu
a 2pe Saas ci
ae since (1,2) and (
Ee but | # 2.
and (2, 1) € R TS Risa
it
| eo Let os) Om

Scanned by
Scanned by TapScanner
TapoScanner
_ pt ation ASD a
SPECTRUM DISCRETE MA rip ian
r + a — *i

60
but (I, NER sagan
(x) Let A*={I,2).
—_—_—an Nowwil EN a
Then AX A= {(1, 1), (1.2),@, 1)2, ,2)}- a _ isnot reflexive
| ; 2 c (l 30) € R but G9, HNeéeR
Let R= (C Q, 2)}.
1).1,
7 g is not symmetric
RGA * A, SoR isa relation on the set A.
hm (20, Cs 9 ¢ ROA DINER be
e
iv)ER
ex(a,a
R is reflsince V aGA,
a " R is not transitive, aoa iisce i
etri
R is symmsince (4,bc
)ER (6, a)ER
J e) " qifRand R’ are reflexive relations on a setett on ae
R is transitive and antisymmetric clearly, ‘7
relatrion R defined in the set (1, 2, 3, 4, 5, 6) as R= {(a, b): bas,
thehe gxample + and Rare relations ona set A.
Example2. Check whet
reflexive, symmet c
ritive.
or transi gol. Since

Sol. Let A= (1, 2.3.4, 5, 6) , REAAsdR' SA% As


| ‘ RURTGAX Aand RRS AXA, wai
R= {(a, bi beat ly (aa D) = (0, 2), 3), G, 4), (4, 5), (5, 69}
VaeGAa ‘ RUR is RON ee
(/) Ris notreflexive as(a,a)@R
(i) (a, b)ER » (bayER [ (a4, b)ER © beat! @ guy)! We now show that RU Rls redlexive iseen
* Ris not symmetric, ae peta EA. isRs )
(ii) (4 VER, (0ER D (4 QER aa i (aa) € Rand(a, a) ER’.

(+ (GO). ).ER & dmatl ends) cma > and RARVGEA,


(aa ERUR
R U R' and R fa) R’ are only
' efl
_. Ris not transitive.
je 3. Let R be the relation defined on the set of natural numbers
N as
“rte xGEN, yEN, 2x+y"41)} hb Ra. Prove that
(i) ae
oO R is symmetric if and on
1. (/) Kw dand by Eb

Scanned by
Scanned by TapScanner
TapoScanner
cs

SPECTRUM Disc 2
62 ned
er e R is in th e se Z
t of integers.
des a5}, wh
Sol. R= {(a, 6): 5 divi . ai i
) g-a=0=5°0 —
> R is reflexive.
= §dividesa-a > ER . ys
(ii) Let (a, 5) ER aah
. Sdivides a—6 eee as
Z> b-a=5(-n) > S divides b—a
= a—b=S5nfor some nE€ aa
~ (a bKER > (6 a)ER
= oe an 4
.. Ris symmetric. ‘ a
‘i Chae
R
(iii) Let (a, 6) and (6, c) € MAES toe
= §dividesa—b and b-—cboth
Rea 2
ore a)
gata sn) and b= e= 5m t Oe ves “Ag ae

(a-b)+(b-= 50, + 5m ia

c > @oer™
wich big

+ n ) > Sd iv id es a—
ae c= 5 (ny
> (4, c) ER
ni aol pe ABER,

Scanned by
Scanned by TapScanner
TapoScanner
= SPECTRUM DISCRETE MATHp, } 10N AND FuncTics
9, If R is the relation
Example by (a, 6) R (ce, @) if and only if a+d=b +c, show t
in N X N defined ee ‘ Let 4 2 b (mod m) and aa im) ober
(i! mand b=c+hm |
is equivalence relation. aatk ab+hm fe
Sol. Here (a, 6) R(c,d) @ at d=bte. pm thm | eae
ee ifa
+ b= 6+ a, which is true. : +(ko+h)m s oh
relation R is reflexive. gle = a=c(modm) Re ®
a=¢ ita eel
(4) Now (a, 6) R(e, d)
=> a+d=6i¢ = dt+a=c+b > c+b=dta => (c, d) R (a, *) : ae
~. relation R is symmetric. : e Hence ae be a relation define d on the set cree
aa a 12. R is a partial order relation. FA.
ji
(iii) Now (a, b) R (c, d) and (c, a) R(e, f) Exam Pen
=> a+d=b+candc+f=d+e > (a+d)+(ct+f)= (b+c)+(d+e) => a+f=bte 4 caretexive since @ = for any ral number a

> (a, 6) R(e,f) ( op Rieaattermmtse: a0 Bea


relation R is transitive. in f gebcand sa, * Ome eRe eee
oo seat ae easly 2 az til) Ris transitive. Let 4, b, c be any real numbers
es eee eee a . asband bse = ascie,aRe > Ri ere
_ Example 10. In N X_N, show that the relation defined by (a, 6) R (c, d)if ad=4cis an equivalency + Ris reflexive, i-S) ic and | oe FQ |

| ew a Ris a parca order relation onthe sto


Becp3 Seren. YR (cd) ad=bc (i) Now (a, b) R (a, 6) if ab=b a, which
is true ae gut GEE:
J i Say H
= relation R is reflexive. t da x
. *7 Sie

: @) Now (a. b)R(c, d) ay si

= ad=be 2» da=ch > ch=da = (c,ad)R(ad) viae


soe +: rela symmetric.
R istion int
i ) Now (a, 6) R(c, d) and (c, d) R (e, f) od
seree tees cf=de => (ad)(cf)=(bc)(de) > adcf=bcede ' Valea

{* ; ‘i
Tay

Semin re al doe
{ey 42} 2) hae) a) ie =

Scanned by
Scanned by TapScanner
TapoScanner

ER and (d, T
e
= {(2,
: esses ana
= (ao ER, and (a,cE )Ry
= @AER OR
Thus R, M R: is transitive.
order relation.
Hence R, 1 Rz Is partial
ial order relation
But R, U R; need not be part
For example : Let X = (1, 2,3} then
B, 39}
Xx X= {(, DA, 2.043) @ Ds 2,2). 23) GB, Ds G2), |
Let R, = {(1. 1), 2.2). G, 3), (2), G, 1}
= {(1, 1, 2, 2). G, 3). 1, 3). @ DF
casas R are both relations o
x X, Therefore R, and
Then RySe
R, is reflexive since (a, a)ER,V aExX
ha = b,
R, is anti-symmetric since for no (a, b) € R, wit
we have (6, a)
© Ry
R, is transitive since (a, 6) € R, and (6, c) E Ry > (a, c)ER, 0 v-taid , : a
~. R, is partial order relations on X seed Sinko =
Similarly, R, is also partial order relationon X M4 at ena “edo “Sal
But Ry U Ro= {(1, 1), @, 2), G, 3), (1, 2,6, 1, 9) Oey 2 hae
1 (1,2) E east DERN ae
= a Katee or} saa

Scanned by
Scanned by TapScanner
TapoScanner
SPECTRUM DISCRETE MATHEM,,.
21. Is inclusion of a subset in another, in the context of a universal
class of subsets of the sets ? Justify your answer.
set, an equivalence relationin. }
22. Show that the relation R in the set Z of integers given by R = {(a, b) :2 divides ab} ee

23. The relation R © N X N is defined by (a, 6) € R if and only if 5 divides b—


equivalence relation. ae na = 7: is % peli tion of eons ty sets A s bset rof X x
24. In the set N of all natural numbers. Let relation R be defined by
R={(. yx EN VEN: x —y is divisible by m}
| 35 iat andnd Nisa
¥ be unique
two” y Y: such that x,y) =F fi ¢
: : i eX ‘1:25, 4}, Y = {a,5, ah ikl
1 os :
7 a ce eee et For wen y ~« foreachx © X, we have y\ E Y sucht
or a,
cg set o numbers,bers defined aaS § 4bif|a|=|b|thenS
j = is: an equivalence
35 relat, ’ gnctio” from
mee Ost.
is also represented by following i iae ae ae
x oe : r 7 a: i 4
Abo

26. Let R be a relation on the set of A of ordered pairs of positive inte


=

ts defin : Vs hing ube


a
and only if x v = y u. Show that R is an equivalence relation. = sd by G9) R(w, “dif 1—- Te ee x a ,
a fala: ie
SY For
or Ag € Q ~=the set of relational
i numbers, define bGe eS iG if and
eee. only ifad=be 2 a
Shoy, = (an i
that R is an equivalence relation on Q. a 3 }
28. Let N denote the set of all natural numbers and R_ be the relatj 4 Nenei © Ae4
N X N defined by (a, 6) R (c,d) ad(b +c) = =be(a+d). Cheek witether'R is ah enuiva ’ :
_ relation on N X N. oe another Definition of Function ;
29. IfR is an equivalence relation on a set A, then so is R™ | Let X and Y be two non-empty sets. Thena rule
—} element of Y is called a function Kip P ccied
For any a, 4 EN, the set of natural numbers, define aR b if and only if aaiviaesBo Thehtn
# es ee i isa The other terms used for aoe tions 2 © ma
artial order relation. ie
pX> Yor x—4>Y. a > |
The set X is called the domainn offa ne
; Ifan element y € Y is associa ated wil )

7 Doses: M4 iTpuei2 oe aie ube Bs under the: ruled peur’ ag) @). |
“ FP Sa SasTY en
ge aie
aoe a

" 198 = vt

Scanned by
Scanned by TapScanner
TapoScanner
CTION. “s $$
SPECTRUM DISCRETE MATHEMay, | _. p51A HON AND FUN
en [" cel s, BE of Aes ta Rts Gad if ¢ Se pe

XX Y= {0,5),
92,0,
2 2B,0.
9).B,0,
5.G,7, 4,5)
2. 4,6),24, } “a MG
Let R be a subset ofX x Y where R = {(1, 5), (2, 6), (2, 7), (3, 6), (4, 5)}
_ Here R is not a function from X to Y as 2 € X is associated to two different elements 6,7 of Y a4.|
@ function no two distinct ordered pairs have the same first element. But R is a relation as RCX xy 7
_Agaim
take R = {(I, 5), @, 2), G, ), (4, 9)}- In this case R is a function from X to Y as each eps
of X appears in the first element in one and only one ordered pair in R. R is also a relation from X toy. a
Remark : Oo Ee 3 a uni €Y such y=. : * ne. oo: ge ge wae

ee Saino nerdl SAN


The saan ciorenhiae < Vis aly yy The Tule hove ae fa
(i) Different element of X may be associated with the same element of Y. D eoent
of ¥- Side
(iii) Theremay be elements of Y which are not associated with any element of X. aly
of function fat x.
(7) We refer to a function as fand not as f(x) which is the value

we 7 Sr me Sh a a
However, by an abuse of language it has become customary to call f(x) as function instead of f ig

sa e.g., y=f(x) =x’, x is independent variable and y is dependent variable.


___ (vi) Functions are generally denoted by f, g, h, ,......
Exampl s. The rule shown in the figure is not a function as each element of X is not associated. Here 5 ¢} . j) The rule shown in the figure.
r) A i

a =
~ Sve
fe 7 7

ite esis Ai

Scanned by
Scanned by TapScanner
TapoScanner
SPECTRUM DISCRETE
1 110 [ T ean
MATE, Epi All soe
A function /trom X to Y is called onto iff every element of Y is an image of at least one since "
. y uh y be ‘aie is os ds it very tun
sp citer words wo can say that for every y € Y, there exist x @ X such that y = /(y), nn nell on on re ota : _ eines Nemes

Into Function : A function which is not onto ts called into, Function in ns


fig. 3 ts into Ninetlon, 1 yw! _ valued functions /4ond. & wes “7 a
Remark : In case of onto function Ky Y where as in case of into
funetion Ky is proper subset of Y, r {V8 (x) Vix € Dy, We w
Examples, () Let X= (1,2, 3,4), Y= (2,4, 6, 8, 10)
Then the function /depicted by the diagram is tL into
4 constant tu we Viveaieda sil‘ % WA
‘ 10-© ¥ has no pre-image in X. i aa A function / she : ere
x y ‘an - pynetion shown in figure a
|
Se oe fe Mappi
g X-»ng X be dined by, Hy (=x LeVr eX,ya
eee aaa 4 called the identity mapping ox,
paseo a- | Mapping ip
an
Lag ontoe mappir
Pa x» ¥ be # one-on
Fig. 6 an x @t X
eioment y © ¥ the unique elemen
(i) LotX = (1, 2,3,4), ¥ = (4, 8, 12, 16)
Then the functi / depion
cted by the diagram is one-one onto. Lat X= (12,3) Y=(a be} nd fi
a aa e:
exlat and Is dofined by f! :
: P= DODD
Method (0 cheek one-one (injectis
Lot /: X= Ye yn a
(/) ‘Take two arbitrary ¢ lom
) mes
pe Solveof a) ‘ / (,

Scanned by
Scanned by TapScanner
TapoScanner
*
ee ors = ® *
J lila Pa

14
Pied ens ia reais

ya
: L i Por

: = - ‘ 5 = an ‘ ND)

at f is not one-one.
*

x) = x2 , Pr ov e th
fined by f(
Example 2. Let f:Z > Z 4
e, We take one example.
Sol. To show fis not one-on
ay sae
Let us take two integers 2 and —2 ee
then f@) =@y=4 | abs
a f(-2) =(-2=4
ae Sinc2e# —2 but f(2) =f(-2)
= So fis not one-one.
ctive) :
E Method to check onto (Surje
| Letf: X > Y be any function.
Y.
: hs (i) Take one arbitrary elements yin
‘Teta 31a
ie (if) Take y=f(x) 2? ae
rms of y.
(iii) Solve this equation and find x in te
.
Gy) Ifcorresponding to everyy € Y, there existx X thefisncalled
Peas
| — *D

Scanned by
Scanned by TapScanner
TapoScanner
f(m) =f (m)

3 | m| =]m| > m= #2
+. _fis not one-one function
hgeemk S962 pa 7 ead
= afey=5, (35 Peep tee
So fis not one-one. Te
{0} £2

function f:Z > NU {0} definec


ok Hoe et age ai. rel Avhabee

wd

Scanned by
Scanned by TapScanner
TapoScanner
78 : a SPECTRUM DISCRETE
MA | .
f(x) = 17!
i |-— 7 (if xy ER suchthat f(x) =f(x,)
- SF Y@ON= ait in [> ofq)) gne-one Peas ys ee Sa
rasa " |x| = 1%2 | > % = %X2 or x “—e Tee hee
Piers (3) =3 and f(-
| aant one-one For example
ai ined 0 wiever* ree Path see
~ SLY @)] =x. sg y7 oy ER then VyER a
Example 4. Specify the types (one-to-one or onto or both or neither) of the following function : 4 x €R such that F(x) = y | :

(i) f:N—>WN and/(j) =J(mod 4) at sat: ‘ eel

(ii) g:N XN-—N such that g(x, y)=x+yp | le 5. Let A= B=4{1,2,3,4;5)— Define fictions
(iif) X=R, Y= {x: x © Rand x> 0} and f(x) =| x|. | : ae fis one-to-one and onto a Shes 4
, (i fis neither one-to-one nor onto a a9 i
Sol. () f: N>N
| One-one: /(/j) =J (mod 4) LC) = J@mod 4) means J is remainder when J is divided by, (iii) fis one-one but not onto. san 2 1
4 fis not one-one : (iv) fis onto
but not one-to-one. eres.
: gS £3) =3 (mod 4)=3, f(7)=7 (mod 4) =3 ¢ : sol.() Letf: A>B defined by f(x) =x, Vx GA.
Now £3) =f) ie f={(, 1), 2, 2), G, 3), 4, 4), G, 5D}.
But 3 #7 / (ii) Letf: A> B Seton 3 ene

es
Onto Eero ia a
} #FGnenen¢
<i e
bse
‘<i eg Merr;

5 EN therevisnojsuhihay/)=5 tenia “ection fom Ato then tw

Scanned by
Scanned by TapScanner
TapoScanner
80
Onto : Let yeER
such that y =s@) |
: * oe
> yextTexryol ea
|
that y = 8) Le 2ie
go Vy ERAxERsuch 7
Hence g is onto.
(ii) g(x) =x +*
g+0=0, (Da tGD=0
_ gisnot one-one as 8 (0) =
But 0=#-!l

Again let yy ER suchthat y-e@)

a =

al al
“i os
er A lees

Scanned by
Scanned by TapScanner
TapoScanner
|
a

ee Y. oot oto see

p o p e 8 O N V O M E E R
2 XG

os
we eee eiee
5 aomwob
. i

44.0%ae MAES shake thd


Oe) aa
r 99 -* p n= oa of is as so ci at iv e, — +
24 . i

comp os it io n of ma ps
e as
,

C are both,1 on=e


Same

Bais 2 eh ae f a > B a n d g : B >


oe ako
+ B a ndg:B-> Can
Z such that — “si og. since s A
sandy sg
g be a fu nc ti on f r Y
o to
m
to Y and
Let f be a function from X
f= {(, 3 @ , 9). 6. 15 )} .8 = 1 , 2 ) @ 8) , (1 5, 14 ), 21 , ae | (sof) wi=(eoNG 4
. 0) Fes)=f G2), since
then go f= {(1,2), 3,8). 6, 14)} a, ae Eey oft a one-one map. —
at
Tt must be noted th
oe ks | | a A> Band g:
(i) gofis defined only when RyC Dg. s see. uy gareonto a A
s pos sib le that one of fog may be defined while the ai
(i) Iti l. Tide 25 ent, then 3
be def ined but may not be qua
(ui) go fan d fo g bot h ma y
tf be a function witdomah indomaX inand Xranandge ran ge 7

posite Function: Le in
eae The funtion with

Scanned by
Scanned by TapScanner
TapoScanner
8)
,
a\
er s
Pat
helt

g may not be o,
La i

that ifago, is one- one, then


igo a
aes

e to illustrate
dl

,
es

We, now give an exampl Oy


ao

|
al
=

>) eee

B= {4
ie
ee

Let A=pi 2},B= = {7,8


rh. tl

oo
oy.
The

bens
i

A to B sn fr om B to C respectively.
from
then fandg are functions
es

Ry S Dg
f

> gofisdefined and Dgof =Dy= A= {1,23}


ae

aLeer

4)=7
gof(i)=g (f())= =g( o 4) se:
=

>=g(6)= 8
g of (2) =2 (f(2)
a
gos t t. 7)(2.8)3
= ;

perf g of is on e- on e ma p sin ce ofeAn


different elem ve s
hat different i
E fe Here,
Butg is not one-one since ie
k
t 54 6 104
g(5) =g (6 )= 8. Bu
n e s m p meee then |
1 B e r e t s B a r

Scanned by
Scanned by TapScanner
TapoScanner
_ Topo onto ide hh
. eso there exists x © X such that g (”) = *-
S@) > ya¥Gyli
= f(g())=/@ * (fog) ()=/@) > WO)= |
-. fis onto.
j
(i) Assume that f: X = Y is one-one and onto. We are to show that fis invert ible,

and onto
4
ae x= %2
oman Vn,x
aaaheter exist one and only onex €X such that f(s) =y, goer a
we can define a functiong : Y > X such that g (y) = x iff f(x) = y on é ge
ae

fs* gofis!— "


= are onto
= i.
7 a

Now (g of) (x)=g(f@)) =g 0) =x, Vx EX f: X F a t e


(ii) Here
Bi gof=lx X to Z
gofis defined from
Again (fo g)(y)=f(g (0) =f@) =v, VV EY g from YZ
“ gis an onto mappin
. fog=ly uc!
FS _Fis invertible and g is inverse of J. al unre, io
F
1.47. Ifa unction f: X = Y be one-one and onto then f is also one-one and onto.
Again as f1 X such that f(x)
th er e ex is ts x €
Braet sf: X + Y is one-one sooty | to each y € Y,
at j
2 Ix fof =ly
F + Y > X exists andf- ' 9 f=lx to eachz € Z, there exists x exe
Barn nme z=g()=g(f@)= «e
ne
ee,
ees
, Now fOn=F' 09
*
ee.

aes i O) = (fof YOn= (of )od


t pcheens OS ee
Bt ae |

Scanned by
Scanned by TapScanner
TapoScanner
88
=!
so. (Foo W=/(g()) =f4)
(fog @=/(¢@) =/M =3 wae
(fog) @)=f(e@) =f =? ”
(fog) (4)=f(g (4) =f) =>
(fog) (S)=f(g 5) =f@) =3
=!
(gof)(=2(f()) =s@)
=8 5) =3
(g of) (2)=2 (F2))
(g of ))=¢ (f(3)) = G3) =!
(gof)(4)=2 (f(4)) = () =4
Sstt ie
(g of) (5)=g(f(5)) = 2) =! |
Ss fogand go fare not equal in general. = eae ig
Example 2. Iff,g:R > Rare defined respectively by f (x)= e+ 3x+ hoe ¥ le 9 . ‘ a

for (i) fo g (ii) g of iii) fof (wv) gog. eh Beek 1 er om


Sol. Here f(x) =x +3x+1,g(:)=2x-3
fp nell (fog) =f(g @) =f2x- 3)=(2x-3/+3(2x- 3)+1
oe TA =4y-12x+9+6x-9+1=4x-6x+1
(x) =g(=f@) ))=2@74+3x41)-3
gO? +3x+I
-6x+2-3=2x" WERT ey ba $34

Scanned by
Scanned by TapScanner
TapoScanner
@ y=, 2)
2. fisone-one ie, fis injective. | |
eo
A

sty Let (s, £) be an element of the codomain R X R. We determine (x, Y) such that ig
a F&N=G.D > &+YR X-WY= Oo) ae ‘ =) a y
i!

e tReF
—o

a
foam

el
aiepa

and x-y=t
a
nny

> x=“ ER
avy

Adding (3)and(4),2x=s+t
St+t
fre 5 kets
. e

ie
ot

AD ER
= Y= SP
bar

sb cting (4) from(3),2y=s-t


.

Scanned by
Scanned by TapScanner
TapoScanner
D Ss a] 1 S

92
ngeasl
val ued fun ction defined byf@=x,x ER. la fRi
Example 8. Let f: R > Rbereal
sol First we check weather f is 8 bijection or 00%
For this let us check f'to be one-one-
Let x27 ER

such that f(x) =f@)


> x? = xy 2% 7%

=X, or ey ey
x

nots
So fi I-I.
Hence fis not a bijection ~- fis not invertible.
ee
lteX = Y = Z= eet
Exam9.pLe
f(x) =2x+ Land gy) =y3- Verify that (g of)"'=f~'og™'
So. f:ix>y f(x)=2x71

g0)=5me

2x+l
wHS.: gof(x)==8(f@)=82x+) = ace
Now we find (¢ of”

Scanned by
Scanned by TapScanner
TapoScanner
A
94 R > R be real valued functions defined pee s
by f(x) = -
x eS

1/3 « ective and each is inverse of othae — 3


1 c+0| _x ER Show that fand g are Py ®

g(x)= B
g both are one-one
sol First we show fand | a
Let Xy*2 € R such that
esa

g(x) =8(%2)
f(x) =f @)
a eat RU)

Ln eA1/3 cee +ee 41/


2x}-1 = 2x3-1 re +| ae a ae eae :
a

;
i

es

a # 4

ee.
7. +a
rs

a
rid

= 2x3-1-2x3+1=0 TD
el it ’ RA

-
ete. is

so

>i.
—* aU
ori
i

an atta
=

= 2(xp-x}) =0
ni

—_

:
-

if
ae
7

gat
Sree,
PR,
a ee,” se ee

Soe =
:
«
et eee ede ia
=

ie

4
aa
SA,
age
es

Sect

ae
4

i)
ul

Scanned by
Scanned by TapScanner
TapoScanner
Now V yER (y #0)
3 xER st. y= SO)
so f is onto.
f is one-one and onto.
itsiv
so _f is bijeinct e.
domain
if ais even sna me
&
* (=f FO) SO)"
ee
eee

SOLNE . Find fog andgof. a


Example is. f(a) =a*1 80 a if ais odd Ee 0) =f FO) -L0-2
pes PesRNS 222

nm
Se

if ais even
itainna
oe
st fos -FE") 5-1, eginodd | 2.
SS peers

2
eo

gof(a) =2(f(a))
Pies deidg
aniiliieaiabaaiea

;
E

| “2 i flis even a
Te

se
ae
hy
eae

paper

salen
a

1a
iv

F
"s

F
*

3
ev) ng

: | fos itfoisots
-~

ee
a

5
*
Arh

an,
eT
e
Is ‘i coe ar

le |
eS
ee

ege
sn
|
he Pied

eh
eee
ra
Pro

Scanned by
Scanned by TapScanner
TapoScanner
SPECTRUM DISCRETE ae
Mar i
jons. let fand g be two functions from Rp ! NYta gets oF
ition of two functions. 3

oP eee
y=(x+2 —x). Letg: RR} fa: caer
3 = 1). Is composition commutative *
ae Lou Kn, BER be det
-RXR?RXR be defined by [3 ) sais! |
Find FORMS hoy
re. j

as’ eLet h=R X R> Rbe defined by 1 hy) =x*2y.


0
ie
14. rp mo Risgiventy f@)~32— bint J 9 :| Sees
Why 3 gp introduction
se The fmetion f(@) = 2x7 ei Obes en inverse el to every one that an infinite set has infini ioe apie
7 i
So, find the in verse functi
num
in its domainin? ¢ IfIT so,
infinite
invertible cn} ber of elements 4s there’ ar
known
. _ _*_ 4% : ae two infinite sets ?
16. Isthe function f(x) = <q" numbers N. sek CFTR men
J: A > B be a function define, example : Is there any relationship between the set of natural
17. Let A={-2,-1,0,1, 2 3}, B= where {0, 1, a sous ,10} =
C = {0, l 2, 4} ‘h, Bes R, set of rational numbers Q etc. ? In this direction the concept ered Re piace
f(x)=2 for allx € A, find f~! (C) :
comes into pict
| el om and consequently the concept of countable set and uncountable set
18. Prove that each of the following function is a bijection : ©
(a) f:RXR > RX Redefined by [&y)= (+2 =)) we will study countable set and uncountable set and some oftheir properties. were.
n+m) | ‘10. Equivalent Sets
(5) fi ZX LZ x Z defined by f(m,n)=Gn*m Aset Ais called equivalent to a set B, written as
A ~ B, if there existsa func
fis a function from S into S, then Prove
19. IfSisaset containing finite number of elements and “iaae
and onto.
~*F ope-one
1: following oe . : .i
Iffis one-one,pes fis onto ; example |, Show that the following sets are equivalent
e () | ia feb (ie
: : ( A=(2n:n€Z} andB=2n-1:n@Z}
= (ii) If fis onto, then fis one-one. be
oe l« ho
) =. | ae + as ia 2
Prove that if fis aab) bijection then (f-
Gat ae (ii) A={n:n EN} andB= {2n:nEN}
mp. that iffis in nhta Wit MO ia
isl * Sig (ii) A= {nin €N} and B= {nin € 2} oy Uy ar
ee Prove that f (x) == 2 27 par ee hes Hanes dees Tt a
x. Determini |
22, Let f and g be functions from R to R defined by f (x) = [x] and g (x) =| E R such that3
{x:x < an
eh Lisa (vi) A={x:x R such that 0 < x < 1} and B= cee EPS Gatiy Cash
~\ efogegos.
33, If #A and #B are both finite, how many different func from A intoB,
thereon
areti , () A= {x:x © R such that—1 <x <1} and B= lars Shs
V xe. an = Pr tie
i a a rane
5 2 2 0,7
2
ae i be é “4 -
ol. () Define a function fe :A>B as S(x)= x1
ae
\ os ee
isd oh ig ate ee ae ae
ey: 2 “ ‘an £5 co
ba oe at ff se
} a | '
-_ ’ ,
or Tape! So
" fs a=
s

Scanned by
Scanned by TapScanner
TapScanner
ene am pO 5 ea Bae
4 we show that f is biject!

—raromane: ats SASERIT HRs


si | Me 252 aa

ae | se
inte
i>

fisoneone
i =

c.
e s e e ed
y el em en t e e o a
For onto Let y€ B be an y

io “hus B= € A suchihat f= FO)“ 20-9 ee Gh gies


tt ‘
was kG 338 glemnage: yy:
avieemoan. bert
= f is onto s
oe qe prioy Litt Fat siclnastarspst
So, fee
axe ~B.

The eee

Scanned by
Scanned by TapScanner
TapoScanner
ax
YxEA
= tan
f(x)
We show that f is bijective.
For one-one Let x),%, € A such thst f @) Fp) hess ae
JF"2 222nd, VnEZ
-

ide!7 BX2
soe=

=> tan = tan J calle


aeons

> % =x,+20,V nELk * %—%2 ssn a rey


col
> “i ne
eee
te ak
But x,, x5 EA=(-1,1) > i
: gaily Be eS ee eit as

YS eee

Scanned by
Scanned by TapScanner
TapoScanner
, Nee EEE
‘104

pe ete apes
Proof, Define a map f(a, 6) > (6 @ Py
ig ee
seyze+ = &-aV x € (a, b) . i
b-a
map.
We first show that fis well-defined
nih Gi
Letx € (a, b) be any element
> a<x<b > 0<x-a<b-a

x-a x=a ea eee ie 9.4


> te eee A
= NSS

2 exes (x-a)<d ie c<f(x)<d


dep 2
pe deutaetnea oe
Rar docten

Scanned by
Scanned by TapScanner
TapoScanner
a+b
a ele
@ kone ipa Se i=;
2 b-a 2

a+b
———

= r<o ie. f (x)= tan


@e -o<tfan
-a

:. _f is well-defined map.
map.
‘Next, we show that f is bijective
one -on e. Let *), 2 E (a, b) be any tw o points such that f(x)=
- For
®

Scanned by
Scanned by TapScanner
TapoScanner
e v e n in te ge rs is equivalent to
pl
Exam1. ove that the set of
Pre th e se t o f e v e n integers nd
h
{2nt+1in € Z} be
€ Z} an d B= och eee.
Sol. Let A= {( 2n :n A
yext1,VxE
Bas /@:
Define a function f: A> ee
ive.
We show that f is biject
t x, ,% ) © As uc h th at f(x) = f' (x5)
For one-one : Le
eee tar ey
> ti

Thus f(x,) =f) 7 *1 = Xy


“(fis one-one.
element
For onto: Let y © B be any
Fire \s
+ Asomen€Zsuchthaty=2n+1
t 1 = y
ae

M = 2 n
=

f 2
iy

u c h t h a t f e ) =
‘Tus 3x=2neAs
oF

Scanned by
Scanned by TapScanner
TapoScanner
110
** | x , - % ) / < ! > son=0
But xx, €0, 1)

x Fa)efopr ar”
So, f is one-one.
For onto For any y € R, we have

! such that f(x)tan= |x->]=Yo


If y20 thn 3xe

5 if y<0 then 3x€ 05 ee 5 a

Thus V yER, 3x E (0, 1) such that f(x) =y


ed

». f isonto
Stet
Ze

e
ononto
So, f is one-and
owt
me?

Hence(@,~Ro iA cosh
.
a‘,

4 oe
-

ea (qc atht. 9 = || 7
"

ie. Show hatfcEoRO r,—


rr
=
ie

aan
+c

a+ned1)by
rthe map f:(0,1)*(adefi
apg

hes
ro

hi
i
a

Scanned by
Scanned by TapScanner
TapoScanner
:
POY : =

ng AB
| (iy Deine she mapi EAx OR ce e‘eea =

|
(0) V (a, 5)
h' (a, 6)= (7 (a), g | on
.
We show that i’ is one-one an d onto
su chthat
Let (a), b,), (4), b,) © AxBbeanyelement ==
For one-one i a*
oF

h! (a,,b,) = H (ays)
vis yg
* * wi

et

> (f(a).¢0)) = Fm)8 CH)


> f(a) = f(a) and g(b,) = 8(b,)
> a,-a, and b=5,
> (a,b) = @,by)
.. hi’ is one-one. . =
For onto Let (y,,9,)EPxQ be any element sonal 2.2 btw ,

Where y,EPandy,€Q
As f:A>Pandg:B>Qonto | .¢

J x, €Aand x, © B such that f (x,) = Ve


(eee
ar oe

: = a

a
er areal K
Sey ee ee ee
ne x.) = (x), r(x.) = (yy, 75) F
— ah a See eee aa 4 ‘i

n F
=. ih - ee sd et: i iy L
t +
,

Scanned by
Scanned by TapScanner
TapoScanner
14

— rable or Enumerable i set): An infi Ait


ite set (or Denume ark 2
(@ Countably infin set | eee
nu me ra bl e or en um er ab le
countably infinite set or de A is eit her fin ite se t or | :
A is said to be countable if
(iiTa) eCountable set: A set mo st co un ta bl e if A is ether a fin
t Ais sa id to be at
(ii?) Almost Coun table : _A se r
:
t An inf ini te set , wh ic h is not countably infinite (o
(iv) Uncountable se Bie sake
be uncountable set. i
nta bly inf ini te or den ume rab le iff its enene can be Put in
1.55. A set A is cou x a
sequence of distinct elements.
Then by defi niti on A ~ N. a ee rt
Proof: Let A be a coun tabl y infin ite set.
re
Theref funce,
3 aor tion f: N + Awhichisbijective =
i ie Vn€N,a’sa, €Asu f(n)= a, and
thatch
Boer a
| fn) = f(r) > m= 7M, [orm AM
A= fe teen a
{a,, a; creme y a, > matits where a,, | a5, junds
Thus

Conversel of Aen
y, elem
let every put in the forofm an in
can bets ud a, a But | ti :
where ay a
A= {a * Gass eed) da net
Let
te
- rid *

Define a function g: Ne Assos Bs V.0eN. =


bat
ee ie
per stiestae
Fe
12) Aag ites

For one-one st nny # ;


ae
SS
Tx?
fo
ad
ae |
i
Reig +
,
s
a,
a |=
Pi le”

Scanned by
Scanned by TapScanner
TapoScanner
er the sequence
such that |
{= \o if f,(n) =!
. :
i t h e 0
r or 1, th er efore he re : |
sequence f are e ae :
Since all the elements of the
i E is é (m, ne
| % ' : ct 5 ( “ 1 » | : When b
But 7. = 7 n? Vv n Ee N
c
ae era
F | So, we arrive at a contradiction.
ble. | s
F Hence our supposition is wrong. Therefore A is uncounta
{x ER: 0 es l} is uncountable. S 2
Example 2. Prove that the set [0, \j=

:
Sol. Let A=[0, ]={xER:05x5 1}
can be a1 ‘anged in ¢
If possible, let A be denumerable, therefore its elements
} ; aa fy.
sequence with distinct elements. a leech ban

We Let A= (x4, %25.%3 yp eeeeees no Say

Expanding each x, in decimals, we have .

a2 ayy OLg farenrennsee


xy = 0 " ayy
ejeeorih hae wy ySahi¢ iy =|
7 Ma as ar)
Mae he a»

oe ass r aie Re ee Me Lee eed .


I AUR ort eS | —

: z aa . fae A eee 5
SESMUORINCCC SESE
TA SMASRONAPOrroesNenaaeieeesReeeee tens:
Sl a EE ee eg Tat eve i Te ile
A
tren ies
4

-i

sesvesensenenueeneen eA sennee . sas


lt ‘ | bi aaa Potts

Scanned by
Scanned by TapScanner
TapoScanner
SPECTRUM DISCRETE y 5
3 any Funes
iow
118 . : . ey = - —— ae ae J oe ae = <2

ie 2 pore: — |
> B be the map definited by f(m) =a, (Ymen
; E Z

is 1-1 Let f(m) = f (m,), for ™m sca


To shfow a bt “
aa
Sea
wee

ere ti
Se ty

sof is1—1
each y= 2" GA, 3 mez
To show f is onto For
such that f(m)= 2” =y
*. f is onto
Z~A
Also we know Z~N
So that A~N = A is countable.
is uncountable.
Cor. 1. The set [a, 6] = &« ER: a $x <b} deni
one i) ee 4 possible, let R, be
Proof: As {0,1} > (a, b] defined by f(x) = 4+ (6-a)x is one-anod n to.
, im
a », Therefore
0, 1} ~ [@, 8)
ble.
But the set[0, 1] is uncountable- *. [a, b] is uncounta
1.57. Prove that every subset of a denumerable set is countable.
nite sequence with dis
Proof: Let A be a denumerable set. Then A can be written as an infi
:
Let A= { cy, dy, Ghyy cssoenAgsy woree} «
Let B C A. If B = @, then B is finite set and hence countable.
And, if B # @, let A; be the least positive integer such that a, € B.
Mo ec
Consider, the set B, = B —{ ay, }.

9, th en B = {a y } is fi ni te an
IfB, =

Scanned by
Scanned by TapScanner
TapoScanner
SPECTR UM DISCRETE May, 4a E {AND FUNCTION

al
ear denumerable sets, then A x ia
Eg aeunnee
Now, we list the elements of ss E, as follo and B= {5,, 5, 5;,. } be
two d enumer; a
A= {ayy 29 939°" J
== ay ad gm) = by, vam
ey geno A ands: :N > B definedby f(n) =
eS
4
APF gad onto _ (ae
fon
in defined by Stade ‘ cece
Bbe amap
| a N xNAX

en 7 (mm
ite ath Ne
H((mmm)= (4 eh
e5

4 &23 °32 41
es &24 €33 ©42 ©51

e p+ q= 2. Se co nd row contains aljfeat A


elements ¢, _ wh er
Here, first row contain all a‘ts
we re mo ve an y el em en t ejif it has alread
q with p+q=3 and so on,
of course, positic a is,
and only once and it occupies a definite
ts of ~~ E, oc cu rs on e
every elemen
2%. is an infinite set. Thus the elements of U Ey
*
nen
Sot kee
v_ E, neN
mneN
uence with distinct elements
cain the form of an infinite seq
Hence v E,, is a denumerable set.
neN

rable.
1.59. Prove that the set N x N is denume a
Proof. Since N X N= {(a. b):a, bE N}
Let Ay = {(1. 1). (1, 2). (1, 3), (Us ADs oneeensd
A2= {2, 1), (2, 2), 2, 3), 2, A), veesveee }
As= 3. 19-G.26G, 348.4 weed

Scanned by
Scanned by TapScanner
TapoScanner
SPECTRUM DISCRETE My 5d
< Mary 3 wt onan? FUNCTION
122 Pe, -
: dental Numbers we can define @ function f:.N ALUB by
Be a es re,
f a polynomial equation with integral Coefficients the
Aa
Gt. Alge h
braic Numbers : The roots of a po 4k if Isk<n Ee) ee ite pee
Re ey
algebraic numbers
2* nie FO x” =0
ie. if p(x)= dy + ax +4
on , wh er e ea ch a, € Z. Then the roots of p(x) are algebraic digs
be a polynomial equati er is called .
- A nu mb er which is not an algebraic numb
s : n
Transcendental Number po ly no mi al eq ua ti on wi th integral |
mber which is not a roo
t of a
number ie. a nu
transcendental number.
nsce ndental numbers.
For example: e, 7, log 2 etc. are tra
Pro ve that the set of alg ebr aic num bers is countable set.
1.61. \ een
P = {p (x): p (x) is pol yno mia l wit h integral coefficients}
Proof : Let
aex , where a; EZ
where p (x)= ag+ ayx + ax" tine
7
We first show that the set P is countable. _ Hey
n)
For each ordered pair of natural number (m, ja =a i
|+| ay | +] 2 [Foe +1 4y =I
Let P, = (Pp @): p(x) EP st. |g
finite number of terms is finite ; | aa -
since degree mo a polynomial is fixed and sum of La a me 3
sete bers awe eat;
-. P.,, is finite setand ~. P,,, iscountabl
P =U {Poe wie alee Si sg es A
Now
ee ee oe
ana ape ea

Scanned by
Scanned by TapScanner
TapoScanner
ax

SPECTRUM DISCRETE yy
TE May,
}

|
124
n, EN such that "> -1 ai 9 s Prvere ahatibe set of rational numbers is denumerbl, nswit
:
that the set of integers is Aaaibiting = das
such that > my E f(A). We get
’ cee }.
f(A)= tb, Pn o fa s equence
in th e f o r m
e e l e m e n t of f ( A ) can be arranged
In other word th
ai ‘y
= £(A)~Nalso A~ SA)
(> — israsteg 4
> A-~N
sete.
Ais countabl or a countable set “‘
ie
er A is a fi ni te set
ef or e co mb in di ng th e tw o cases, We find that eith
Ther i
Hence A is atmost countable .
set of itself. ? its
set is equ iva len t to a pro per sub
Example 2. Prove that every infinite chee |
Sol. Let A be infinite set Fs
ble set (say) B.
*. by previous theorem A has a counta
C=¢
Let C=A-B > A==BUC and BN
bie
Since B is countable set “is lemens can be ranged inthe form ofan init
ee at a > atl)tu

distinctelements. =
eS eee * oe A 14 oe oo
ae tel be cou tat eset. 7s is saree —
Let B= ) by,
{6), ,

Scanned by
Scanned by TapScanner
TapoScanner
126
that
,n
Thus 3 (mEN )h
x Nsuc
h(m,n) = Ff (m), g(n)) = (@®)

- his onto
ct io n be tw ee n N x Nto AXB
Hence h is a bi je
NXN~AXB
A x Bi s al so countable set. a
sete --
AsN X Nis countabl set.
the pr od uc t of tw o co un table sets is a countable =
Thus bl e sets is a countable set.
ite nu mb er of co un ta
ion of fin
Example 5. Prove that un
E= {E, , E>, E3 00 » E, } be a fin ite collection of countable sets
Sol. Let

Let By = (pyr gree}


ae
E,= {45 &59» age}
nu enee snakes eke eres
eeeeerseesse ~~

saree eee aeadee she eee re eee


eeeebe

b= fen,» en,’ ye

Take, the union Es the element of which are as follows : iat


‘| I é fe
i=l

ey =O

% &>1 ct. +
Pai
a OS semen aie acai
a|
“14 3 a2. an) aS CN dary
oe oe + oon ag ;_

Scanned by
Scanned by TapScanner
TapoScanner
CRETE ! 4 ad
SPECTRUM DIS
ston ANP -
128
on ar gu me nt , di ag on al sl ag , writderiaes
to e' s di ng un sl A T E D R C R E (Diagonalisati va proveety car's S« diago
un
+60, ‘G od)
diagonal meth Ge or g Ca nt or as a mathematica
1891 by
en t was published ‘ne-to-one correspondence with the infinite
Cantor's diagonal argumcannot be put into on
ts which
there are infinite se w known uncounta
ble sets.
ts are no :x
numbers. Such se
C a n t o r ’ s d i a g o n a l argument
1.61. Working of na ry digits (i.e. each dip i Whee}
ni te se qu en ce s of bi
h e o r e m : Ca nt or co ns id ered the set T of all infi
T
\ ‘

ents from trom T, then there is always an ele,


0

;BY enumeration of ele men ts


LF 5), 855-0 Sye
co rr es po nd s to no S,, in the enumeration.
T whic h
si a ze
Sp re e is an y en um er at ion of elements from T,
Proof: 5,.5y 50-9
er )
We can consider s, = (0, 0, 0, 0, 9, 1 | Meats

s, —( Lilies eal)

s, =, 45.0; 120) 1, Op ce.-n0)


i
s, =(1,0, 1, 0, 1,0; 1) evens)

+) |
§, =(1, 1,0, 1,0, 1, 1,
s_ = (0,0, 1, 1,0; 141, 3ena)

5 = (150,0,0, 1,0, 05-00)

Scanned by
Scanned by TapScanner
TapoScanner
i oe ,
DISCRETE
SPECTRUM
Ma ‘h i ed AND FUNCTION
130 SW e335)
wer set theorem) 3 e the funct Ows ;
(PO t
Theorem : Cantor’s Power set theorem
en re but no bijection, So |S|<|p weee defin 1a) 7 Gi tee
an injection from S pOt0 P(S)(9 vues
edas | (S)|.
Statement: ISisay so BENE ETT h(a) = t (a) _ Seihervise
ool ot

S ‘ "¢> ee
Proof : For any set S, the map P :
Clearly this map is well defined and injection
from S to P (S) ot be lonel
y itself. If
f(a) is descendent ofa ey
se uae FO)
3 an injection from S to P (S)- 6 ; since g is injective, the clement g g(a) i soa
f(@*=fo8g (b) for some
to B. a
Now we shall prove that 3 any bijection from
S to P (S) sdefined, we claim that it is a bijection from A ae es
w de ed ifb € B is
a map / is su rj ec ti ve . In lor
Let qi > P(S) be We first prove that een
scendent of a lonely point, then 6 is not ee
de ch

put Y={xES:x€q@)} h is surjective.


+d Ls
bis no
ai of h, h (a)= 6. Thus
a ae

subset of S we will pro ve


that 3 y€Ssuch that g(y)=y Jee ae . a
e, we first note that for oi
1) eet si]

Clearly Y is well defined Now we wil l pr ov e A is in je ct iv


that ich @
su ae st
re isy ES with g(y)=Y f( a) is a de sc en de nt of a lonely point. Now
Suppose the contrary So the jonelyly point iff
ef in
ini it
tio n io nofof Y
Y we
we h hav e y € Y, a contradict
fallse, so by Dy def
7 ()) is fa
if yEq()= =Y, then y ¢ On,
such that
Y, then y & q (9) is true, so by definition of Y we have y € Y, a contradicti h(a) h(a,)
If y€q(y)= On 999 ME NR
ndy¢ Y. aaa
get contradiction in both the cases y © Ya We consider two cases :
|
ae

S leads to a contradiction.
aor ro “L

So the assumption that g (”)= Y fory © (a,). Lsegiplg Sah aa


case 1: If f (4) is descendent of a lonely point then so is f v= ee
So no suchy exists
P (S)
Y €t 2 g(a) =h (a) = h(a.) = g(a)
So Y ¢ range of function q Bu
A (ai) ~&
lq)
\4,
(a
§ gis not surjective g

>
ee > a, = a,
Be a ait hk 3
Thus we have proved that for any set S, there is no surjection from S + P (S), so no bijection.
ie. pow er set of N is uncountable. a ince gis wel l defined
ber , then P (N) ,
Example : IFN is set of natural num
* oe h(a)= h(a,)= f (ay)

Scanned by
Scanned by TapScanner
TapScanner
SPECTRUM DISCRETE yy
ae | ATH, |
132 cept
n=k+ |
a 1 7

SECTION-V ia irra ees

OF M A T H E M A T I C S I N D U C T ,
PRINCIPLES
en t of Pr in ci pl e of Ma th ematical Induction
1.62, Statem
Let P (n) be a statement such that
(a) P(1)istrue re, :
re =r.
(6) P(r + 1) is true whenever P (r) is true whe ie
hate
s 77.
Then P (7) is true for all natural number t—*
The Well-Ordering Principle 3
a least element.
Every nonempty set of non-negative integers has r,
ive Def ini tio n: Som eti mes we fin d it dif ficult to define an object explicitly. Howeve :
Recurs oT May) g2k+2
ect in ter ms of its elf . Thi s pro ces s is cal led rec urs ion.
easy to define this obj ee Sie Sie ee
sets. In previous discussions, we “a
We can use recursion to define sequence, functions, and l Fe Se ae esa le ae
terms of a sequences using an explicit formula.
two steps to define a function
For example: We define recursively defined functions: We use
of nonnegative integers as its domain: ‘wie
Basic step : Specify the value of the function at zero.
a rule
Recursive step ; Give de ee ee at

ILLUST. ve *i
ee

=. rai
Tle. PY
oe
er: ee

Sol. Let P(n)= on _3! ign


P(1)=9-8- ‘oie

Scanned by
Scanned by TapScanner
TapoScanner
TIN, AnD FUNCTION
SPECTRUM DISCRETE MA gant
‘ oaeal a Paa. keateg1- 2
Assume that result (1) is true for” ©nsr
; G5 iT is‘dag

(2k +3) iy ‘wae tae


pe?
ig ((k +2) (2k+3))= este ee .
6 a
iN Pa Pe? Foonerest P= <— Ta

we eee cas +h?+ (k+l)?= (caumnanee


Adding (k +1)° to both sidof
es(2), we BF! ie 6
3 ‘
k(k +1 ®

vesult is true for 7~ k+l]


Et)
3 4B: P+ ape

: n=
Beware ult is true for n== k, then it is also true for
ke 4+4k+4
a for0 n= 1. mths
= (kD? [Esaue0}- K+)" nested yt the e result
But is tue

the res is
ulttrue for nl
al € N. |
by method of induc tion,
ch
is obtainbyed
5 (kt D74
|
2 nm We have added (k +1)" to both sides of (2). This
De BePein tht a= mere
(1)-
em of LH 's, of
al induction to prove that
result is true forn=4*! sampleje 5. Use the princ iple of mathe matic
it is also true for n= k+l. VnE N are ae
if the result is true for n= 4, then
i
+n(n+2) _ Mati) (2n+7) n 7
oe, ol Se
out)
» 6

n+))
(2a Sol. , 7
ae1)r
n(n+
ase Bg —Net N Vn&EN, 1-3+2°44+3-5+ series + n(n+ 2) » een)
6
ines
Sol. We are to prove that or n=!,
1:3 =3
her n+1)(2n+41 EN.
LHS. =
Van

:
Pe2ese nti = maser

= sEDG+6 ) - he 6
pa =3 | aoe
For+3a= 1, Tune 7 ' Leth

L.H.S., = R.H.S.
~ result (1) is true for m= I.
m
thatures
Ass ulte(1) is true forn=k.

Scanned by
Scanned by TapScanner
TapoScanner
= SPECTRUM DISCRETE MATHEY,
result is true form =k+ |
-. if the result is true for n = &, then it is also true for »=k+ I.
But the result is true for 7 = 1.
“. by method of induction, the result is true for all n © N.
Example 6. Use the principle of mathematical induction to prove that
l n
eel l 1
age ~ 2n4l
VnEN. | 1. yn _1 is divisible by 8V2 EN.
1-3 3-5 -5: 7" *@n-)Qn+) I
ha #4
4

e that
to prove
Sol. We hav 0 nv? em is divisible by 43 Vn EN. *
l feay it (iif) pe eo ee
] I i mane
: the principle to
of math ematical induction
OPE prove that —
— +—— + —67> +...» *Qn-)(2n+)) FT, 2n+1 “(Iee ; vse
ae os

For n=1

, 1 1 eee
lao 4
or
[42
- re
1
— —
prove by induction that
peta eS He AT +-— +

+ LHS.=RHES.
-. result (1) is true for n= 1.
Assume that result (1) is true for =k.
- , oa
fo Bisa
gs a6
<7 3,
6) OK=NCZE+TD 2K41
ed
to both sides of (2), e
l

VOENIEE
L Ading

as Barey-nEevea

Scanned by
Scanned by TapScanner
TapoScanner
hl

s -
138 |
en Qe}
From (1) and (2), we get pee 2 ie
aqt+r=aqth a ge
| — r-lal<lal-lal ‘pita (

Phe Fae
“. r-r, =a(q,-)
< r,-|a|<0 ies
=> adivides r-7

<r<0
Foie 0
,
ey oe =() as az0

from (3), a(q,-4) = 9 ee = aq +r+|al, at =sr<0 st

q~4 r such thas | =aq,q|+jal|t+r ee‘7.93


ae aa
; there exist uniqu e integers g and | oe me ae
for any two intege rs a and b, with a> 0, q q uch that >) To Be A § : ’ y
=a(q, + We a '

b=aqtr,0sr<i|al. oe =
suchthat
andrrs __ a: 62 o> 0
: ith , a ¥ 0, then there exist unique integ ge that ig aor Ga, of oes ea
I. If a and bare any inte
Cor with a
gers
b=agtr0sr<|a|
ve
Proof. We have proved the result for a>0 combining (2) and (3), we ha
If a<0, then |a|>0
~. such that
r d
there exists unique integers g> an fy *tt}

oe
eh b on b= 4) lal+r, 0<r<|a\= q(-a) +r,
;
0 <r<|ja| i AS qs"
ere
are unique so q, rare also unique,
lca ai Oo Uae

Hence the result. Rg Bea ES =


i =(-q))atr, 0sr<|a|

es =qatr, 0Sr<|a| where q=-q EZ. ro Cae


baat Vs swe” eR A %

Scanned by
Scanned by TapScanner
TapoScanner
aE ae

eee:
ad

em
1 el 7

1 a aml
AME fu
f= é
_ anh
40
4 Py _ _

a Ate a mek ’ ‘ A : :
» any integet
lf b= $.q+1 then 6° = ($q-l)"
3 G+ C, <
°C. (Sq)* + *c,(Sqy° + *c,(59) b=a
we|
a. .
Sgn d8
aaoe
; 23a 4°C.5 2,4¢ 3
q)+1 =
, 5 + ? ~
59 +
-s(*C
.
=$k+1 (say), FEZ
oa
lf b=5q+ =2 then 6° = ($q+2)" ee
1 , 6 4
2) + 6 9 ay ? +4 C5 64 )2 " + °C, 2
= 4c, (5q)*+
)+ 4¢,(20)q" + *C; 8g+3)+1 =Sk+1 (ayy 4.ke
= 5¢4C,5° ‘;* Cea

if b=Sq>3 then bt = (5q+3)"


+
- *¢, (59/4 = 2C59)°@) “C 25 00 3° + 4
C,( 5q) 3° +
3,4
4C, 43 4

“Ske Lom, Aa
= 5(4C,5°q*+*C,57¢° G)+ #597 (9)+ *C3 9(27)+16)+1
=
if 6=5q~4 then b* = (Sq+4)"
ae
- 4c, (sat+ *C\Sah + *C205)?4?+ *C,604°+ *C, 44
°q
+ 40,5
= 5( 4C45 (4)+ tease,
74 * abs S0eh=84
Hence fourth power of any integer is either ¢
Example 2. Prove that if o and b arei eget ao:
satisfying a= bg+r.where26< ee
Sol. Given a and b are imtegers with b> |
By division algorithm, we have is bs
.?
a= bd +rw ere
a
and
3 onthe Ae:
Pu ry; 2 4 ; 4 , < Z ; ’

4 Arey Hs ao ie. aCe

Scanned by
Scanned by TapScanner
TapoScanner
142

6 q+ 1, Then
When m= are

m? +23 = (6q+l)? +23


SERGE 36g? +12q+2=412 Gq’ +4+2) ~ 1D(@2.
= 12[q(q+ 1)+2(q 2+1))
e integers.
g.q* | are consecutiv = An integer
= q (q+ 1) is even integer = q(qt
1)+ 2(q? +1) is also even integer |
2 |q(qt +209 +) ee jan integer for alln EN.
+1)+2(q7 +1)) > 24| m? +23 a rE 0 2h,
> 12.2 |12(¢@¢

Whme q+ 5, Then
= 6n
m? +23 = (6q+5) +23
= 36.2 +60g+25+23 = 36g? +60q+48
+4q+4)
= 12(3qg2 +5q+4) =12 (g2 +q+29q7
= 12 (g (q+ 1) +29? +24+2))
ide - mi : in ¥ 2
e eas er or 3 N ee
B leavesthes oaths
c a

- g,q+Lare consecutive integers > 9 (q+ I) is even integer


g(qt+1)+2(q?+2 also q+ intege
even 2) is r. —
>
> Ygq(qtW+2(g2+2g+2) % 122|12@@+1+2@? +2442)
> 24| m*+23 [- of Q)]
Hen rese
thec ult.

Scanned by
Scanned by TapScanner
TapoScanner
SPECTRUM DISCRETE jy
\44

not
1.65. If @ and 4 are any two integers, both zero, 8. of aand b exists ana
that g.c.d
then prove that
we shall prove that g.c.d. (2, b) exists.
Proof: Firstly,
W 1Lo.g. assume that @ and b both are positive and b2a “a
ed by sion
[ g.c.d. (a, bis not affect

by division algorithm “a zz a
i eu
b= aq,t+h where 0S 7; <@
4

If r, =Othen b= ag, > a|b and g.c. b)= a


(a, d. and pee ‘ hatsae as ah
ged (a, d) exists. Hence
=
_ god (@ p) is unique".Al
If r, = 0, againby division algorithm
a=n97+">5 0S rr <7 b) where a, b not both zero, thenUSp yr
166 nee e |
e
If r,=0, then a=79, > 419
set
the er
if: Consid
Putting value of a in (1), we have A= esa
b = (7192) +7 = 4929 +1) Asa, bare not both zero | oe
> nib
Whogiet a0 [al>O
Thus 7, \a and 7, |b
Take p|a and p| 6. Then p| b- am,
- P\n

Hence g.c.d.
(a, 6) = | 1 aR ee
so that g.c.d. (a, 6) exists.

Scanned by
Scanned by TapScanner
TapoScanner

SPECTRUM DISCRETE yy
146 yb.
a e
Now we will prove th r h e f o r m xa +y .
member 0 t
Let mat nbbeany other

Since (a. 6)=d ee


dja and d/ » = di(matnd) d<(matnb)
Than B R ae
if 4 | koA 5 dha
si t ive e xa +yb.
valuof
> — disthe lea st po
then prove that a positive integer id dibs
Cor. 1, Ifa and b are two integers not both 2=r0, : fee f
. “an nee i
a,iff () dja, d\b
d=ec.d(6) (ii) if pene pl omene le Sate ce Cn
as a al |k blk
(a, b) » dijaand d|b.
Proof: Firstly, let d= g.c.d.
And as c/a an d
c| bs o that
p,g EZ
=peand b= qe where
t
Let =(a,6) s0 2x,yE Z such tha
d=ax+by=pextqcy
pxt+qy€Z
=c(pxt+qy)=er where r=

. d=cr » c\|d > c\(,9).


s (i) and (ii)
Conversely : Let dbe positive integer satisfying condition
By condition (i/) if c is a common divisor of a and5, then
cid =csd 2d2c d.
of aandb.
> disgc.
thece
Hen result.
Cor. 2. Ifa and bare given integers, not both zero, then prove that the set
B= Mm |
Proof; Let aes

= Sire eee B a

ie FachelementofBisamulipleofd
A,

Scanned by
Scanned by TapScanner
TapoScanner
ae,

™ y — ‘ene
SPECTRUM DISCRETE Mar
‘ ee
i integers, not both
(a+ by, 6) = 4) thatt
prove tha
mur€r,6 oo — Sag
b = d,\atby and d, \by 10. F ae gna, mB m(a b)
nytt) and 4, = (a, m8)
> d,\a+by and d, |

> d,\atby-by and d,\a Fot eiag


oe A) a
Pr we want t0 prove dy =m d.
d,\a and d,|b > d, \(a,b) > d,\d
“ As d= (a, 5) = dja and d|b it e Pee. |
From (3) and (4), we get ic
=> md|ma and md|mb
sil
> (a,b) =(a+ by, b) V yEZ. = md\(ma, mb) i.e
> mdld, a See Hes ck
(a, b)=(a,b+ax)=(at by, b) Vny EZ. a uae
both zero, prove that
not ers,
1.69. For a. b any two integ eyrtheras d= (a, b) so there exist integers x,y such that
oi a & Fe
= a,—b).
(a, b)=(—a, b) =(a,—b)
d=axt by : “a 5
Proof: Let d=(a,b) and d, =(-a,6) | ae
2 md =m (ax+by)
As d=(a,6) 2 dja and d|b md =(ma)x+(mb)y | | = aah Z
»dj\—-aand d\b
~
Jet thie ee
nO
= d\(-a,b) > d\d,
> dj |ma and d,|mb a
Further, as d, =(Ca,b)
d, \(m a)x+(m b)y ii
>
er dy |-a and d |b
d,|a and d, |b d, |md
>
aSar . d, \(a,b) » dy \d
d= d, > (@6)= (445) J From (1) and (3), we get |
—— From(i)and@2),
eG Di ks \ tm rsi o fe078 Sean \ d, =md SAA

Scanned by
Scanned by TapScanner
TapoScanner
ya Ti, a =e 1
150 SPECTRUM DISCRETE MA
a (6 60) = C45) 2) Suit
a b, ©) = (a, (6, = iene
Cor. 2. If d|a, d|b and d>0. Prove (¢-3) = = (a,b).

Further more if d, = (a, 5). Prove [z. +). =I.


qd, 4 ee i: eo
i then show that (a, pc er oe
Proof : Since (m a, m b)= m(a, 6) for positive integer m. fe by

ke ae
(3}43)|-G-4)
N=2

d, /(a—bq) -—
worded
=
- 3 ala dib
a a! jpand 4 /r
r of 6 and r
a b 1 ,6 val) if
d, isacommon diviso
of b andr.
=> — | =

d, | 4, 4s dy is the g.c.d.
in (2.5) ae

thec
Hen ult.
rese

Further given d; =(a,6) > 4, >0.


Putting d= d, in(1), we get

aoe Tay =1
(+.2| = Gonna d,

Scanned by
Scanned by TapScanner
TapoScanner
FUNCTION_ =i
2 we eo
Tad\(a,6) > dil > ene 2 See
(ii) mn) =1
that (a,me
Assu gottat sive so that d=1 » (ca)= 1. satege 2 e ;
|
-. there exist integers u and v such that

eat hs
(c, b) =¢ : ke Bie ise

ee
putas |
aut(mn)v =1

i
>
sea (a, m) =]
=

Give?
: \a+b

ejath and efb


“ad oo

ee
.

0
Similarly (a, n) =} 4 c la+b-b ala i
Note. Let (a, m)=1 er ee
* Fan) ri
-. we have (m, a)= 1, (m, a)=1 “8 e
en
> (ma@)=1 > (@,m)=1
. os . » Gbyak ie
pres =] aE : (c, a) = (C,D)= ae , ei soon
=> (m, a) ni oy a8 x,
sence the result
and so On.
= 1, then prove that (a, bc) = a 4). | | oe #
example 5. If (c,
g way, we get
in this
Proceedin
ca Let (a, 6) ==d and (a,bc)=e aad Jal PONE 2am
(m,a’) =1 or (a’,m) =1
“sy d ja, d|b and ela,e|bc = visiaieae itn
- if(a, m)=1, then Wha de |
2 d |(a,bc)
m),= 1.
=1, (a, m) = 1, ----9 (a
(am)2,
i # d |e bi apts egg
Example 3.Ifa/b, c/d and (b, d)= 1, then prove that (a, c) = I. 8) 0S ee ae
+ ela and (c,a)=1
Sol. Since a/b, c/d.
“, there exist integers m, such that
b =am,d=cn

Scanned by
Scanned by TapScanner
TapoScanner
184 re —<—« ——————
_ SPECTRUM DISCRETE MATHEN,
Now (x,y) =d d|x, d\|y
eens

d|x+y
d| 100
d= 10, 20,25, 50, 100. 1,m+2are 5c
; mm
e

As |e and’ xis oad [os 5 | (@m—2)(m=1)


=> d must
be odd
5 | 5(m> —m)
dd =25 | ee
= dx > 25|5p | 5 | (m—2)(m—1)m(m
~

at
=

> 5\|p
as ( p, 5) = 1
which is impossible ;
. is wrong so that d=5 > (x,y)=5.
our supposition bee 5, | ee
are infinitely manyp such that (p, 5) = 1
As there from (1) and (3), we get pata
.. x and so infinitely many pairs x, y satisfying
there are infinitely many 6x5 | m-m as (6,5)=1_ ve

x+y=100and (x,y) =5. a


Example 7. For any integers a, b, c, prove that (a, bc) =(a, (a, 6) c). |
Sol. Let (a,bc) =d and (a,(a,b)c)=e
a d|a and djbe =» d|lac and dljbc
> d|(ac,bc) > d{|(a,b)c «i
d|aand d|(a,b)c => d|(a,(a,b)c)

ela, el(a,b)c > ela, el(ac,bc) > ela, elbc— ©.’ 2 oa

h & Ls
>.
i

Scanned by
Scanned by TapScanner
TapoScanner
156 SPECTRUM DISCRETE MATHpy
(ii) Let (S5p+2,7p+3)ed
fe d|Sp+2and d|7p+3 <tr
* d|5(7p+3)-7(5p+*2) b?)=10r2
> djl 2@ d=tl ] Ee _gb+b = l or3

But
d is positive
a d=1 @ (S$p+2,7p+3)=1,
(iil) Given p be odd so let p= 2k+1, KE]
2 3p -3Qk+I)+1=6k+4 | e,, (2,2}=1 then (a, 6)=c.
and 3p+2 =3(2k+1)+2=6k45 ig, Ie! ce Se
Let (p,3pt2) =d prove that every tWO a )
ie. (6k+4,6k+5)=d 4 prove tat one of any three conscoutive Hiiegeay asim by 3
° d|6k+4 and d|6k+5 a 42 be tinctions such a Cae -.o=1 ae
2 d|(6k+5)-(6k+4) a ad ae
° djl = d=+1 ap, If ayy @qo-9 In ei
d is positive,
But Js converse true ?
d=1 * Gp, 3pt2)=1. | 23, meee yer ee sate

EXEKCISE 1.12 e (if) Ifg.c.d. (a, 6)= 1 and c| a then g.c.

1. If(a, b)=1 and c/a, then (c, b) = 1. (a, 6) = 1, then g.c.d. (ae,
iti) If g.c.d.
2. Ifaand b are relatively prime, then any Y common divisorvisor ofofaac¢ and b is is aa divi (iv) If g.c.d. (a, 6) =1, pliner 3
divisor of c. | “ Ha ad b are non zerooie
3. If(a, 6) =I, the(an¢, b) = (c, 6)
se f(a, c)=d, a/b and c/ b, then show that ac/b d.
Show that (a, 6) = (a+,b).
» EKG Bee eatin,

Scanned by
Scanned by TapScanner
TapoScanner
158

Proof : We have for any integers * = (at by, b)


(a, b) = (4, b+ax)
Using equations (1)
(a, b) = (4 b-aq,)
= (a,r,) = (a-1,99%)
= (r nm) = (7557 = 593) = (7,573)

*
Continuing like this, we get ‘a
(a,b) = (4%) = Vy) = 15)

seseseecseeeeee

eeecebanneaeeee

= (rela)
= (7,0) =%
of a, b.
-. 7, the last non zero remainder is g.c.d.
combination of a and b.
Cor. 1. Express G.C.D. ofa and basa linear
Proof : From Euclidean Algorithm, we have
b= aq,t", 0< n <a

0< ta iiaini’ —te


a="9,+%,
. ~ «in ea
ye Lee ae
fy 3050&
a
aS phe
teanaee maa ——

CL at lll a
: “
a
mi. alae ‘i 4

Ta n-3Tnd nd
typ 2 8ES ee
a ete
eo a

Scanned by
Scanned by TapScanner
TapoScanner
Peis
ae

SPECTRUM DISCRETE MATHEy, = sions F SUNCTION -


160
A ve Ber
asi Se) f° psa and | eee
we can take yD pid by=d oe). aan ae
anabenay
" and (a, ) oth ee Z 225

gh ° pdand dla, d/o “eae


os from (1), we get
p/a, D
ye have D/ a, D/b, Die
egas
i aye common divisor of a, 8, c. 4
apb=a"y"
: ; other common divisor of a, b, Gu
a ee i}. Let p bee
) pia, D'/b => D’/(aa,,b) >D'/dBod eon

2S y=! “ pig Die = DUG) > DID exiaas


n Now ; ; ‘aan -
(8. y =i . (a, 5.e)=P 3 $s
o ; : b, ; es
52 from (3), y"lb [* y" /B6 and ("B= > Disgc-4. es rs <
ve that
(m a, mb, mc)=m(4, b, c).
hs
ec nee ~( an pet at
From (3) and (4), we get, . @h0=D vs alee
py"6 -a")" Aon(m d, m= ™2 age
| mc)= mD
= Bd =a™! of) . (ma, mb, mc)=mD
* (a, b)=1 and @/a, 6/b | From (1) and (2), we get
@d)=1 = (@'5) =! (m a, mb, mc) = m(q, 6, c) |

cont miaim/ bm eben (5,55)


a b sah ; ie
Wh _m/b,m/c,then | —,—,
from (5), a”!/B =
- wecantake B= a"'A Proof is quite simple.
Fr
(5)om
and (6), we get,
a™ 46 =a"!
2 46=1 = A=1,6=1

Scanned by
Scanned by TapScanner
TapoScanner
; - ya é +3054 = 6 +7 ae
| Example 2 Find iegersxand) sta 12978 eo: hea :
Sol. Firstly show g.¢.d. (12378, 3054) © = ee a es : a
Here = 3054 (4) + 162
12378 Soha “a “an prea Su a
- =es162(18) + 138 12 162)3054(18 The 21 ee
baa:
sre
1 a he 162 | yea it 8 =5 ae i, I 7oi

138 = 24 (5) 1434 1 : ry 5 =3(1) ers cs

24 = 18(1)+6 1296 on ia re a nace eee Ao


18 =6(3)+0 | “T38) 1627 % 2aes, “a Ng a4 Re:

and 6 =24-18 24) Bes ee - ee 2


= 24 — (138 - 24 (5)) ‘vk a a & 1=3- -2=3- (5- 3)=2

=6 (24)- 138 “Tey Now 29 x (8-5)-5=-3 X54


= 6 (162 — 138) - 138 | aya =-3 x (21-8(2))+2*
=-7% 138+6X 162 =-3X21+8X8=-3 x:
; 2-7x (3054-162 x 18) +6 X 162 =8
x 50-19 X21 =e 5
0
my | = 132 x 162-7 3054
a = 132 (12378 - (3054) 4)-7 * 3054 |
ae =— 535 x 3054 + 132 X12378 ale
- a:
= 12378
:

x + 3054 y where x= 132 and y=—535 Re:


ea |S
ae => for x= 132 and y=- 535.
12378x+3054y=6
Se example 3, Use the Euclidean Algorithm ind integers x and suc that
ag)[ae ke

x + 2378 y
g.c.d. (1769, 2378)= 1769
iti

ER apae ie
ae |

» Her ot
mt
(eal.

Ga ise Treat96
- Ea
i

‘,
a
Sie ri.ak

pets be e :
a

é
ne ‘ha
|
are

re

ae
athe Ff
i

io il te

Scanned by
Scanned by TapScanner
TapoScanner
ee
hos te
oad
=18x57- ~2 x 512

= (3 (198)— 2(288)) 57 -aE Kal sep + $950


=p wey 1d 288 17 EP CN nati’ aitia
= 512 (—2) + 288 114) + 198 17) =e ae
53 | ‘a aise
~~ 114 andz=17]
=5]2x +288 y+ 198 z where x =— 2,9
. a Re (v) Te
g.c.d. = 9=512x+288 y+ 1982. |4 ee 36,y=71
Example 6. Evaluate |.c.m. (306, 657]. a , ~ Name
Sol. Firstly find g.c.d. (657, 306) : I. any positive integer greater than 1 is calleda
By Euclidean ee Sait
ee
306 = 45 (6+)36 Any positive integer> | called prin number if
45 =36(1)+9
2,3,5+1 1, 1s Vie Eae number
36 =9(4) +0 fs. Composite Number. Any positive integer reat
1 ch
g.c.d, (657, 306) = 9.
= ty 3,9, 10,....- are composite numbers. eas nae
Now using (a, 6) [a, b]=a6
% es
, ode Now we can divide natural mumbee ait tebee
we have [a, 5] = am
ase =
er =ae
is (ii)aPrime Numbers ea
306 —x- —657 = 34. 657 = 22338.
= [306, 657] = Chae 3
‘2 x 4 (iii) Composite Numbers

2st only even number a 5%


ir stad.
in the form 252 m+ 595 n.
1. Find the g.c.d. of 595 and 252 and expresd s hich ii
726 x + 276 y.
2. Find the g.c.d. of 726 and 275 and express it in the form
3. Find the g.c.d. of 858 and 325, and express it in the form 858 x + 325 y.
4. Find the g.e. o 1109 an 499, an exrestn th fom 110944999. a

6. Find y Gateges) oe
@ 8 at NNN
w 68 +710 ye

Scanned by
Scanned by TapScanner
TapoScanner
ano NCS srsieninsina tn
SrocTRUM DISCRETE My f at
ene Ps a te Pe

;
age Capen eae Neh P ™. ae ast ple farsome 1 Sitser
UTR. Sper peeendaee

Reatsiewie gcd Qans > See ee & 2 ww go its only divisors


are l, p, ;
op 2 cl ar? * * Shy ae
‘ee
Pr 2 | Sem Eos nm we p= Pe title at ae ao ee

tf 2~o te G2?
? * Oe ee ao
mmc esd EP eon rem ; ae
LRA Pr seem © sh dapeede pis = 2 gp B® af primes is infinite = sab aa
| ae prumes a ee
Se ee

2° >» 2 fetes ses goers go ond to the Sequence primes aS


STS ret arte ost ae
for he @egct ew!
ec a) byw Bess tsar eee N defined Kae SAS
Wuteptong dah SWS Te 2 WE ET N=2*3*5*.~..... x p)+1 scan soa a
$2.2~ a0? ~& gxsprame, Gea N>P ; ae
Gea 22> = 2 “= aae. then it has atleast one prime factor. But none of prienes fi
a r2z = PPPs eNS
- _giegrsabep > Die C= ety =. Nhasat least one prime factor greater than p.
ieniiitien phe ar pet ss in buck the cases, 3 ¢ prime which is greater than p, which is impossible. 3
Os. LF 7 a2 2 terme 2 a Brsmeci=rsf iz 7 dads ees one heer 2 jie — i ale : eee sed 4a 22 a

Seek Fel te 2 weiter swe ee

sr 42. >
elie FH TT

@e+ Ge +D=bar s4ns4etl EY.


See See st eee at ae Tatnns
; 5
es AY, =
ris,
iy sme = ~
ys oe pe Te

= gilt.
wee o- 42,4, > Pib ep ae JAI, €
Sprata=
: 4: +
ee 2 is i

Scanned by
Scanned by TapScanner
TapoScanner
168 SPECTRUM DISCRETE MATHEM,,, |

ou
uv se 3
- S
ms

If N is a prime number, then certainly N > p and N is of type


4 k + 3. a 7 von! " #27 meg ad tie

IfN is composite, then it has a prime factor of4 k + 3 type.


[< peers ee ri ibers oF the tots 4 i+. is agnlh Ot eo aan
But none of the primes
2, 3. 5, ...... . p divides
N.
this prime divisor of N is greater than p. ee «there are infinitely many prime numbers of the fe1 os
7 bes ¢
in both the cases, we have got a prime which is of form 4 & + 3 and is greater than p, Which jg oe psi serorpimes the om 8 kEZbe: inite.
impossible.
es . our supposition is wrong 3 ap ve
rae gest posible prime ofthe form | 8+ 1 kEZC -onsid <a
number of primes of the form 4 k + 3 is infinite. cca 72 11? +13? sesssee p? +1
Cor. Prove that number of primes of the form 4 & — | are infinite. : of odd integer is of the form 8 k+ 1 and
Proof. Primes of the form 4 k— 1 are of the form 4 k +3. sa om 8k 1.
imreger °
" E
1.83. Prove that primes of the form 6 & + 5 are infinite.
.
: p eight aston is of the form 8 K+ I and let it be &’.
Proof. If possible, suppose that, primes of the form 6 & + 5 are finite, and let p be the greatest prime of this | wise 2 > malt a
type. | 8 k’.
332-97 11? -13? ...... p”) is.of the form siaieoiese La
Let N=2x3*x5*.......... x p-1 = et : “ee
a.
*
* Nis of the form 8 fs five}
7 seat
ae
Clearly N is of 6 &— 1 type and hence of 6 k + 5 type.
| Noweither N isa composite number or N is a pri e number:*
> p.
If N is prime, then certainly N is of type 6 k + 5 and
If N is composite, then it has at least one prime factor of 6 k + 5 type. | ifN isa composite number, then clearly none ot t rime
p divides N.
2, 3, 5, ........,
But none of the primes 3 «odd number, not divisible by 2.
. Nhas got a prime factor of 6 k + 5 type and which is > p. a 2 pinta ienced ee
te me bob He pee pam orem geS E+ 3 Wessel ch .
is impossible
2. whi

he: number of primes of the type 6 £-+5 is infinite Pact


; Cor. Pee ey eons see S/S

Scanned by
Scanned by TapScanner
TapoScanner
170
If Nis prime, then clearly N> p and Nis foe 8 F*> 4k+1
If N is composite, then N has at least one prime factor TPS
Take a=3-5-7......p,6=2
“ (@5)=1
Also
N= a? +8?
=. Nhas atleast one prime factor of 4 &~ | type.
pete theorem is complete.
= N has at least prime factor of 8 k+ 1 or 8 &* 1 type.
If %
[- every number of ype 4k + 1 is of type 8 F~ I or 8k+5 a5 kis even,
fthe aT is composite, proceed as above. |
same type. ifm,
But the product of any number of primes of 8 k + | type is again 0 ad
er759
3

a sequence of decreasing numbers


. * ef - 1

pepeating this argument, WE B


j * et

Since N is of 8 k+ 5 type. v4
none of primes 3/595 1, Ths Myre , M1 (say), all > 1
-. N has atleast one prime factor of 8 k+ 5 type and it is > p as ai tena“sp
-
divides N. that m,_, is prime
we shall have to accept '
op es | Py (say)
Gm both the cases, there is a prime of 8 k+ 5 type which is > p, which is impossible, Uikimately
~
> 8 « n= Pp Po P3
”. is wrong.
Our supposition
“number of primes of type 8 + 5 is infinite. Z i every positive integer ~ 1 is the presi eta.
rrwigge os the above primes in the in $2*. |
‘Cor. Prove that the number of primes of the form 8 k — 3 is infinite.
ail 4 1 sur, then we can represent n as pal Pa
ei Proof. The primes of the form 8 k—3 are primes of the form 8 k + 5.
ie , | ‘ne CP ie
eid
;
i Or Let n be any positive integer > 1. " 1 wag

ats, let n= py Pp,” os


"Statement : Every integer n> 1 an be exes a prot of primes and

- vie a's >> 0, p's> Oand

Scanned by
Scanned by TapScanner
TapoScanner
=>

> a-% =0, a-%2a i othe sano i =F Tk =0

> = N- = I: a tee<tt + 2 =k . | a = ‘ H
% 1 ate ene of riesin ios wy. a
. Lp eaiecomte nib e
“every natural number > | is
242 = m+2 RH = -

a rs
ag

Example :() 720=8x90=16%45=16%9%°= a pega te P


24 «3? x5

lase 43
in Sota 16 x 315 = 16 x9 x 35-21 *3 2947,
= 9m’ piles

x=3 x 38
9 5=
5x
) 9xRS
= 3 33 x 5x77=3=3 XT7x
X LI
:
(iit) 10 x
3 9
11 55 5
=9
= 3xS? xT? x1 is a composite number.
(iv) 40425 = 3 x 13475 $3 x25x 539=3 X25 X49K bie aa: 242

es p >
ea all
:Hence pt 21 isa ce number for

Surrey -q=2 ofaan {


by pq
show that p? +94 isdlvisb(b>le mar 2i
ie el ad

: Feat ee tata tehmtnticr bm+S: meZ. ie p? +q4 is a composite nul bee wil

ne = be put in the form 6 m, 6 m+ 1,6m+2,6m+ 3, 6 m+ 4,1.6 Sol. Given p,q are primes such that p— q 3 =
Hae
| Now pe +q7= (p? -1)
For oie td and 6 m+3=3. ‘pies gsi % a = ,

es ei Gm+1,6m+3=3Qm+1),6m+4= 2G m+2) ar
- plas Bs ae. eS
iw fy «fs7% i
pe P_\= ? PP =(p J
if,SOT on

eel n6m+1 or 6m+5. est oe .

ae agar ia
% as

Scanned by
Scanned by TapScanner
TapoScanner
NN

__SPECTRUM DISCRE
i74
7 a ta
prove p?-l is divisible by 24.
Example 5. For any prime p > 3,

61 G I 0 Or
L ? G 07P°0
4 7 NS ae
Sol. Any in te oft
ge he
rsfra
out of which 6/, 6/+2,6/+ com6/
+4 are3, posite
(2 6/6/, 2[6/+2, 3/6744
aA

eform 6/+1 or 61+5


L

“. i
any prp> is of
3m
q
If p=6/+l,then p2—1 = (6/41)? -1=360" +121
: nes oy =
=12/3/+))

If p=6/+5,then p? +4 251-1 = 3617 +607.


~1 = 61+5)?-1=36/? +60
= 12(3/7 +5/+2)=12G/+2) 0+)
If/ is even, then /=2m

pt=-1 =12@m)6m+D=24m6m+) ae
= | Luo to et ee
wad = 12(6m+2)(Qm+l)=24G m+) Ome) ise sie
xe eh jen SS + hae
= by 24.
p*—1 is divisible a
If / is odd, then /=2m+1 ae is a
is Pp -l =12 (2m+1) (3(2m+ I+)=12(2 m-

=24(2m+)Gm+2)

p?=1 = 12(6m+3+2)0
= 246m-
hi

Us a is
© P

Scanned by
Scanned by TapScanner
TapoScanner
eae q

FA |

E

SPECTRUM DISCR TE MATH, |


176

EXERCISE 1 - 1 4
b by 1 2 if p > 3 , o d d .
q .
's d idi
v vi
i si
s bl
i e b l e
i m e s , p r o v e t h a t p +
If p , q a r e t w i n p r
1.
c o m p o s ite.
g e r n > I, p r o v e n t + 4 " is
2. For any int e
i s a c o m p o s i te n u m b e r.
s h o w t h a t n’ 4 . 4 + 4 i s a c omp
3. For any integer n> 1,
number.
g e r n 2 1 , s h o w t h a t 8 P +] i‘< saaccoommpposite
4. For any int e
is @p e r f e c t s q u are.
a r e tw in pr im es , sh ow th at pq t+ 1
5. If pandq
+n 2 + 1
+1 is a c o m p o s i t e n u m be r.
y in te ge r n> |, sh ow th at nt
6. For an
3 :
e of th e fo rm n° —1 is 7.
7. (i) Show that only prim
; 2
Show that only prime of the form n* —4 Is).
(i?)

8. If pisaprime and p| a’, prove that p’ la’.

ve each inte ger n> 11 can be writ ten as a sum of two com pos ite num ber s.
9, Pro

10. If 2” +1 isan odd prime, show 7 is equal to a power of 2.

oe
fia

Scanned by
Scanned by TapScanner
TapScanner
Ls
Ee r Py re F i 7 i - F
a . i
.- u
See ee = A 5 ee - oe i » 4 Ey ym
7 Lace ieee ee Se ee ee ee ee a) ee eee fe ee ee a ee
7

Scanned by
Scanned by TapScanner
TapScanner
[6=1.2.3.4.5.6

gina ia Sh aa Men ce
|20
An Important Sor
[n =n(n—1)(n—2)(n—3)...3.2.1=n[ (@-1) (2-2) (n-3) ste ean

Pee ee ee ee ee | itig@t
& # & & @ SES Ce RR ee oe 'tiheteiaasas

‘tei es @

similarly
(n + 2) (a+ In =| n+2
ote 1. Since| 7 =n|n-1

Put n=4
l1=1L0
1=(1)( 10). ma fOd
We will
also prove at a later stage that | 0 = 1
Note 2. Factorial of proper fraction and negative integer is not defined.

be a ee

Scanned by
Scanned by TapScanner
TaoScanner
SPECTRUM DIscre TE Ma
is Ty

Py lens 52 =a §
180 ions ANP ee
_ WwW shen
Example 2. Evaluate
Be
Pee

(iii) n= 15,7,=12
eee et i
: (ii) eho
(7) n=6
]
= r=2
. sot £32123. ee 6 I :
r=2 i
Sol. L. (i) .
1 =6
(7) When
,
» Teno De! (n-2)(n-3) J Me

(2x |4 (n—2) (n—3)=6 - (n ~2) (n—3) = (3) (2) j
| 2| 4
_

mar [| 2|,6=2 n-2=3, or n=5


[rp

citi.
oe
=4 EXERCISE 1.1 i
‘ie

7,7
(ii) When n=

at 35 s ey cine
| 4x@Gx2x1)
n-r [4 5 7-4 | 4| 3 , Evaluate [8 [4 -|3. ae
[7
8 if
(iit) When n=
15,7 =|?12
n peace
5 15 15x14x13x] 1212
_15x14x13] _ 2 Comoe Ta a
[| 12] 3 | 12xGx2xI)
45S [a aes
ib

ee
ea oa
Se conpte [| 25s) 4 eee

th,
Rr
]

ae
Example 3. If [6 [7 [8 is

a iad ig
5, Evaluatefe. E L ay: hen
Wen Ss n=5, r r=2 t!
id

is
erie

a
it:
matey (te
ati
n ae
when (i) n= 6, r=? (ii) n=9, p=
6. Evaluate ieee
rt : [meen apg me 5 hae ae
| 6 7x| 6 8x7x| 6 7 56 7 56 ig
7 beech k d
S|? 35
oa
r= 3x56
7 =64 F

8 If —+—=—:: find ié
| ] xr

[9 10 [nee sae
H Example 4. If n+ 2 = 2550 |n, find x, :
=]

On
=

iI

el
o.
se

=
=

|
=

Sol. [n+2 =2250 |n = (n+2)(n+1) [nm =2550 [n


iN. Prove that = = 1.3. 5.:....2n-—1)).2%,
+2
> = (n(n+ =2550
1) ) > n+3n—2548 =0

3 on ba
ES
+ x
ceed ”
ae , oe 12. Basic Counting Princi
ples
There are mainly two counting pr
inciples namely
Rejecting n=— 52 as n cannot be negative, we get n = 49. () Sum Rule (ii) Product Rule
iH These ae y
Example 5. If Ls an Ln are in the ratio 2 : 1, find the value of n. Counting SHR a form the basis of permutations and combinations and so are known as basic
2|n-2 n-4
[x

Sum Rule

Sol. Here Ln Ln =2:] If there :


‘[4|n-4 ot ways 'n‘Wowhich
operat ion s such that they can be per for med ind epe nde ntl in m and n wayways,s, ther
m then es
2|n-2 -
either of the two operation y s can be performed is m + n. me

AMS Aa aoe

Scanned by
Scanned by TapScanner
TapScanner
ee i

ow many 3-digit numbers can be formed fj


TEN \-

;Exetoducat ae 3 NF
vy that one of them can be performed in m ways and Whenwhe. ipo Ui P <agplt
op ion tthe
0 digits| is allowed?PW(in)
| ee Fr or of given digits = 5
ways. Fundamental Principle of Counting
Le. FPC (la ay ae of places to be filled =3 gre

in ‘m’ different ways and if corresponding to each of, % ; z here are 5 ways Of filling up the first place,
| . + Nelo
a

If one operation can be perforth are ‘n’ different ways of performing the second operatic. oe
"Va
Be ofca cette - othe two operations taken together is m X n. ; then : ir 00 by fundamental principle of counting, — 668 ak

ie there are ‘p’ different ways of performing the third operation then the number of differen wi here are 5 ways of filling upthe first plac
performing the three operat togethersism X1” X p andso on.
takenion (i
iid lace. % he
Note : Fundamental Principle of Counting is known as Fundamental Principle of S * a
by fundamental principle of counting, |
t
ital number of 3-digit numbers = 5 x 4 x 3 * 60 ease
We give some example to illustrate the above principle.
-

(ILLUSTRATIVE EXAMPLES) © ) : <a ‘i sample4. How many numbers can be formed from the digits “3
med from the digs
Example 1. Find the number of 4 letter words, with or without meaning, which can be formed ) : J vite help of these digits. | si , E
letters of the word ROSE, where the repetition of the letters is not allowed. e. Hence, number of one digitnumbers=4. = ae

Sol. ROSE (6) Numbers with two digits : First place of two ¢ or 3
Number of letters = 4 in 3 ways.
| pice can be filled oe i ebenia
Number of places tobe filled up = 4 | =4x
Hence, number of two digit numbers ee
‘e
| i.

re
ee

as a! a:
The first place can be filled up in 4 ways as any one of the 4 letters can be placed there. Af if ho

ae: 7 in ae nee of the 4 ways, there are 3 different ways of filling up the second Number of three digits number = 4 ;
5 one of the remaining 3 letters can be placed there. Therefore, by the principle of counting,the ty (@ Number with four disits - va:
Bs taken together can be filled up in 4 x3 ways. After filling
the two places in 4 x 3 ways,the third BP ie ope hate .
nee abe : eo as any ane of the remaining two letters can be placed there. So, the hr bY ee mt offour digits numbers = 4 Se
me rat aide icgeeenea x 3 x 2 ways. After filling the three places in 4 x 3 x ec, *, total number of digits formed with
—— euived up in | ways as the remaining|1=24letter can be placed there. a Fun §belowFind
"(oe the numberonaof
the aiken ee signals
different tha
na "+ Fequired number of words = 4 x 3 x2 x : ‘of flags = 5 | seal
ie: Example 2, In how many ways can 3 itd Sol)
. people be seated in a row containing 7 seats ? :
Sol. First person can be seat
ed in 7 ways
aa
Second person can be seated in
6 ways
and the third person can be
seated in 5 ways.
eee By the fundamental princ
iple of counting total number of ways
Seven seats in a row in which three p
=7x6x 5=210 ao | ait date Rg mit
a
Jit ale ake ae

hs os hae . ed -
ain
erg = eat. \
per rks
a es
es
ie
atie
e OU eeta. 4
‘ i
.
4
nat eS Se a

Scanned by
Scanned by TapScanner
TapoScanner
SPECTRUM DISCRET, ue ’ OMBINATIONS
‘ A
. ie \
be made by arranging at least sts of 40 girls and : 60 boys. In how
184
' ber of sigterent signals
: that 628 three Ags
NI ;
Nea A C ass p
nsi
d secre tary be chosen if the tre asurer
many
must be
wa:
ves :
s
YS can a President, vice pre
Vice ros +
Example 6. we pope flags are available. in, -m 6 yess e not bol d more than one office 9 4 girl, the secretary n : bea boy and ¢ eee

one eer etude to seated on a bench, 6k Gen the Se ao ee ieee


Sol. Number of flags © ‘ ight children a al a el ages y
9 Bi in how many ways can the children be seated 2 Maiti ¥
(a)a Number of signals with “1 three flags aa
u 6 ways (i) n sts rit Ts 268
First place can be filled ws y How many arrangement are possible if the youngest child sits at Gon pe

Second place can be Biecin eee w pench ? ata ee


: , i
of differen t colours, i how many different
: si: rae
can be filled in 4 ways
Third
’ Given ee use of 2 flags, one below the other ? Snals can be ent
mi . =6 x 5X 4= 120 ways. 1. requires cae oe
“total number of signals = 6 ieee
(6) Number of signals with four is “
* 6, 5; f wa
u
ofa ni‘a4 . th ia
a arrangement that can be made by taking some or all
“ es +h
13:
ae ae ed=6X5X4X3= 360
= total number of signals form Mp» _."p, means the number of permutations of » arer....
Meal a re ledteeeiig Bote: Permutations ofn different
(c) Number of signals with ; five flags
Five places can be filled in 6, 5, 4, 3, 2 Ways ath a wejons- utations of three letters taken two at a time are- ae=2

; total number of signals formed = 6 x 5X 4 x 3 * a u ab,be,ca e cin ci


(a) Number of signals with six lags ie ba,cb,ac oe
Six places can be filled in 6, 5.4, 2,2, © wi3®
total number of signals formed = 6 x 5 x 4 x 3 me 1 5 720 ea Suits
eae aie "p is also written as P
two at at
rhe number of arrangements of three letters taken
(n, r). s

required number of signals = 120 + 360 + 720 + 720 = 1920 ‘ci 4g Bia Combination e aie aos it s

town which is connected with his. ai


1. Ram proposes to go to his friend Ali’s house in a
three different routes. From there he will go to the city where his uncle lives. The city ic-
with the town by two different routes. List out the various possible routes which Rar
from his village to go to the city. “

ould
:-

2. If there are 20 steamers playing between places A and B, in how many ways c
from A be made if the return was made on tata
i 1.

= a
(i) thesame steamer, (ii) a different steamer ?
| 3. A coin is tossed 3 times and the outcomes are recorded. How many possible o " .
How many 4-letter codes can be formed using the first 10 letters of the Gnelich <
ah
letter can be repeated ?
5. How many 3-letter code words are possible using the first 10 letters of English a
() nolettercan be repeated? (ii) _ letters are repeated ? 5 ae
|
6. How many 5-digit telephone numbers can be constructed using the digits ( :
starts with 67 and no digit appears more than once 2 weve te

ommed from the digits 1,2, 3, 4,5, 6 ifthe


f 7. How many 3-digit even numbers can be fi mPa he digit
| rae
eee em, Se

Scanned by
Scanned by TapScanner
TapScanner
are
xmite 2
as e i 2‘ isi | | ; ‘ rs
;
fi

> te fae “f bo ne ari


pie
‘ce re
-= n(n— Nn Or a= r+) @~p & oe :
aber (n-1)...3.2. a (ear Se) e Qiks
"ea | ; {7 GnG-n CG one ee
a 1 wid r+24=0 = (7-8) r—=aG is Vents ema ae Ree

on, We kane tat =O DOLD


IGS oe : r=8,3 o Se
n
and “ey = = Lie aa al ny, “ah pec r 28 [vot
8 P09, S5) Swe get, P(10,m,
+5P,4)= 3 00e
7 | : rs

ape ann tied Cb °> o n teeta a P,5)+5P(%4)=P(10m)


9 F3) oie lS ee" aa OP a cee nie
"p Sata i @-2) ~ 57 se coer i
n
2 |
=n (n-1)(n-2) l=|n
n
and mT

From (3) and (4), we get,


Lo > | 0-1.

| ii 5;
ILLUSTRATIVE EXAMPLES
a ys é
Example 1. Prove that

; () "Py=2-"Py2 (i) P,= 9P, +3 x OP, |


|
Sol. (i) LHS.="P, =|n ae h |

, RHS,
= 2+ "P,-9= sax

“ LHS.=RHS. is |
(ii) LHS.="P=10x9 x 8=720 nese
f | R.H.S.=?P3+3. °Pr=9X BX 743.0 x 8)= 504 +21
L.H.S, = R.H.S,

Scanned by
Scanned by TapScanner
TapoScanner
Find nif p(n, 4) = 20 P (en 2)- | i
Find nif PC n.3)= 100 P(
", 2): eee
:
Find nif 30P(n,6)=P"™+27) oo - tra a
that
Find the value of such es 1 ’
n>4
3"
ZS
(i) Gap,
"Py

(y "Ps = 42 "Ps 07
on
‘iL Ree
5. Findr if: : 6
() 5P=2°P,-4 25
Cited wit Phe + a Hage
ane
P r a g : P r=o30 s
80 0: | find r.
5, If ‘
3
—— )

n
ac ti ca l Pr ob le ms involving Perm utatio
1.6. Pr
Now we will apply the formula
to practical problems, 3
7) =n (n —- 1) ( n - 2 ) (n-r+1)= a n= .
P(n,

e 1. How man y 3 -lett er word s can be mad e using the letters of the words. 0) with Dowowel
Exampl n= 7
a4 , eC a

Sol. Given word is ORIENTAL


en)

-. number of letters = 8 ie <p


= 3
Number of letters to be taken at a time
”. required number of words = 5p,
= 8X7 6=336 cane
Ri
Be Exampl2.e Find the number of differen8-t letter words formed from th
each word is to
ie (i) begin with T (ii) end with E feebeer |
MEY | Sol. Te on Oe
Number of letters = 8 i hd

Scanned by
Scanned by TapScanner
TapoScanner
SPECTRUM DISCRETE May oi AND COMBINATIONS’ ise
| th wow many numbers lying between 100
190 a sf ned from the letters of the word "pr a 1000 sat
:
| ind the number of different 8-lertet t Se ion?of the garantie = a iad
ea nae vowels occupyiné te | perween 100 and 1000 consist of'three digits, rplits Pe
Sol. T R I A N Baer ; iM nt 1, 2, 3, 4,5 ’ haus eee
| 2 3 pasta
.< x S given qumber of BV en digits= 6 NE fa eit. ete ae
A

i . ‘ cages a oe aa fie i
N bers formed of three digits = ° P; =6x5x4= = ieae
‘2 YO ee ranged in four x marked places in *P; ways.
-. the three vowe ed among themselves in | 5 ways. those numbers which have ‘0° on their.
bo ee os
Also the five consonants can be arrang
required number of words = "Ps x|5 a ot of three digits.
=(4x3x2)xO*X 4x 3X 2 X1)=24 X 120= 2880. ig we have to exclude these from the total.
s ba |
an in g, can be fo rmed using all the letter ‘ee
ore find numbers which have 0 on the extreme left position, weefi 0 a
out me
_ How many words, with or wl ith ?
Wor
together
soa ti me 4 tha t the vo wels and consonants occur
ure
Sol. EQUATION sips
| peremaininein two places out of five digits at our disposal which can be done in 5p
Consonants
are Q, T,N —E poways:
Vowels are E, U, A, 1,0
letter.
fa the
greater than 80000 can be formed using
one
5 voweasls
Consider 3 consonants as one letter and also
two letters can be arrangedin | 2 ways. ce
| wachampledigit is10.usedHow onlymanyonceoddin numbers
a number ? al

Also 3 consonants can be arranged among themselves in | 3 ways and 5 vowels in themselves in| Sol Given digits are 2, 3,4, 5, and 8.
fick. | number of given digits = 5 a
= | 2 X | 3 x [5
number of wordsd
require | ery a time = 5
of digits to be taken at
Number ideal
=(2K 1)XGX2*1I)XGX4X3X2X 1)=2 x 6 x 120-1440 Since number is to greater than 80000 2 inh: eae
Example 7. In how many ways can 5 girls and 3 boys be seated in a row so that no two boys are together, | first digit from left should be 8 od ea aa
Sol. Let 5 girls be G), Gy, Gy, Gy, Gs ‘. fix 8 in the beginning.
Gi Mia Gs MGe %, Ogi he Now 4 places
are to be filled with4 digits. eh ea
‘’ no two boys
are together aah ' Again numbers are odd z hee a Pee pase oo
3 boys can be arranged in 6 *X’ marked places in Sp, ways. ‘numbers should have either 3 or5 in teeta
Also5 girls can be arranged among themselves in |5 ways. * end’s place can be filled in 2 ways,
required number of ways= °P; x | 5
ee places with3 digits can besfilled in >3p
x3 x2x
4x3 l= 20 x 120.=O= 14400 mud numbers “ben
) x (5x
(6 x5x4)x
=(6 ( x2 x 1)=1
Example8. How many 4-digit numbers can be formed by using the digits 1 to 9 if repetition
not allowed ? ems le 11,1 Taniamany different pcan m
| "rik | signals can
Sol. Digits are 1, 2,3, 4: 5, 6, 7, 8,9
ae ih tof flags = 5
total number of digits=9 = yo a Tg
Number of digits to be taken = 4 Y aeniaaind
j dae e
“. numbers formed= °P, =9 x g
x. 1x6=3004 By

nts . oe Cus

Scanned by
Scanned by TapScanner
TapoScanner
a".

: 192
fagsatatime= Ps Pe anh diferent Sleter w
Number of signals by hoistne aime =P - a oe is tohave vawelsen ccupy
.* Me
:
*

fate dine of signals by hoisting four flags


five flags at a time= Ps
Number of signals by hoisting a < periods in each ‘working «
that each subject ishe
ad er.
total number of signals formed
= 5p, +‘P+ sp, + °P,+ Ps 3x2+5X4xX3X2Xx1
is cere at
.+6 4
0- + 1
2 2 0 0
4 12 0= 32 5-
=5 r arangeme
8-letteent
pe number of differ
EXERCISE 1A = he rote iemlaeraag
iatU gare
, In ho w many ways can these horses come;in the f Mym pe many ways can 5 books on Chem
nn in g a ra ce
(i) Ten horses are ru no ties the books on same subject remain ‘
suming
an third place, as
(ii) Seven songs are to be rendered in @ programme. Ina how many different orders no hy h
ee rendered ? a
(iif) There are six candidates contesting a ot — office in a municipal election. In ho
ways can their names be listed on a
Bio log y and Mat hematics are to bes |
h in Che mis try , Phy sic s,
(iv) Four books, one eac are~ he seated next to each other, in cae
done ? 8e
a shelf. In how many ways can this be if all ibe ladies sit
toge
oftheir seating
of different colours ite
(v) How many different signals can be generated from 6 flagsothe
the r ?
makes use of all the flags at a time, placed one below each word is to
med using all the letters of t ee
How many words, with or without meaning, can be for (i) have no two vowel Re |
th
EQUATION, using each letter exactly once ? (ii) have both consonants and voy i te 4
There are 6 items in column A and 6 items in column B, A student is asked to match¢ |
an (iii) have the relative position of ie vowe
s rect or incorrect)
column A with an item in column B, How many possible answer (cor
7 How many 3-digit numbers can be fmed by) Isit
the question ?
How many different 4-digit numbers can be:
4 4 zi
choose a chi
(a) From a committee of 8 persons, in how many ways can we used only once ina as —
2 ae any ¢
more than one position? =
chairman assuming one person cannot hold
(b) From a pool of 12 candidates, in how many ways can we select president,:
secretary and a treasurer if each of the 12 candidates can hold any office ?
‘How many different 5-letters words can be formed out of the letters of the eee ;
many of these will begin with D and end with E ?
eas we cone! i 6 line, each arrangems
arr nts are possible ? How many of them start w
of the )
of different ‘Seleters words formed from the letters atthe

Scanned by
Scanned by TapScanner
TapoScanner
SPECTRUM DISCRETE y,
De aa ae |
as different. nk
| ‘
if = 4
of n;
® pine pe a

ade
# = tn Fee

umber of permutations :
ede and of et ing alike and denoted by *a’, 4 of th
er ‘Sm |

ened fo
Proof.f£. Let n things be denoted by Ee
and denoted by ‘6’ and the

Let x be the req uir edmum ber 0°7 9" a


od
p . Th
ar
es
ch
e
new pet ers
arttem tet
canbe aranged an &aoathey
letters Re a 78

e x X |P
Now, consider one of thes s. A:
emg tes
bi, th --- ce uca changes mag _
peut wi eve rise gpemmutaonandifs
; «sch
te to ta l nu mb er of pe rm ta ti ons will Pe * [P|
|p permutations then
een ings lena} 3 8NE BT
ou number of peut of

wee xx[exlel2 7 #7 Tpthgeg a ‘eidtal


er things are alike
tion to
Extension. This rule can be extended if in addi
a1 to 7

ber
of permutat
Example 1. Find the num ions of the letters of the word ALLAHABAD. —
ALLAHABAD oT a ge a 6
Sol.

ue oe gat ¢ va
aig
eaeof icy a / pi had F% Pie ia
A’s=4
|
=2
Number of L’s

Er ae
mindomioen=5

Example 2. In how many ways can 5 flags, in which ys


ame
a staff, one blow the other, if flags of one col
Sol. Total number of flags =s.-

ud ©

Scanned by
Scanned by TapScanner
TapoScanner
{ul
Remainin11g letters can be arrangedin px[4x22

=
11x10X9x8x7XOX5*14
Gx
_ 138609
2x1)x[4x@xD

one letter.
(if) Consider five vowels as

wm a
* sens canbe arnglin =<

in + ways.
Also five vowels can be arranged in themselves

ah
of
*
number of arrangements in which vowels are always together
[8 : [5 _ 8xTX6x5%4%]3 X14 Same

3.12 [4 |3x (x2) [4 3360 x :

(ii) Required number of arrangements= 1663200— 16800 = 1646400 eC


and gP in the end.
(iv) _ Fix| in the beginnin
10
-“. required number of arrangements= iB [412

10x9x8x7x6x5x|4 ‘sage a.
- —————— = 1960)“aed
(3x2x1)x| 4x (2x1) Se

Example 5. How many numbers greater than 1000000 can be formed by using hodient
Sol. Given digits are 1,2,0,2,4,2,4 ae
“. total number of digits = 7
Number of 2’s = 3
Number of 4’s =2
Number of digits to be taken at a time =7

Scanned by
Scanned by TapScanner
TapoScanner
6 be ad s of di ff er en t co lo ur s form a necklace ?
Example 1. In how many
ways can , i
—- = ie
ay
Sol. Number of beads = 6 "|
“5 =— x(5 xX 4x
* required number of necklaces Oe > D6 “|
ie
i Th

arn sisisae In how =


C and D are to SE mong
Example2. Four persoA,nsB,
seated?
Sol. Number of given persons = 4
them in a circular table = [a1 = |3 =3x2x1=6
Number of ways of seating
boys and 5 girls be seated at a round table, so that hoe i |
Example3. In how many ways can 5
together?
boys .
Sol. Let the boys be seated firstly leaving one seat vacant in between each of two

ways.
This can be donein [5-1 i.e., in| 4
[5 ways.
Now5 girls can’be arranged in5 vacant seat ‘Ps i.e,
iS; required number of ways = |4x|5
=(4x3xK2X1)X(5X4X3X2X 1)=24x 120
EXERCISE 1.6
a round tableprovided Parveen
In how many ways can8 girls be seaatted
together ? adit ia
In how many ways 4 boys and 4 girls be seated at a round table p Se:
betwee ns?"
two girl ets
The Principals of six colleges seat themselves round a table
‘problem. in hey eae Oe i ee e an

1.10. Combination

RS ory ee
’ SSeS

Scanned by
Scanned by TapScanner
TapoScanner
“ Ss ugh gee tol eae “A ate
: ons 2 ,
nies
saanlyest®combinationsowen thing.
“nthings taken (r— 1) ata time.
things taken rat a time.
+ number of com in | : L. verify that ad ata get orm a at
| “a pont sec 4)+C (8, 3)= apes oe os
ovG = ae se" C, | oy B | m oP gx 7x 6x5 | 8x 1x6,

or We "C=" "GC ) ae 5. e ee x 3x 4 a1 x a x an: Z

Another method
[n+l Oy
RHS.="*'¢,=——
[r n-r+l
Nae
_—
—_—_—_—

LHS.="C,-1+ *
Ln Ln
erreite -r+l “Tr [nor - [r=1.@e Pri ee

< La , +t]. [n ti
[r-1|n-r[n-r+l or [z=-1 [n= 219 Th r+) ce
[n n+1 |- (n+ 1)|n_
‘fete [roar] CED Gar+ dpe
L.H.S. = R.H5, a=

OR “Aa
om
Tete pa

Find the value of Cy + "Cy +"Cy+...+ "Cy


Proof: Here each thing can be dealt in two ways. Either it is winded a
ae
Hence each thing can be disposed in 2 ways. rot shoMh pie
But each way of dealing with a thing is associated with each way of dealing
total number of ways of dealing with » things am
1 RIK2 nie
But these include the case where none of the
things is selected, |
the required num
of b
waye
s=2r
"”-]
Again, we may select | or 2 or 3 or4..or nt hing: out of th

Scanned by
Scanned by TapScanner
TapoScanner
“te
i. of - ie

Pare eee *:
eae SEB ‘oom
at 3 ah ay? yz
re Py

+90)+ 9G+9Oy,

3 n- I3r+13= onto
4n-13r+4=0

“ais
Raligt 2 wir -
ee

hb pote Aangve
7 =6:9: 13, find ante

nWebe Sac

SPARES Fy oF

Scanned by
Scanned by TapScanner
TapoScanner
vena + ph 5: fr

|
Ss
:

| IFC (wm, 10)=C Gr, 12), determine mandhence GE


jee Ci n, 5).

6. If "c= §C,4;, find Fr oe de

7. Prove that C(27,n)=

8. If m=C(n,2), prove that C(m,2)=3 C(n+1,4). eee


9 If "Cpi"Cpey=1;2and "Cr+i: "Cr+2= 2:3, find m:
1.15. Practical Problems Involving Combinations Pik
i acne

ILLUSTRAILV
be selected fro
Example 1. In how many ways can a committee
(i) 3 members. i) 13members. wg
= ee ot ‘+ amk «
Sol. Total number of persons =15 ae, So 3
hry = Bile
() Number
of persons to be selected=3

Scanned by
Scanned by TapScanner
TapoScanner
206 SPECTRUM DISCRETE
‘ Le
. Example 6. What is the number of ways of choosing 4 cards from a pack of 52 Play
E many of these
| (i) four cards are of the same suit,
(i) four cards belongsto four different suits,
(tii) are face cards
(iv) _ two are red cards and two are black cards, Fs: .
(v) cards are of the same colour ? | pont nena al
Sol. Total number of given cards = 52 id ona a 9 3
Number of cards to be selected at atime=4 ag ag ciao
: 2c. =- - 52x51x50%49 _5 a
- in
rey

= Pat
“: required of ways = “C4
number “TxaK ax 4
biapedl ‘sndhhe Z my
(7) efou
Thare suits namely diam eH chu
rre

, way s of cho osi ng 4 dia mon ds, C4 ways 0


There are '3C
des and 'C, ways of choosing 4hearts.
4 spang
choosi
“ required number of ways = eo. + Popa F
eb ms

Scanned by
Scanned by TapScanner
TapoScanner
(6) lboy, 4 girls Pues ; Bik dost
4
es inchivea sider eGR 1G Ep Cass ae
1x6 AT <a |
= Cy
Tr. x. 4 °C)+ 7 Cc; x1; ee
PG.basta

=§4+7=91 — se

Example 8. From 5 consonants and 4 vowels, how many words can De ¢ nstructed
2 vowels ? me eg te “rte
Sol. 5 consonant ou canbe sels in", way and vowel ot of canbe
5x4x3 Ax 3
. total number of groups fixie, x ‘o= 1x2x3" ix =10 :
Now each group contains 5 letters, which can be at = ged amo!
required number of words formed = 60 x|5 = 60 )
Example 9. How many words, with or without meaning,¢
from the letters of the word DAUGHTER?
Sol. DAUGHTER cient
8
Pree

Scanned by
Scanned by TapScanner
TapoScanner
a Saar eaeaiael mea

SPECTRUM DISCREYR. 2
——=—_—_—

3. In an-examination, a student is:tovanswer 4 questions out of 5. Question 1 ang>


the student can make the cho;
compulsory, Determine the number of ways in which
4. In an examination paper on mathematics 10 questions are set. In how many dj
number | is made compuls x
choose 6 questions to answer? If, however, question
stions in all ?
Ways can you select to answer 6 que A is chosen, then B must bea
5. We wish to select 6 persons from 8, but if the person
‘ e;
made ?
many ways can the selection be
k of 52 cards st a ,
6. Determine the number of 5-card combinations out ofa dec
has exactly one king. -
7. a
A father with eight children takes them 3 at a time to zoological garden as c
without the same three children together more than once. How often will each
often will the father go?
8. In how many ways can a cricket eleven be selected out of 15 players ?
Poi? a
(a) In how many of them a particular player be excluded ?
(5) In how many of them will he be included ? 63
9. Show that the total number of ways in which six ‘+’ and four “—’* signs can
such that no two ‘— signs occur together is 35.
10. A question paper has two parts, part A and part B, each containing10 que. |
choose 8 questions from part A and 5 from part B, in how many ways can he ;
li. A bag contains 5 black and 6 red balls. Determine the number of ways in v hic
balls can be selected from lot. = ota!
12. A bookshelf contains 7 different mathematics cock ea do d
many groups of 3 mathematics and 3 physics textbooks can be selec
There are 6 boys and 3 girls in a class. An entertai nmittee |
13.
there are:4:boys end 2 girls inth e commit ee
14. For the posts of 5 teachers, there
there are 7 SC candidates among
15. (a) In how many ways ca
(6) How many ¢
girls and 5b

Scanned by
Scanned by TapScanner
TapoScanner
212
1.16. Introduction to Elementary Combinatorics
Discrete mathematics or combinatorics deals with existence, enumeration, Classifica,
"
©ptimisation of arrangements or selection of discrete structure. birdy Cviok FES ep aad mE
since; ig
Le hy isteD
e 4

1.17. Two Basic Counting Principles


tag
(1) Sum Principle Per see
(a) IfS= A, VA, Vin UA, td A, re mutually dso ss Le,
<1) tect nae sa
A, NA, =@ifi#j, then | 3d oy ee

Nitta al rales |

(6) An alomatve my of ting i pip a


x have?
\f Ev
.

Scanned by
Scanned by TapScanner
TapoScanner
214 : SPECTRUM] )

Example 4. If the lincense plates of a state require3 english letters followed by 4 r


can be manufactured if | suse ha t aaa
(a) repetitions of letters and digits are allowed ee hg
(b) letters are repeated but not digits ca
(c) No repetition are allowed. ae 1 her | : . = “ A

uu ih SayA—ia
Sol. (a) Three english letters can be choosen in 26*26*26 ways.
Four digits can be filled in 1010x1010 way ai Ta
as
Lt
M

. By the product principle iO eau ‘seach


Total number of plates manufactured= 267 210%0 hi few boas
(6) Three english letters can be choosen in 26x26*26 ways. anita
Four digits can be filled in 10*9*8*7 ways ss Nw
. By the product principle — a al Ca
Total no. of plates manufactured = OS eAE
(c) Three english letters can be choosen in 26x25*24 ways. 2h
Four digits can be filled in 10x9x8x7 ways 3 on
“. By the product principle <9 Wake ae
=
Total no. of plates manufactured 26%25*24x10x9x8x7
Sometimes we need to combine the to f na )

Scanned by
Scanned by TapScanner
TapoScanner
216
: = snletaersda table ti udents }
a P Rate | eed Q
= epudentsene
ier 32 SES
ee will have :
m5) 5 books each ad
ton of objects in whch objets rallied 10 erpete i clled amu
cllecs
grAee
= SS St Oma ae B
e.g. S={a,a,a,b,b,c}={3*% a,2%b,1*¢}
§ Wy) os rhe tc Yeh se ,
In general S = {ny Hy Pa yy ereesreny My MARS its,

isa multi set in which a, occur 7 times, 4) OCCUF 7p Het and anon
1.19. r-permutation ar ita: a
elements from J
By an r-permutations, we mean an ordered arrangement of
of nS is just called a permutati
n-permutatio on
ofS.
1.20. IfS is a multi. set with infinite repetitions then the number of
where k = number of distinct objects in S. uae el
Proof. Since each of the r-place in the arrangement can be filled in k ways. by
objects. | 7 ae
” By multiplication principle f i # a ; il ed da Diet ae has ae : Fe

; =k. Fhe Bept orev bens a hae


minh toge: odio Ae. OE Bah
Remark: If the repetition number aa i maa tenste
ae i aye Qfos

Scanned by
Scanned by TapScanner
TapoScanner
218 —_— — =
SPECTRUM DiscrE

Proof. There is a one-one correspondence between the above solutions and the choice of
as follows
from {1, 2, ....... m—1} a
Let Hy + Xy Hone. +x, =m, x, 21.

Let g 0 -cees Vey =


Wy =e, Ya = Hy HX +Xy Fervent poy = TO Sy.

Then I<yy < yp <...< yyy <m—l Since x21 0 aRea


Conversely if Vs Vor eonee Py Satisfy (*), ior we take
:
riiy Ty‘ahr
:

~ YI: oe i3-- Fir aheneny x, = vp jen Xp] Opi Spas '

Xe yey a aa Sg hay; a
This gives solutions ofae ae I; 21 ES Eo

Scanned by
Scanned by TapScanner
TapoScanner
(0) ” they are an or aitnacat Ravous cm
(7) they are not necessarily of different flavours ae
(tif) they contain only 3 different flavours Ob sat og:
(wv) they contain only 2 or 3 different flavours ?
urs
be denoted
Sol. Let the flavo by A, B,C, D,Eand F.
()Here we want the numberof ways of
of 6 different flavours.

Hence the
| Rencrniees aa 2 7 soot
pet a ; Aa au
requir: ed number of‘ofdidifferent ol flavours is C4 > oe actS e
(i) The number of 4-combinations of 6 Seen alloy
(7) The number of ways of choo: ing 4 cones of
F Mpaiegtame sts
eB f ' cS ys ‘-? ae dyed
_

Scanned by
Scanned by TapScanner
TapoScanner
222 SPECTRUM Discrey ss eet ee Sa i gen ee greta fh oe
es “ — ———S

ire tie RA ime ae arth ore anaesr Ga F


Proof. This follows from the above theorem a
. ai ; ‘ : . s oO glee eek
Since each unordered partition give rise to &! ordered partitions of the types (g, qin ¢ INES ae peta aneed fe:
a

‘ie pico n! n! a Fe.

& ! (Required no. of partitions)= (qk | ae \


qq!

*. Required no. of partitions = nt ; ole


k1(qt)* ; aa
Example 1. In how many ways an 12 out of 14 people be distributed into
3 terms ¢
has3 members, 2nd team has 5 and the 3rd team has 4 members. Grr Aas :
cal ae in iene. a) 3 st
Sol. Let S= A, UA, UAj, where |Ay| =3,|A21=5,1A31=4
SETA: saat
[Sj=3+5+4=12 is

The number of ordered partitions of S of type (3, 5, 4) Trav Tie Io

Also the 12 person can be choosen out of 14 peoples


in C (14,
7 = 121 48 Ly ee

14 p
of g
n
Example 2. Find the number of ways of choo12souti
ih
i)

Sol. Here S = Ay UA UA3, where |Ay| =|Az| 14x 31 -4


“¥
t
7

et
Spy ts j Vie

Scanned by
Scanned by TapScanner
TapoScanner
224

Multiple of other
Sol. Each number can be represented
by (2") X n where 1s oc
So now as there are only 10 odd numbers between “4 hg “select
eee 11.
1, 2. 3. ccsouy 18, 19, 20 then by Pigeon Hole Principle wo
two of then
thom ae bound
Hence one of them can be divide by other. Hence proved.
Pumpe een yorar ns eprbnge obo ar
no farther apart than 1 unit. oc wsioglnosgig went Dg
Sol. We must find the pigeons and the pigeonholes.. ‘So divide the
Will be an equilateral triangle of length of each side as one 1unit, Now
po pve
points as our pigeons. So by Pigeon Hole Principle two
be at a distance less than one unit, Hence Prove.

coach rae

oes hee
(+2+3)= fesse aan
fro ofthe tl ia ae ae

Scanned by
Scanned by TapScanner
TapoScanner
226
oi SPECTRUM Discy
Hence at least one of the numbers Gy, F75+
09 Gy is greater than or equal to k,
nee carne:
ia ‘ha Ni 6 an, ‘EXERCISE
Similarly if each of the numbers a), 4,-.,4, iS greater than km, then the sum of | Zak
cs a rs) es 5 aia |
ace | ; ee An est a
Would be greater thank which again ona (1)
less than or equal to
thal ther is at easton ofthe
k
2.4. Pigeonhole Principle (Strong
Form) ‘
Let F1>92+--»4, be positive integers. Ifgq,
+ q2 + ...... Qun—nt1 etiam
objects are put into n boxes, then either the Ist box contains at least q,
objects, or ¢ .
least g, objects, ....., the mth box
contains at least g, objects.
Proof : Suppose it is not true, that is, the ith box contains Veepmies
at most gq, —“1 objects, i= |1
total number of objects contained in the n boxes can
be at most
(91 1) + (go -1) +... + (Qn—1) = 91 + Qa tn + n—1,
whic
is one
h less than the number of objects distri
This isbu
a contr
te adict
d. ion, —
Fre, Pie form of the pigeonhole principles obtained from the strong form by a
ng
Then N+ 92 +..+9,—n+1=2n—nt+l=n+l
aS ee
Cor: In elementary mathematics the strong
form « ) the pis onho
Special case when g, = gq, =. = qn =r. In this case the principle
b
° ifnzc¢-— 1) 1 objet repaint
boxes, :
objects.E

Scanned by
Scanned by TapScanner
TapoScanner
228 SPECTRUM DISCRETE y
~ es | x :
Example 2, If 11 integers are selected from {1, 2, 3, 100} then prove that there are at teas

y 6, such that Se -Vb| <1.


@ and
Sa y . ¢ 1 eee + c i eee dia san we ae

Sol, Let x € $=
{1, 2,3,..., 100}. ef

Then 1 < Je <10 = 1<[Vx] <10, were fs Li te ner pal


Thus for elements
x of S, [Vx ] must be one of 1, 2, ....
10 .
from S, Ext coe principle, at at
Therefore, if 11 numbers are selected
and 5, must have same integral parts.
> Vb =i+ fy where0< fi, <1 mts
Va =i+ f and eee hi
. Rs
Hence the proof.
Example 3. Let n be an odd integer. If ij,i,..,/,, is a permutation of 1, 5m r
| oma
(1 — 4,)(-i,)...(/-i,) is an even integer.
Sol. Let m = 2 m+ | where m is non-negative integer.
Then the set S = a, 2, ... n) contains (m + 1) odd numbers, namely 1
numbers, namely 2, 4, ..., 2m
-. There are (m+ aed numbers and m even numbers to the tint iyi
Consider the (m+ 1) numbers | tse
1— i. 3-— By oeeey na tire -

Since i, (l =r <n)is cay a EIN are


er
* lh

by pigeonhole principle, el aS = iy,isodd. Se oe


gal aK

Scanned by
Scanned by TapScanner
TapoScanner
230 SPECTRUM Disey
tn ae

Example 7, A storekeeper’s list consists of 115 items, each marked “available” or “unaya:
if there are 60 available items then there are at least 2 available items in the list exactly
4
Sol. Let the positions of available items be A] +29. GQ:
Since agq < 115, therefore, the 120 numbers
ay < a> < veg

and) +4 <a, +4 <n < Aggy +4


lie between 1 and 119.
| a
~- By pigeonhole principle, two of the numbers in(1) and (2) must be equal,
But the numbers in (1) are all distinct and similarly numbers in (2) are all dis
number in (1) must be equal to a number in (2). * ‘:
=> Asome
i, j for which a;~ a; +4 = a;-a, = 4, as required. O 2a See
Example 8. Each student of a class of 27 students
go swimming on some of the nes
Friday in a certain ; week. IfOieach student goes at leastaa: twice
students who go eee _ cera’ I the same days. an then I show
a that“ar
- sh
at

Scanned by
Scanned by TapScanner
TapoScanner
oe

232

aes 12. Let numbers | to 20 be placed in any


numb ers
must be atleast32, 0 z
Sol Let aia.2 placed around
ae ee me, ane a! tee pron» Slt Io wal
= al

Scanned by
Scanned by TapScanner
TapoScanner
_ SPECTRUM DIscre
= . then 1 Ao. elie 3 Ladi vigeavce
Thus if p is| the smallest prime divisor of any _
= . 1 ivi -of My

7“ Me ss
| pins of BC and AC respectively. id ex
psn, <19.
17,
But there are only 7 primes namely 2, ae ste i, 13, 17 which are less than
eo
Hence by pigprinc therh
iple,n oatl
e are etwo 1;'s which havea common p
least
means there are at least two composite numbers which are not relatively prime. ee
one of two colours t
Example 16. If every point ona straight line is coloured with
| * aa z
segment whose ends and mid point have the same colour.
Sol. Let every pont ofthe straight ine be coloured with two colours re and black, 4
Then there exist2 points, say A and B , withthe same colour, say red.
os ete © ee aah
Let C be the mid-point of AB.
a Bs je
We have two cases
Case I: When C is red. nemo? aa ane to) a
is the r seg en ene tee ca vt radt ‘fuse ie aie
In this case ACB

aes ag ee pe patihets 9 e |
Case II. When C is black. Ca

, x— =
A’

In this ease take point A’ on the A


CsuchthatBB’=AB,
WA‘and B both are black then A’

Scanned by
Scanned by TapScanner
TapoScanner
236 ee =

re I ah for some i= 1, 2,.....7


which contradicts (1).
‘fe ay = hy. Vi= 1, 2: ony FT

Hence Bp Ap verry is a decreasing , se


A Theorem of Ramsey oy, Fits :

example

Siet hg:a
O 6E in oma
umca ci ese
pore

Scanned by
Scanned by TapScanner
TapoScanner
5

8 A The Inclusion —Eacieswin Principle


The principle of inclusion and cues ote ae
enumeration. Let A and B be two subsets of a set S, then to count
« cot the
count the number of elements of A and those of B—A and add.

But |B—A| =|B|-|ANB|. 5


Hence JAUB|=|A|+|BJ/-|ANB|.

In other words, while counting elements of A and of E |


ate Pounted Syiee and 0 iy ordex 10 ut cul
|A|+|B}.

Scanned by
Scanned by TapScanner
TapoScanner
3.2. General Principle of Inclusion— Exclusion |
Let A,,A.,,...A,, bez finite sets. Then

IA, UA, U...UA,I= DIAN] IAAL


i=l lsi<jSn (a

$ DAQAP SO Ag mt C O M AY
sels
<i<j<ksn Pee
Proof : We prove the result by induction on 7.
For m = 1, the result is obvioustrue.
ly
For n = 2, the result becomes

|A,VA,| = SIA\| = SlAi OA | | a


i=l i<j
he. = |A,UA,| =|A, [+142 1-14, 9A2 | 7
which is true. ee
Let the result be true for sets lessthanm,

a
a
are tev.
sae
=
-
=—ee
a
oe
a
ee =
er

Scanned by
Scanned by TapScanner
TapoScanner
ane SPECTRUM D
Sol. Let M denot € the set of teachers who teach mathematics and
P denote th
teach physics.
nee

“(MU P)=20, n (M) = 12,n(MNP)=4 oe


Now 7 (M U P)=n(M)+n(P)—n(MNP)
> 20=12+n(@)-4 = n(P)=20-12+4
s n(P)= 12
. 12 teachers teach physics,
Example 4. There are 200 individuals with a skin disorder, 120 had
Ci, 50 to chemical C3, and 30 to both the ch been expo
exposed to emicalC,s and C). Find the te ae
a ay oe Ee trl bee
tee hes
a

(1) Chemical C; but not chemicalC,


(7) Chemical C, but not chemicalC; a
(ii) Chemical C, orchemicalC, a hee
Sol. Let U denote the universal
set consisting of individuals
denote the set of individuals exposed to the chemical «

exposed to the chemical

‘_-m(U)=200, n(A)= 120, n(B)=50, (A. B)=


Cr. il CRUST af

oie

Scanned by
Scanned by TapScanner
TapoScanner
244 SPECTRUM DISCRETE y a
: Now (TO C*)=n(T)-n(TAC)
l4= 30- n(T AC) = (TNC)=16
number of persons drinking both tea and coffee = 16
- Also n(TUC)=n(T)+n(C)-n(TNC) |
50= 30+n(C)-16 wo
n(C)=36 Ute is
number of persons drinking coffee but not tea ‘even a ad
=n(CN T)=n(C)-n(CNT)=nC)-n(TNC)=36-,,
Example 9. A town has a total population of 60000. Out of it 32000 read ‘T
Paper and 35000 read ‘Times of India’ paper, while 7500 read both the news
many read neither the Hindustan Times nor Times of Indian —
Sol. Let U denote the set of population of town, H denote the se e0ph
Times and T the set of people of reading Times of India.
tty naats
(=H )
32 00 0, n( T)= 35 000, MOT)=7500
n(U)= 60000, n
Number of people reading neither Hindustan Times nor imes of In Fp
U) - nH UT) Bi
=n(H' VT’)= n(H U T)' = n(
ae n(H TY] = 60
= aK)—UNE) t0=
= 60000-5950 500

Scanned by
Scanned by TapScanner
TapoScanner
246 ee a zs SPECTRUM Discrrpr:.
RETF <_
Also total number of viewers = 500
Now » (F UH UB)* =50
> 500—n(F UHUB)=50 > n(F UHUB)=450
= B)— si
(HM n
n(F) + n(H) + n(B) —n(F MN H)- E) den cri
> 285+ 195+ 115—70-50-—45 +n(F NHMB)= 1502 1H z
viewers watall
of er
-. numb chithe threegam
ng es=20. = | 1
Number of viewers watching football alone="(FAH®A B®)
=n ©)- —n(FN a —n(FNB)+n Oe HN ma eee ‘aps |

= n(B)—n(BNH)- nBN ME NHN B)= is

= 190
+ 95 + 40
= 325
Example 14. In a survey of 260 collene ents,

Scanned by
Scanned by TapScanner
TapoScanner
|
5
-ExcLUsION PE

(6) Number of students who study French and German but not Russian eel
=n(E UG) —n(F AG A R)=20-8=12 reas m2 3] = 10.
Number of students who study German and Russian but not French
=n(GAR)-nEAGAR)=15-8=7 |
Number of students who study French and Russian but not Geran

=F NR) -ME NG A
ARBBAT n c been we
ja
2 hoe aan
Number of students studying only French A i aa ay fa
9 G)- 7 Osea
= n(F)—n(F ee | ae "ve Beh
Number of students studying onlyGerman
=n(G)—n(G MN R)-n(FNG) +nNG
(F AR)=
Number of students studying only Russian Shae me

A R)-—m(G 1 R) + mF 1G A R)=4 A?
= n(R)—n(F
Number of students who do not study any of the lan; wages
= n(F AG AR)') =n(U)-n(F UGU
ees

Scanned by
Scanned by TapScanner
TapoScanner
y
Iscre: TE
hotieane SPECTRUM D
oe
-
A haa it shi
Also n(U) = 1000,
() n(A UBUC)=n(A) +n B)+2(C)—M ANB) ists

543
= 333 +200 + 142- 66-28-47 +9 =
= n (A S AB E nC ") =" (A UB UC )* )
Required numbers = 1000— 543 = ASI, 3 ;

=n
enthne Ben
229
=333 —66-47+om9=1 10 1000 w Pee
mb erof integers fr
Example 19. Find the nu
.
A; be th s integers from
e setof
, A > ,
Sol. Let A}
pay) = [32 =
Then 7

Scanned by
Scanned by TapScanner
TapoScanner
252 SPECTRUM DISCRETE yy.
= |Ay|—A, NAVA, NA;)| : ye
= |A,|-[|A,;OA2 | +14, MA;(A,
|—- NA2) N (A, OA3)] ™ ¥g

= JA, | ~[Ay Ag] — 1A, OA31 +14) A043 |= 166-35 aaa a


EXERCISE 3.1
1. If X and Y are two sets such that X U Y has 50 elements, X has 28 elem 2 os
32 elements, how many elements does X M Y have? '
2. IfX and Y are two sets such that » (X)= 17, (Y) = 23, n(X U Y)=38, §; a
3. In a group of students, 100 students know Hindi, 50 know English and
Each of the students knows either Hindi or English. How many students
group ?

4. Let A and B be two finite disjoint sets such that (A U B)=400 and (A) =:
5. In a group of people, 50 speak both English and Hindi and 30peo
not Hindi. All the people speak at least one of the two A ,
speak English ?
6. Let A and B be two finite sets such that m (A— B) = 25, ms UB)
Find n (B).
in a class of 50 B.A. IBemein 12,students hav

Scanned by
Scanned by TapScanner
TapoScanner
In a town of 10,000 families, it was found that 40% families.
newspaper B and 10% buy newspaper C. 5 % families uy
4 % buy
A and C. If 2% families
buy all the newsp
which buy () A only (ii) Bonly (i) noofne
A, B,andC,
In a survey of 60 people, it was found that 25 people read N
Newspaper T, 26 xed Newspaper 1 2 soe vend ad te
both T and I, 3 read all three news papers. Find
(4) the number of people who read atleast one ofthe ne anedtt
(i) the number of people who read exactly one newsp:
In a survey of 100 persons, itwas found that 28 ead m né

read all the three magazines. Find


(4) how many read none of the dats nagazine
and C2) SOE wea Ia Ey : ly. a tan

Scanned by
Scanned by TapScanner
TapoScanner
256
A , | is eq ua l to th e n u er permutation
mbof
Again | A, O
MATH, IS, F,U,N; a ae
|A, NA, | cate umber ofprmton fis

MATH, I, S, FUN ; 7 | |
of permutation of six symb
|A> OA3 | is equal to the number
M, A, T, H, IS, FUN
and |A) NA, OA;| js equal tothe number of pen sinasiof aeyitihces
UL at lee Og
MATH, IS, FUN -

so that |Ay AA, | =5!= 120, VAL A3 |= 41-24, areas


|A, OA3|=6!=720and | A, Shara! a =6.
:
Pan

=
ee
bark |

EOIN See”
="

.
reat

oJ
F
A

|
a
Ce

rt
es
ad

et
5
ial
mi

Scanned by
Scanned by TapScanner
TapoScanner
258

Example 2. Find the numofber


integralsolutions of xy +X +5524 su
12 Sx) 5 18,-1< x; = 12. ore 277% studies rae
Sol. Let Y= 1, Yg = Xz 12, Yy
= 43 1 Ve pp rgiino sib tr iy
Then given equation transforms to ay tert tras dag
Vp tlt Yo t1Wt Py Sl S2h oe a ‘annie
OF Vy tV2
+ V3 = 12. e "
Let be te set ofall non-negative integra solutions of
. Sb eenshy s3enee
oct bees
Then|S|='2#3-I¢,, = 4c, =91. ‘
Let Aj;,A,,A3 be the sets representi
¥3 = 14 respectively. | die eters
To find | A, Lowe abt cteep ane ae -
* Fi et a 22 - Y2> 44> 3

Scanned by
Scanned by TapScanner
TapoScanner
ag"
=e
ve
i- a fee e re
- . es

is en
{1, 2, ...,2} of n elem ts
a permu tati ion ee
autht > Wee
NS hate ceacea ri
TRE fete 2, oy be aoe Thus a derangement of eer
Gy coh aang age eae nt osition coma
We denote the number of derangements
of {1, 2, ...,1} Dae

3.5. Forn
> | ee D mle Hepeiniyg jag
M a ear

f aes 3 5 Pw ee ie
PEs . 1A)|=0-01 Visisn i
ay | The set ierie Lat |

Scanned by
Scanned by TapScanner
TapoScanner
262
SPECTRUM Disc RET
e dium Dt i 5: i Futd aepe i = ei
a

Similarly d” is equal to the number of derangements of {1,3, ...,} so that


d’ =D a-I*

From (3), (4) and (5), we have


Daa) (D,_» + D,,-1)

> D,-nD,-; =— [D,4-(@-DD,_9].

i
ik

‘ “1
; (Ts a
cece ereeeeneeeen eens steesieenrestecnrsseees 3 Tre ro :
. > >


i
SOSeRESEE
Ph cheee ee ene eee ceenes

Scanned by
Scanned by TapScanner
TapoScanner
ATE TN
264
di st in ct p r i m e di vi so rs o f n,
-s si sn }- Le t Pj » P22 p, be
Sol. Let S = {1 , 2, 3, «u

vi si bl e by Pj} i. Si k
Write A, = {x E S : x i s di
s A i , Ay es | e
hy a|
ar e th os e in no ne of th e su bs et
ti ve ly pr im e to n
The integers in S rela
e | f
A, UA, eesees
| = | S |—|
= | Ay nA, (O) sasens nA,
>. p(n)

e - mu lt ip le s of d in S
If d\n = then there ar

n
aw di y N A , | = o s | Ay OV Ag Deve
5 id A d te bewene
P,P;
Pj

pr in ci pl e of in cl us io n an d ex cl us io n
By the
n
p(n) =n- ee— + >. bee sseee
P\P2
l<ij<k PiP;
i=] Pj

= fy ei a
Py P2 Pr

Scanned by
Scanned by TapScanner
TapScanner
Scanned with CamScanner
SPECTRUM Discrey P
268 EM
. . AM
i statement Is any meaningful, unambiguo,s RY
Note 1. Some authors define statement as - A AMbiguoys d Clara , aio
but not both.
sentence which is either true or false "

Note 2. A statement cannot be true and false at the same time. This fact is known aS the J, Ww

excluded middle. yy
pound Statements
1.3. Logical Connectives and Com
Any statement whose truth or otherwise does not explicitly depend on another Statement jg
. : $a]
Vio,
simple. For instance,
8 is an even number.
ents.
The set of real numbers is infinite are simple statem
ents.
A compound statement is a combination of two or more simple statem
The pharases or words which connect two simple statements are called sentential connectives 1,
tives are “and”, “op te ae ,
connectives, logical operators or simply connectives. Some of the connec
‘it
then”, “if and only if”.
When simple statements are combined to make compound statements, then simple statements»
called components. Our problem is to determine the truth value of a compound statement from the ms
values of their components.
Note. Simple statements are generally denoted by small letters p, q, r, 5, f,...

~
1.4. Truth Tables

It is a table giving the truth values of a compound statement. It has a number of columns (venig
lines), and rows (horizontal lines). The number of columns depends upon the number of simple statemea
and how involved are their relationships. The number of rows in a truth table depends only upon
number of simple statements. /n case of n statements there are 2” rows. The truth tables are very helpfila
finding out the validity ofa report.
1.5. Basic Logical Operations
There are three basic logical operations :
1. Conjunction
2. Disjunction
3. Negation
which correspond respectively to “and”, “or” and “not”.
Conjunction i
Any two statements can be combined by the connective “and” to form compound statem
A ate ent called

“conjuction” of the original statements,


For example, consider the two statements :
He is practical. He is sensitive.
The conjuction is “He is practical and sensitive.”

Scanned with CamScanner


yr

269
oct AND PROPOSITIONAL CALCULUS
false even if one of the
The conjuction will be true when both the statements will be true and
wcponents iS false.
in symbols, two statements are denoted by p, q and their conjuction by p A q. This is read is “p and g™.
Rule. p A q is true when both p and q are true.
Truth Table

q P A q
7
T
F F
T F
F F
Examples
1 Let p:35+7=12
q:2 is a prime number.
pAq:5+7= 12 and 2 is prime number.
Now pis true and
gq is true
p A qis true.
Let p: Every even number is divisible by 2
~

q : 12 is an odd number.
p Aq: Every even number is divisible by 2 and 12 is an odd number.
Nowp is true and gq is false.
PA qis false.
Disjunction
Any two statements can be combined by the connective “or” to form a compound statement called the
“disjunction” of the original statements.
For example, consider the two statements :
p: There is something wrong with the teacher
q: There is something wrong with the student.
Then p V q: There is something wrong with the teacher or with the student.
This ‘or’ is inclusive or, that is, there may be something wrong with the teacher or with the student or
with both.
The disjunction will be false when both the components are false.
In symbols, the disjunction of two statements p and q is denoted by p V q. This is read as * p or q”.
Rule: p V q is false when both p and q are false.
Consider another two statements :
p: shall watch the game on television
q: 1 shall go to college
PY q:\shall watch the game on television or go to college.
This is exclusive ‘or’, both p and q cannot happen together,
Exclusive OR or X-OR has the symbol V .

Scanned with CamScanner


SPECTRUM DISCRETE y
M4
270 i
both.
p or g Is Tue, but not
Rule: pv gisuu 2 when either Truth Table for 7
Truth Table for V

T T T qT t F
F T qT F T
T
E T T F T T
F F F F F F
Examples _

p¥q:5<12 or §+3=12
Here p is tue and q is false.
pY qis te.
Let p: Every even inteisge re
prim
id

q:5<3
re or5 <3.
prim
pV q: Every even inteisge
Now p is false and q is also false.
p \ @is false.
3. Let p:3+5=8
gridtée=2
pv q=(3+5=8) Vv (1+6=9)
p isw
No true and q is false.
pY qistrue.
Negation or Denial *
Negation aree
fs

there corresponds a statement which is its negation.


To every statement,
careful while writing the Sal)
contradiction and not to a contrary statement. We should be very
r place or to put the phrase
given statement. The best way is to put in the word “not” at the prope
pi
the case that” in the beginning.
For example, if p stands for “He is a good student.” Negation of p, denoted by ~ One
a good student.” We canno t say tha
“He is not a good student” or “It is not the case that he is
student” is the negation of p.
Role. Jfp is true, then ~ pis false.
and ifp is false, then ~ p is true.
Truth Table

P ~p
T F
F T

Scanned with CamScanner


Pe
[oGIC AND PROPOSITIONAL CALCULUS 271
—_—
xote : Translating from English to Symbols
We should have the following points in mind while translating from English to Symbols.
1. In logic, the words “but” and “and” mean the same thing. Generally, “but” is used in place of and
when the part of the sentence that follows is in some way unexpected.
2. The phrase neither p nor q means the same as not p and not q.
fumple : You are given the following statements :
p:ltis hot
qi lItis sunny
Write each of the following sentences symbolically :
(a) It is not hot but it is sunny
(b) Itis neither hot nor sunny
SL (2) The given sentence is equivalent to “It is not hot and it is sunny” which can be written
gmbolicallyas~p A q.
(5) “It is neither hot nor sunny” means that it is not hot and it is not sunny. Therefore, the given
semence can be written symbolically as ~p a ~q.
Note : Statement Variables
A “Statement form” or “Propositional form” is an expression made up of statement variables ~, a, v
tat becomes a statement when actual statements are substituted for the component statement variable.
L6. Tautologies and Contradictions (or Fallacies)
A tautology is a proposition which is true for all the truth values of its components. In a truth table of
txtology there will be only T’s in the last column.
(a) Truth Table (5) Truth Table

P ~P pV~p P ~P pA~p
T F T T F F
F T T F F
Trample : Consider the proposition p V ~ p. Its truth table is (a).
wea

the proposition is always true whatever be the truth value of its components.
it is a tautology.
A contradiction (or fallacy) is proposition which is false for all truth values of its components.
Mh

Consider the proposition p A ~ p. Its truth table is (5).


vw

p A ~ pis a contradiction.
Note : A compound proposition which can be either true or false depending on the truth values of its
‘onponent propositions is called a Contingency.
Logically Equivalent
Two different compound propositions (or statement forms) are said to be logically equivalent if they
have identical truth tables.
The symbol = is used for logical equivalence.

Scanned with CamScanner


272 SPECTRUM DISCRETE MATHEMATIg

Example. Negation of the negation of a statement is equal to the statement.
Symbolically, ~ (~ p) = p.
Sol. Truth Table
P ~pP ~(~p)
T F T
F T F
Now truth values for p and ~ (~ p) are same and hence p and ~ (~ p) are logically equivalent. Th
logical equivalence ~ (~ p) = p is called involution law.
Example : The sentences p > g and ~ q > ~ pare logically equivalent

P p | p>q P q ~q | ~p|~q7~pP
T T T T T F F T
T F F T F z F F
F T T F T F | T T
F F i F F T | T T
p>q ~q>~p
p>q =~q>—p.
1.7. Prove that

(i) ~(~p)=p
a ...[De-Morgan’s Lav]
(iii) ~(pvq)
= ~pa~q
Proof.
(i) Truth Table

p ~p (=p)
T F T
F T F
A comparison of the first and third column shows that they are identical.
~(~p)=p.
(ii) Truth Table

Pp q ~P ~q P\q \~(pAq)|~PV~q
T T F _ FF T F F
T FO F |T F T 7
F T T F F T T
F F T T F T T
A comparison of the last two columns in the truth table shows that they are identical.

Scanned with CamScanner


qe
y jociC AND PROPOSITIONAL CALCULUS
273
~(pAQ)=~pV~q
(iii) Truth Table

P q ~p ~q PVq |~(PV4)|~pA~q
T T F F T F F
T F F T T F Fr
F T T F T F F
he F F T T F T eo
Acomparison of the last two columns in the truth table shows that they are identical.
~(PV Q=~pA~q
j&Provethat (pA q)Ar=pA(qAr) -.. [Associative Law]
Proof.
Truth Table

Pp q r paAq qr (PAQ)Ar| PAAD


T T T T T + T
T F T) F F F Fo 7
F T T F T F F
F F T F F F F
tT | T F T F F F
ri T F F F F F F
F T F F F F F
F F F F F F F
A comparison of the last two columns show that they are identical.
Hence(pAqg)Ar=pA(qAr).
19. Conditional Statement
Any statement of the form “if p then q”, where p, q are statements, is called a conditional
statement.
Herep is sufficient for q but not essential. There can be g even without p.
Let p: you work hard
q: you will pass.
Now it is possible that a student may pass who has not worked hard. Although p is not necessary for
#918 necessary for p. It will not happen that one who works hard will not pass.
The conditional statement “if p then q” is denoted by p > q (to be read as p conditional q) or ( p
Dplies q).

The conditional p > q is also read as “ if p then q” p implies gq, p only if q, p is sufficient for g, q is
&cessary
for p, q if p.
Rule. p > q is true in all cases except when p is true and q is false.

Scanned with CamScanner


214 SPECTRUM DISCRETE MATHEy,, 7
Truth Table 4

P q p>q :
T T qT |
T F F |
F T T
F F T

1.10. Prove that:


(i) p>q=(~p)Vq (ii)~(p>Q=pA~q
Proof.
(i) Truth Table

Pp q ~p p>q (~p) Vq
T T F T T
T F Fo F F
F T a rr T
F E T T T
A comparison of the last two columns in the truth table shows that they are identical.
P>q=(-p)Vq
(ii) Truth Table
Pp q ~q p>q ~(p>q) pA~q
T T F T F F
T F T F T T
F T F T F F
F F T T F F
A comparison of the last two columns in the truth table shows that they are identical.
~(p>qg=pA~q.
1.11. Biconditional statement or equivalence
The statement “p if an only if q” is called a biconditional statement and is denoted by pq.
The biconditional is also read as
()) qifand only ifp (ii) pimplies q and q impliesp
(iii) _p is necessary and sufficient for q (iv) q is necessary and sufficient for p
(v) piffg (vi) qiffp
Rule. pg
(i) true if both p and q have the same truth value i.e., either both are true or both are false.

Scanned with CamScanner


ll
ci AND PROPOSITIONAL CALCULUS 275

(ii) false if p and q have opposite truth values.


Truth Table

4
am} a);4
na/o4j4);3

nial
soln}
N)o,
12. Prove that (pq) r=p + (qr)
proof.
Truth Table

p q r pq qr (pq)r p(qer)
T T T T T T T
T F T F F F F
F T T F T F F
F F T T F T T
T T F T F F F
T F F F T T T
F T F F F ‘3 T
F F F T T F F
The comparison of the last two columns shows that they are identical.
(po q)er=p>(qer).
L13, Precedence to Logical Operators
We formed formulas are fully paranthesised, so there is no ambiguity in their interpretition. Often,
however, it is more convenient to omit some of the paranthesis for the sake of readability. e.g we would
Prefer
to write
P—+Q A R rather than

(P + ((~ Q)* R)).


The syntax rules given below define what an expression means when some of the paranthesis are
omitted. These conventions are analogous to those of elementary algebra, as well as most programming
knguages, where there is a precedure value that says
a+b X cmeans a+ (6 X c) rather than (a+ 5) X c.
The syntax values for propositional logic are straightforward.
|. The most tightly binding operator is ~. e.g. ~ P A Q means (~ P) A Q. Furthermore, ~ P means ~
(~ P).

Scanned with CamScanner


SPECTRUM DISCRETE M
276 . . ATHy MY
xpressions combinj ‘
Ming and y typ
The second highest precedence is than A operator. In exp

hw

operations comes first. !

eg. PV QARmeansP V (QAR).


are several A operations in a sequence, they are perfo rmed le ft to right,
i
If there
e.g. PAQARA S means (((P A Q) AR) AS) .
associates to the left,
3. The V operator has the next level of precedence and it
4. The > operator has the next lower level of precedence.
e.g. PAQ~>PV Qmeans (P. AQ) > (PV Q):
The - operator associates to the right :
thus P+Q—+R-~>Smeans(P > (Q>(R> S))):
5. The « operator has the lowest level of precedence, and it associates to the right,
EXAMPLES
1. Let p:12+5=17
qg:5+2=7

pwg:l2+5=17 iff 5+2=7


Now p is true and q is true
p qis true.
2. Let p:5=4
q:6=5
pq:5=4 iff 6=5
Now p is false and q is false
pq is true.
3. Let p:Only one even integer is prime
q: All odd integers are divisible by 5
p< q: Only one even integer is prime iff all odd integers are divisible by 5.
Now p is true and q is false.
p@ qis false.
1.14. Laws of the Algebra of Propositions
Here 0 stands for contradiction, 1 for tautology.
Commutative Laws

PpVq?qVp PAq*qAp

Associative Laws
(PV QVrepV(qvr) (pPAQgArepA(gan

Scanned with CamScanner


ma ia
ataciacinemsratemerenti meget
L CALCULUS

nt eee
A nD PROPOSITIONA 277
Distributive Laws

vne(pAgVy (PAN PY GAN (PY DA(PVY)

ead a
ph

PAA
Identity Laws

pvorP _ pdlep
Negation Laws

ph~p?O . PN “pei
Idempotent Laws

pYp?P _ DA pep
— Null Laws
prowl pVlel -
ne Absorbtion Laws
pAPVg?P ; PV (pA g)ep
DeMorgan’s Laws
~(pV a (~p) A (~q) rE ~(PAD*Cp)V &~@q)
Involution Laws -
_~CpP)ep
1.15. Converse, Inverse and Contrapositive
Ifp > q is a direct statement, then
(‘) q->pis called its converse (ii) ~ p > ~q is called its inverse
and (iii) ~ q > ~ pis called its contrapositive.
Note Since P?>q=~q>~p and gq>p=~p+~q
contrapositive = direct statement and converse = inverse.
Note. If the direct statement is true, then its converse
and inverse may or may not be true.
1,16, Duality
We know that dual relationship between ‘line’
and ‘point’ exists through the interchange of the
Meet’ and words
‘join’, For example ;
‘A line is the join of two points’
‘A point is the meet of two lines’
Similarly there exists dual relationship in logic. We first interc
hange A and V.
For example :

~(pA q)=~pV~q
~(pV g)=~pA~q

Scanned with CamScanner


|
278 SPECTRUM DISCRETE MAT HEMATIC

EQUIVALENCE
Let S be a set of propositions and p, q be propositions generated by S. p and q are equivalent if p« 4
is a tautology. The equivalence of p and q is denoted by pq.
IMPLICATION
Let S be a set of propositions and p, q be propositions generated by S. p implies q if p> gis,
tautology. p + q is written to indicate the implication.
1.17. Statement Patterns or Well-formed Formulas
If p, q, r,... are statements, which can be treated as variables, then any statement involving thes
statement and logical connectives A, V, >, «,~ is called a statement pattern or a well formed formula,
Note. Brackets can be removed or introduced in statement patterns, but with care. |
() pPAQ@Ar=pAgar (ii) ~(pV q)#~pVq |
[ILLUSTRATIVE EXAMPLES]
Example 1. For each of the following sentences, state whether it is a statement and indicate its truth value
if it is a statement.
(i) 3X5+4=19, (ij) Moon is not a heavenly body.
(iii) Today is Sunday. (iv) Why are you smoking ?
(v) Do you like reading ? (vi) Logic is a very interesting subject.
(vif) There are only 100 positive integers.
(viii) The sum of three angles of triangle is two right angles.
(ix) May God bless you with success !
Sol. (i) This is a true statement.
(if) This is a false statement.
(ii) This is a statement which is true on Sunday but false on other days.
(iv) This is not a statement as the sentence is declarative.
(v) This is not a statement.
(vi) This is not a statement as it is an open sentence.
(vii) This is a false statement as the number of positive integers is infinite.
(viii) This is a true statement.
(ix) This is not a statement.
Example 2, You are given the following statements :
p:5 xX 7=335.
q: Moon is a heavenly body.
r: Jammu is the most populated city in India.
s : Food is necessary for life. |
State the truth values of the followings : |
() pAgpArpaAsgqgAnrgqhrsrAs. (i) PVGQPVAnpVs,qVrqVs,rVs. |
|

Scanned with CamScanner


Ww

pot AND PROPOSITIONAL CALCULUS 279

| Here Ps qs are true statements and r is false.


¥
() pA q@p A s,q A sare truc and others are false.
(if) All statements are true as there is only one false statement.
ample 3, Let p be the statement “the south-west monsoon is very good this year” and q be the statement
ae rivers are rising”. Give the verbal translation for (i) p V ~q (ii) ~ (~p V ~@q).
wh () The south-west monsoon is very good this year but the rivers are not rising.
(ii) It is not true that the south -west monsoon is not very good or the rivers are not rising.
o ~(~PV~Q=~(~P)A~CQ)=prgq
the above verbal translation can also be written as : The south-west monsoon is very good this
yarand rivers are rising.
gumple 4. Write the following statements in symbolic form and give their negations :
() If you work hard, you will get the first division. (ji) If it rains, he will not go to Kathua.
(iii) If Mahatma Gandhi was a saint then Sardar Patel was as iron man.
Aloe,
wl. (i) Let the symbols for the statements be :
p: you work hard
q: first division
the statements is p > q

Its negation is ~(p> g)=~(~p V q)=~(~p) N~q=pA~gq.


In words : Even if you work hard, you will not get first division.
(ii) Let p: It rains.
q: He will go to Kathua.
symbolic expressionis p> ~q
Itsnegationis ~(p=>~q)=~(p > ~q)=~(~p V ~q)
=~(~p)A~CQ=pAgq.
In words : Even if it rains , he will go to Kathua.
(iit) Let the symbols for the statements be :
p: Mahatma Gandhi was a saint
q: Sardar Patel was an iron man
the statement is p > q
Itsnegationis ~( p> q)=~(~p V q)=~(~p) A (~q)
=pN~q
In words : Even if Mahatma Gandhi was a saint man, Sardar Patel was not an iron man.
Eample 5, Write the following statements in symbolic form :
“You can not ride the roller coaster if you are under 4 feet tall unless you are older than 16.”

Scanned with CamScanner


SPECTRUM DISCRETE MATHEW),
4

€ \2
j
p= “you can ride the roller coaster” |
/

gq = “you are under 4 feet tall”


7 = “you are older than 16.~
(ga-r)>7P
Example 6. Write the following statements in symbolic form
“Every student in this class has studied calculus.”
Sol S(x): xis in this class, C(x) : x studied calculus
V¥ x(S(x) + C(x))
Y x (S(x) A C(x)) means “Every student is in this class and has studied calculus.”

Example 7. Write the following statements in symbolic form


“Some student in this class has studied programming.”
SoL S(x) : x is in this class, P(x) : x studied programming.
3 x(S(x) A P(x))
Example 8. (/) Find the truth values of ~(~ p V q) if p is true and gqis false.
(ii) If p is true and qis false, find the truth values of ~( p A ~ q).

Sol. (1) ~(~p V q)=~(~p) A~q=pA~q


Now p is true and
gq is false
p is true and ~ q is true
p A ~qistrue.
truth values of ~(~p V q) is T.

(ji) ~(pN~q)=~pV~(~q)=~PVq
Nowp is true and q is false
~ pis false and
q is false.
~pV qis false.
Example 9, Determine which of the following statements are true or false :

(i) [(6<8 A (8<6)] +6=8) (i) [REQ) +(QSR)] + Q=R


(iii) [(¥2 is rational ) V (2 is irrational)] -> (1 = 0)
Sol. (i) Let the symbols for the statements be
p:6<8
yi8<6
r:6=8
given statement is (p A q)r

Scanned with CamScanner


er proProstmiONAL CALCULUS 231
iD
ne wt ;
cow PS

: pagqis false
is false.
a ‘7 is false and r is false.
me ag)erris ave.
[(6< 8) A (8 <6)] + 6=8 is true.

“ Let the symbols for the statements be


p:RSQ, q:QCR
r:Q=R
given statement is (p>q)>r
vow pis false, qis true and ris false.
p> qis true.
(p> q > ris false
- [REQ>(QER)] > Q=Ris false.
(iii) Let the symbols for the statements be :
p: ¥2 is rational
q: 2 is irrational
r:1=0
given statement is (p V q)>r
Now pis false, q is false and r is false.
pV qis false
(pV q) > ris true.
as [(v2 is rational) V (2 is irrational) } + (1=0) is true.
[umple 10. Write down the truth table of the following statement :

[p>@vn]
Vv l[po~-)
SL
Truth Table

p>(qVr) | pwn? [p>7 qV


nl] Al[po ~r]
|

l

oo

F
ala[nj3a}/4/4)3lai<
nhalalalaojafala/s

alfalalalpajayayays3

F T
4/4]
anlafpa~aydal/a

F
albania) a,4), 4,7),

F
E
T
T
T
T

Scanned with CamScanner


282 SPECTRUM DISCRETE M ATHEN,
oN
Exam11.
ple Prove that A q) V(~p A~q).
p< q=(p

Sol.
Truth Table

p | a |[~p|~a| pq | ~pA~q | ped | (PADVCPA~q)



T F T T
IT T F F
F F F —
T | F F T F
F F F —
F 7 T F F
T F T T T
F F T

A comparison of the last two columns in the truth table shows that they are identical.

pq=(pAq)V (~pA~q).
Example 12. Prove that (p > r) >(q>s)=(pAq)>(rV 5).
Sol.
Truth Table

p q r s |p>r\|q>s|parq\|rVvs |(p>nV@>s) |PAQ Vs)


T|T|T IT T T + T T ,
T| FIT {T T T F T T T
F|T|T|T T T F T T T
Fi] F|T|tT T T F T T T
T|T {Flt F T T T T TO
Tlrpileldrle T F T T T
Fi] T|FIT T T F T T T
F | FF] FIT {T T F T T T
T|T{|TIF T F T T T T
T | F | TF T T F T T T |
Fi} T]T]F T F F T T T 4
F | F | T]F T T F T T Ti]
T| TIFF F F T F F F
T | F | FF F T F F T Tad
F |} T|] FF T F, F F T T.} J
F | F | F | F T T F F T T 1
A comparison of the last two columns
in the truth table shows that they are identical.
(p>r)>(q>s)=(pAqr(rVs)

Scanned with CamScanner


a

pciC AND PROPOSITIONAL CALCULUS 2


—_— —.
rample 13. Prove that p->(q Ar)=(p>q)A(p=> r).
wl.
Truth Table
p q r qr | p>(qAr) p?q por |(p7qga(ps=n
T T T T T T T £
T | F T F FO F T F ]
F f T T T T T T
F F T F T T x T
T | T F F F T F F i
T F F F F F F F
F T F F T T T T
F F F F ac T T T
A comparison of the fifth and eighth columns shows that they are identical.
P>(QAr)=(p>q)A (pr).
Example 14. Prove by means ofa truth table that p«q=(p—>q)A (q>p).
%l.
Truth Table

P q p->q q>pP (pP>Q) A (q>p) pq


T T T T T T
T F F T F F
F T + F F F
F F T T T T
The like truth values of the last two columns prove the validity of the statement.
Erample 15. Prove that (p A q) > (p A q) isa tautology but ( p V q) > (p A q) is not.
Sol.
Truth Table

Pp q |PVQ pAq (pA q)>(pV a) | (pV q)>(pnq)


T T TT T T T
T F T F T FF
F T T F T° F
F F F F TL F
. From the above table, it is clear that (p A q) > (p V q) is a tautology while (p V q) > (p A q) is

Scanned with CamScanner


y

284 SPECTRUM DISCRETE MATE, ;

re, Sa hehcoi s yyf


Example 16, Prove that {[ (p> 9) ¥ p] Ags > gis a tautology.
Sol.
Truth Table a

n g | PY | (prayVvp Kp>aVplaag tle) V pl Ag} +g

T|T T | T T T
T F F T F T

F T T T T 7
F F T T F T

From the shove table, it is clear thar {[( p> >@) V p] Ag} >@q isa tautology.
Example 17. Prove that if pg and g>rthen pr.
SoL Here we are given that p > g. g > r and we have to prove that p > r. The result will be established 7
we show that

[(p>g)A(g>n] > (p> nisa tautology.


Truth Table

pia|ripralerslpor| earn [pg ag>n]o(p>n


T{TI|T] TT] T T T T
TYFLT | F dT T F T
FiT{/T{] Tt | tT | T T T
FlFl]7T}]31rj]3rttgt T T
rite ia F F F T
TI FU FF T F F T
F/T{F{ T F | F T
FLFU FI T T T T T
j
. fp>gand gorthen p>r [- (peg Algor) > (pr visa tauoke i

Example 18. Wrne cown


{i) Contrapositive of p>~g (ii) Contrapositive of converse of p> ~4
(iif) Inverse of converse of p> 9
SoL (ij Contrapositive
of p> ~ gis
~(-qy>-p=q>~p
(it) Converse of p>~qis-~q->p
“. comrapositiveof ~g>pis
ae

~p>-(~q)
—“~p-@q
(iti) Converse of p> qgis g>p
Inverse of g>pis-gr-p

Scanned with CamScanner


act AND PROPOSITIONAL CALCULUS 285
gumple 19, State the Converse and Contrapositive of the implication “If it snows tonight, then I will stay
ghoame”.
wi Let ps Itsnows tonight

q: Iwill stay at home.


Given statement is p > q
Converse of statement is g > p
ie. “If [stay at home then it Snows tonight”.
Contrapositive of statement is ~qg = ~p
ie “If do not stay at home then It will not Snow tonight”.

EXERCISE 1.1
1. Which of the following is a statement (or proposition) ? Justify your answer :
@) — Listen ta me, Krishna ! (ii) 17 ts a prime number.
(iii) °+5x+6=0. (iv) 6 has three prime factors.
(v) Two non-empty sets have always a non-empty intersection.
(v) The real numberx is less than 1. (vil) Two individuals are always related.
State the truth values of the following :
2

(1) There are only finite number of rational numbers.

(ii) ee iS a rational number.


(uz) There is only one triangle apart from the triangles (congruent to it) with prescribed lengths
for sides a,b, cwitha<d-~ ce.
(iv) The quadratic equation ax” + hx +c=0. a ¥ 0. has always two real roots.
(v) A triangle one of whose vertices lies on a circle and whose side opposite to this vertex is a
diameter of the circle is a right angled triangle.
(vi) There is always a real root for any quadratic equation.
nin
(vii) :
The number of ways of selecting 2 persons in
- . .
two chairs out of n persons is
n
P2
—_
(vii) (@ +6)? =a" +B + 2a.b.
3, (f) Give the truth table for the statement ~ p V g
(ii) Write down the truth table for ~ p A ~q
(ii) Write down the truth table for the statement (~ p V g) A (~p A ~@q)
(7) Give the truth table for the statement (p— q)(~p V y).
(v) Write down the truth table for the statement (p A qg) >~p
(vf) Give the truth table for the statement (p A q)>(p V q).
(vii) Give the truth table for /<> m where 1=(p—>q) A (g>p)and m=p<q.

Scanned with CamScanner


286 SPECTRUM DISCRETE MATHEMATIC,
es

4. (i) If p: lines / and m are perpendicular to each other


q: Aisa point on m,
write down in symbols the statement r= A is a point on the line m which is perpendicular to /.
What is the negation of this statement ?
(if) If p: 1 study
q: | fail,
What is the symbolism for the statement
r: [study or I fail.
What is the negation of this statement ?
(iii) ‘Ram is smart and healthy’
“Ram is neither smart nor healthy’
Are these statements negations of each other ?
(iv) Are the following statements negation of each other 7
‘x is not a rational number’
‘x iS not an irrational number.’
(v) Write down the statement. ‘Two congruent triangles are precisely those which have
corresponding sides equal’ as an equivalence and write its negation also.
(vi) Write down the negation of the statement : ‘All the sides of an equiangular triangle are of te
same length’.
5. (i) If p stands for the statement, ‘I do not like chocolates’ and q for the statement. ‘I like ice-cream,
then what does ~ p A q stand for ?
(ii) If s stands for the statement, ‘I will not go to school’ and f for the statement, ‘I will watcha
movie’, then what does ~s V ¢stand for ?
(iif) If p stands for the statement, ‘I like tennis’ and q stands for the statement, ‘I like football’
then what does ~ p A ~q stand for?
6. Prove that

(i) pAg=qnp (ii) pV q=qVp


(iii) pV(qVr)=(pVqg)Vr (vy) ~(pV~g=~PpAq
(v) ~(pA~qg)=~pVq
7. Write down the truth table for

() (pAqg>p (ii) p+ (p>4)


(iif) (pNq)>(pV4) "es (iv) pA(q>p)
(vy) ~(pNQV~(qep) (vi) (p>qV~(p*9)
(vii) ~(p>q)(pA~q) *' (viii) pV ~q
(ix) pV(gAr)*(pVqg)A(pVr)

Scanned with CamScanner


oy
D PROPOSITIONAL CALCULUS
i oe the following distributive laws - : : 287

PAAVD=AOVEDA
g,

ATP
ci (PVG AT=(DANVGAN (ii) PY Q@Ar=(pVgA(pvn)
. (iv) (PAQVr=(pvHNAQqVN
(i) Show that (~PA~q)>(ps
q) IS a tautology -
(i) Show that (PA Q)
>(pV a)isa tautology
(ii) Show that (p A q)
+(~py q) is a tautolog
y
(v) (PA 9) > (P © @) isa tautology,
() (pA 4) > Pp isa tautology,
(i) (PAG) A(~pV~a)isa
contradiction,
(ii) Show that (pV gq) Vrepy
(9 V ris a tautology.
(vii) Show that p> (pV g)isa
tautology,
(ix) Show that (p V q) A (~p a ~ q) is a contr
adiction.
0, Construct: truth table of ~( p A q) > ~?p V ~q. Is
it Contradiction or Tautology
.
Il. (i) Write down the truth table for
7A m where /=~q>~p man~po
n q.
(ii) Write down the truth table for J <>
yy where 1=~(p V gq), m=~ pA~q.
12. Construct the Truth table for the proposition

(p> 9) >(@>p)
13. Write down the truth table for the followin
g statement patterns :
() (p>q)(~q>~p) @) (p>9)>[G=n+(p=Q)]
14. Prove that:

P>(~qVr)=(parq>r.
I. Simplify

() ~(~p>~q) (ii) ~(~p*q) (iil) ~e~pV~q)


6. Find the converse, inverse and contrapositive of the following statement
:
“If 4x-2=10, then x= 3”.

I, ma Converse and Contrapositive of the implication “if today is Thursday,


to ay”,
then I have a rest
,

(i) Nota statement


l,
(ii) Statement (iii) Not a statement (iv) Statement
(v) Statement (vi) NotaStatement (vii) Statement.
2

OF WG) FP Gi T
:

()F 0) T ODF (i) Tit) T

Scanned with CamScanner


SPECTRUM DISCRETE MATHEY ng

Scanned with CamScanner


S 7_-~
t &
< > =
o _ e & >
tr Pai Se |e + ' =
i > t < >
A, Q, QR, <
Oyu LL ass

=
oO
o
MY ta l > . &
<— | te i be > lm |e |e fe
a y | x ~
— a4 !
a,
o l a
< >
ty | Le an Le > 5 <felafote
fe t x
a
ty | | be Y > rfe fel e fo
G, | ke ble
’ ha |
le le q, a ale le epee
"
fe . = => i ~
~ is a & © =
wy

proPrositiONAL CALCULUS 289

~ ———, ~>g|q>P ! m™ lom

‘Eeprfelet
; F F Tr F
aF T
; T T F F F T
: F T T T iT T
ag; ‘lisnot perpendicular to m or A is not a point on nr

7 “ V q; ‘I do not study and I do not fail’.


ia No (iv) Yes (vy) peq:
are congruent
Two triangles are not congruent and ave corresponding sides equal or two triangles
jhe ® pair of corresponding sides equal.
i) ‘No all the sides of an equiangular triangle are of the same length’.

5 0 | like chocolates and ice-cream. (ii) Either 1 will go to school or I will watch a movie.

(ii) 1 like neither tennis nor football.


. 0
Truth Tables

P q pAq (pA q)>pP


T , ¥ T T

T F F T

F T F F
F F F Z
i) Truth Table

p q p>?q p>(p>4)
T T T T
T F F F
F T T T
F F T :
(i) Truth Tables
; i paq pVvq |(pAgrPva

T T T = ,
T | F F Ag T
F T F T
F F F E :

Scanned with CamScanner


q
299 SPECTRUM DISCRETE May;
(m) Truth Table
p g q>p PA(q->p)
T T T T
T F T T
F T F F
F F T _F
(7) Troth Table
Pp | ¢ pral ep |-(prq) ~(g*p)| ~@AQY~(qep)
TIT] Tr {TT F F F
TjFilrl- T T T
Fi{tTjrlr | T T
Fi[rFi[ri{3r| it F T
(vi) Trath Table
Pp gq [p29 ~(pea)| p>q| (p>9)¥ ~(p*q)
TL, TT] T F | tf TO
Tlefer T F T
Fi T F T T T
F | oF T F T T
(ii) Truth Table
p | ai ~g|prgb (pra) pr~q|~-w>g-A-~9)
TIT| Fi T F F T
TiF{[T] F T T T
PUT] FF]Tt | F-|--B-.] -.--%
Fir|/ Tt} 3tt{r {er T
(viii) Trath Table
P q ~q pVv~q
T T F ir
T F T T
F T FO F
F F T T

Scanned with CamScanner


291

Ce

Scanned with CamScanner


t e> R
> t
aS
IR IR le Ie fm : ; J
STR]

~pN~qlleom
Be : eetle
bw
>SY a,> ®
tleleie 7 >
hh t
< Q
- a, —
1
©
>
& E g,
[uy
l
> lade le tlebe &

~q
2
< ta TL fe fe Se
=
Q, 1! |
>
#|~

&

~P
Truth Table

Sel EIR IE] 2f


©
-
< be TE fe [te RL ly .
tile
Q,
we
&

IPVq|~@Vq)
v-.|
2]
.gy,
: 4
<li Pe Py G
=
be TR TR TR Je (4, &
>
4
“ ole oe
p PROPOSITIONAL CALCULUS

Pf tee | | be bi
es
> fee | ta PR fee PR Lo 3| «.
5

q
: :p pe pe Pe Fe‘“

P
<<
iy
'w
fame | be | fee pte] fete] fete
ole fair
2jae
fe t
fe fe fe |e |
|e e Rifbee be
a

(i)

TR oy a [|e |
Y fe fe Pe Pe
&
ee be
Wa, ens Veo, Vee

uw aN
292 SPECTRUM DISCRETE MATHEMATIcs
i, |

13. (i) p q pq ~9>~P |\(P>9e?(~9>~p)


T T T T T
7. F F F A
F T T F T
F F T T r

Gi) | P| a)? pra ger | @>)-6-9| (prgrlqrn+o-Q)]


T|T|T|T T T T
T|T| FT F T T
T|/F|T]F T F T
T|F]F] F T F T
FIT|T |] T T T T
F]|T|F{T F T T
F)F|T|T T T T
F\/F|F|T T T T
15. () ~pAq (ii) peq (ii)pAq
16. (i) Converse:
If x=3, then 4x-2=10 (ii) Inverse:If 4x—2# 10, then x #3
(iii) Contrapositive : If x # 3, then 4x-—2 # 10
17. Converse : If I have a rest today, then Today is Thursday.
Contrapositive : If I do not have a rest today, then today is not Thursday.

1.18. Arguments
Def. of argument : An argument is a statement which asserts that given set of propositions
P1; P2s P3s-+-s Pn taken together gives another proposition P.
These are expressed as pj, 2, P3y.+++- » Pn/— P. The sign “/-” is spoken at turnstile. The propositions
Pls Pas P3rreecereee , Pn are called “premises” or “assumptions” and P is called the “conclusion”.
Valid argument : An argument pj, P2, P3,....+ P,/-P is true if P is true whenever all the premises
P1s Pay P3y-s++++y Pn are true, otherwise the argument is false. A true argument is called valid argument, ands
false argument is called a fallacy.
Note. If is important to realise that the truth or the conclusion is irrelevant as far as the validity of argument
is concerned. A true conclusion is neither necessary nor sufficient for the validity of argument.
The validity can also be judged by the relationship Pi A pr A Py vee A pa > P provided it is a tautology.

Scanned with CamScanner


Ww
gclC AND PROPOSITIONAL CALCULUS 293
l

\ILLUSTRATIVE EXAMPLES}
gsample 1. Test the validity of : If he works hard then he will be successful. If he is successful then he will
we happy: Therefore, hard work leads to happiness.
al. Let the symbols for the statements be :
phe works hard.
gq: he is successful
r: he is happy.
The argument is :
(p> D)A@>r")7>prr
Truth Table
Tp | a | r | pea | aor] (pega@en| por | (eed ACq> D> (p>
T|T|T T T T T T
T F T F T F T T

F T T T T T T T
Fr F T T T T T T
T T F T F F F T
T F F F T F F T

r | T | F T F F T T
F F F T 7 T T T

From the table, it is clear that [((p — q) A (q >r)] > (p > 7) is tautology.
given argument is valid.
Note. Another method for testing the validity of argument.
In the last three examples, there were only two or there statements and consequently 4 to 8 rows in the
mth table. But if there are four or more statements then the truth table will have 16 or more rows and the
ns chance of the making a mistake will be more. To over come this difficulty ie., to reduce the size of the
uble we have another method, infact above method stated in another way, which follows as :
“Assume that the conclusion is false. Now if p,; A pA ceeeee A pp is a fallacy, then the argument is
ns
valid, otherwise the argument is Invalid.
Example 2. Test the validity of :
“If my brother stands first in the class, I will give him a watch. Either he stood first or | was out of
sation. | did not give my brother a watch this time. Therefore I was out of station.”
Sol. Let the symbols for the statements be :
p: my brother stands first in the class.
q: 1 give him a watch.
r: Twas out of station.
St

Scanned with CamScanner


294 SPECTRUM DISCRETE MATHEMA),
C
The argument is p > q,pV r,~q/-r.
o™
Assume that r is false.
Now there will be only four rows as there are only two variables p,q
Truth Table

Pp q r P>q|pVq| ~4 (p>q)A(pVrnA(~Qq)
T T F T T F F
T F F F T T F
F T F T T F F
F F F T F T F
| Since (pq) A(p Vr) A (~q) isa fallacy.
the argument is valid.
Example 3. Prove the validity of following arguments :
If man is a bachelor, he is unhappy.
If a man is unhappy, he dies young.
Therefore, bachelors die young.
Sol. Let p: man is a bachelor
g : man is unhappy
r:man dies young.
The given statementis [(p>q)A(q>n]>(p>r)
Truth Table
J II

P q r |prq|qrr|(prgAq>r)| por I>ll


T T T T T T T T
T T F T F F F T
T F, tT F T E T T
T F | F F T F F T
“F | tT | tT | tT | T T T T
F T F 7 F F T T
F F Z T T T T T
F F F T T T T T
: Since given statement is a tautology, so argument is valid.
Example 4. Check the validity of argument : . ;
If I work, I cannot study. Either I work or pass mantheniatiey,
] passed mathematics. Therefore, I study.

Scanned with CamScanner


ONAL CALCULUS 295
nD prorostt!

thematics
y: pass m4 °


«ven ven statemen t is

(pr Dhevor I> 4 Truth Table


I II
17 r ~F |P*~Q| PVP | (p>~qAa@vratr)| Iq
= l7 |T F F i F T
ra | T F F F T F T
rr | OF” T T T T T F
rr |sF F T T T F T
> |r| T F T = T T
F T F F T F F T
TF fe | T T _T TO | T F
+ lrlrFft T F | F T
The given statement is not a tautology.
So argument is not valid.

EXERCISE 1.2
|. Test the validity of:
If it rains then crop will be good.
It did not rain, therefore the crop will not be good.
1. Test the validity of:
Unless we control population, all advances resulting from planning will be nullified. But this must
not be allowed to happen. Therefore we must somehow control population.
Are the following arguments valid ? If valid, construct a formal proof ; if not valid, explain why.
(a) If wages increase, then there will be inflation. The cost of living will not increase if there is
No inflation. Wages will increase. Therefore, the cost of living will increase.
| i) If the races are fixed or the casinos are crooked, then the tourist trade will decline. If the
: tourist trade decreases, then the police will be happy. The police force is never happy.
: Therefore, the races are not fixed.

ANSWEKS >
Not vatia 2. Valid 3. (a) Not valid (b) Valid

a
Scanned with CamScanner
296 SPECTRUM DISCRETE MATHEMATIC,
ee,

1.19. Proposition Generated by a Set


Let S be any set of propositions. A proposition generated by S is any valid combination of
propositions in S with conjunction, disjunction and negation.
Note. The conditional and biconditional operators are not included as they can be obtained frop,
conjunction, disjunction and negation.
Equivalence

Let S be a set of propositions and p, q be propositions generated by S. p and q are equivalent if p *¢


is a tautology. The equivalence of p and q is denoted by p © q.
Implication
Let S be a set of propositions and p, q be propositions generated by S. p implies g if p > qisy
tautology. p > q is written to indicate the implication.
1, 20. Laws of Logic
Here 0 stands for contradiction, | for tautology.

Commutative Laws

PVqeqVp PANq@*qAp
Associative Laws
(pVq)VrepVvi(qvr) (pAQgAr@pa(qarr)
Distributive Laws

PAV) (PAQV(PAr, PVGAre(pVQA(pVr)


.
Identity Laws I.
PA0sp pV lop
Negation Laws Pr
pA~ps0 pV~pol

Idempotent Laws

PV pop pA p*p
Null Laws
pA0s0 pvlel

Absorbtion Laws
PA(PVq)ep PV (PA Q)ep
DeMorgan’s Laws

~(p Vg) (~p) A (~q) ~(p\q)e(~p)V


@@)
Involution Laws

~(~p)*p

Scanned with CamScanner


10 NAL CALCULUS
prorostt
wo! can - {mplication and Equivalence
wt Detachment
-

(p>q)Np>q
———

————
Contrapositive

(p7>QA~q>~P
——_— Disjunctive Addition
p> (pV 4)
TO Conjunctive Simplification
(praqg>p and (PAD)4
—_— Disjunctive Simplification

(pV gA~p > 4 and (pV


gq) A~q =P
—_ Chain Rule
(p7Qr@grn>prr
re CONDITIONAL EQUIVALENCES
(p27Deeq>~P) eCPV49
—_——
Biconditional Equivalences

(pege(praA(q> p) e(Pragy (~pA~q))


|
122. Chain Rule
| Prove that (p> q) A (q>r) > (P>”)
‘Proof. Truth Table
(p2g A(q>n | por | (P-DAG PFN
D7 ?z
plq|ritprq|q>r
T T T T T
T| T|T
F T F T 7
T| F | T
T T T T T
F|T|T
T T T T
F| F | T T
F F F T
T| T | F T
F F T
T| F | F F T
T F F T T
F/T | F
T T T T T
BE F
)) isa tautology
> (p>
Since (p-> g) A (q>r
(P>g) A (q>rn>(p>n

Scanned with CamScanner


298 SPECTRUM DISCRETE MATHEM,y,
1.23. Detachment Law oS
Prove that (p>q) A p> q
Proof. Truth Table
q P q (p>q)Ap (p>qAp>q

aAialjoala)s

aja) m/ ole
T

sioaia/4
F

a] om] om
T
F
Since (p > q) A p> qisa tautology
All oo"

(p>qgAp>q —
1.24. Mathematical System
A mathematical system consists of
1. A set or universe, U.

2. Definitions: sentences that explain the meaning of concepts that relate to the universe. Any term
used in describing the universe itself is said to be undefined. All definitions are given in terms of these
undefined concepts of objects.
3. Axions: assertions about the properties of the universe and rules for creating and justifying more
assertions. These rules always include the system of logic that we have developed to this point.
4. Theorems — the additional assertions mentioned above.
Example 1. In Euclidean geometry the universe consists of points and lines (two undefined terms). Among
the definitions is a definition of parallel lines and among the axioms is the axiom that two distinct paralle
lines never meet.
Example 2. In Propositional calculus, the universe consists of propositions. The axioms are the truth tables
for the logical operators and the key definitions are those of implication and equivalence.
Theorem : A true proposition derived from axioms of mathematical system is called a theorem.
All theorems can be expressed in terms of a finite number of propositions Pi, Pr»

P3 A ssaSveie' AP, > C.

Proof : A proof of a theorem is a finite sequence of logically valid steps that demonstrate that the premises
ofa theorem imply the conclusion.
There are two important types of proofs namely direct and indirect.
Direct Proof: It is a proof in which the truth of the premises of a theorem are shown to directly imply the
truth of the theorem’s conclusion.
Rules for Direct Proof
Maen ft
te

1. [t must terminate in a finite number of steps.


2. Each step must be either a premise or a proposition that is implied from previous steps using 4")
valid equivalence or implication.
3. The last step must be the conclusion of the theorem.

Scanned with CamScanner


yc {C AND PROPOSITIONAL CALCULUS

irect Poof
299

Negate the conclusion of the theorem and add this negation to the premises. If this set of propositions
plies 8 contradiction, then the proof is complete.
gales for Indirect Proof
1. The first step is the negated conclusions.
1, The last step must be a contradiction

[ILLUSTRATIVE EXAMPLES]
sample 1. Prove that the following are equivalences :
(i) PVG *qVPp (ii) p>q *~q->p (ij) (pPAQVCPAQ)?q
ol. (i)
Truth Table
P (pV q)>(q Vp)
T T
T T
F T
F F T
Since (p V q) > (q V p) isa tautology
pV qand q V pare equivalent
(pV q)#(q V p).
(ii)
Truth Table
q ~P ~q (p > q) >(~q>-~p)
T F F T T T
5 F T F F T
T T F T T T
F F T T T T T
Since ( p > q) > (~ q > ~p) is a tautology
( p > q) and (~q > ~ p) are equivalent
(p> 9)? (~q>~p)
(iti)
Truth Table

P q qqAgVCPAQq) (pPAQV(~pAq>q
T
T
T
1

Scanned with CamScanner


aaa . SPECTRUM DISCRETy
=
300 MATHEs4
i
tantology
Since(pAQ)A(~pA q)>qisa
valent
(pAgQv(~prD and gq are equi

(pAQV(~PAQD?ed
Example 2. Give direct proof of
~pVqsVpP~-q> 5
Proposition Justification
Sol. Step
(1) ~pVq Premise
(2) ~q Premise

(3) ~p Disjunctive simplification (1), (2).

(4) sVp Premise


(5) Ss Disjunctive simplification (3), (4). #
Note. Conditional Conclusion
The conclusion of a theorem is often a conditional proposition. The condition of the concly Si
sis the consequence of the ¢ ON can
included as a premise in the proof of the theorem. Then we are to prove
a Onclusion
Example 3. Give indirect proof of
p7rgqg>s,pVvVqrsVr

Sol.
Step Proposition Justification
qd) ~(s Vr) Negated conclusion

(2) ~sN ~r De Morgan’s Law, (1)

(3) ~Ss Conjunctive simplification, (2)

(4) q>s Premise


(5) ~q Contrapositive (3), (4)
(6) ~r Conjunctive simplification, (2)

(7) por Premise


(8) ~p Contrapositive, (6), (7)
(9) (~p) A (~q) Conductive, (5), (8)
(10) ~(pVvq) De Morgan’s Law, (9)
ql) PVq Premise
(12) 0 (10), (11) 4

EXERCISE 1.3
1. Show that
(i) pA q logically implies p + q
(ii) p~q does not logically implies p > g

Scanned with CamScanner


oi AND PROPOSITIONAL CALCULUS 301

, Give direct proofofthe theorem p-+r,qg>s,pVq>sVr.


i Give direct proofof p-(q->s),~rVp, qzer-s.

4 Give indirect proofof a+bh,.~(bhVc)> ~a


s Give direct and indirect proofof p->q,q~>r,~(pArn.pVr2=r
5, Give direct and indirect proof of
(p>q A(r>s),(qr>n A (s>u),~(ta uj)prr>-~p

133
38, Proposition over a Universe
Let U be anon-empty set. A proposition over U is a sentence that contains a variable that can take on
gy value in U and which has a definite truth value as a result of any such substitution.
Examples : Consider

(a 1x -6x=0 > x(7x-—6)=0

be > x =0,
7
6

On.

4 $ . 7
If we take Q as universe, then truth set (i.e., solution set) of 7 x7 —6 x =O is {02} .

Ifwe take Z as universe, then truth set of 7x*—6x =O is {0}.


Ifwe take N as universe, then truth set of 7x°-6x=0 is¢.
(ij) 2 =5
Ifwe take Q as universe, then truth set of =7=5 is @.
2 =5 is a contradiction over the rationals.
(iif) (x +3) (x-3)=x°-9
If we take Q as universe, then truth set is Q as (x + 3) (x — 3) = x* —9 is true for all rational numbers
(x + 3) (x —3) =x* —9 is a tautology over the rationals.
Truth Set If p (72) is a proposition over U, then the truth set of p (1) is
T p(n)= {a © U/p (a) is true}
Example Consider the set {1, 2, 3, 4}

Its power set is {@, {1}, {2}. {3}, {4}, (1, 2}, C133, (1, 4, {2, 3}, {2, 4}, (3, 43, {1, 2,3}, 1, 2, 4},
{13,4}, {2, 3.43, {1,2,3, 43}
Let proposition be {1,2} NA=%
truth set of proposition taken over the power se of {1, 2, 3, 4} is

{, {3}, {4}, (3,


4} }.
Tautology and contradiction : A proposition over U is a tautology if its truth set is U. It is a
Contradiction if its truth set is empty.

es
Scanned with CamScanner
302 SPECTRUM DISCRETE MATHEMATi¢

Equivalence: Two propositions are equivalent if p q is a tautology. In other words p and q te
equivalent if T, = T,-
Example : x + 7 = 12 and x=5 are equivalent propositions over the integers.
Implication : Ifp and g are propositions over U, then p implies q if p > q is a tautology. In Othe
words p> gwhen T,C Tp
Example : Over the natural numbers,
n<=3 > n8as {0,1,2,3} € {0, 1, 2, 3,4,
5, 6, 7, 8}
Troth Set of compound Propositions
The truth sets of compound propositions can be expressed in terms of the truth sets of simple
propositions. The following list gives the connection between compound and simple truth sets :
1. Tpagm Ip NT, 2. Tpvqg=IpUTy, 3. T.p= Tp”

4. Tpeg=O,NTJUCSAT,) 5 Tp.g=T,fUTy.

[ILLUSTRATIVE EXAMPLES]
Example 1. IfU = P{1,2, 3, 4}, what are the truth sets of the following propositions 7
(if) AN {2,44=¢ (ii) 3E€Aand
| EA (ii) AULIJ=A
(fv) Aisa proper subset of {2, 3, 4} (v) #AS=#A
Sol. (i) Truth set is {9, {1},£3}, f1, a
(ii) Truth set is {{3), {3,2}, £3, 43, {2,3,4})
(iii) Truth seris {41}, f1, 2, {1, 3}, £1,4}, {1, 2, 3}, {1,2,4}, (1,3, 4}, (1,2, 3,43}
(iv) Truth set is {12}, {3}, £44, {2,3}, {2,4}, (3,4}}
(v) Truthsetis {A CU,#A=2;
Example 2. Given the propositions over the natural numbers
pin<4
g:2n>17
and r:nisa divisor of 18
What
are the truth sets of
(a) g (6) png (c) r (4) q>r
Sol. We have
Tp {1,2,3},
T, #0, 10, 1, 12, ...... }, T, = 1,2, 3, 6,9, 18}
(a) T,= 19,10, 1, 12,0...)
(by Typ g7 Ty fiT, = 11,2,34 9 19, 10, HH, 12, ...) =
(c) T,~11,2,3,6,9, 18)
(d) T,.,*To UT,= {1,2,3,4, 5,6, 7, 8) U {1, 2, 3, 6, 9, 18)= {1, 2, 3, 4, 5, 6, 7, 8, 9, 18}
aT

Scanned with CamScanner


ic AND PROPOSITIONAL CALCULUS 303
1 ople 3, Let the universe be Z, the set of integers. Which of the following propositions are equivalent
* 9
gL:
. @ o<rs4 (b) 0<r <8 (c) O<n<2
(6) Truthsetis {1,2} (c) Truthsetis {1,2}
ot (2) Truth set is {-2,-—1,1,2}
sets are equal
=)" We know that two propositions are equivalent if their truth
(b) and (c) are equivalent.

EXERCISE 1.4
1, Over the universe of positive integers
Ne p(n):
4° -3n=0
q(n): nisa perfect
square and n < 90
r(n): nisa divisor of 36
What are the truth sets of these propositions ?
Over the universe of positive integers :
y

p(n): nis prime and n< 32


q(n): nis a power of3
r(n): nis a divisor of 27
(a) What are the truth sets of these propositions ?
(b) Which of the three propositions implies one of the others ?
3. If U = {0, 1, 2}, how many propositions over U could you list without listing two that are
equivalent?
4. (a) Determine the truth sets of the following propositions over the positive integers :
p(n): nis a perfect square and n< 100
q(n):n=# P (A) for some set A.
(6) Determine T, A q forp and q above.

ANSWERS
1. {1,4,9, 16, 25, 26, 49, 64, 81}, (1,2, 3, 4. 6,9, 12, 18, 36}
2 (a) Tp= {2,3, 5,7, 11, 13, 17, 19, 23, 29, 31}
T, = (1, 3, 6,9, 12, 15, 18, 21,......}
T,= {1, 3,9, 27} (6) r impliesq 3. 256
4, T= {1,4,9, 16, 25, 36, 49, 64,81}; T, = (1,2, 4,8, 16, 32,...), (1,4, 16, 64}

1.26. Predicates
The predicate is the part of a sentence that gives information about the subject. For example, in the
sentence “Ramesh is a resident of Amritsar”, the word Ramesh is the subject and the phrase “is a resident of
Amritsar” is the predicate. So, predicate is the part of the sentence from which the subject has been
Rmoved,

Scanned with CamScanner


sat

304
SPECTRUM DISCRETE
. Mar Hey
In logic, predicates can be obtained by removing any nouns from a statement. For example . ANicg
for “is a resident of Amritsar” and Q stands for “is a resident of”, then both P and Qar Ple, if p
The sentences “x is a resident of Amritsar” and “x is a resident of y” are denoted as © Predi cate ..8 and,
respectively, where x and y are predicate variables that take values in appropriate sets. P(x) ang Qnes
y),
The statement “x is greater than 5” has two parts. The first part, the variable
x, is the Sub;
Statement. The second part—the predicate, “is greater than 5”—refers to a Property
that the sie OF the
Statement can have. We can denote the statement “x is greater than 5” by P(x), wh Jeet of th
er ep den, e
predicate “‘is greater than 5” and x is the variable. The statement P(x) is also Said to
be
propositional function P at x. Once a value has been assigned to the ec of ‘
variable x, the Statement
Proposition and has a truth value.
P (x) become,
4

Predicate. A predicate is a sentence that contains a finite number


of variables and bee
Statement when specific values are substituted for the variables. Omes g
The domain of a predicate variable is the
set of all values which may be substituted
variables. The predicates are also known as in pla
“propositional function or open Sentences”, “© Of the
Truth Set : Let P(x) be a predicate and x has domain D. Then the
set {x ED-p
(x) is true}
called the truth set of P(x). is
ne

Example : Let P(x) be “x is an integer


less than 6” and suppose the domain Of x
is the Set of
positive integers. Then the truth set of P (x) is {1, 2,
8 .

3,4, 5}, AY
.
0

Let P (x) and Q (x) be predicates with common domain D of x. The notation
P (x) =>
element in the truth set of P(x) is in the truth set of Q (x). Q (x) means that every
Also, P(x) <> Q(x) means that P (x) and Q (x) have Identical truth sets.
For example, let the domain ofx be the set of positive integers and let
P(x) : “x is an integer less than 6”,
Q (x) : “x is a factor of 4”.
Truth set ofP (x) is {1, 2,3, 4, 5, 6, 7}
and Truth set of Q (x) is {1, 2, 4}.
Now every element in the truth set of Q (x) is in the truth set of P(x), So Q (x) > P(x).
A statement involving the » variables X11 Xy, eX, can be denoted by P (x), x,,) A statementof
the form P (x,, Xy;.00%,,) is the value of the propositional function P at the n-tuple (x), Xyyou ty
and P is also called an —_n-place predicate ora n-ary predicate.
The symbolic analysis of predicates and quantified statements is called the predicate calculus
whereas the symbolic analysis of ordinary compound statements is called the statement calculus
propositional calculus.
Example. Let P (x) denote the statement “x > 3”, What are the truth values of P (4) and P (2)?
Sol. Here P (x) denotes the statement “x > 3”, wich is
For obtaining the statement P(4), we replace x by 4 in the statement “x > 3”. Therefore P (4), W
statement “4 > 3”, is true. Similarly P(2), which is the statement “2 > 3”, is false.

Scanned with CamScanner


—_
0 prorOstTiONAL, CALCULUS
“AN
je det Oh y) denote the statement y= 45", What sre the ruth values of the propositions
( (1, 2) anid
fi

it eX) denotes the utatement "= y 4.4"


le r”
of obta fly x by J andy by 2. in the statement % = ys O", Werefore
ining the statement € (1,2), we fyreplace
nl | 2), whic trig the statement “124-3 c Shtlarly Q (4,0), whieh ts the statemen
t “3 = 04 3", by
i slat (x,y, 2) denote the stateme net yo 2, What
are the truth values Of the propositions
4

mp a
¢ yy
@ (1,2, 3)
wn)!
| lee R (x,y, 2) denotes the statement “4 y= 2"
ror obtaining, the statement R (1, 2,3), we replace x by ly by2 and z by 3 in the statement
yt R (1 2, 3), which is the statement “| 42 = 3”, 14 true, § “s 4-y = 2”,
imilarly (0, 0, | ), Which is the statement
+021", 1s false,
127, Quantifiers
If p(n) is a propositions over U with Ty) % a, then we way “Shere exists ann in U such that n(n) is
mt” We abbreviate this sentence as (An), (p(n), 3 is known ay exintential quantifier,
Itis clear that if p (n) is a propositions over a universe U, its truth set Tpiy)
19 a subset of U,
framples
(1) (A4);,(5 4 = 100) means that there is an integer
k such that 100 is a multiple of 5. This is true,
(2) (A x)p (x - 3 = 0) means that there is a rational numbe
r * such that x” = 3, This is fal we a3 the
glution set of the equation x? ~3 = 0 over () is empty, We write it as (Ax), (x? 3
x= ())
If p (n) is a propositions over U , with Tp on = UL Them
we say “for all in U, p (n) is true.” We
ibbreviate this as (Wn), , ( p (n)). V is known as universal quanti
fier,
Jk: P(x) means, “There exists an x such that P(x) holds.

| Ve: P(x) means, “For all x, it is the case that P(x) holds.

So for example, if x denotes a real number, then

dx: x* =9 jg true, since 3 is an x for which x2 = 9,


’ 2 ~~ ‘ e ’ . .

On the other hand, Wx: x2 =9 ig clearly false; not all numbers,


when squared, are equal to 3,
rol one
Ver x? +1 > 0 is true, but Wx: x* > 2 is, false, since
'
for examplex = | does not satisfy the predicate,
On the other hand, Sx: x
* 3 2is true, since x = 2 is an example that satisfies it.
Nepati .
“bation of Quantified Proposition
When we negate a quantified proposition, then
Gy the universal and existential quantifiers become
™Plement of one anot her. In simple words negation of an existentially quantifi
hiversaly ¥ quantified ed proposition is a
proposition and negation of a universally quantified proposition is an existentially
(Mantifje
4 proposition, In symbols,
“Wau (p(n) # (Amy (~p (n)) and ~ (A ny (p(n) © (W ayy (~p (nj)

te
Scanned with CamScanner
> 306 |
eee SEEcTRUM Discrer,
Wy,
he scope of the other. \
Nested Quantifiers in t
if one is with qumber larger than it.”

lel
ed
Twé ‘quantifiers are nest 1si 4 re al
ev er y re al number, there
e.g. “F or
5 Wx dy: y7 >
This can be written a
Example Vex Ay (x +y = 9) (x,y): x+y= 0.
Q(x ), whe re Q(x ) is ay P (x, y), where P
Example : vx ddition
ative law for a
Example : Wx Vy @ +y=yt x) Commut ty
proper
Additive inverse
Example : Wx dy(x+y=0 ) on
= @ +) 42) Associative law for additi
Example : Vx Wy Vz (x + (9 +2)
variables js real —
ry <0)) where the domain of both
>
Example : Vx Wy ((x > 0) A (y <0)
if x is positive and y js negatiy e, then
r eve ry real num ber x and every rea | number y,
“Fo Xp:

negative.”
num ber and a positive real e snumber
is always negative»
“The product of a negative real .
.
Example: “The sum of two positive
integers 1S always positive.
ve.”
are both pos itive then their sum is positi
“For every two integers, if they . re ”
y,x + is positive.
“For all positive integers x and
Wx Wy (x > 0) A (p> 0) > + y> 9), where domain is all integers.
positive integers.
Wx Vy (x + y> 0), where domain is all
Order of Quantifiers
tifiers are universal or all quantifiers»
The order of the quantifiers is important, unless all quan
existential.
Example : Vx Vy P(x, y) = Vy Vx PG, y)
Example : ax 3 y P(x, y) = Ay ax P@, y)
Example : Vx 5 y P(x, y) is NOT equivalent to A yVx Pi, y)
ists of all real numbers. Find thet So
Example : P(x, y) : x + y= 0, where the domain of both x and y cons
values of Wx dy P(x, y) and Jy Vx P(@, y).
Fy Vx P(x, y) denotes
“There is areal number y such that for every real number x, x + y = 0”
This statement is False.
Vx dy P(x, y) denotes
“For every real number x, there is a real number y such that x + y = 0.”
This statement is true. shy

Binding Variables : When a quantifier is used on the variable x, Then this oc currence of theral
is bound, An occurrence of a variable that is not bound by a quantifier or set equal toa particular
said to be free. All the variables that occur in a propositional function must be b ound or set e@
particular value to turn it into a proposition.

Scanned with CamScanner


poGic AND PROPOSITIONAL CALCULUS 307
—_—_
example. In the statement 3 x (x + y = 1), the variable x is bound by the existential quantification 3 x, but
the variable y is free because it is not bound by a quantifier and no value is assigned to this variable. So the
satement dx (x +y= 1), x is bound, but » is free.

ILLUSTRATIVE EXAMPLES]
pxample 1. Translate into your own words and indicate whether it is true or false that (G w)z (4 uw—9= 0).

sol. Consider 4 u2-—-9=0

9 3 cs, . .
4W=9 > w= => u= + —, whichare not integers.
10

the equation 4 u° —9=0 has a solution in integers is false.


~d

Example 2. Use quantifier to say that V3 is not a rational number.

Sol. ~ (A x)g (x7 = 3).


Example 3. Over the universe of Books, define the propositions B (x) : x has a blue cover, M (x): xisa
mathematics book, U(x) : x is published in the United States and R(x, y) : The bibliography of x includes y.
Translate into words.

() (Ax) (M@) A ~BQ&)), (6) (Wx) (MQ) A U(@)>BQ@)


© GUxCB@)), (4) Gy) (YX) (M&) >RG,y))
Express using quantifiers :
(e) Every book with a blue cover is a mathematics book.
(f) There are mathematics books that are published outside the United States
(g) Not all books have bibliographies.
Sol. We have
B(x) : xhasablue cover
M(x) : x isa mathematics book
U(x) _: xis published in the United States
R (x,y) : The bibliography ofx includes ».
(a) There exists a mathematics book which has not a blue cover.

(6). Every mathematics book that is published in the United States has a blue cover.

(c) There exists a book whose cover is not blue.


‘ (d) There exist a book that appears in the bibliograph of every mathematics book.
(e—) Wx) B@>M() 1) GX (M@A~U@) &) GX WHER)

Scanned with CamScanner


308 SPECTRUM DISCRETE MATHEMAty,
—S
Example 4. Let M (x) be “x is a mammal.” Let A (x) be “x is an animal” and let W (x) be, “x is warm bloodeq»
(a) Translate into formula : Every mammal is warm blooded.

(4) Translate into English (5 x) (A (x) A(~ M (x)).


Sol. We have

M(x): xisamammal

A(x) : xis an animal


W (x): xis warm blooded

(a2) (Wx)(M@) A W@))


(6) There is an animal which is not mammal.

EXERCISE 1.5
Translate in your own words and indicate whether it is true or false that :
(A x)g (3 x°-12 = 0)
Use quantifier to say that V5 isnot a rational number.

Use universal quantifiers to state that the sum of two rational numbers is rational
Use universal quantifiers to state that the sum of any two real numbers is real.
Over the universe of real numbers, use quantifiers to say that the equation a+2x=b,hasq
solution for all values of a and 8.

Let C (x) be “x is cold blooded.” Let F (x) be “‘x is a fish and let S (x) be “‘x lives in the sea.”
(a) Translate into a formula : Every fish is cold blooded.
(6) Translate into English: (4 x) (S (x) A ~ F (x)) and (V x) (F (x) > S (x)).

ANSWERS
1. False 2. ~ (Ax (x = 5) 3. (WV a)g (V })g (a+ bis a rational number)
4. (Va)yp (V d)p (a+ bis a real number) 5. (Wa)
(V b)p(Ax)p (€+2x=5)
6. (a) (Wx)(F (x)A C@))
(6) There exist animals that live in the sea that are not fish.

Every fish lives in the sea.

1.28. The Theory of Inference for Statement Calculus


We study logic to give us rules of inference, or principles of reasoning. The theory associated wi
such rules in known as inference theory as it is concerned with the inferring of a conclusion from certa!
premises. When a conclusion is derived from a set of premises by using the accepted rules of reasonit!
then such a process of derivation is called a deduction or a formal proof. In a formal proof.

Scanned with CamScanner


Ww

po rcic AND PROPOSITIONAL CALCULUS 309


The rules of inference are criteria for determining the validity of an argument. These rules are stated
ig

*
wens of the forms of the statements (premises and conclusions) involved rather than in terms of the
=
_-3| statements or their truth values. Therefore, the rules will be given in terms of statement formulas
sa than in terms of any specific statements.
— .

In any argument, a conclusion is admitted to be true provided that the premises i.e. assumptions,
som, hypotheses are accepted as true and the reasoning used in deriving the conclusion from the
semises follows certain accepted rules of logical inference. Such an argument is called sound. In any
cgoment, we are always concerned with its soundness. In logic, we concentrate our attention on the study
wf the rules of inference by which conclusions are derived from premises. Any conclusion which is arrived
gty following these rules is called a valid conclusion, and the argument is called a valid argument. The
qual truth values of the premises do not play any part in the determination of the validity of the argument.
jn short, in logic we are concerned with the validity but not necessarily with the soundness of the argument.
VALIDITY USING TRUTH TABLES

Let A and B be two statement formulas. We say that “B logically follows from A” or “B is a valid
zonclusion (consequence) of the premise A” iffA — B is a tautology, that is, A > B.
Similarly a set of premises {H,,H,,.......H,} a conclusion C follows logically iff
HAH, A... AH, >C.

RULES OF INFERENCE

84 Here are two rules of inference which are called rules P and T.
Rule P: A premise may be introduced at any point in the derivation.
Rule T: A formula S may be introduced in a derivation if S is tautologically implied by any one or
more of the preceding formulas in the derivation.

Example. Demonstrate that R is a valid inference from the premises P > Q, Q > Rand P.

Sol.

{1} dd) P+Q Rule P


{2} (2) P Rule P

{1,2} (3) QQ Rule T, (1), (2)

{4} (4) Q-R Rule P


{12,44 (5) R | Rule T, (3), (4)
The second column of numbers designates the formula as well as the line of derivation in which it
.,4 curs. The set of numbers in braces (the first column) for each line shows the premises on which the
wy formula in the line depends. On the right, P or T represents the rule of inference, followed by a comment
ra showing from which formulas and tautology that particular formula has been obtained For example, if we
m=} follow this notation, the third line shows that the formula in this line is numbered (3) and has been obtained

Scanned with CamScanner


310 SPECTRUM DISCRETE a

from premises in (1) and (2). The comment on the right says that the formula Q has been introduced usj
rule T and also indicates the details of the application of rule T.
Table of Implications
PAQ=P PAQ=Q
P>PVQ Q=PVvQ
]P=P+Q Q>P+Q
l, 1@+Q=P 1 P+ Q)=1Q
P,Q=PAQ 1P,PVQ=Q
P,P>Q2Q 1QP>Q=]P
P>+Q,Q>R>P>R PVQ,P>R,Q7?>R>R

Table of Equivalences
|] PoP E, PAQSQAP

PVQSQVP (PAQVARSPA(QAR)

(PV Q) VREPV(QVR) PA(QVR)@(PAQ)V (PAR)


PV(QAR)@(PVQ)APVR) 1TPAQ>]PV1Q
TP VQeaTPAT1Q PVPSP

PAPSP RV(PA|P)@R

RA(PV |P)oR RV(PV |P)@T

RA(PA |P)F P+Qa|PVQ

](P+Q)ePA1Q P+Q<]Q~+]P

P+(Q+R)@>(PAQ)>R (PQ) P1Q

Peo Qeo(P+Q)A (QP) (P <> Q) <> (P AQ)V (JPA 1Q)

Scanned with CamScanner


» <a

aciC AND PROPOSITIONAL CALCULUS 3it


l
Ihe 129. Theory of Inference for the Predicate Calculus
The method of derivation involving predicate formulas follows the rules of inference given for the
ement calculus and also certain additional rules which are required to dea! with the formulas involving
tifiers. The rule P, regarding the introduction of a premise at any stage of derivation and T, the
‘| duction of any formula which follows logically from the formulas already introduced, remain the
J xe Ifthe conclusion is given in the form of a conditional, we use the rule of conditional proof called CP.
gmetimes, We use the indirect method of proof in introducing the negation of the conclusion as an
wuitional premise in order to arrive at a contradiction.
In order to use the equivalences and implications, we need some rules on how to eliminate quantifiers
gring the course of derivation. This elimination is done by rules of specification called rules US and ES.
once the quantifiers are eliminated, the derivation proceeds as in the case of the statement calculus. and the
qaclusion is reached. It may happen that the desired conclusion is quantified. In this case, we need rules of
| generalization called rules UG and EG, which can be used to attach a quantifier.
The rules of generalization and specification follow. Here A (x) is used to denote a formula with a free
xcurence of x. A (7) denotes a formula obtained by the substitution of » for x in A (x). For such a
sbstitution A (x) must be free for y.

Rule US (Universal Specification) From (x) A (x) one can conclude A (¥).

Rule ES (Existential Specification) From (3 x) A (x) one can conclude A (¥) provided that vis not
fe in any given premise and also not free in any prior step of the derivation. These requirements can easily
temet by choosing a new variable each time ES is used.
Rule EG (Existential Generalization) From A (x) one can conclude (4 5) A (y).
Rule UG (Universal Generalization) From A (x) one can conclude (¥) A (y) provided that x is not free
|nany of the given premises and provided that if x is free in a prior step which resulted from use of ES.
fien no variables introduced by that use of ES appear free in A (x).
An invalid conclusion can be arrived at if the second restriction on rule UG was nat imposed. We
ilustrate this by as example.

Let D (u, v) : u is divisible by v. Assume that the universe of discourse is {$, 7, 10, 11}, so that the
satement (A u) D (u, 5) is true because both D (5, 5) and D (10, 5) are true. On the other hand, (») D (y. 5)
$false because D (7, 5) and D (11, 5) are false. Consider now the following derivation.
{1} (1) (A u)
D (uy, 5) P
{1} (2) D (x, 5) ES, (1)
{1} (3) (vy) D @. 5) UG, (2) (neglecting second restriction)
In step 3 we have obtained from D (x, 5) the conclusion (vy) D (v. 5). Obviously x is not free in the
Remise, and so the first restriction is satisfied. But x is free in step 2 which resulted by use of ES, and that x
‘ss been introduced by use of ES and appears free in D (x, 5); hence it cannot be generalized. This is the reason
“iy we obtained a false conclusion from a true premise.

Scanned with CamScanner


312 SPECTRUM DISCRETE MATHEMATIQ
as,

Following examples explain the method of derivation.

[ILLUSTRATIVE EXAMPLES]
Example 1. Show that (x) (H (x) > M (x)) A H (s) > M (s).

Sol.

{I} (1) (x) (H @) > M (x) P


{1} (2) H (s) > M(s) US, (1)
{3} G) H(s) P
{1,3} (4) M (s) T, (2), (3), 1,
Note : In step 2 first we remove the universal quantifier.
Example 2. Show that (4 x) M (x) follows logically from the premises

(x) (H (*)>M(x)) and (Qx)H(@)


Sol.

yo) (Ax) H(@) P


{1} 2) H (y) ES, (1)
3} (3) (x) (H)(%)>M(x)) P
3} 4) Hy) > MQ) US, (3)
(1,3) (5) = MQ) T, (2), (4) 1,
{1,3} (6) (4x) M (x) EG, (5)
Note : In step 2 the variabley is introduced by ES. So a conclusion such as (x) M (x) could not follow fre
step 5 as it would violate the rules given for UG.

EXERCISE 1.6
1. Show that: (x) (P (x) > Q (x)) A (x) (Q(x) > R(x) > (P(*) > R ()).
2. Prove that : (3 x) (P(x) A Q(x)) = (Ax) P(x) A (A x) Q (a).

1.30. Introduction to Proofs

Now we introduce the notion ofa proof and describe methods for constructing proofs. A proof ®
valid argument that establishes the truth of a mathematical statement. A proof can use the hypotheses
the theorem, if any, axioms assumed to be true, and previously proven theorems. Using these points
tules of inference, the final step of the proof establishes the truth of the statement being proved.

Scanned with CamScanner


y

joi AND PROPOSITIONAL CALCULUS 313


cote Terminology
; 1. Theorem: A theorem is a statement that can be shown to be true.
2, Proof: A proof is a valid argument that establishes the truth of a mathematical statement.

3. Conjecture : A conjecture is a statement that is being proposed to be a true statement but that
ys not been proved. When a proof of a conjecture is found, the conjecture becomes a theorem. Many times
sonjectures are shown to be false, so they are not theorems.

4. Axiom (Postulate) : A statement that is assumed to be true and can be used as a basis for
goving theorems.
s, Lemma: A theorem used to prove other theorems.
6. Corollary : A corollary is a theorem that can be established directly from a theorem that has
heen proved.
steps for Proofs :
The first step of the proof usually involves selecting a general element of the domain. Subsequent
geps show that this element has the property in question. Finally, universal generalization implies that the
teorem holds for all members of the domain.
131. Methods of Proving Theorems
We will discuss the following methods of proofs.
Direct Proofs
A direct proof of a conditional statement p — q is constructed when the first step is the assumption
fut is p is true ; subsequent steps are constructed using rules of inference, with the final step showing that g
must also be true. A direct proof shows that a conditional statement p — q is true by showing that if p is
tue, then g must also be true and so that the combination p true and q false never occurs. In a direct proof,
we assume that p is true and use axioms, definitions, and previously proven theorems, together, with rules
ofinference, to show that q must also be true.

Example. Give a direct proof of the theorem “If 7 is an odd integer, then n? is odd”.

Sol. Note that this theorem state V 7 (P (71) > Q (n)), where P (n) is “n is an odd integer” and Q (n) is “nis
odd”. As we have said, we will follow the usual convention in mathematical proofs by showing that P (ni)
inplies Q (n), and not explicitly using universal instantiation. To begin a direct proof of Extra this theorem,
¥e assume that the hypothesis of this conditional statement is true, namely, we
Assume that n is odd. Son = 2 k + 1, where k is some integer.

n* = (2k4+1)? = 4k2 44k41=2(2k2 42441 = n? isodd


-. ifn is an odd integer, then n“ 2 is an odd integer.
Proof by Contraposition
Direct proofs lead from the hypothesis of a theorem to the conclusion. They begin with the premises,
‘ntinue with a sequence of deductions, and end with the conclusion. Proofs of theorems of the type that
me not direct proofs, that is, they do not start with the hypothesis and end with the conclusion, are called
“direct proofs.

Scanned with CamScanner


314
]
SPECTRUM DISCRETE MATHEMATIg

ss
—— 8
An extremely useful type of indirect proof is known as proof by contraposition. Proof,

—_—
contraposition make use of the fact that the conditional statement p > q is equivalent to its contrapositj
~ q > ~ p. This means that the conditional statement p - q can be proved by showing that ity
contrapositive, ~ q > ~ p, is true. In a proof by contraposition of p > q, we take ~ qg as a hypothesis, and
using axioms, definition, and previously proven theorems, together with rules of inference, we show thay
~ p must follow,
Example. Prove that ifm is an integer and 3 n+ 2 is odd, then 7 is odd.
Sol. Assume that “If3 1+ 2 is odd, then 1 is odd” is false ; ie. nis even. Son =2 k for some integer k.
3n+2=3(2k4)+2=6k+2=2(34+1) ©3n+2
is eveni.c. not odd
This is the negation of the hypothesis of the theorem. Because the negation of the conclusion Of the
conditional statement implies that the hypothesis is false, the original conditional statement is true.
“. by contraposition, we have proved the result that “If3 7 + 2 is odd, then 7 is odd”.
Vacuous Proof: A proof that p q is true based on the fact that p is false.
Trivial proof: A proof that p - q is true based on the fact that q is false.
Proof by Contradiction
Suppose we want to prove that a statement p is truce. Also, suppose that we can find a contradiction g
such that ~ p + q is true, Because q is false, but ~ p > q is true, we can conclude that ~ p is false, whic
show that p is true. How can we find a contradiction q that might help us prove that p is true in this way?
Since the statement r A ~ ris a contradiction whenever r is a proposition, we can prove thatp is try
if we can show that ~ p > (r A ~r) is true for some proposition r. Proofs of this type are called proofsby
contradiction.
In simple words, a proof that p is true based on the truth on the conditional statement ~ p - g, whereq
is a contradiction.
Example. Prove that : 2 is irrational by giving a proof by contradiction.

Sol. Let p be the proposition * 2 is irrational. We suppose that ~ p is true.


re 3 ‘ a
If j2 is rational, there exist integers a and 5 with fz =e where a and b have no common factors
4.
an 2 .iseven
2° — => 2b* =a? => a’ => ais‘ even
b
Let a= 2c, for some integer c

a=de = 2b*=4c0 & bt =2¢7% & b? is even => bis even

2 divides a and 6, which contradicts the fact that a and 6 have no common factors
Hence the result.
We will introduce several other important proof methods, including proofs where we considé
Smee

different cases separately and proofs where we prove the existence of objects with desired properties.
Strategy behind constructing proofs. Includes selectinga proof method and then successfull
constructing an argument step by step, based on this method. '

Scanned with CamScanner


pocic AND PROPOSITIONAL CALCULUS

guhaustive Proof 315

Some theorems can be proved by examining a relatively small number of exam


ples, Such Proofs are
alled exhaustive proofs, because these proofs proceed by exhausting all Possibilities
. . - An exh .
saspecial type of proof by cases where each use involves checking a Single example. aUStive Proof

gyample. Prove that (#7 + 1)? = 3” if nis a positive integer with n < 4.

gL Forn = 1, (n +1)? =2*=4 and


3” = 3! = 3; form =2, we have
:
(n+1)2 = 3? =9 and 3” 2
=3 =9
-forn
= 3, we have (n+1)3 = 43 = 64 and 3” =33 -27 ; and fo nr= 4, we have (n+1)3 = 53
j=3 34 = 81. ~~ = 125 ang
;*= . In cach of these four cases, , we see that ( n+l)?) = 3” ifni
718 tes integer with
a positive n <4
Proof by Cases

A proof cases must cover all possible cases that arise in a theorem. We
illustrate Proof by ca Z
couple of examples. In each example, you should check that all possible cases are coy * eases with a
ered,

Example. Prove that if7 is an integer, then n* 2 =n.

Sol. We can prove that n2 = n for every integer by considering three cases, when n = Q
whennp > l, and
when n S -1. We split the proof into three cases because it is straight
forward to pr Ove the
considering zero, positive integers, and negative integers separately ; result by

(ij) When» = 0, because 02 =0. It follows that n? = nis true in this case,

(i) When n & 1, when we multiply both sides of the inequality n = | by the positive integer n
, we
obtain 2-n 2 n+ 1. This implies that n© 2 =n forn = 1.

(iii) In this case n S —1. However, 2


n* = 0. It follows that n2 > nN,

Because the inequality n? = holds in all three cases, we can conclude that iff
is an integer, then
NZ.

Forward Proof

We may choose any method, we need a starting point for our proof. To begin a direct proof of a
conditional statement, we start with the premises. Using these premises,
together with axioms and known
theorems, we can construct a proof using a sequence of steps that leads to the conclusion. This type of
Proof, called forward proof. Similarly, with indirect reasoning we can start with the negation of the
conclusion and using a sequence of steps, obtain the negation of the premises.

Proof of Necessity and Sufficiency

In logic and mathematics, necessity and sufficiency are terms used to describe a conditional or
‘ - 4 a.e oa 4
Implicational relationship between two statements. For example, in the conditional statement : “Ifp then q",

Scanned with CamScanner


SPECTRUM Discrey, my
nil
ty
Similarly, p is Suffig:
ayarantees the truth of q.not always .imply that 'ie Mf,
ue ofp g true does
q= is necessary for p, be cause the truth but p not bein Is
15 "IY"
p being true always implies that q go. hy
i g one Whi c h must be present in order for another
condit“enion 1s 0 n. The assertion thar Mion
In general, a necessary produ ces the said conditioer
hich
nich: her mean s that the form statement js true _ Ma
while a sufficient condition is one “
«ant? iti n 0 al et be either simultaneously true, or simultaneous, at
“necessary any sufficient conditio

EXERCISE 1.7
‘ the two statements mt 1S
latter is true. That is,

gPete,
if 7 yan d 7 are bot h per fect squares, then # mm is also
1. Give a direct proof that
b, where a and b are posi
tive integers, then a s yn orbs eo
Prove that : If =a
ne

t fall on the same day of the week.


Show that at least four of any 22 da ys mus
orem “If3 7+ 2 is odd, then » is odd
&

Give a proof by contradiction of the the ”


~-

*) \=|x] [yl , where x and y are real numbers


Use a proof by cases to show that |
wn

> and equals ~ g when g :


(Recall that | a | , the absolute value of a, equals a when a = 0

Scanned with CamScanner


Scanned with CamScanner
I
ALGEBRAIC STRUCTURES AND
MORPHISM
wG FBRAIC STRUCTURES WITH ONE BINARY OPERATION, SEMI
GROUPS, MONOIDS, GROUPS
i Introducti
on
In the lower classes we have studied the concepts of binary compositions, relation
s and mappings. In
| ais chapter we shall study an algebraic system with 2 binary operation — on its aleve and
| gisfying some postulates (or axioms). This algebraic system which
occurs naturally in various
| gghematical situations is called a Group. The structure ofa group is one of th
2 simplest mathematical
' gacture. Group is considered as the starting point of the study of various al
gebraic structures. The study of
| poops is the study of single algebraic operation in its purest form.
' pefition. Binary Composition (operation)
Let A be a non-empty set. A mapping 7: A X A= A is called = binary composition (or internal
| emposition) or simply (a composition) on A.
| The mapping f corresponds to each ordered pair (x.y) € Aa unique element Fy).
where x vEa
| Note 1. We may use any convenient notation for a compositicn but the most common
ly wed are 0. G ©. = _
zc. Let + be a composition on a non-empty set A. Then *(x.yyor x*¥, xve€ A
denotes the image of
fy) under *.
2. If* is a composition on a set A, then we write it as (A, *). Here A
is 2 set with operon *_ IFS. ©.
ztw0 compositions on A, then we write it as (A, S, ©). Here A is aset
with two operations S and ©.
ftample (i) Let R be the set of reals. and f:R X R—> R be defined es
i( (x, y))=x yforall (x,y) RXR where x vER
Then fis a binary composition on R.
(i) Let N be the set of naturals
and * : N x N > N be defined
as
x*¥y=x+y, x yen.
Weknow forall x,yEN > xtyEN > x*yEN
*- addition is a binary operation on N.
(ii) Ifwedefine *:N x NN as
x*y=x-y, x,yEN.
And if we take x=3, p=5
Then 3*5=3_5=_ EN
* =*¥EN forallx,y EN
~Sbtraction is not a binary operation on N.
319

. | etd
Scanned with CamScanner
320 SPECTRUM DISCRETE MATHEMAy, 4

Definition. (ALGEBRAIC STRUCTURE)


A set having one or more binary composition is called Algebraic Structure.
Types of Compositions.

We shall now discuss some important types of binary compositions which will be Used j,
defining algebraic structures such as Group, Rings, Fields, Vector spaces.
(a) Commutative Composition. A binary composition * on a set A is called commutative composition,
iff x*y=y*x VxypEA.
Example. The addition composition in the set of real numbers is commutative ie, x+y=ytx, for al
x,y ER.
aT

(5) Associative Composition. A binary composition * on a set A is called associative compositio,


iff (x * y)*z=x*(y»*2z) forall x,y,z EA.
Example (i). The addition composition in the set of real numbers is associative

ie, (xty)tz=x+(y+z) Vx,yzER.

(ii) The composition * defined on R (set of reals)as *:R X R>R

by x *y=x+2y,x,y,
ER is not associative.

Since
(x *y)*z =(x+2y)*z =(e+2y)t+2z =x+2y+2z,

and x*(y*z)=x*(yt2z) =x+2(yt2z)=x+2y+4z


Thus (x *y)*z #x*(y*z).
(c) Composition with identity element. A binary composition * on a set A is called a compositia
with identity element iff Je € A, suchthat e*x=x=x* e forall x GA.
Then the element e is called identity element of A, which is always unique.
Example (i). In the set of reals, 0 is the identity element under addition composition since x + 0=x=0+!
for all x © A.
(ii) The set N, of natural numbers, does not possess the identity element under addition composition
since there is no natural number e such that

ate=aforallaEN. . o€N

(d) Invertible Elements. Let e be the identity element of set A under the composition ‘*’ on the#
A. Let a € A, then f € A is called an inverse element ofa iff a*Bp=e=B*a
Then the composition * is a composition with inverse element, which is always unique.

Example (i). In the set I of integers, 0 is the identity element under addition composition and each cle
a & [has its additive inverse (- a) EI, since a + (— a) = 0 = (— a) + a. Thus every element of integt”
invertible.
‘nl
(ii) In the set N of naturals, ‘1’ is the identity element under multiplication composition but there ish
element other than ‘1’ which is invertible.

Scanned with CamScanner


(fg
hes\

\.GEBRAIC STRUCTURES AND MORPHISM 321


a
eae

(e) Distributive Operations. Let * and O be two binary operations on a set A, Then we say that
operation * is distributive with D0 if
x*(yO2=@*yO@*z) VxyzEA (Left distributive Law)
and (yOz)*x=(y*x)O(e*x) V x,y,zEA (Right distributive Law)
Note. If the composition * is commutative. Then
Left distributive law <> Right distributive law.
trample (f). In the set of naturals multiplication composition is distributive over addition composition
snce x. (ytzj)=x.yt+x.z Vxy,zENn.

|.1. Definition (Group)


Let G be a non-empty set together with a binary operation * defined on it, then the algebraic structure
<G,*> is called a group if it satisfies the following axioms
() a*b€GVabEG (Closure Property)
(ii) (a* b)*c=a*(b*c),Va,b,cEG (Associative Property)
(iif) J anelement e © G suchthat e+ a=a=a*e,V a &G. Then eis called the identity element
of G w.r.t. the operation *. (Existence of identity)
(iv) For alla © G, 36 © G such that a * b= e = b * a then b is called the inverse of a and is
denoted by a7, (Existence of inverse)
Note 1. Ifthe operation ‘*’ is denoted by ‘+’, the group is denoted by <G,+>.
2. If the operation * is denoted by ‘-’, the group is denoted by <G, - >.56.
1.1.0. Finite and Infinite Groups
If the set G in the group <G, * > is a finite set, then it is called a finite group otherwise it is called an
infinite group.
1.1.1, Order of a Group
The order of a finite group < G, * > is defined as the number of distinct elements in G. It is denoted
by o(G) or| G |. Ifa group G has elements, then o(G) =n.
Remark : The order of an infinite group is not defined or we say that the order is infinite.
1.1.2, Abelian and Non-abelian Groups

A group <G, *> is called an abelian group or commutative group


a ll

iff axb=bea, Va,bEG.


Ifax b # b*a,V a,b &G, then the group <G, *> is called a non-abelian group.
1.1.3. Groupoid, Semi-Group and Monoid
Groupoid : A non empty set G together with a binary operation * defined on it is called a Groupoid
- ifit satisfies the followimg axiom
a

a*bEG Va,beEG.

Scanned with CamScanner


322 SPECTRUM DISCRETE MATHEMaTig
. a y
Semi-Group : A non empty set G together with a binary operation * defined on it is called a Sem.
group if it satisfies the following axioms :
(i) at*b5EG VabEG (ii) (a*b)*c=a*(b*c)V a,b,cEG.

Monoid : A non empty set G together with a binary operation * defined on it is called a Monoid if
satisfies the following axioms
(i) atbEG VabeEG. (if) (ax b)*c=a*(b*c)V aybhc EG
(iii) A anelement e © Gsuchthat a *e=a=exa VaeG.
Here ¢ is called the identity element of G w.r.t. the binary operation *.

ILLUSTRATIVE EXAMPLES
Example 1. Show that the set of all natural numbers form a semi-group under the composition of addition.
Sol. Let N = {1, 2, 3, 4, ...... } be the set of natural numbers.
(i) Closure Property : Sincen+mEN, Vn,mEN
N is closed under addition.
(ii) Associative Property : Since
(n+ m)+p=n+(m+p), VnmpeENn.
Associative property hold in N under addition.
Hence N is a semi-group under addition.
Note : (N, +) is not a monoid, as (N, +) do not have identity (zero) element.
i.
Example 2. Show that the set G = ( i > x,y GR, st. x+y # 0} form a semi-group under the
x y
operation of matrix multiplication.
Sol. The G satisfies the following under multiplication of matrices.

(é) Closure Property : Let A aE


= 4 Py | B=ee
yl? 8 be any two elements of G, where x, + ¥1 * )
1% ; X22
and X3+ 3 x 0.

> (xy + yy) 2 + ¥2) = Xy X02 + 1 XQ +3 V2 +H) V2 FO

AB=|"! x, +» I x2 2] Xj Xo+yV)X.
_}*1%2+ I X2 x X ¥2+ + YM 2 , EG
X, Vy} [2 2 XpX2+ Mp XQ AX Y2+ Yi V2

for x, x2 + yy x2 + Hy Yat ¥2 = 0.
G is closed under multiplication.

|
Scanned with CamScanner
e
sm CTURES AND MORPHISM
I zed
ie gh Ca Property :.oSince matrix: multiplica
wate tion
is associative.
(i) associate property hold in G also,
‘ ¥G form 2 semi-group under multiplicati
on,
ne
ye above set do not form a monoid under multiplication, Since it has no identity element.
a!
Ee a | be the element of G such that
ah a b
x y
AB=A=EA, va>| € G, where x+y ¥ 0,

xatya
Ieee
ea x yl _|ax+bx aytby
ie |ig+ya xbtyb x yp axtbx ay+by

Taking first two, we get

(x+y) a=x > a=—x+y

(xty)
+y) b= b=y => b=__/y
t+y

Also, taking , last two, we get


(a+ b)x=x > (at+b-1)x=0

(a+ b)y=y => (a+b-1)y =0, weng


onaddi get
(a+ b-1)(x+y)=0, butx+y+#0
> at+b-1=0

» at+b=]
Thus, the element E in G is not unique.
Hence the identity element in G do not exist.
bample 3. Let M(X) be the set of all mapping of a non-empty set X into itself, then show that M(X) form
‘monoid under the composition of composite of mapping.

Sl Let M(x) = {f | f:X > X is a mapping}


(i) Closure Property : Let £, g © M(X) be any two elements, then

f0g:X-X is.also mapping.


* fOgEM(X) VS, gE M(X).
M(X) is closed under the composite of mapping.

Re
Scanned with CamScanner
YS
324 SPECTRUM DISCRETE MATHEy,
m ATI

VK
(ii) Associative Property : Letx © X be arbitrary element and let f, g , h © M(X) be any el

Fusa
me
Then

Te

(fo g) oh) (x) = (Fog) (h(x) = f(g (A@))) and
(fo (go h)) (x) =f(g oh) (&)) = f(g (A)
(fo
g) oh) (x) =(fo(goh))()VnEX i!

> (fog)oh =fo(goh)


Vf g hE M(X)
Associative law hold in M(X).
(iii) Existence of identity : There exist an element i: X > X defined by i(x) =x, Vx © X such th
a
foi =f=iof VfEM(X)
i is called the identity element of M(X) under composite of mapping.
Hence M(X) form a moniod.
Example 4. Show that the set S = {0}, under the operation of usual addition of integers, is an abelian grow
of order one.

Sol. Closure Property : Forall a,b€S,

a=0, b=0, a+bES as 0+0=-0ES.

Thus closure property holds in S.


Associativity : We know addition of integers is associative
ie. (at+b)+c=at(b+c) Va,b,cES.

Here a=0, b=0, c=0 and(0+0)+0=0+(0+0)


Thus associative property holds in S.
Existence of identity : V a € S, there exists 0 € S such that
a+Q=a=0+a.

Here a=0 and 0+0=0=0+0.


Thus 0 € S works for the identity element of S.
Existence of inverse: V a & S, there exists — a € S such that
at(-a)=0=(-a)ta
Here a=Oand —a=-—0=0.
Thus every element of S has inverse.
is a group.
Since all the axioms of a group are satisfied. Hence <S, +>
Also addition of integers is commutative
le atb=bta VabESs.

Here 0+0=0+0. Scontains only one element.


one.
Therefore, <S, +> isan abelian group of order

Scanned with CamScanner


GEBRAIC STRUCTURES AND MORPHISM 325
a
sample 5, (a) Show that the set natural numbers or the set of positive integers does not form a group
okt addition and multiplication.
(b) Show that the set Z of integers forms an infinite abelian group w.r.t. usual addition of integers.
(c) Show that (Z, — ) is not group where Z is the set of integers
wlation. (a) Let N = { 1, 2, 3.....) be the set of natural numbers of the set of positive integers.
First we consider the operation of addition :
(i) Closure property. Sincen+m EN Wn,mEN, soN is closed under +.
(ii) Associative property. Since (n+ m)+p=n+(m+p)W m,n, p€N, so the associativity holds

(iii) Existence of identity. e € N will be identity of N under + if


| e+n=n WnEN
: But e + n= 7n is possible if e =0
But 0 € N
So identity under + does not exist in N
Hence (N, + ) does not form a group.
Now we consider the operation of multiplication :
(i) Closure property. Sincenm EN Wn,m€EN,soN is closed under multiplication.
(ii) Associativity. Since (a b) c=a(bc) V a,b,c EN, so associativity holds in N.
(iii) Existence of identity. Since 1 © N and l.n=n=n.1Wn€EN,s0 1 is the identity element.
(iv) Existence of inverse. Take 4 € N, 4 € N. But there does not exist any € N such that
4.n=1l=n.4.
So inverse of 4 does not exist in N.
N is not a group under multiplication.
(5) Closure Property : We know that the sum of two integers is also an integer
ie. a+bEZ Va bEZ.
Thus Closure Property holds in Z.
Associativity : We know that addition of integers is an associative operation
ie, (a+ b)+c=at+(b+c), VabhcEZ
Thus Associative Property holds in Z.
Existence of identity : There exist 0 © Z such that
a+0=a=0+a WaEZ
Thus, the element 0 € Z works for the identity element in Z.
Existence of inverse : For all a € ZA — a & Z such that

a+(-a)=0=(-a)+a

Scanned with CamScanner


326 SPECTRUM
Thus, the elemen — a Discrey
t is the inverse of the el
ement a in Z.
TE AB
Since all the axioms of a group are sati
sfied. Hence <Z, +> is a
integers is co
mmut ative Itign
ie. at+b=b+a VWa,bEZ t
~ mt
< Z, +> is an abelian group. “

Also, Z contains an infinite number of elements.


Therefore, < Z, + > is an infinite abelian group.
Remark : A group is always a semi-group (or groupoid, monoid), where as the converse is NOt tr,
general. Mey

(c) (4 Closure property. Sincen—m EZ Wn, m &Z, so closure property holds.

(ii) Associativity. Take a= 1, 5=2,c=3


Thena, 6, cEZ
(a—b)-c=(1-2)-3=-1-3=-4
a—(b-c)=1-(2-3)=1-(-1)=2
(a—b)—c # a—(b-—c) in this case.

associative law does not hold in Z under subtraction.


-“. (Z,—) is not a group. additiond
el i
ia n gr ou p W.F t. usual
an infinite ab
Example 6. (a) Show that the set Q of rational s form . derthe
rationals. infinitefinte a
abelian erouP "
bers forms 4”
(b) Show that the set R* of all non zero real num “> under the operat
Operation of multiplication of real numbers an infinite abelian groUP
(c) The set C* of all non-zero complex numbers forms
of multiplication of complex numbers. nfl
98
Solution. (a) Let Q be the set of rational numbers - Then P2 for some P! par
= 7

. b € Q so that a= ot and? 2
(i) Closure property. Let a, q 20,04
and ql
q2 * 0. closed ynder +
and a, # 0,
s
P, 9+ F241q EQ; since 2!
Then a+6=/1, 2 2-13 ——~~

implies that Ni G2 # 0. 20:


. . £049
(ii) Associativity. Let a, b,c © 2 20,72
rE Z and q
' dqs
a= Pr pa Ph oa P3 for some 9'5 4M
q; 9 93

Scanned with CamScanner


nn a FACE Ma AS SP het a aah Se en,
Py

( cp RUCT JRES AND MORPHIsng


Me
i 327
hen (4 + b) + 05ie Py
[4 P»4 p
4 23 Py 15 +p g

No)” gh14 a,
= (p, fy + P, 4) I + p, 44,
( 119) q3 '

= 142 43+ py 43 + q, dy p
3
. . qy q> q3

§
ince commutativly and associativity holds in Z *

Py | P )
a + (b+c)
Mf tat +2)43) PL4, Pat 494try gy
u

- Py\9r
2, AG 93) : +(p, 95+ Ps 4 )4,
1 (4 4, )
_ P\ 4 Gy + Pp, 439, + P3 45 q)
49 43

Py I q4 + P qy q3 4: q5 qP3

4) 97 93
(atb)+c=at(bt+c)V a,b,cEQ,
So, the associative law holds.
(il) Existence of identity. The number 0 € Qand a+0=a=0+q VaEQ
0 is the identity element.

(iv) Existence of inverse. Let a € Q so that a= ; for some p,q € Z and q #0,

Thn b= —- & Q
4 10 Joa.
a+ b = p 42 _Pq-Pq_ 9 _,
q q Pq Pq
Similarly b+@=0
at+b=0=hbt+a
bis the inverse of a
(Q, +) is a group.
Moreover the addition of rationals is commutative.

So ath= Lan Py Po 9. _ Poh *P\%n


_ Pydat _ Pr Pt

N 4 M49, 9,99 47 4
And =“b+aVabEQ —
naQ Contains an infinite number of elements.
er
efore (Q, +) isBone an infin
topesite
abelian group.

be

Scanned with CamScanner


ie
e

m
=

328 SPECTRUM DISCRETE Matip


M4

NG
(b) Closure Property : We know that product of two reals is also a real number SS
ie, a.bER* ¥Y abER*

Associativity : We know that multiplication of reals is an associative operation as


(a. b).c=a.(b.c) WV abhcER*

Existence of inverse : For all a € R*,3 1 € R* suchthat a (2) =|= (+) a


a

1. .
Thus, the element — is the inverse of the element a € R*.
a

Since all the axioms of a group are satisfied. Hence <R*, -> is a group. Moreover, the Multiplicar
of reals is commutative
sre!

ie. a-‘b=b-a,V¥
a bER*.
Ee

.. <R*, -> isan abelian group.


Also, R* contains an infinite number of elements.
Therefore, < R*, -> is an infinite abelian group.
(c) Let C, be set of all non-zero complex nos.
ave

y are not both zero and x,y © R}+


ie., Cy = {x + iy|x,
(i) Let zpzatib, m=ct+idEC,

Then 2,22=(ac-bd)+i(ad+ bc)


And 2,22 =0
if ac-bd=0 and ad+bce=0
which is possible when cither a=0=6
or c=0=d
> either at+/h or c+id isa zero complex number.
But a+ib and ct id both are non-zero complex numbers.

Thus z)Z2 is anon-zero complex number


2422 EC
Thus Cy is closed under multiplication.
(ii) CL et z,=atib, 2=ctid, y3=et+ifEC,
then (2172) 23 = [at 15) (c+ id|(e+if)
=[(ac—bd)+i(adt+ bo] (e+if)
=[(ac-bdje+ad+ be)f] tiflac-bast(adtboel

And 21 (2273) =(a+ib)[(e+id(et+if)]


=(a+ib)[(ce-df)+i(cft+ de]
i


Scanned with CamScanner
| -

ganat STRUCTURES AND MORPHISM

fad
he
— =[a(ce—df)-b(cf-~ de)]-ilbl(ce-df)- alef+ de}}
=[ace-adf-bcf-bdel-ilbce—-—bdf+acf-ade}
=[ace—-bde-adf—bcfj-ifacf-hdf-ade-bce]
=[(ac-bdje-(ad~+be)f]~if[l(ac-baj-(ad-beje]
(x) 22 )=3 = 21 E22; )-
Ttus multiplication in C; is associative 1 ~ 10 © C; such that
(ay (at ib)= (1+ 10)=a+ib=(a~ib)(1 = iO)
- 1+i0is the identity element.
(a) Let z= a+ ibe, => a, bare reals and are not both zero.

| — a-ib _ a-ib
Now TF G+ib (a+ib)(a—-ib)
32 +B-
_ a ( —6 )
Pap a? +b7/
=> a+b +0.
Since aand bare not both zero

Thus ec,

—b
* uu (25) +i (*,}> is multiplicative inverse of z&C,
z a +h~
Hence C, is a group under multiplication.
imple 7. Let Q* denote the set of all rational numbers except — 1. Show that Q* forms an infinite
telian group under the operation * defined by
a*b=a+b+ab,
fora, bE Q*.
SL Here Q* = The set of all rational numbers other than —1.
For a, b, c E Q*
> a,b, care rational numbers other than - 1.
(i) Closure property : Since a and / are rational numbers so a + 6 + a@ bis also a rational number
Ifa+b+ab=—I1
then a+b+ab+1=0
> atab+b+1=0
> a(l+6b)+(1+4)=0
= (a+1)(b+1)=0
> a=-1 or 6 =—1,
which is not true
at+tbt+abx#-1 = atbt+abE
Q*.
closure property holds in Q*.

Scanned with CamScanner


SPECTRUM DiscR ¥
330 ETE N
ATE,
(ii) Associative property : Ny
(atb)*c=(atbtabjec

=atbrabtcr(atbtabjc=atbt+ctabtacsy.. a b c
ae(b*c)=ax(bictbe)
=atb+ctbeta(btctbc)=atbt+ct+bhetabig., ;
[0¢
-: (a*b)*c =a*(b*C)
associative law holds in Q*.

(iif) Existence of identity : Let e © Q* be the identity element of Q*. Then


axe =-a=era

> at+etae=a

> (i +a)e=0
sore

=> e=0
(~~ | > d+ "ase

(iv) Existence of inverse : For any a € Q*, let

a*b=0
> a+b+ab=0
> a+(l+a)b=0 |
> (l+a)b =-a

=> b=-— eQ:.


l+a

as b=- a #—]
l+a
j
1
E if — = =-1 > -a=-l-a => 0=-1 whichisnotrm
l+a J

is the inverse of a.
lea

(v) Commutative property :

a*b=a+b+ab

=bt+at+ha
=bea.
OE!)

Hence Q* forms an abelian group


under the composition «,
mark : We b
Remark : V can check closed Property, commutative
i property, existence of identity
i . ene

existence of inverse from the composition tab! i


¢, which we define as :

| ee

Scanned with CamScanner


a
S
FBRAIC STRUCTURES AND MORPHISM 331
ALG
composition Table or Operation Table
A composition table is a type of square array which indicates all the possible ‘products’ in the system.
The system must be finite). The elements to the left of the operation are arranged on the left of the table
eithe elements on the right of the operation are arranged on the top of the table. Of course the order of the
eements on the top should be kept the same as the order of the elements on the left.
The composition table can be constructed by the rule
(i, /)th entry in the table
= (ith entry on the left)-(jth entry on the top).
Further Let S be finite set and * be a binary operation on S. From composition table, we can draw the
following conclusions :
(i) Ifall the elements in the composition table are the elements of S then S is closed under * operation.
(i) If any row is same as the first row in the composition table then the exteme left element in the 2™
row is the left identity of S. Similarly, if any column is some as the first column. Then the element at
the top of 2" column is the right identity of S.
(ii) If every row contains only one identity element, then the elemen headed by that row works as the
]

left inverse of the element headed by that column, in which the identity element lies. Also then the
element headed by the column is the right inverse of the element headed by that row.
(iv) Ifall the entries in a row are different, then the left concellation law holds. Similarly if all the entries
ina column are different, then right cancellation law holds.
(v) If the entries in the composition table are symmetrical above the principal diagonal, then S is
commutative w.r.t. the operation *.
Example 8. Show that the set G = {1, @, a} of cube roots of unity forms a finite abelian group of order 3
der multiplication of complex numbers.
So. Here G = {1, , 0°}.
To prove that <G, -> is an abelian group. We form the composition table using = 1 as given
telow:
* ] @ o

1 | a ow

@ o 1
2 2
@ l w

(i) Closure Property : Since all the elements in composition table are elements of G, so G is closed
mder multiplication.
(i) Associalivity : Since the elements of G are complex numbers and multiplication of complex
tumbers is associative, so multiplication is associative in G also.
(iii) Existence of identity : Since 2™ row is same as the first row
“. Lis the left identity. Also 2™ column is same as the first column
| is the right identity. So 1 is the identity of G.
1 e
le, la=a=al WaeEG.

Scanned with CamScanner


332 SPECTRUM DISCRETE MATHE MAT,
'
(iv) Existence of inverse ; Here each row (column) of the composition table contains idenes
element | once and only once. So the element left to | is the lefl inverse of the element above J. Simi
the element above | is the right inverse of element left to 1. “
Thus we see that
l=1L=tlsol! =),
Also w-@ = 1 =0"- a, soa! =a" and(w)' =a.
(¥) Abelian : Since the entries in the composition table are symmetrical about the principal diagonal,
Hence G is an abelian group under mulitplication.
Example 9. Show that the set G of all n-th roots of unity forms an abelian group of order n, under the
usual multiplication of complex numbers, where nz is a fixed positive integer.
Sol. LetG = The set of all n-nth roots of unity

izke lea 4a 22 i(n-1)


=ce " 10S5k<n>=lle” .e 2 0 seceeees ye 7
|
\
ft
The operation is multiplication of complex numbers.
Closure property :

Zhe Die
Ve" ,e 7 €G6,05 k,k, <n
2thyx 2ikyx 21h) +hy)

Nowe" +-e@"% =e a EG if kj+k,


<n
and the closure property is satisfied.
If k, +k, 2 n, then by division algorithm,

ky +k, =nq+r,where OS r<nand q €Z.

2ihys ths lthsb)e Qingerie


Here
e * -@ * Se ” =e .
Zire GF lire
% - — 7 ~~
ogi » A ry) on"

lirgr
= 7 ‘
~(I)% 0 4 (But et!" =cos2a+isinda <1eided
"

fire

~“e¢ © EG ws OS r<n.
.
the closure property is satisfied
Associativity and Commatativity : Since the elements ofG are complex numbers and mulnp!<2*
of complex numbers is associative and commutative,
“multiplication is associative and commutative in G.

Scanned with CamScanner


.

HIS M
CTURES AND MORP
.
prt
ics
oe AMopr
reser »
ce of identity :WhenkeO,thene 7% =e =).
jsten :
orks as the identity element in G
ere 1 a
, of inverse : The inverse of| is | itself.
ristene rence
: 2k 2i(a-hie
Y 7 eG, k=1,2,3,..a-l,thhen e« * EG

2k Zi{n-dj) et lta
n e n = ¢ % wget! wt
such tha! t ¢
Qik Zitm-a’)er

inverseof ¢ " Be * [-: the operation is commutative]

Hence <G. *> forms a finite abelian group.

{ ! Oo} fe oO
> - ~
} fo af}
poaple 10. LetG~ ) = oy +
oll; of

cove thatG forms a finite non-abdelian growp of order 3 under the composition of matr
ree

The operation denoted by multiplicatively is defined as


Pasi ek? =-1,

ipehkho-jigk hy him
mre pe -tak

Clearly the system is closed under the given operations and associative law holds.

ie VatbEG e2@a-AEG and YahcEG @ (ce: db)-c=a(o- ao


The element | © G works as the Kientity element a5

VeGG, alee tea.

Inverse of Lis 2 and inverse


of ~1 ix— 1 [vo t-te band (-1(-1)= 8)
laverse of fis ~ Zand inverse of = 4 is / [- ft-T

laverse of » is —/ and inverse of-y is) Iv fesyr

bers of & is ~ A and inverse of - A is A [s &i(- 4 1}


Pesther since ij epi {°° @~
A and ji=- A]
|
; ke YubGG, ahe 4 a (in general).
:
Thas
G is a fini te non abelian group of order 8.

Scanned with CamScanner


ss SreCTRUM DISCRETE Matiy Mites

QUARTERNION GROUP
Lf we fet

wit a)OP ads[ OF» Fen


8 Claw] fo we have-e the identities 7 =“fo j? 2ak’.42
4] | then,
6 te 10 4}? iol Of ff 0|
tee hyp heikhiejigic- Rr en aike nd
then the ct G- trl, 27,2), 2 A} forma finte non-abelian group. This grou
p is called Quarters,

Group and is pencralls denotedby Qe r ; 7


cos@ -—-8ma)

Example 11, (2) Show that the set of all matrices of the form a |
[ sin . a} 7
here @ 5 a ny

.
number forms an abelizn group under the operation of matrix multiplication.
(4) Sthow that the act G of all m & m matrices over Z (or Q. R or C ) forms an infinite shetiag oy
water the operation of addition of matrices
I 7

Selk(a) LeG={A,° Ay * sin S| : -3inda


where
a € R}.
sin a cos a

The binary operation defined on G is multiphcation of matrices.


() Closure Property ; Let A,,, ae Ag © G be any two elements
r
(oma -sina

. cos —sinBi a.peER
where A, =
| ote a oo8 a }
tS
sings cosf

foosa =sin *] cosfs -sinfi


Now AJAg ©
sin a cos a jisinf cosft
yp

cosa cos § sina sin fs -cosa@ sin B- sin a cos f5


on
ge

sin a cos i+ cosasinff{ -sina sin B+cosa cosf


cos(a +f) -sin(a + B)}
a

we

sin(a +B) costa + B))


“Ay.g EG, for ceFER@ ar+fER.
Closure property hold in G.
{u} Associative Property : For all A... Ag, A, €G,

= (Ag.p)Ay * Ajasprey * Aas(pey) es (a + B)+y <a> (2+


(AgAgiA,

= Ag -Ag. yAy)
™ Ag -(Ag-
“. Associative property holds in G.
(ii) Existence of identity : 3 an element
% _{eos0 -sind)
ft oO}, eli ti
. , sin 0 cos 0 lo i ue ‘
AgAg “ Aqso ™ Aq 7 Ap: Ag. ¥ Ag EG
Ag works for the identity element for G.

ee

Scanned with CamScanner


ee SC A el
ey A ee A
oN [eee —

ne
ai
epRUCTURES AND MORPHISM
4 > :
we G ~The set of all mr X nm matrices over integers Z.
ay Lett erty. Let A,B, EG
am

Closure prop
\

iti B=(b,,) _ wherea, ,.b,, eZ Vif

x
2.0,
BEG
E2 v¥ A.BEG.
ds in G.
clessure property hol
ajo
. , Associative property. Since the set of all matrices is associative under addition, so G is
ive under +.
|) Existence of identity. Take O = [0 Jn.» Where 0 is zero integer

es E Gso that a, , € Z.
tat OA (4,,) *d
pen O+ A =(0) wxA +(a,,) =(o+a,,)

- (a, ) =A~A +O similarly.


. mx

O is identity of G.
(wv) Existence of inverse.
Let A™ Ca GG, sothata,, EZ

1,€Z* -a,€Z * B=(-a,) EG


J men

a+B=(a,) mar
+(-a,, man

-(a,,+(-a,,)) =(0) n= O=B+A, similarly.


mewn

B is inverse ofA and B E G.


é ‘so G contains an infinite number of elements
) Let A,B
& Gasin(/)
ThenA + B=(a,, +b,,) as in (/)
mean

* (4, +a } , since + is commutative in Z


, f : J man

=(4,,) mxn +(a, ,)


man
=B+taA.
Gis commutative und
er
Hence G is an infinite commutative group under +
+.

Scanned with CamScanner


al

336 SPECTRUM DISCRETE MATHEMATIC g


—,

ADDITION MODULO nu
Now we define another type of addition known as ‘Addition modulo 2° and written as ‘a +, b° where
aand } are any integers and nis a fixed positive integer,
We define a +, b= 7,0 Sr<n where r is least non-negative remainder when a + 6 (usual addition)
is divided by 7, Also itis writtenas a+b r(modn), eg,
(i) 18 +,9= 2 as 18 +9= 2755 (5) +2, ie, 2 is least non-negative remainder when 18 + 9 jg
divided by 5.
(ii) -37 +, 2=1as -37+2= —35 = —9(4) + |, ie, 1 is least non-negative remainder when —37
LS

+ 2 is divided by 4.
Note : When a and 6 are integers such that a — b is divisible by » (a fixed positive integer) then we write q
= 6 (mod ») and read it as a is congruent to 5 modulo 7.
e.g., 21 = 1 (mod 5), 33 = 1 (mod 4)

MULTIPLICATION MODULO n
Further we define another type of multiplication known as ‘Multiplication modulo nm’ and written as
a x, 6 where a and b are any integers and n is a fixed positive integer.

We define a x,0 =r, 0S r<n_ where r is least non-negative remainder when ab (usual
multiplication) is divided by n. Also it is written as ab = r (mod n), e.g.,
() 8 Xy 3=3as8 X 3 =24 = 3(7) +3, ie, 3 is least non-negative remainder when 8 X 3 = 24 is
divided by 7.
(ii) 3 X6 5=3as3 X 5=15=2(6)+3 as 6 divides3 X 5 —3, i.e., 3.5 = 3 (mod 6).

ADDITIVE GROUP OF INTEGERS MODULO


Example 12. Show that the set Z,, or J,= (0,1, 2,3 y...08 ,n-1}, n>1, n €Z forms a finite abelian
group under the composition of addition (congruence) modulo n.
Sol. Given Z, or J,= {0, 1, 2,3, ....,2—-1}, n> 1, n EZ.
The composition defined is addition modulo n.
Va,b6]J,, a*b or a+,,5= least non negative remainder r when a + b is divided by n
ne. axbor at,b =r > atb-—ris divisible byn.
Le. a+b =r(mod pn).
Closure property: V a,b€ J, OsSa,b<n
a+b =r(modn), where 0 = r<n.
Now r&J,,.
the closure property is satisfied.

Scanned with CamScanner


wen —-- ’
NE LY LAREN A | w
7s

ghBRAIC STRUCTURES AND MORPHISM


Ve 337
associativity : VabcE]J,

the least non-negative remainder redmain s the same if


di vi de by n.
(at byte (b + c) are
or at

(a* b)*c =a*(b*c)


ty holds in J,,.
Thus associative proper
Ee J,,
Commutativity: V a,b
The least non-negative remainder remains the same ifa+6or 6+ a is divided by n.
ie. a+b =r(modn) and b6+a=r(modn)
a*x*b=b*a.
Thus commutative property holds in J,,.
Existence of identity: V a€&J,, OS a<n.
Here a is the least non-negative remainder when a+ 0 or 0 + aare divided by n
a*x*Q =a=0*a
Thus 0 & J,, is the identity element.
[iistence of inverse : Inverse of 0 © J,, is O itself.

Also forall a€&J,, a# 0, n—a €J,, such that


a+(n—a) = 0(modn)
and (n-—a)+a = 0 (modn)
ie. eed) —0= —a ¥e
Thus n — a is the inverse of a.
Hence <J_,*> is an abelian group of order 7.

Note: J, under addition modulo n is called additive group of integers modulo rr.
ftample 13. Show that the set J, = {1, 2, 3, ..., 2 — 1}, where p is a prime number forms a finite abelian
foup of order p, under the composition of multiplication (congruence) modulo p.
Sol. Given J, = {1, 2, 3, ...., 9— 1}, pis a prime number, p> 1. The composition defined is multiplication
nodulo p.
Va,b&J,, a* bor a X, 6= least non negative remainder r when a b is divided by p.
ie a*xboraxX,b=r => ab-—ris divisible byp

ie. ab =r(modp). *
Closure property: V a,b€J,, 1 =a,b<p
ab=r(mod
p) where 0 Sr<p
If possible let r= 0, then a 5 = 0 (mod p)
> pflab-0O => p/ab

1 >

Scanned with CamScanner


LAN
—— ee
STECTHUM DIsciun re MATHEMATICS

Prat pas pine


either pf or psd where Ds ade p.
ve get ait adwurd resull,

thes Pe Vee TET, ® Joisclosed under «,


|
Comautativity : Yad GI,
The beast ton-negative fenainder remains the same if ab or ba is divided by p
ie ad ar(modp) and bam (mod p)
aehedbea |
Thos commutative property holds in J). |
Associativity : Vo a, dc € J,
The least noa-negalive remainder remains the same if (ad) ¢ or a (dc) is divided by p.
(a*A)eceae(dbec)
Thus associative property holds in J).
Existence of entity : Vo a € J,
e¢t=a=teg as a. and]. leave the same remainder a when divided by p.
1 € 1, works as an identity element for the set J,
Existofence
invense: Wo ),, ls ax<p
Consid
a set
erS= {le a2ea,3ea,...,(p-l)* a}
As }, is closed, therefore S 4,
Further all the elements of S are different.
If possible
et dea=peatsiy<p;ixj andict i>y > 0<i-j<P

2 j2Bja(modp)
* plia-ja @ p is prime
piso, wher e
> either ? p/
= Wa -_ /a

But iSax<p and 0 <i-j <p which are absurd.


all the eleof
menS are different
ts .
XS) =p-1=0I,) © S~),
i)

4 Now 1 € J, ® 1ES
ue Re heguy
te kaw) (mod p) ;
cxists.
inverse of every element of J,
py Where k & J, is an inverse ofa Le.
Be. Hence x
ee “MCC < J, , «> ig an abelian group of order p - |.

Scanned with CamScanner


LyLT ee ees Las ea ey

gant QC TURES AND MOuritinet AM


43
7h
it pis net prime, then J, is not a group
«et
, oe fl2, 4. 3) under multiplication (congruence) mexlule 6 i met a grow
/ Y
é s

jiere We find 2,3 G, but 26 1-0

a 2°) m O(mod 6), where OF C)

thus G is not closed under the given operation


ea L The group J, is known as Multiplication group of integers modalo p.
sbelaan group
[a 1} of resictue chanses modulo n forms a fiete
gaark: the set G = ((O), C1). (2). 2
ake the operation defined by
modulo nm
we blel + j], where [/] denotes the residue class of (
Here in fact the operation is

wou ( ioren
where it+f Manger Le i*y B r{mod mn).

+ {0} {1} {2} ee {rz -1}


(0) [0} {1} Cae! be Cen [n- 1}
{1} {t] {2} (31 Seacimehldheds ida sntemesiosmcei (@}
2) (2) [3] {4} een genase {i}

[rr-l} | [ej [0] {t] ~nmesvanshnaenaatemammenmenansmmaciecsin | [= 2)


Closure Property : Since all the elements in the composition table are the elements of G, so G is
ase under the composition +.
Associativity : Since the elements of G are residue classes over integers and addition of integers is
tecutive. So addition of residue classes in G ts also associative.
Existence of identity : Since the second row is same as the first row and also the second column is
&e a5 the first column.
[0] works as the identity element of G.

ie (O}+(j-()=[) > [O]. VUE G.

Existence of inverse : Here cach row (column) of the composition table contains identity element {0]
*s ant only once. So the element left of [0] is the left inverse of the element above [0]. Similarly, the
“ment above [0] is the right inverse of the element left to [0]. Thus we see that inverse of [/] is {» - 1.
“st isa sn 1 and inverse of [OJ is [C].

Scanned with CamScanner


ra

340 : - SPECTRUM DISCRETE MATHEMATIcg


—_ i.)

Commutativity : Since the entries in the composition table are symmetrical about the principa|
diagonal.
Commutative law hold in G.
Moreover, as the set G is finite set. Hence (G, +) form a finite abelian group.
Note : The above group is known as additive group of residue classes modulo m and is generally denoted
by I/< n>, the quotient group.
Remark : The set G = {[1], [2],........ [ p — 1]} of residue classes modulo p forms a finite abelian group
under the operation defined by
[‘] - '] = [i], where [i] denotes the residue class ofi modulo p and p is a prime number.
Here in fact the operation is
9558

ij) if ij<
IS

awe {i if op,
where ij=npt+r ie. if =r(modp) and lsr<p-1
CW) | 2) |B) | ceceecteceeessccssssseseeee | 2-1]
CDP CUD | 2D | DBD | cesesssssssssesesseescssesssssereesessseeeeeen [p-1]
Pe
Bl | B) | | 19) [p-3]

[p-2] | [p-2] | (p-4] | (2-6) | eeeeeeeeceeeccccciscccseeeecssssneeeess [2]


[p-1] | [p-1] | [2-2] | [2-3] [1]
Closure Property : Since all the elements in the composition table are the elements of G, so Gis
closed under the composition.
Associativity : Since the elements of G are residue classes over integers and multiplication of integers
is associative, so multiplication of residue classes in G is also associative.
Existence of identity : Since the second row is same as the first row and also the second column is
same as the first column.
[1] works as the identity element of G.

te. (1) - ()=P=11- 0, V0 EG.


Existence of inverse : Here each row (column) of the composition table contain identity element [I] |
once and only once. So the element left of [1] is the left inverse of the element above [1]. Similarly, the
element above [1] is the right inverse of the element left to [1]. Thus we see that the inverse of [p-—r] is [P-
r'] where 1 < r,r’ <p such that [r] [r’] = [1].

Scanned with CamScanner


SSS SY

.GEBRAIC STRUCTURES AND MORPHISM 341


foe

Commutativity : Since the entries in the composition table are symmetrical about the principal
agonal.

commutative law hold in G.


Moreover, as the set G is finite set. Hence (G, -) form a finite abelian group.
p and is generally
vote : The above group is known as multiplicative group of residue classes modulo
group.
sroted by I/<p >, the quotient
11.5. Some Special Groups
|, General linear group of degree n |
frample 14. Show that the set of all 7 x n matrices having non-zero determinant over the set of real
gumbers under the operation of matrix multiplication is a non-abelian group. This group is known as
ceneral linear group of degree # and is denoted by GL (rn, R).
OR
Show that the set of all 7 * nm non-singular matrices over the set of reals under matrix multiplication is
jnon-abelian group.
sol. Let G be the set of all » X m non-singular matrices over the reals.
Closure Property : Let A, B be any two numbers of G.
A Bisalsoan X nreal matrix.
Since |AB|=|A||B|
#0 (Since | A| # 0,|B|
#0)
A Bis non-singular n X n matrix over the reals.
ABEG,VA,BEG.
Thus G is closed under matrix multiplication.
Associativity : Since the set of all matrices is associative under multiplication, so G is also associative
under multiplication.
Existence of Identity : Let | be the identity matrix of order n X n, where|I| =1 # 0.
Also AI=A=IA, VW AEG.
Lis the identity element of G.
Existence of Inverse : Let A & G so that A isan X mnon-singular matrix over the reals.

Let B= a adj A, then B is an n « n matrix over the reals.

|A|"! _
l . n-2
|B| = ——|adj A|= |A\" #0,
|A| |A|
BEG.

Now AB ath adjA


|A|
e

= jaye [- Aadj A=|A]1]


JA]

Scanned with CamScanner


w
SPECTRUM DISCRETE MATH
EM 4p
Similarly BA =I
AB =I=BA
A'=BEG
G is a group under matrix multiplication.
Since.G has an infinite number of elements, therefore G is an infinite group.
Also since, in general matrix multiplication is non-commutative, i.e. AB # BA, therefore g is
infinite non abelian group. a
Note : The set GL(n, F) of all n X n matrices having non-zero determinant over the field F Under thy
operation of matrix multiplication is a non-abelian group.
Il. SPECIAL LINEAR GROUP OF DEGREE n
Example 15. The set of all » X nm matrices having unit determinant over the set of real numbers under the
operation of matrix multiplication is a non-abelian group. This group is known as Special linear group
‘degree 7 and is denoted by SL(n, R).
Sol. Let G = SL(x, R) = set of all n X n matrices having unit determinant over the set of real numbers
To show that G forms a non-abelian group under the composition of multiplication of matrices.
Closure Property : Let A, B be any two members of G.
|A|=1, |Bl=1
Now |AB| =|A||B|=1-1=1
> ABEG, VA,BEG.
Thus closure property holds in G.
Associativity : Since the multiplication of matrices is an associative operation. Thus the associative
property holds in G also.
Existence of Identity : 4 a unit matrix [= I,, © G such that

A-]=A=I-A, VAEG
1 € G works for the identity element for G.
Existence of inverse : Let A & G be any element, then 3

B = —- adj (A) €G such that


|A|
AB =I=BA

B| = AL i aca] = |adj.(A)|
2S _= LAT
|A["”

|A{ 1
1 . A.adj(A) _ |A|I
=A- — adj (A) = ———— _ = ——= 1.
Also AB = Av 7 a0 [Al [Al

Scanned with CamScanner


ae Sy ee Oe

\LGEBRAIC STRUCTURES AND MORPHISM 343

similarly, BA =I
AB =I=BA
B= A! is the inverse of the element A in G.

Non-commutativity : Since in general matrix multiplication is non-commutative. In particular matrix


<ultiplication in G is also non-commutative.
»

Hence G = SL (”, R) forms a non-abelian group under multiplication of matrices.


Note: The set SL(m, F) of all n X n matrices having determinant | over the field F under the operation of
garix multiplication is a non-abelian group.
nl
i]. GROUP OF PERMUTATIONS
Definition : Permutation

Let S be a non-empty set. A permutation on S is defined as a map from S to S which is both one-one
and onto.
Example 16. Let A(S) denotes the set of all permutations on a non-empty set S. Then A(S) forms a group
under the operation of composition of maps. Moreover if S contains » elements, then the group A(S)
contains | m elements. This group A(S) is called Permutation Group.

Sol. Here, A(S) = The set of all the permutations on S.


A(S) = {f/:S > Sis a one-one onto map.}
Let f,g,h€ A(S).
J,g%, hare one-one onto maps from S to S.
Closure property : Since f, g are maps from S to S,so fog is also a map from S to S.
fog:S-—>S is defined by (fog) (x)=f/(g(x)), VxES.
We prove that fo g is one-one as well as onto.
For one-one :
Let x,,x2
© S such that ( fog) (x )=( fog) (x2)
> f(g (1))=fC
g(x ))
> 2(x,)=2(x2), since fis one-one

> Xx, =xX2, since gis also one-one


fo gis one-one
For onto
Let zeEsS.
Since f:S—->S isontoand z€&S,sody€Ssuch ) =z.
that/(y
Since g:S > S is onto and y € S, sod = y.
x € S such that g(x)
Consider( fog) (x)=f(g(x)) =S(y) =z.
fog is onto.

Scanned with CamScanner


We

344 SPECTRUM Discre


“TE
So, fog isa one-one map of§ onto S.
NG
Jo gis a permutation on S.
> fogEA(S), Vi,g EAS).
Thus A(S) is closed under composition of maps.

Associativity : Let x € § be an arbitrary element.


Forall f, g,4 € A(S),

((fog)oh)(x)=(fog)(h(x))=f(g(h(sx)))
(fo(goh))(x)=f ((goh)@))=S(g(A(*))
((fog)oh)(x)=( fo(goh))(x ), VxES.
~ (fog)oh=fo(goh)
Thus associativity property holds in A(S).
Existence of identity: Defineamapi:S +S by
i(xy) =x, VxreEs

Let x,.x; ES. Then (x,)=i(x,)) * x, "xp

i is one-one.
If x€S, then (x) =x

iis onto
i is a one-one map of § onto itself.
iis a permutation on S.
So, i € A(S).

Also” ( for)(x)=f/(i(x))=f(x),
Vx ES.

foi=f. Similarly iof=f,V fEA(S)


i is identity element of A(S ).

Existence of inverse : For all f€ A(S ),f is one- one and onto map of S to S.
J is invertible map and f' is also one
- one and onto.
Pa f~' isaalso
a permutation on S.
f 'EAS)
Also f-':S—Sis definedbyf''( y)=x iff f(x)=y
Ee x5.

f(xyES. Let f(x)=y + yes.


Also f(x)=y > f'(y)=x.

Scanned with CamScanner


cEBRAlC STRUCTURES AND MORPHISM 345

Now ( of (x)RS' (Sx) =x =i (x)


) Hf! (y)
(fof) (x)=i(x ), Vx ES,
>» flof=i, similarly, fof =i
f lis inverse off
A(S) forms a group under the operation of composition of maps.
further if S contains elements, then
The number of elements in A(S )
= The number of permutations of S.
= The number of arrangements ofn elements
=[n.

oefinition : Symmetric group.


Ifa set S has » elements, then the group A(S) of permutations on S having | elements is called a
ymmetric group of degree 7. It is also denoted by S,.

Symmetric group S,,= The group of permutations ofa set of n elements.


forexample. Let / be a permutation
on S = { 1, 2,3, 4} suchthat f( 1)=3, f/(2)=4, f(3)=2,
faye.

123 4
We write the above permutation in a better way as f£ -[ . Here in the first line write the
3 42 1
ements ofS and in the second line we write the images of the elements ofS such that the image of each
ement in first row occurs below it.
Multiplication of the two permutations.
tn ga(i 234 (1234
2341)’
* \3 214
Then f(1)=2,f(2)
=3,/(3 )=4,f(4)= 1.
g(1)=3,¢g(2)=2,¢(3)=1,g(4)=4.
Sincef takes 1 to 2 and g takes 2 to 2, so fg takes | to 2.
f takes 2 to 3 and g takes 3 to 1, so fg takes 2 to I.
f takes 3 to 4 and g takes 4 to 4, so fg takes
3 to 4.
Finally,f takes 4 to 1 and g takes | to 3, so fg takes 4 to 3.
Here, to find fg, we first applied fand then g.

1 2 3 4/1 2 3 4) (1 23 4
fe-|, 5 a4 als 2 1 a) le a 4

Scanned with CamScanner


SPECTRUM DISCRETE MATHEMATICy
346
Cyclic permutation.
A permutation fon a set S is called a cyclic permutation of length / if 3 x), x2,.-.--- xX, ES such

XX ey
= X35 ey LGD =xe, I= m and fR)=x V x E Sif x F
that f(t) =X2 SQ2)
We write it as (xX) X2...... X71).

Here the image of each element in the row is the next element and the image of the last elemen
is the first element.
123
4 5 6
For example, if f= be a permutation of degree 6.
r f (; 241 5 6

Then f(1)= 3, £(3) = 4, f(4) = 1, £(2) = 2, £(5) = 5, f(6) = 6.


Thus {= (134) is a cyclic permutation of length 3.
Here the element whose image is the element itself is called an invariant element. In the above
example 2, 5, 6 are invariant elements.
Remark : (i) The length of a cyclic permutation is the number of objects permuted by the cycle.
(ii) Acyclic permutation of length ? is also called £-cycle.
(iii) A cycle of length | is the identity permutation.
(iv) A cycle of length 2 is called transposition or bi-cycle or 2-cycle.
Example 17. Write down all the elements of the permutation group (or symmetric group) S3 on three
elements 1, 2 and 3.
Sol. Let S= {I, 2,3}.
Then there are |3 = 6 elements in S;

(CSC T GG SG GT
This can be written as

S3 = {i, (12), (13), (23), (123), (132)}.

EXERCISE 1.1
1. Show that the set of natural numbers form a monoid under the composition of multiplication.
2. Let X be any non-empty set, let P(X) denote the power set of X. Then show that
(a) P(X) form a monoid under the operation NM, intersection of sets.
(b) P(X) form a monoid under the operation U, union of sets.
3. Let M2(I) be the set of all 2 x 2 matrices over the set of integers. Show that the set M,(I) form!
monoid under the composition of multiplication of matrices.
4. Show that the set S = {— 1, 1} under the operation of usual multiplication of integers, is an abeli*
group of order two.
5, Show that the set Z of integers does not form a group under multiplication.

Scanned with CamScanner


a iy
\LGEBRAIC STRUCTURES AND MORPHISM

E
Show that the set of rational numbers does not form a group under multiplication.
kN

. Show that the set of all non-zero rational numbers forms a group under multiplication.
. Show that the set R of reals form an infinite abelian group w.r.t. usual addition of reals.
so

. Show that the set of reals R does not form a group under multiplication.
we

. Show that the set C of all complex nu8mbers forms an infinite abelian group under the Operation of
addition of complex numbers.
IL. Check whether the set E of all even integers forms a group under the binary operation a * b=2g + 2
b
12. Does the set E of all even integers form a group under usual addition ?
13. Check whether the set O of all odd integers forms a group under addition.
14. Prove that the set of complex numbers z, such that | z | = 1 forms a group under multiplication of
complex numbers.
15. Show that the set of Q” of all positive rational numbers forms an abelian group under the operation
defined by

asb= 5° V abEQ’.

16. Show that the set of all positive rational numbers under the composition defined by a * 6 = ab

forms an infinite abelian group.


i7. Let m be an arbitrary but a fixed non-zero integer. Then show that the set G = {ma: a © Z} of all
integral multiple of m, is an infinite abelian group w.r.t. ordinary addition of integers.
18. Let Q* denotes the set of all rational numbers except 1, then show that Q* forms an infinite abelian
group under the operation o defined by aob=a+b-abforalla,b€& Q*.
19. (a) Does the set of all integers form group under the operation a * b=a+5+1.
(5) Examine if Z, the set of integers, is a group under the binary operation o givenaob=a+5h— 1.
(c) Examine if Z, the set of integers, is a group under the binary operation o given by aob=a+bh+2 W
abEeZ.
20. Let a set G = {e, a, b, c} under a composition defined as below by given composition table

elalbic
elela|lble
aljale|lc{6b
b clela
elc|lbJlale

Is G a group ? If it is, whether abelian ?


21. Show that the set G = { 2”: nis an integer } forms an infinite abelian group under multiplication.

22, Let a be an arbitrary but a fixed non-zero integer. Show that the set G = { a” :ne€ Z} ofall integral
powers of a, form an infinite abelian group w.r.t. multiplication.

Scanned with CamScanner


348 SPECTRUM DISCRETE MATHEMATICy
——a,

23. LetG= {a+b J2:4,b € Q}. Show that G is an infinite abelian group w.r.t. addition composition
defined by, for all a+b V2, c+d 42 E G,(a+b¥2)+(c+d V2) =(a+c)+ (b+ ad) V2,
where a, b,c,
d EQ.

24. Show that the set G = {x : x is a rational number, 0 <x < 1} does not form a group w.r.t. ordinary
multiplication of rational numbers.
25. Let X be any non-empty set, let P(X) denotes the power set of X. Show that
(i) P(X) does not form a group under the operation M, intersection of sets.
(ii) P(X) does not form a group under the operation U, union of sets.

26. Show that the set of all 2 x 2 non-singular matrices over the set of real numbers R, forms an infinite
non-abelian group under the composition of matrix multiplication.

x
27. Show that the set G = | ‘ »x,yeER,st.x+y#0} does not form a group under the operation
x y

of matrix multiplication.
b
28. Show that the set G = i( A where a, b,c,dERst. ad—bc= | forms a non-abelian group,
c

29. Show that the set of all 2 x 2 matrices of the form | a | where a, 6 are real numbers, not both
al a

zero simultaneously, forms an infinite abelian group.

ee
x . . ,
30. Prove that all matrices of the form 2 | where x is a non-zero real, is a group with respect to
x x

matrix multiplication.
31. Show that the following set with the given binary operation is a group. Find the identity element and
the inverse of each element and check whether it is abelian group or not.
(a) LetS = {(a, 6): a,b ER, s.t. a + O}, the binary operation « on S is defined as
(a, b) x (c, d)=(ae,be+ d).
(b) Let S = {(a, b): a, b EI st. either a # 0 or b # 0}, the binary operation ~ on S is defined as
(a, b)« (c, d)=(ac-bd,ad+bec)

0
(c) Let G = (6 | where a # 0, be a real number}, the binary operation defined as usual

multiplication of matrices.
1 ab
(d) LetG = {}0 1 c|, where a, 5, ¢ are real numbers}, the binary operation on G as the usual
aS
a

00 1
multiplication of matrices.
es

Scanned with CamScanner


en 21 eee A 2 A

youssBRAIC STR
UCTUR ES AND M
orpiisy
349
3. Show that the set G = 60.1.9
=

ui. Show that the set £1 > 4,5} isa finite abelian group of order 6 under addition modulo 6.
G =
‘multiplication modulo 6". “> 9. 4, 5, 6} is a finite abelian group of order 6 under the composition
vy. Prove that the set of

:
of multiplica zSers which are less than 7: and¢
: co-prime
P
ime to 7 forms
fe aman 2dellan
abeli grou :
P
tion modulo n.

Snow that the set U(


a) =
- {x -x €& zZ S.t
the operation of multiplication maiuiot. ° = 1 i ' ae
————————————————
:
33. Show that the set G = {0.1
2.3} forms a group under addition modulo 4.

x. Let S be a non-empty set and nto maps from set A to A forms a group.
P(S) denote the power setofS. Then (P(X). A) is an abelion group under
the binary operation of symmet
ric difference A, given by AA B=(A\B)U(B* A) VA,BEP(S)
Show that the set of all rational
: numbers of the form —~ isa group under addition, where p and q
34 =
are .integers. ~

39. Show a that t me setS == {27 427735.


3° : where a. bare integers} under the binary operation
i defined as usual
multiplic ation of rational is an abelian
group
40. —
i el
a - all the elements of the permutation group (or symmetric group) Ss on four
elements 1, 2,

ANSWERS
ll. Not a group 12. Group 13. Nota group 19. (a) group (b) group (c) group
20. Abelian group

1 5
i. (a) (1, 0), [4.—2) not abetian group (4) (1,0). — 7 _ 8. . abelian group
a a 2 2 2 Z -
a~+h a~ +b

1 ol [1 o ‘1 0 0] fi -a@ acd]
(c) ie . abelian group (a4) 10 1 O},/0 l =e|. pot ubelion
0 O 0 Oo
10 O 1) jo oO 1}
12. Elementary Properties of a Group
Let< G, * > be a group under the operation *. Then G has the following elementary properties.
LUniqueness of identity element
The identity element of a group is unique.
Proof: if possible, suppose thate,. e: are two identity elements ofa group.
(Since e, is identity element) 1)
e,* &2 = ey
=H LY (Since e; is identity element) .-.(2)
also =e, FP
Thus ‘ =e , [From (1) and (2)]
"7 2
unique.
the identity element ofa group is

Scanned with CamScanner


SS
nd

SPECTRUM DISCRETs
350
ATHEy Rg

G
II. Uniqueness of inverse element
The inverse of each element of a group is unique.
Proof. Let ¢ be the identity element of the group (G, *) and a € G be an arbitrary element.
If possible, let b,, b, € G, be two inverses of a
a*b,=e=b*a (-. 5, is inverse of a)
3
~(l
and a* b,=e=b,*a (-- bp is inverse of a) .
++6(2
(Since e js
Now b, =bh,*e identity wd

= b, + (a* bp) [-- of (2)


= (b; * a) * bp (By ASSOCiativity ing
le A
ae

=e*b, [-: of (1)]


= by

by =b;
Hence each element of a group has unique inverse.
III. Cancellation laws hold in a group
For a, b, c € G, we have
ax b=a*c => b=c (Left cancellation law)

b*ea=c*a => b=c (Right cancellation lav)

Proof: Let a, b, c € G. Since a € G so a' € G such that


a'*a=e=a*a" Al)

Now suppose that a* b=a*c


> a'«(a*b)=a'*(a*c)
)
>
(a! * a)*b=(a' *a)*c, (By associative law inG
=> e*b=er*c

=> b=c.
a*b=a*ec => b=c

Similarly, we can prove that


b*ea=c*a => b=c.

IV. For everya EG, (a')' =a.

Proof: V¥ aE&G > a EG,

then aa! =¢e= aa

=> inverse of ais a”!


—_

Scanned with CamScanner


ES AND MORPHISM
351
gic STRUTT =
i
ee qa =e=aa-
Cain Age -l tga
inverse of 4 I
(a7!) | =a.
je.
the product
ersal law for inverse of
Rev Wa,beEG.
v.Re (a* by = bi«xa'

rsince a6 EG “ a*Fee _)
» cEG where c=a*b
> b'*a'EG
Also b,aEG 2 b',a'EG 2)
» deEG_ where d=b'*a".
[ *-" of (1) ]
Consider c*#d=(axb)*d
(Associative law in G)
=a*(b*d)
!
~ ax (b+ (6a") [ -- of (2)]
(By associativity in G)
=q+*((b*b')*a"'),

=qe(e#a')=a*a'=e.

cxd=e.
[.- of (2)]
Now consider d* c=(b' * a')*c
(Associative law in G)
= p'«(a'*c)
[-- of (2)]
=b'*(a'*(a*b))
(Associative law in G)
=p'« ((a!*a)*b)

=b'«(e*b)=b'*b=e
d*c=e
c*xd=e=d*c
> cl=d
> (a* by! = be a.
que solution in G.
be any elements. Then the equat ions a* x= bandy * a= b have uni
VL Ifa, b E G
has a solution in G.
Proof. We first prove that the equation a * x = b
Since a EG G, so 9 a! & G such that (1)
ata'=e=aq'*a
Sine a’ bEG so a'*bEG.
Take xagiedb “ xEG

Scanned with CamScanner


352 SPECTRUM DISCRETE MATHEMATIcg
ha

Now a*x =a*(q'*b)


=(aea')#b (Associative lawin G)
=e*b
=5
the equation a*x=b5 hasasolutioninG.
Uniqueness. Let x, x2 be two solutions of the equation a * x = b inG.
a*x,=b and a*x,=b
> a¥x;=a*x, > X, =X (By left cancellation law ina group)
Hence the equation a*x=6 has a unique solution in G.
Similarly, we can prove that the equation y * a= b has a unique solution in G.
(Solution is 6 * a!)
VIL. Left identity and right identity are the same in a group
Let e and e’ be the left identity and right identity in the group (G, *). Then
eve’ =e’ (Here ¢ is the left identity)
also exe’ —e (Here e’ is the right identity)
Thus e’ =z,
Hence left identity and right identity in a group are same.
VIII. Left inverse and right inverse of every element in a group is same
Let e be the identity of the group (G, *) and let 6 and c be the left and right inverse of the element a EG
respectively. Then
b*a=e and a*c=e
Now b=b*e
=b«(a*c)=(b*a)*c
=e*c
=<¢.
Hence the left inverse and the right inverse of every element in a group is same.
1.2.1. Theorem : Let G be a non-empty set together with a binary operation * such that closure propery
and associative law hold in G. Then the existence of left identity and left inverse in G implies the existence
of same right identity and same right inverse in G.
1.2.2. Definition ofa Group based on Left axioms
Let G be a non-empty set together with a binary operation * defined on it, then the algebraic structurt
<G, *> is a group if it satisfies the following axioms
() a*bEG,VabEG (Closure Property).

(ii) (a+b)*c=as(b*c),VabceEG (Associative Property)

Scanned with CamScanner


353
yGEBRAIC STRUCTURES AND MORPHISM
(i) Jan element e € G such that
exa=a ,VWaEeG (Existence of left identity)
(iv) For all a€ G, A an element 4 € G such that

b*a =e. (Existence of left inverse)

123. Definition ofa Group based on Right axioms


tion * defined on it, then the algebraic structure
Let G be a non-empty set together with a binary opera
¢.«> isa group if it satisfies the following axioms
(9 at*bEG,VabEG (Closure Property)
(i) (a*b)*c=a*(b*c), Va,b,cEG (Associative property)
(iii) 3 an element e © G such that

a*xe=a,VaEG (Existence of right identity)

(v) Foralla € G, 3 an element d € G such that


a*xb=e (Existence of right inverse)

vote: If< G, * > be an algebraic system in which closure property, associative property holds. Then G
xed not be a group if left identity and right inverse exist in G (or right identity and left inverse exist in G).

For example : Let G be any set containing atleast two elements.

Define a binary operation * on G bya*5=65,Va,bEG.


Clearly, closure property, associative law holds in G.
Also the element e € G be the left identity in G fore* a=a, Va EG.

Moreover,a*e=e => e is the right inverse of a.


But <G, +> is not a group, for if a, b be two distinct elements of G then a*b=balsob* b=bsoa
*b=b*b = a=b (by right cancellation law), a contradiction.

12.4. Theorem. A semi-group in which both the equations a x = b andy a = b have a unique solution, is a
Zoup. Prove it.
(It is also called a definition of a group)
Or

LetG be a set with binary operation which is associative. Assume that for all elements a and 0 in G,
te equations a x = 6 and y a= b have unique solution in G, then prove that G is a group
Note : If in a semi-group G only one of the equation has a solution. Then G may not be a group.

For example. Consider the algebraic system <G, -> defined by a-b=6, V a,b EG.
i)
Here <G, -> is a semi-group in which only ax = b has a solution in G. But G is not a group.
(Already proved)

Scanned with CamScanner


’ Ms N
me *" )
SPECTRUM DISCRETE M
354
ogd
aWs sthol
ion av :
fin ite se mi -g ro up js a group! ff both the cancellat
t any
1.2.5. Theorem, Prove tha
for finite sets)
(It is also called a definition o fa group, but
an , not be a group.
Note : If one cancellation law holds, then the system ma)

set conta ining at leas t two ele men ts. Define a binary operation y
For example : Let G be any oH
axb=b,V abeG.
Clearly closed property and associative law holds.
ie. Gis asemi group.
Here V a,b,c EG, a*b=b and axc=e.
axb=asc > bec ie. leftcancellation law holds.
But G is not a group under *.
Here right cancellation law does not hold.
hold.
Remark 1. Ina semi-group, the cancellation laws may not
integers.
For example : Let S = Set of 2 X 2 matrices over
Now S is a semi-group under multiplication.
1 0 0 0 0 0
Here A = B= and C= are (any) three elements of S,
0 O 1 0O 0

0
AB =AC each =
0 0
But B #C.
Left cancellation law does not hold.
Similarly we can check right cancellation law does not hold.
Remark 2. There are semi-groups which are not groups but they satisfy cancellation laws.
For example : Consider N = Set of all natural numbers.
= {1, 2,3, 4,5, ....}
Under the operation of ordinary multiplication, N is closed and associative law holds.
ie. Nis asemi-group.
Also both the cancellation laws hold
ie Wa,bcEN
ab=ac > b=ec
and ba=ca > b=c.
But this system is not a group. (Here inverse of its elements do not exist)
Note: <N, - >is an infinite semi-group with cancellation laws hold.
<N, ‘> is nota group. |
Thus the above theorem cannot be generalized to infinite semi-groups
elds

Scanned with CamScanner


—— HERE.
S —
eUCTURES AND MORPHEM ass
a>
a ofa group.
"pet
a ag rleG&s group under the composition multiplication,
eee

> ~

saoge (or WT) 27 8S 27 Sad (c-e)-2as o* and son.


— Th

yee.

| say “too!
ce OB is denoted
enote as 2a ~ andand (a (a7! °-a™")-a7"
a74).27! bybya a3 and soon.
| See
g-a”! =e, therefore
we denote a =e.
sxe
a" =eae....... a
Thess
nm times
a” = (a!) = (ayaa) Wn EN
—_—vY
numes
tes a =(a')” V nEN.Here ao” E€G Wn El
W<G, +> is a group, then
a+a=2a ie. ais operated
two times.
n@=a+a+....+ a(ntimes)
0a=0
(On left side 0 is zero integer. On right side O is zero element of the group)
(-l)a =-a (additive inverse of a)
(-n)a =n(-—a) V n€l
Here na ©€GVneEl.
Remark: 1f<G, ->isa gro
up.
Y a€G, V m,n &l, we can prove by the method of mathematical induction
() a™ g™ = qgmtn

(ii) (a™)" =qmn

(iii) a"= (a“!y" = (a")7!.

' Here (i) and (ii) are also called Laws of indices in a group.
3. Order of an Element
Definiti
Mtion :Let a be an element of a group G. If there exists a win integer m such
positive that a” = e, then a is
*dto have finite o
and the smallest such positive integer n with this property such that a” = e is called
© ord
*t of @ and isTder,
denoted by O(a).
wig does not exist a positive integer n such that a"
=e, then ais said to have infinite order or the
0€$ not exist or the order
is zero.

é: ‘ 1
\

Scanned with CamScanner


356 SPECTRUM DISCRETE MATHEMAT
Tsig

~—
Note : (i) Ina group G, order of identity element is always 1.
ie. O(e)=1.
(ii) In case of additive notation the above terminology is stated as :
Let a be an element ofa group G. If there exists a positive integer n such that 7 a = 0, then a is saiq to
have finite order, and the smallest such positive integer n with this property such that na = 0 is called the
order of a, and is denoted by O(a). 1
If there does not exist a positive integer 1 such that n a= 0, then a is said to have infinite order. OF the
order does not exist or the order is zero.
Examples : 1. In a group <Q — {0}, ->, we have O(1) = 1 (as 1'= 1), and O(-1) = 2 (as (-1)° = 1), By
the order of any other element does not exist.
2. In the group ({0, 1, 2, 3, 4, 5}, +6) the order of every element exists.
e.g. O(0)= 1, O(1) = 6, O(2) = 3, O(3) = 2, O(4) = 3, O(5S) = 6.
Note : The order of an element in an infinite group may or may not exist. But there do exist infinite groups
in which order of every element exists.
Example : Give an example of an infinite group each element of which has a finite order. Co
Sol. Let G={z;z€Cand z” =1 for some positive integer 1}
G = Union of all n, nth roots of unity, where n EN.

Under the composition of multiplication of complex numbers, G is an infinite group with identity
element z= 1.
The order of each element is » which is finite, where n & N.
Then G is an infinite group in which order of every element exists.
1.3.1. Theorem (A) Ina finite group the order of every element exists.
Theorem (ii) If G is a finite group of order n then show that for any a © G, J some positive integer r,
1<rsn,suchthat a’ =e.
1.3.2. Theorem. Let G be a group and a € G be of order m. Prove that

(i) a® =e, a, a’,....., a”! are all different.

(ii) WV n€&l, a” is equal to some one from the above list. Pr

1.3.3. Theorem. Let G be a finite group and let a € G be an element of order n. Then a” =e iff nisa
divisor of m.
Proof. Firstly, let n be a divisor of mie. n|m, where O(a) =n.
there exists a positive integer g such that
m=nq
* O(a) ="
Now a” = q"% = (a")4 = ef =e, (a) |

Scanned with CamScanner


————

& -
357
c gTRUCTURES AND MORPHISM

or a a" e, where O(a)=n.


verse
conte j theorem
BS vision algorithm
ength whereg, rG@I and Osr<n
m=
-g’ = ef - a” = a’
= aI. aq” = (a")?
aa = grit
ee

a’ =e where OSr<an
2?
the least positive integer such that a” =e.
ih is not possible, because O(a) = n and nis
hic
if r=0
Above result holds only

je when m=nq+O0=nq
ie, when 7 is a divisor of m.
ky
=
O(a") Gb ’
_ mm
where KEN.
Let G be a group and leta EG be order m. Then
34. Theorem.

oetlf O(a)=m, then O(at) =m iff (m, k= 1.


By above theorem,
ky m

o@’) (mm, k)

k ;iff ——mo— os iff _


O
em
=
OW
=1 ie
mah
(m,A)=1.

2.1f O(a) =p,P where pP is a Pp prime number, then


(. (Pp, k)=1)
O(a‘) =P, for all k= 1,2, aeeee »p—l.
:

135.Theorem. Let a, b and x be any elements of a group G. Then prove that

= O(a) (ii) (xa x)* = x lak x, forallkEI


(i) O(a!)

il) O(@)= OC ax) (iv) O(ab)=0 (ba)


Proof, (i) Let O (a) =m and O(a!) =n

> m, nare the least +ve integers such that


a™ =e and (a7')" =e

=e! =e, but O(a 'y=n


=|

Now (artym = go™ = (a)!


1)
"im
Bain, a= (g-!y-™ <q"! y"y! =e“! =e, but O(a)=m
Apai =
.Q)
m | n

Bite

Scanned with CamScanner


358 SPECTRUM DISCRETE MATHEMAT
1G
From (1) and (2), we get

mn

O{a') =OXa).
(ii) We shall prove by induction that

(x axyt = x71 gt x, forall KEI


when k= 1, LHS.= (x7! ax)! = x! a! x =RHS.
the result
is true for n= 1.
Let the result holds for k= m, where
m is a positive integer.

ee (x7) ax)” =x 'a™ x istme.

Now (x! axy™! = tax)" (7! ax) = (x a x(x ax) = a (xx )ax
=x) g™ eax =x at! x,
“. The result is true for k= m + | also.
Hence
the result is true for all positive integers.
Also when
& = 0, then

LES. = (x7! ax)? =e=x ex=x ax


=RHS.
Now, let k=—m, where m is a positive integer.

as (x ax)t = (27! oxy = (x axy"¥! = fc! a” xy!


att lyt

[By Reversal law (a5)! = 6 'a7!]


=x gt x. te Gey ter]

-. The result is true for zero and negative integers also. Hence the result is proved for all integers.

(iit) Let O¢x7! ax) =m and Ofa)=n

Now (cl ax)"=x a"x =x! ex=x'x =e

But O(x7! ax)=m

min Al)

Again -- O(x!ax)=m
nb

= (x ax)" =e

Scanned with CamScanner


——— en cr

gsnntC STRUCTURES AND MORPHISM 359

a op, oe oe ol
for

> x a"x =e=x x

> rla™x =x ex
: a” =e [Using left and right cancellation laws]
aut Q{a) =n
aim (2)
From (1) and (2), we get
m=n.

2 O(x tax) =O(a).


(ix) From (iii) we have
O(a) = O(x ax), VaxEG
Replacing a by a b and x by a, we get
0 (ab) = O(a!(a b) a) = O(a! aba)
= O(e b a)=O (ba)
Aliter: Since ab= eab= (b' b)ab=5"' (bab
O(a b) = O(b7! (6 a) b)
> O(a b) = O(b a). [Using (ii’]
ieeark : If a, 6 € G be elements of finite order of a group G, then O({a5) may not be finite and if it is
++ even then it need not be equal to O(a) O(6).
fumple (i). Let G = {f; {: R > R is one-one and onto function} be a group under the operation of
=oousition of functions.
Let fi, f2 © G be two elements such that f}(x) =—- x and f(x) = I-x. Then O(f;) = 2 = O(/5).
2 O(f| fo) does not exist.

SFr f(x) = AUG) = ACs) =-Cd=x = OY) =2


ad f(x) = fo(f2@) = A(l-x) = 1-(-)=x
2 Op) =2
But fi fax) = AU2()) = AC-x) =-(-x)=-1+x
Also, (f, f2)"(x) 4x, Vn EN.
O(f, f2) does not exist.
a b
tumplen. Let = (| ] i 6 deRsuchthatad—be 0
c a
le. Gis a group of all non-singular 2 x 2 matrices under the operation of multiplication of matrices.

Let a=) 1 ana B=| | be wo element oF


QO -l
Prove that O(A) =O(B)=2 but O(AB) does not exist.

Scanned with CamScanner


360 SPECTRUM DISCRETE MATHEMATicg

te ens
——,

Sol. Here aa Te Tl 1 > O(A)=2

a
Ea AL) +
e ee
BAAR ae enema ee tenn ewes ee eseeeseeeseeeeeesanecaeanes

Onn eee ee een nena nen emeeenneesensenereseneeneenseennas

O (AB) does not exist.


Example (iif). Let G = {+ 1, + i, + j, = A} be the group of Quaternions under multiplication. Then
O(i/)=4=O({), but O(i7) = O(k) = 4.
[se it = 74 =k4 =1 and ij =H
Thus O(i /) # O(/) O(/).
Example (iv). Let G = {(0, 1, 2, 3, 4, 5), +.} be a group under the composition of multiplication modulo 6.
Here O(2) =3 and O(4) =3, but O(2 +44) = 0(0) =1
Therefore, O(2 +, 4) # O(2) O(4).
Example (v). Let S; = {#, (12), (13), (23), (123), (132)} be the symmetric group on three elements 1, 2 and 3.
Let a =(12) and b=(123). Then
O(a) =2, O(b) =3. [ a? =i and } =i]
Now ab =(12) (123) =(13) and 6 a= (123) (12) = (23) ly
But O(a b) = O((13)) = 2 # O(a)-O(6).
1.3.6. Theorem. If a, 6 be any two elements ofa group G such that a 6 = b a and (O(a), O(b)) = 1. Thea
prove that O(a b) = O(a) O(6).
Proof. Let O(a) = m and O(b)=n, where (m, n) = 1.
Let O(a b) =k. To show that k= mn, where ab=ba.

Now e = (aby = a™* pt* = a™* (pny

ie. a"* =e, but O(a)=m


> mink, but (mn=!1
mk. (l)

Scanned with CamScanner


«wGEBRAIC STRUCTURES AND MORPHISM

Sinilarly, e=(aby™* = g™*pm* = (a” ¥ aad

ef b™r = ebm = pmt


tl

te b™* =e but O(b)=n


= ni[mk, but (mn)=1 {2)
~~ om[k.
From (1) and (2), we get ..A{3)
m\|k and n|[k => [m,n]|k
But [m, n]-(m, n) =mn
at [m, n]-1 =mn
- From (3), we have
(4)
mn\|k.
Again (a b)"" = g™" B™" = (q™)" (b")™ = e” e™ =ee=e
but O(ab) =k
k|mn
*. from (4) and (5), we get
k=mn
ie. O (a 6) = O(a)-O(8).
13.7. Definition : Idempotent element.
J . . - ,
In a semi-group G, an element a is called an idempotent element if a~ =a.

| ILLUSTRATIVE EXAMPLES]
Example 1. Show that if G is a group then a € G is an idempotent if and only if a = e, the identity ofG
SoL Given G is a group.
Let a & G is an idempotent element
a
2 =a
>

> aa=ae
a =é. . ;
=>
es [Using left cancellation law)
Conversely let a= e, the identity of G
aa=ae
= é e = ¢ = a

[v @= e]
2
> a” =a

= ais an idempotent element.

Scanned with CamScanner


SPECTRUM DISCRETE M
362 ATHpy,
A
a € G. Show that Gis sbelim, =ST
Example 2, Let G be a group such that a? =e, forall
Or
its own inverse is an abelian group.
Show that a group in which every element is
Or
element, is of order 2, show that the group js ae}
_ Ifeach element of a group, except the identity
= ab#e. an,
e a # ¢, b#e
Sol. Let a, b & G be any two elements, wher
2 (or O(a) = 2, 0
a” =e and b* 2 =e (6)s,
Le. every e ic 3
- ~
> asa and b= (d.e, every element is IS OWniny
Also a, bEG > abEG (By Closure Prope
(ab)? =e (or O(a)

> (aby! =ab


But (ab)! = ba"!
ba! =ab
« pole .
[~ 6 = and a7! 24
> ba=ab

G is abelian group.
a, b € G. Prove that if b # e, then O(b)=3)
Example 3. If in a group G, a> =e and a bay! = b? forall

Sol. Now b? =aba™ --()

b4 =(ab a7!)

=aba! [ve (tax =xtaty

= a(aba™!)a! [Using(1)

=a* ba”

b8 = (a* bay?

= q? hb? a? = a*(ab a”) a” (Using (If

=? ba?

66 = (aba)? =a ba
(Using
a(ab aja?

=a'ba*

Scanned with CamScanner


NS EE eSa

- Mid
qurcstRuCtuns. AND MORPHISM

iw p? =a ba
= ebe!=b {4 “cet
pt

> bib! =bhe

(Hy left cancellation law}


° pi =e,
O(b) must divide 31. But 31 is a prime number,
O(b)
= 31.

le 4 IfG is an abel ian grou p, then (a b)" = a” hb”, holds forall a,b & Gand for all nf
gsamp
n group,
sal Given G is an abelia

a"b" by Mathematical Induction,


Let 4,6 EG, We shall prove the result. (a 6)" =
if on =9, then (ab Y=c=ee=a’ hp

the result is true for n = 0,


i on=i,then (ab)!=ab=a'
+, the result is true for = 1,

Suppose that the result is true for n= k 2 1,

(aby edb
Consider (ab) he =(ab)‘ (ab)=(a b) (a
b)
> = ( (ai bf ) a) b, by associativity in G

= (ak (a4 a)) b, by associativity in G


= (a‘ (ab' )) b, since G is abelian

= ((a‘ a)b*) b, by associativity


= (ak ok) b
=a'*!(p*b) by associativity
a ght! phtl

the result is true forn =k + |, if it is true for n= k,


But we have already proved the result for n= 1.
the result is true for every positive integer n.
When n is a negative integer
Let_n=~— m for some positive integer m.

Scanned with CamScanner


364
tig

Then (ab)"=(ad)™
=((ab)")" . ; a
4 since misa positive integer
=(a7b") ,
= (6” a”™ y! , since G is abelian

=(a"
=q" Bb"
(om yt = am 8
Hence (ab) =a"b",V n€l.
Example 5. Show that the equation ya y=a"! is solvable for y in a group G if and only if g ig the cube
of some element in G.

Sol. Firstly, let equation ya y= a 1 issolvable inG

= 3b€Gsuchthat bab =a!


-l
> bbaba=a'a=e >= baba=b 'e=b > babab=e
> bababa=ea=a

=> (b a)? = a, where ¢ is identity element of G


a is cube of some element in G.

Conversely : Let a= m° for some elementm € G

Now y ay = (m*)*a m for y= m7


oe npctetn <9? a ont

> y? ay =a"! is satisfied fory= m "EG

equation y? ay= a”! is solvable for yina group G. . nfGe


wh er e is the identity of G
Example 6. If in a group G, xy? =y xand yx’ =x y, then show that x=) = ¢
So. sx) =y x mxeypxy? a
ee xy’ *: i]

Peaxyxy?ax~ysy ayryry? |
yay xyxy! fuse (Il

Now yx =x yp @ pxrexyxyry'!
=> veylry xyxy! a

> vryayplxyxype
By (1) and (2), we obtain,
veyxy lay lx xyr @ yryrexyyyry
> yrpxaxyyxyx=y xyxpry
= (yxy =(ry)
Interchanging x and y in (3), we get,
(x »? = (yx)

Scanned with CamScanner


1) re .

yceBRAIC STRUCTURES AND MORPHISM


‘ ;
Now (3) and (4) imply
(ey¥ = (yxy = Cy xr (yx) ~ Oy) (yx)
2 e=xyx ax?=y
futher xy? e yor @ xx Ze yx ry
> xleyy! > ye
Lastly yx" =xy xp exra=xve > xe
Hence the result.
tumple 7. IfG is an abelian group then show that fora,6 €G
(i) a and 67! commute ap a! and
6 commute (in aand 47! commute

wl - Gis abelian group


So ab=baV abEG

() Here ab=ba => (ab)! = (ba)!

2 b'at=a'b! & a! and 67! commute

(i) Here ab=ba => a! (a é) = a! (6a)

= (@!ab=(a'b)a = eb=(a'da
=> b=(a'bya = b(a'a)= (adda
2 (6a')a=(a' ba = ba'=a'b (by nght cancellation law)

= a! and dcommute
(ii) Here ab=ba = (ab)h@! =(ba)h"!
> a(bo"')
= b(ab"') = ae~ d(ab')
> a=b(ab"') = (66 ')a = 6(@h"') = &(67'
a) = (ab!)
> b!'a=aab"! (by teft cancellation law)

aand 67! commute.


tumple 8. Find order of each element of group {(0, Ll, 2, 3,4, 5), +6}, Le. composition is addition modulo 6.
SL Let (G. +5) bea group where G = {0, 1, 2.3.4, 5} with composition as addition modulo 6
Here e¢=0
We know oa = ¢, mr is least positive integer
> Olas
Now order of identity element of each group is 1
Oc) = OM I
Ta find O(1)
A= Leg 12, 31 = bee 2) = b +6275
HL= 1 +6 301) = 1493 74,501) 4 25
= 1 te 4h) = hte
Lee 5S = OF e
Ml)=t-pSC1I)@

Scanned with CamScanner


F
a
Oy
SS

we

366 SPECTRUM DISCRETE Man


# e form<6
6(1)=e and m(1) Rg
> O(l)=6
To find O(2)
2(2) =2 +62 =4 , 3(2) =2 +6 2(2) =2 +64 =0
3 (2)=e and m(2)#e
for m<3
=> O(2)=3
To find O(3)
2(3)=3
+63 =O=e > O(3)=2
To find O(4)
1(4) = 4, 2(4) =4 +64 =2, 3(4)=4 +6 2(4)=4+62=0=e

3 (4)=eand
m(4) #e form<3
=> O(4)=3
To find O(5)
1(5) = 5, 2(5) = 5 +65 = 4, 3 (5) =5 +6 2(5) =5+64=3

4(5) = 5 +6 3(5) =5 +6 3 =2, 5(5)=5 +6 4(5) =5t+62=1


6(5) =5 +6 5(5) =5 +6 1=0=e
6(5)=e and m(5) #e form<6
=> O(5)=6

EXERCISE 1.2
1. Show that a group of even order has an element of order 2.
2. Show that in a group of even order the number of elements whose order is 2 are odd.
3. Give an example of a group G and elements a, b € G such that O(a) and O(b) are finite, but O(2 bs
not finite.
. 6.
4. Let G be a group such that (a 5)” = ab", for three consecutive integers n and for all 4, bE
Show that G is abelian.

. If (ab)* =a? b*,VabEG,a group. Then show that G must be an abelian group.
OW

. Find the order of each element of the group {(0, 1, 2, 3, 4), +s}.
onKN

. Find the order of each element of the group of four 4th roots of unity.
. Find the order of each element of the following group

off }b eL dE df
under matrix multiplication.

Scanned with CamScanner


|
IC STR
y ceBte
g, consider the element A = s 1 in G, the group of 2 x 2 non-singular matrices having unit
de terminant ov er R. What is the order of A ?

9, show th at in a group of non-zero rational numbers under multiplication, only 1 and —1 are elements of
gnite ord er and all other elements are of infinite order.
2, provethat ab=ba.
_ Ifthe elements a, 5 and a } of a group are each of order
6)? # a? b?.
in Ss give an example of two elements a, b such that (a
f+) is a group such that 2 a= 0 for all a & G, then show G is an abelian group.
prove that every group of order 4 and less is commutative.
1s, Show that if G is a group of even order then it has at least one element different from identity element
which is its own inverse.

Anon-empty set G with a binary composition denoted multiplicatively is a group if and only if
a

(i) (ab)c=albc) V ab,cEG


(ii) Aan element e € G such thatae=G
(iii) For each a € G, J and element b € G such that a b= e
\1,Anon-empty set G with a binary composition denoted multiplicatively is a group if and only if
(i) (ab)c=albc) VabhcEG.
(ii) Janelement e € G suchthatea=a VaEG.
(iii) Foreaha &G, A 6&6 € Gsuchthatba=e
18. Prove that the equations ax = band y a=b have unique solutions in a group GV a,beEG.
9. If fora group G, a” 6” =ba for a,b €G.showthat a” b"~*,a"~-2 5", ab! have same order.
0. If a2 b = ba® =5 forall a,bEG (a semi group), then prove that G is abelian.
21. A non-empty set G is a group under multiplication if and only if
(i) (ab)c=a(bc) V ab,cEG.
(ii) The equations a x = b andy a= b have solutions in G for any a, b EG.
22. Given G be a finite semi group. Prove that 3 x € G such that x? =x.
Or
Prove a finite semi gorup has an idempotent element.

ANSWERS
6.1,5,5,5,5
7001) =| » 0-1)
= 2, 0()= 4, O(-D =4
8.
°D=1,D=2, O(+i)= 4, 9(+,)= 4 and OCGA) = 4
. OA) does not exist
WS {i.02), (13), (23), (123), (132)}

Scanned with CamScanner


368 SPECTRUM DISCRETE MATHEMATICg
ee

CONGRUENCE RELATIONS AND QUOTIENT STRUCTURES


1.4. Congruence Relation on Integers
Definition :
For a fixed positive integer n in Z . Define a relation (say) =, on Z as follows :
Foralla,b€Z, a=, b ifand only if n|(a—)d)
ie. a—b=nk forsome k EZ.
Remark : (1) This relation on Z is called congruence modulo n and a=,, 6 is also denoted by a = 5
(mod rt). However, if a — 6 is not divisible by n, then we write itas a # b (mod 7).
(2) Weread a = b (mod n) as ‘ais congruent to 5 modulo n’.
(3) Two integers a and b are congruent modulo a positive integer n, if and only if a and 6 leave the
same remainder when divided by n.
Proof. Let r,;, 2 be the remainders when a and 6 are respectively divided by , then
a=np+r, and b=nq+rz, forsomep,gEZ.
> a—b=(np+r)-(nq+r)
a—b=n(p—q)+(ri-72)
(r; —r2) is the remainder when a — b is divided by n.
Nowgiven d=b(modn)_ iff mja—b
ie. iff ry;—m=0
ie. iff rp=r
Hence the result.
Theorem : Congruence relation =,, on integers is an equivalence relation.
Proof. Since for all a € Z, we have a—a=0=n0 > a=, a forallaEGZ.
=, 1s reflexive.
Now, leta,5
€ Z suchthat a=, b. Thendk
€ Zsuch that

a-b=nk => b-a=(-kj)n


=> n|(b-a)
So b=,a -. =, is symmetric.
Finally , let a,b,c € Z suchthat a=, 5 and b=, c. Then 4G integersp and q such that
a-b=np and b-c=ngq.
On adding,
we get a—c=np+nq=n(p+q)wherep+qEZ
= n|(a-c)
so a=,c -. =,, is transitive.
Hence =, ie. congruence modulo nis an equivalence relation on Z. .

Scanned with CamScanner


palc STRUCTURES AND MORPHISM
—.

properties of Congruence Relation :


Leta, b, c. d© Zand n a fixed positive integer.
lfa=b(modn) and c=d(modn), then

() (at +¢)
= (+d) (mod n) (ii) ac=bd(modn).
(i) Since a = 6(mod
poof. n)
n) and c = d(mod
> n|(a—b) and n|c—d
> n\(a—b)+(c-—d) > n\(at+c)-(6b+d) => +d)n).
(a+c)=(b(mod
(ii) Also a = b(mod n) > nia—-b

and c = d(mod n) => nl\c—d


nla-b > n|c(a—b) + nica-cb
Also njc-d => n|b(c-d) > nibc—bd
Thus n|ca-—cb and n|bc—bd
> n\(ca—cb)+(bc-—bd) => n\jca—bd
> njac—bd > ac=bd(modn).
Cor. If a2 = 6 (mod n) then for any integer c, we have
(i) (atc) =(b+c) (mod n) (ii) ac=bc(modn).

Remark. From above, we find that addition, subtraction or multiplication to both sides of a congruence by
ninteger does not change the congruence. But this may not be true if we divide both sides of a congruence
byan integer.
for example : Consider the congruencies
24 = 12 (mod 4) ie. 6.4 = 62 (mod 4)
If we divide both sides of this congruence by 6, then we obtain 4 = 2 (mod 4) which is not true as
$1(4-2).Hence 4 = 2 (mod 4).
The next theorem will throw some light in this regard.
Theorem : Let a, b, c be integers and n be a positive integer nm.

(i) ab = ac (tod n) ifand only if = ¢{ mod “]


(a, n)
(i) If ab =ac (mod n) and (a, n)=1, then b = c (mod n).
Proof. (i) Let d= (a, n). Sincen>0,d #0.
Aintegers r and tsuch that (r,1)= land a=dr, n= dt.
Now ab=ac(modn)

> njab-ac => dt\drb-dre


> t{|rb-re ie. t\|r(6-c)

Butr and r are relatively prime * (7, =1

Scanned with CamScanner


x > wy
..
370
SpE
Ry, b,
t|b-c ie. b=c(mods
So
Le. =c (mod *)

Le. = {mee =I

(i) The proof of this follows from (/) as (a, n) = 1.

| ILLUSTRATIVE EXANp
Example 1. 26 = 2 (mod 12)
Sol. As = 12|26—2ie. 12/24
LES
26 = 2 (mod 12)
Example 2. Show that n is odd iff n = 1 (mod 2)
Sol. Firstly let 1 is odd
n=2k+1;kEZ > n—-1=2k
> 2|n-1
=> n= 1 (mod 2)
Conversely let 1 = 1 (mod 2)
2|m-—1 >= DkEZst.
n-1=2k
ie. n=2k+1 =~ nis odd.

Example 3. Show that for every prime p> 5 either p?—1 or

p? + Lis divisible by 10
Sol. Since p> Sand p isaprime, so p is odd _(1)

2| p?-1 and 2|p2+!


As 5ft p,wehave
p =1,2,3, or 4 (mod 5)

> p> =1 or 4 (mod 5) a


> p? & +1(mod5)
> 5| p*=-lor 5| p24!
From (1) and (2),

10 | pr-l or 10] p2+I.

Scanned with CamScanner


a -

é gTRUCTURES AND MORPHISM


A
ove hati is composite, then (n— 1)! = 0 (mod n) except for n= 4.
R .
ie

le 7°
is composite

) a l<asb<n

ee ie 1=6 # 0(mod 4)

tet a= b #2
a
Then n=ab=a.a>2
2 (n-1) 22a
in(n—1)!
> both a and 2 a occurs
2 a.2a|(n-1)!
a” | (n-1)!
>

> n\(n-1)!
> (n—1)! =0(mod n).
Lastly let azb

Then l<a<b <n

: l<a<b sn-1
= bothaand boccur in(n—1)!

= ab\(n— 1)!
° (n—1)! = 0 (modn)
bample 5. Prove that an integer is divisible by 3 iff its num of digits is divisible by 3

SL Let N=@,10"+a,_, m-! 107! +....+107 ay +104, +a, be decimal expansion of an integer N
Consider integer polynomial

f(x) = > a, x!
k=0
Then
N =/(10)
Now 10 = 1 (mod 3)
. (10) = £(1) (mod 3)

° N= Sa, (mod3)
2) =O
Thus 3|N

ift 3| da, i

k=0

ye

Scanned with CamScanner


S
MATHEMATIC
SPECTRUM DISCRETE
=
Example 6. For any prime p, prove that (a+b)? = a? +b? (mod p)
Sol, We have ,

(a+b)? =a?+ [PJortoe[2)at- b* 4... { jeer +bhP


p-

Since A(’) for lsrsp-1


r

so @ = 0(modp) for 1s rsp-]

Thus (a+b)? = a? +b? (mod p).

Example 7. If a" =| (mod m) for some positive integer A then


(a, m) =]

Sol. Let a" =1(modm)


Then a" =|+mk, kEZ
> a"-mk =1

> gat! 4m(-k) =1


> (a, m) =1

Example 8, For n 2 1, using congruence, show that 43 |6”"*2


+72n4+]
Sol. We have
ott? 4 q2nel = 6" 67 $7297

67.36 +49" .7
Il

6" .36 + 6" .7(mod 43)

6” .43
= 0 (mod 43)
“ 43 | ott? 4 72nel

Example 9, Show that 2!5 +14" + 1 is divisible by 1)


Sol. We have
2' =5 (mod II)
(24)? = 5? = 3 (mod 11)
le 28 =3 (mod 11)

Scanned with CamScanner


ee
a
| w ~

a3 =
ee AND MORPHISM

we ull (1) and (2)


g!2 = 15 = 4 (mod 11)
312.23 = 4-2? (mod 11)
a
2!5 =32 = — I (mod 11)
cite 14 =3 (mod 11)
I

1440 = 34 (mod 11)

8 32 = — 2(mod 11)
> 34 = — 18 = 4(mo11)
d
> 11)
38 = 16 =5 (mod
2 11)
3!0 = 45 = 1 (mod
(4)
Thus 1440= 3° = 1 (mod 11)
Multiply (3) and (4)
315.1440 = — 1 (mod 11)
ie 25-1499 +1 = 0 (mod 11)
> 2!5.147° +1 is divisible by 11.
by 12.
frample 10. Find the remainder when 1! +2! +3 YF wxaaanenhe + 100 ! is divided
«dl. Form 2 4, we have
n! = 0 (mod 12)
W+2!4+31+
4 t. +100! = 1! +2! +3! (mod 12)
= 9 (mod 12)
The remainder = 9.

ttample 11. Find the remainder when 444444 js divided by 9.


Sol. We have 4444 =9-493 +7

= 7 (mod 9)
= —2 (mod 9)

444g = (2) (mod 9)


As —~§ = 1 (mod 9)
44gae gs (-2) 8"! (mod 9)
= (—8)'484 | (-2) (mod 9)
= | .(—2) (mod 9)
|
27 (mod 9)

Tis the remainder,

Scanned with CamScanner


~<a

374 SPECTRUM DISCRETE


Marup. tg

Example 12. Verify that 27 +1 is divisible by 641..


5

5
Sol. We have 2? 41 = 232 4)
Now 2!! = 2048= 125 (mod 641)
> 27 = (125)? = 241 (mod 641)
2'1 522 = 125 x 241 (mod 641)
=> —2 (mod5641)
233 = 30= 12
Since (2,641) =1

” 232 =_1 (mod 641)


Jett al

5
= 27 +1 = 0(mod 641)

ie. 641 | 2? 41
Example 13. Find the remainder when
(a) 2° is divided by 7 (b) 53'°3 + 103%? is divided by 39.
SoL We have
259 = 23642

= gié . 92

= g'6.4

Since 8 = 1 (mod 7)
= g'© = 1 (mod7)
Thus 259 =4 (mod 7)
= 4 isthe remainder when 2° is divided by 7.
(b) We have
53 = 14 (mod 39) and 103 = — 14 (mod 39)

53!4 10353 = 14! + (-14)*? (mod 39)


= 14° (14% -1) (mod 39)
= 14°3(19675 -1) (mod 39)
Since 196 = J (mod 39)

19675 = I (mod 39)


Thus 53!92 41035? = 14°3(1-1) (mod 39)
= 0 (mod 39)
tet

>» 39divides 53! +103.


ete

Scanned with CamScanner


a0
ceBRAIC STRUCTURES AND MORPHISM
v

EXEKCISE 1.3.
I. prove that if
c> 0 and a = b(mod m), then ac = be (mod mc)

Give an example to show that


yy»

m) and 6 # 0 (mod m)
(i) If ab = 0 (mod m), thena # 0 (mod

(ii) a? = b* (mod m) does not imply a = 6 (mod m)

3. m) and
Ifx= a(mod n) then a = b (mod g)
x = 6 (mod

where g =(m, n).

4, Find the remainder when

() 34° isdivided by23 (if) 4899 and 11173 +333!!! are divided by 7.
s, Showthat 277-1 is divisible by 233.
6. Using congruences, prove the following, form 2 1,

@. 7" 4o"" (ii) = -13]37*7 +427"! (iit) 27;2°7*' +5"°°

ANSWEKS
2 () a=2,b=3andm=6 (if) a=3,b=2and m=5
4. (7) 2 (i) 6,0

15. Quotient Structures


Definition : (Quotient set)
A Quotient set is a set derived from another by an equivalence relation.
Definition : (Quotient Structure)

Let S be a structure, R, an equivalence relation. If the equivalence classes form a structure of the same
gecies as S under relations derived from passing to quotients, R is said to be compatible with the structure
2 §, and this structure on the equivalence classes of S is called the quotient structure, or the derived
Sucture, of S/R.
Definition : (Free Monoid)
In abstract algebra, the free monoid on a set is the monoid whose elements are all the finite sequence
“ stings) of zero or more elements from that set, with string concatenation as the monoid operation and
wh the unique sequence of zero elements, often called the empty string and denoted by ¢ or A, as the
‘tity element. The free monoid on a set A is usually denoted A*. The free semigroup on A is the
xdsemigroup of A* containing all elements except the empty string. It is usually denoted A’.

More generally, an abstract monoid (or semigroup) S is described as free if it is isomorphic to the free
Sonoid (or semigroup) on some set.

Scanned with CamScanner


376
SPROTRUM Die Rey

Definition : (Cyclic Group)


Definition ;

integral power ofa " » inte VK


ke. iPd € G, then d @ EG such that b= a" for some integer n, Mey
ais then called a generator of G.
Note. (’) If G is a cyclic group generated by a, we write Ils G = Sa
(i) If G is a group under addition, then G is called a eyelle group iPeach
vloment op RN
multiple ofa de, if b EG, then d = a tor some integer ‘an eet
(ii) A cyclic group is also called a Monie group.
(iv) UG = <a> be acyclic group of order a, then
G=leaa@ywcan bie, O(G) #0 (= n
. Ifaisa generator ofa cyclic group G, then a ‘is also a generator of G, for MWY Ye Gy
x=a"alsox= (a'Y", where n,—n & Z. Why
Examples of Cyclic groups
Example 1. Show that the multiplicative group {1, wy) uw} formed by the cube Fools Of unityj is
group. Wh
(\q

Sol. Let G be the group of cube roots of unity under multiplication,


G={ lee},
Here, | = @", therefore each element of G is an integral power of,
G is cyclic group generated by@ ie. Ge <a>,
Example 2, Show that the multiplicative group {1, — | vf, ~ 7} formed by the fourth roots of
tunlty iy
cyclic group.
Sol. G={1,~—1,i,-/} is a group under multiplication,
Since 1=(i)', -1l®?, -je75
Therefore each element ofG is an integral power of i,
G is acyclic group generated by i ie. Ge <i>,
Example 3, Show that the group Z of integers under addition is an infinite cyclic group
generated by |
Sol. Since each » & Z can be written as n= 1
im Z=<] >,
Also each n & Z can be written as» = (—
n) (-1) where—n € 7
Z=<-1> ie. Z has two generators | and ~L,
Example 4. Show that the group G = {(0, 1,
2, 3,4, 5}, +e} is a cyclic group under the operation qidi®
&
congruence modulo 6.
Sol. Since 1 = 1 (1)
2=1+612(1)

Scanned with CamScanner


qin SMUT A MO
|
Pye ld tele lead | a7
)
de Vly Madly bet)
Sed ty Ela tty btat =a
cy
(eT ETol tet tel tel 6
c,
, every Clemente OFG oan by
Wellton ay yp (1)
,
Gel® Le Gs aoyelle ktoUup,
ewe is also ponoratod by
ote
he Gs “As
. Aptoup may have more than ong (Can be orally proved)
Boheoratory,
jw & Prove that the five roots of unity
Horm i eyetic proup under inultiptlention,
ay, We Know that the a —ath rooty oF unit
Aah. y
eal |
a7 fy 4 fersnel

jou an abelian group under mullipleation


dre

Lot Gye

ye
4 1() eo?
SO Sr sdb
pe we : Xv
(heh eth glk ildy ) whore y= 22!
v wy ?

5
form an abelian group
Clearly Gy = < ¢'> ds a cyclic Group generated
by ¢'
As ? Ta (e')’, vo (e')’, el Vy (cy!

eg by
eee 2" COS Qe isin Qe Lok JQ) |
Theorem. Let G be a finite group of order 7, IfG contains
an element of order n, then G must be
cyclic,
Proof, Let a © G such that O (a) © n,
Let He {a's r ET} bea subgroup of G,
But O(a) =n > H™ {e, a, Grand Yusag>
ie, His a cyclic subgroup ofG generated by a.
Also. O (IH) =O (G)
» G=H=<q>,
he, G is acyelic group.
Note + In order to show that a finite group
is cyclic or not, Find the order of every
Contains an element Whose order is equa cleme nt of GWG
l to the order of the group, then the group
ment will be the generator of the cyclic group, must be cyclic and that
otherwise the group is not cyclic,
heorem 3 Every cyclic group is abelian,
“Of, Consider a cyclic group G generated by a. ie,
G™ <a>,
Let YEG be arbitrary elements.

ee
Scanned with CamScanner
374 SYECTRUM DISCHETE MATHEMATs
wy

‘xed and yed" for some integers n and m.


‘then syed dod Ma if =d" f =yx,
“, Gis an abelian group,
Hemark ; Whe converse of above theorem need not be true 1, An abelian group need not be cyclic,
Haample 6, Give two examples of abelian yroups which are not cyclic,
Bol. (i) Set, + be the group of rational numbers under addition,
Mis an abelian group which is nat cyclic,
m :
For, suppove that = 0) is 4 yenerator of Q),
n

‘Then every clement of Q should be an integral soultiple of ie

1
[At on EQ he any element,
a)
J m . ‘
[At sh << for some inteyer k,
n n

sake,
3
I
which is not possible as k, m are integers, where ax = is not.
3
Hence no clement can act as generator of Q,
1 0 -—| 9 i OQ -|
(if now lin A th~ C=
. | I 1 \, I . |0 “lt
‘The composition defined on G is usual multiplication of matrices,
‘The composition table is given below :
‘id A |B YC
i {J A BIC
AJA JI [CB
Bim LTC yt A
C\C |B IA TI

Since all the elements in the composition table are the clements of G. Therefore G is closed unde
multiplication, Also the multiplication of2 % 2 rmatrices is associative,
Also 1 is the identity clement ofG and
ana

A? = B’= C= Limplics that AW'= A, B ‘ep, Cl#c,


“inverse of each element exists in G,

Scanned with CamScanner


"e,

cSrnecs UBES AND MORPHinm


ose
y the entries ON both sides of the diagonal are identical ,
i

bY
Gisan abelian yroup of order 4,
pte pia (=f 4% OfAjy=O()=O(C)=2
Sights
pin OC" Le, 4.
G has po element whove order is equal to the order of G
sce Gis nota cyclic yroup.
. » prove that the order of a cyclic group is equal to order of its yenerator.
ey f tP °

LUSTRATIVE EXAMPLES
phe I Chow that the set of n-nth roots of unity forms a cyclic group under multiplication.
{ump Zara |
eG f n :04rsn~ | forms an abelian group
yh We knew that the set of n-nth roots of unity

zi mustiplication.
ial

lt «7e"”,
2a 3a Ae (n-Nja
4 Ge |e“. i pl geneous e

Charly Ga <> isa cyclic group of order n, generated by e’,


ince C'=(¢yY forall O2ran-I.
fumple2. Show that the group <G, X > is cyclic where G = {1, 2, 3,4, 5, 6}. How many generators are
.
here?
= O (G) = 6, then G will be
OL Firstly we are to show that there exists an element x € Gsuch that O (x)
‘ie group and x will be generator.
Here identity element ofG ise = |
Punher 3' = 3, 37 =3X,3~2
P= 3?X,3 = 2x3 6,34 = 3X3 = 6X3 -4
Poa X35 — 4X3 #5, 3° = 3° X,3 = 5X73 =!
¢ and 37 # I for a<6
tothat 3° = |=
“ 0(3)=60(G) so that3 is a generator
det = 4, 3 = 6, 37 =?, 3! = 3
We have 36 = l, 35 = 5, 34

§ 43
G ;can be expressed as (3°, 37, 3',3°.3 3}
le, (3! 32, 33 a 35 3°)

G is cyclic.

Scanned with CamScanner


380 SPECTRUM DISCRETE M ATHEMA
a T Icy

11! Part : To determine number of generators ?

Anelement 3” & G will be generator ofG if g.c.d. of1 and 6 is one


and we have (1, 6)= (5, 6) = |

so there are two generators 3 ; 3°,


Example 3. How many generators are there of cyclic group of order 10?
Sol. Let G be a cyclic group of order 10 generated by ‘a’
= 0(G)=
O(a ) 10
So G= {a,a2,a?,
a4, a, a°, a’, a8, a?, a! =e}

Anelement a” & G will be generator ofG if g.c. dof n and 10 is one


= (3, 10)= (7, 10) = (9, 10)=
and we have (1, 10)
3
So there are four generators ofG namely ala , a’ and a?.
A

EXERCISE 1.4
1. Show that G = {1, 2, 3, 4} forms a cyclic group under multiplication modulo 5.
2. If an abelian group of order 6 contains an element of order 3, then show that it must be cyclic group,
3. Let G be acyclic group of order 8. Find how many generators are there.
4. Define cyclic group. Determine which of the following are cyclic groups and find their generator,
() {1,-1,4-9 (iif) (Z,+) (iif) (R, +)

5. Evaluate 1%, 17> where 1 € Q, the additive group of rational numbers.

6. Show that group {1, 5, 7, 11} of intergers under multiplication moduls 12 is not a cyclic group.

ANSWEKS
3. 4 generators 4. (i) Cyclic (ii) Cyclic (iii) Not Cyclic 5. 1%=6, I>=-5

1.6. Substructures
Definition : In mathematical logic, an (induced) substructure or (induced) subalgebra is a structure whost
domain is a subset of that ofa bigger structure, and whose functions and relations are the traces of the
functions and relations of the bigger structure. Some examples of subalgebras are subgroups
submonoids, subrings, subfileds, subalgebras of algebras over a field, or induced subgraphs. Shifting
the point of view, the larger structure is called an extension or a superstructure of its substructure.
Definition : (Submonoids)
Let (G, *) be a monoid and H © G
Then (H, *) is called a submonoid of (G, *) Iff () H is closed under the operation *.
(ii) There exists an identity element e € H.

Scanned with CamScanner


C STRUCTURES AND MorpitisM 381
yctBRst
example Let Z = set of all integers with * as binary operation, which is monoid, Then
fur Sc ateteca t
z, ta‘ |OSk Sn, ‘a’ positive integer € Z} with binary operation * is a submonoid of (Z, ¢).

foition ¢ (Subgroups)
A non-empty subset H ofa group <G, *> is said to be a subgroup of G if <H, *> is itselfa group.

Here H is a group in itself under the same (or induced) operation of G.


Notice that every group G has atleast two subgroups, viz., {e} and G itself. These two are called
rnvial
or improper
:
subgroups. If H is a subgroup ofa group G such that H # {e} and H # G, then H is
subgroup of G.
alled a non-trivial or Proper
For example
s
(i) We know < R, + >, the set of all reals under addition is a group. Also <Q, + >, the set of rational
so a group andQcR
sal
<Q, +> is a subgroup of< R, + >.
(i) <Q’, :> is a group and Q' (the set of positive rationals) is a subset ofR (the set of real numbers).
\so <R, +> is a group.
But <Q’, -> is not a subgroup of <R, +>.
(iii) <Q— {0}, * > is a group.
<Q’, :> isa subgroup of <Q — {0}, ->
where Po.
(iv) Let G = {1,— 1, i, — #} and H = {1, — 1},
Here G is a group under usual multiplication of complex numbers and H is a subgroup of G.
(v) Let G be the set of all 2 x 2 non-singular matrices over complex numbers, then G is a group under

cob He SJ JL dh
ntrix multiplication.

Clearly H C G. Also H is a group in itself under the same operation of matrix multiplication.
Hence H is a subgroup of G.
Some Observations
|. If is a subgroup ofG and K is a subgroup of H, then K is a subgroup of G.
2. IfH and K are subgroups ofa group G and H C K, then H is a subgroup of K.
Notation: If H is a subgroup of G. Then we write itas H s G (or H <G).
Again H is a proper subgroup of G is denoted as
H<G(orH<G, H # G).
PROPERTIES OF SUBGROUPS
1. The identity element of a subgroup is same as the identity element of the group.
Proof. Let H be a subgroup ofa group G.
Let e and e’ be the identity elements of G and H respectively
Let a € H be any element
[~ e’ is the identity of H]
ae’ a

_
Scanned with CamScanner
= nS
a 7"

382 SPECTRUM DISCRETE MATHEMATICS


——ey

Also « a€ Hand HOG > aEG


oeeg [- ¢ is the identity of G]
we have ae=ae'
« ane’. [by left cancellation lay}
Hence the identity of a group and that ofa subgroup is the same.

Il. The inverse of any element of a subgroup is the same as the inverse of the element regarded
as the element of the group.
Proof. Let e be the identity element of G and H.
Let a € H be any element.
Since HE G - a&G,

Let 5 be the inverse of a in H and c be the inverse of ain G.


ba=e and ca=e
> ba=ca
> b=c. [by right cancellation law]
Hence the inverse of any element of a subgroup is same as the inverse of the same element regarded as
an element of the group.
Ill. The order of any element in a subgroup is the same as the order of the element regarded as
the element of the group.
Proof. Let 2 be the identity element of G and H.
Let a € H such that o(a)=n
> a*=e and a"“#e foreverym<n.
AlsogaEH > aEG andso a =eEG = oa)=n inG.
Hence order of any element in a subgroup is same as the order of the element regarded as the element
of the group.
IV. Subgroup of an abelian group is abelian.
Proof. Let H be a subgroup of an abelian group G
HCG.
Let a, b & H be any two elements

abEG * abba [<s G is abelist]


Va,bGEH wehave ahb=ba
Hence H is an abelien subgroup of G.
The converse of above result is false
ie., A subgroup may be abelian even ifG is not abelian.

Scanned with CamScanner


Tt SAS Pr.

yGEsRAlc STRUCTURES AND MORPHISM 343

yoark: (I) A non-abelian group may also have abelian subgroups.


por example : (i) The sets {1,- 1} and {1,— 1, i, — é} are abelian subgroups of the non-abelian group of
agemion Qs under multiplication.
" qi) The sets H = {1, (12)} and K = {1, (123), (132)} are abelian subgroups of the non-zbelian group
S;, the symmetric group on three numbers 1, 2, 3.
ma non-abelian group may also have a non-abelian subgroup.
for example = (/) SL (2, R) is a non-abelian subgroup of the non-abelian group GL (2, Rj under the
ecposition of multiplication of matrices.
(if) S; and A, are non-abelian subgroups of non-abelian group S, under the composinon of
exposite of mappings. [See Chapter on Permutation groups]
(RITERION FOR A SUBSET TO BE A SUBGROUP OF A GROUP
ynma I. A non-empty subset H ofa group G is a subgroup iff
() abEH, ¥V abEH G@) a@'EH, VY aEH.
poof. Necessary part. Suppose that a non-empty subset H of a group G is its subgroup
Therefore H itself forms a group.
The conditions (i) and (ii) hold in H, by the definition of a group
iafficient Part. Suppose that H is a non- empty subset of a group G such that the cond.tions (/) and (u)
J sold in H.
The closure property and the existence of inverse holds in H.
Now, let a@,6,cEG@H
abcEG [- HOG)
(a6)c=a(bec) since G ts group.
So, the associative law holds in H.
Since H is a non-empty subset of G, so 3 a € H.
By (ii), 2! EH
So, aa'GH
- aa@'EH, froom()
> e€H, where ¢ is identity element of G.
> = The identity element exists in H.
Therefore, H itself is a group
Thus H is a subgroup of G.
Note: The above two conditions can be combined to a single condition ah 'EH Yo. s EH.
Leama IL, A non empty subset H ofa group G is a subgroup iff! @b'EH VW a AEH.
st] Proof; Necessary Part : Suppose that a non empty subset H of a group G is its subgroup.
H itself forms a group.
° WheH >» b'EH.
Alo Va,bEH @ a, b'EH
2 ab'EH. (" His a group)

Scanned with CamScanner


384 SPECTRUM DISCRETE MATHEMA TIcy
omnes
Sufficient part : Suppose that His a non-empty subset ofa group G such that Va,b EH ab! eq,
‘To prove H is a subgroup of G.
Clearly associative law holds in H, as it holds in G and elements of H are also elements of G. Further
VaGH saa eaa'EH *eGH
ie. identity element exists in H.
Nowe,gGH » ea@' EH e a'EH
VaEH 2 a@'EN Le. inverses are there in H.
Also V a,b@H ab'EH (‘* inverses are there in H})
2 a(b'y'ENn
=> ab Eu.
closure property holds in H.
H is a group in itself under the operation of G,
Hence H is a subgroup of G.
Remark : In case of additive notation, the above two lemmas can be stated as :
A non-empty subset H of a group G is a subgroup iff
() a+bEH, Va,bEH
(ij) -aGH, VaeEH.

The above two conditions can be combined to a single condition a—~b GH, Va, b EH.
CRITERION FOR A FINITE SUBSET TO BE A SUBGROUP OF A GROUP
Lemma : A non-empty finite subset H ofa group is a subgroup of Giff abG@H,Va,b EH.
Proof : Necessary Part. Let a non-empty finite subset H of a group G be its subgroup.
H itself is a group
abEH, Va,bEH. (By closure property)
Sufficient Part. Suppose that H is a non-empty finite subset of a group G such that abG@H,Va,bEH.
The operation of multiplication is a binary operation on H.
Let a,bh,cEH > a,b,cEG, since HCG.

=> (ab)c=a(bc), since G isa group.


The associative law holds in H under multiplication.
Firstly we prove that cancellation laws hold in H.
Let a,b,c GH suchthat ab =ac.

Since aE H, so aEG.

a@'€G suchthat aa'=e=a'a

Scanned with CamScanner


a - 0 Ee TGS S55 _

(ctBRAIC STRUCTURES AND MORPIIISM 385


qi

nwab “ae
)
*
al(ab)ea'(ac

* (a"' ajb= (a! a)e


2 ebmec * be
ab =ac® b* ¢@,

similarly barca = b= ¢,
The cancellation laws hold in H.
H is a non-empty finite set with an associative binary operation in H and the cancellation laws
hold in H.
H itself is a group. (already proved)
Thus H is a subgroup of G.
Notice that the above theorem holds for only finite subsets ofa group.
gemark : In case of additive notation, the above lemma can be stated as
Anon-empty finite subset H ofa group G is a subgroup iff
a-bEH, Wa, bEH.
Theorem : (/) Prove that the intersection of two subgroups of a group is again a subgroup of the group.
proof. Let H and K be two sub groups of a group G.
H and K are subsets of G.
> HOOK GG.
Now letx,yEHONK
xyEH and xyEK
=> xy'GH and xy'E€K, sinceH, K are both subgroups of G.
> xy'EHNK
. xy'EHNK, V ~yEHNK
H MK is a subgroup of G.
) Theorem : (ii) The intersection of an arbitrary collection of subgroups of a group is again a subgroup of the
youp.
Sol Let
G be the group and {H, |4 © A} bea family of subgroups of G.

Take H = (7 H,
AEN

Since H, isasubgroup ofG, VA € A

e€ H; VAEA

Scanned with CamScanner


q

386 SPECTRUM DISCRETE MATHE MG


Ny
Alsoas H, SG.VAE A

vw f)H, SG
Len
> HEG
Now let a. FEH

= abe ()h,
den

> abEH,. VAEA


F

2 able H, VAEA (- foreach AG A, Hy is a subgroup of ¢)

2 ab'e f)n,
AGA

-1
=> gh’ EH

> HH itselfis
a group and HCG
so. )60- His a subgroup of G.
Remark. The union of any two subgroups of a group is not necessarily a subgroup of the group.
For example : (/) The sets H = [0,3] and K = (0, 2,4} are subgroups of the group G = (0, 1, 2, 3,4,9
under the operation addinon modulo 6 But the union H U K = {0, 2, 3, 4} is not a subgroup of G, f&
2D3IEGHUK, but 2+3J"5E HUK.

(it) The setn Z* [......, -3n,-2n,-n0,2n,3n,......} of integral multiple of st, is a subgroupof


the group of integers under addition.

ZZ™ {2.0009 -6,-—4,


2,0, 2,4, 6,...... }
and SZ t..0.,-9,-6, -3,0,3,6,9,...} are subgroups of Z, under addition,
But 2Z U 3Z *™ {..... _~ 9,-6,-4,-3,-2, 0, 2, 3, 4, 6, 9,...... } is not a subgroup of Z, fr
HIE UIA bat 4-37 EWUIL
(ia) The set Hee f1,- 1s -tjand Ke= {1,- 1,/- 7) are subgroups of the Quaternion group Q,.
HUK © (1,- tao a7,- 4) is not a subgroup of the Q, for. ,/GEHUK, but 4 j/k EHURK,

(4) The sets H > {1,(12)} and K = (1, (13)) are subgroups of S,, the symmetric group ca &
numbers 12,3 But HU K = (4, 12), (14)) is not a subgroup of Sy for (12), (3) E HUK™
(12) (13) « (123) HU RK.
Theorem : The union of two subgroups of a group ts a subgroup iff one is contained in the other.
Proof. Necessary Part: Let HH, and H, be two subgroups of a group G such that H, UH; is age?!
subgroup of Gi.
We shall prove thateither H,O HU, of HyCH,.

Scanned with CamScanner


csrructl RES AND Morr
uc is
¥
pe
sfpossdle. auppase that H,¢ H, and H; <¢ H,.

cect Hy Hr. 9053 2EG such that EH, but a€ H,


apinsince Hy & Hy, so 3 bEG such that 5 & H, but b€H,
since E Hand H, SH, UH, 80 @EH,UH,.

* bEHUH,
2bEH,UH;
> ab 'EH, UH, since H, U Hy isa subgroup
> eb 'EH, or ch' er,
First cons
theid
caser
e when g'h
€ Hy,
sce @EH, andHyisasudgroup 2 pteyy,
a'(ab EH,
eo (@'a)b'EH,
2 ¢«b'EH,
2 B'EH,
2 (')y'EH,
ie, OEH,, whis
ic not h
truc.
This case is not possible.
Now consid
the er
case og A' EH,
Smee SEH, .
(e@b")SEH, » o(S' HER,
ie,g@eGH, = gE H,, which is again false.
This case is also not possible,
So both the cases are nut possibis. Therefore, our suppo
sition is wrong.
+ either Hi SH: oor HCH,
Safficient Part : Suppose that cither H, SH,
oor HCH,
* NUL=H. or H, UH. =H,
nes Ny UH: is a subgroup of G, since both H, and H, are subg
roups of G.
CTOF TWO SUBGROUPS
radgg LM be 0 subgroups ofa group G, them the st HK defined by HK ~{ha
: for allAEHLEK)
hes Product of the subgroups H and K.
th
A fOn-empty subset H ofa group G is a subgroup, then HH = H.
=A hom-ermpty subset H of a groupG is subgroup
uff HH! H.
a IH and K are two subgroups ofa group G, then HK is asubgroupof G iff HK = KH.


Scanned with CamScanner
388
SPECTRUM DISCRETE MATHEMATICg
—S
Definition. The Centre ofa group G is denoted by Z (G) or C (G) or Z and is defined as
Z=C(G)=Z(G)={gEG|gx=xg, VxrEG}.
Theorem. The centre Z (G) ofa group G is a subgroup of G.
Proof. Let Z(G) = {g € G: gx=x g, Vx € G} be the centre ofa group G.
Clearly Z(G) CG.
Since ex=xe, VWxEG e€ Z(G).
Therefore, Z (G) is a non-empty subset of G.
Let g), g2 © Z(G) be any two elements, then

gix=xg, and gx=xg,, WxEG > x g>'= g5!x.

Now, *(g1g3 -ly_)=(8i) 87-l = (1%) 8p!~- = 810 8)')=g1(83'x)=(e,


=
g5!)x
ie, x(gig5')= (ei g5|)x, VxeEG.

So, g)8)| €Z(G), V gus. €Z(G). \,


Hence , Z(G) is a subgroup of G.
Remark. G is an abelian group iff Z (G) =G.
Proof. Firstly, let Z(G) =G
ie, Z(G)={gEG:gx=xg, VxEG}=G
> xy=yx, VxyEG => G is an abelian group.
Conversely, let G be abelian > xy=yx, Vx,yEG.
To show that Z (G)=G.
Since Z (G) is a subgroup ofG “ Z(G)CG
Now, let x € G be any element

G is abelian “ xy=yx, VWyeEG.


> x€Z(G)
GCZ(G)
Hence Z(G)=G.

ILLUSTRATIVE EXAMPLES
Example 1. Show that the set n Z= {...,—3n,-2n,-n,0,n,2n.3 i
of additi “ } ofof all
is a subgroup of the group Z of all integers under the operation all In
i tegral multipi le
s of n,
Sol. We know that Z, the set of integers forms a group under addition.
Now nZ={nm:mEZ}
Since n,mEZ > nmEZ
nZCZ.

Scanned with CamScanner


ws

-grkU CTURES AND MORPHISM 389


gil
yo "show that 7Z forms a group under addition.
ov
we eEnZso thatx = 1m, and
y = nm for some m, , m2 E Z.
{ *> _ »
Lé yon — ma Om —m2)EnZ. [ Since m,— mz € Z forevery my, mz © Z.]
9
: the closure property holds .in n Z.
“ ssociative law holds in # Z since it holds in ZandnZ C Z.
ea
" gan0EnZ and x+0O=x=O0+x, VxEnZ.
A 50
0 is identity element of 77 Z.
©2nZ.
yenmenet we have y=n(—m)
wow for
nmtn(—-m)=nm—nm=0=y+x,
And x +) =
xinn Z.
yis the inverse of
» inverse of every element in nZ exists.
on.
nZ forms a group under additi
Thusn Z is a subgroup of Z.
x numbers. Show that the set
frample 2. Let C* denote the group of all non-zero comple
g={zEC*s.t. [2] = 1} is a subgroup of C*.
ci Since |E C* and| 1 |= 1, -- 1 eS
ie, Sis non-empty subset of C*
Let 21,22 & Sbe any twoelement > | Z,| = and |z2| = 1

Now ]z122|=|2Zil|Z2l= 1-1=1


> 2z1z72ES5
the closure property hold in S
The associative law holds in S since it holds in C* and S € C*

Since l-z=z=2-1 for allz © C*


In particular l-z =z=2z-l forallz &S
1 is the identity element ofS
Since foreveryz ES >zEC* -. dz’ EC*st.
[- C* is a group]
zz’=l]=2'2z
But |zz’|=|1|=|z'z]

> |ziiz'J=1=]z"[z|
>
hjz'|=1 = |2’|=1 =2z'ES
foreveryz €S,4z' ES s.t.
zz'=l=2'z
inverse of every elements of S exists in S
S is a group under multiplication.
Hence § is a subgroup of C*.

Scanned with CamScanner


- “5

390
SPECTRUM DISCRETE MATHEMATI cg
— Ss
Example 3. Let G be group of 2 2 non singular matrices over R under multiplication. Show

b
W= ° | /saeo is a subgroup
of G.

a b ad+#0
Sol. . Clearly
Clearly W = ° | / abdeR isi non empty

a b a,b,c,deER 1 0
bset of group G=
Stinger OF FOUR ° | ci e «(| ew

i Let A= a 5, and B= a, 5, ©W where a, d, #0, a, d, +0


| 0 d, 0 d,

: - J
Now AB"! = | 72 2 a p-! =! E ~,
L iyo a, d, 7
d,

a a5, eth
=| “22d, d.
2lew a,
yA da
14
d
4
0 1 a, d, ad,
al a,

=> Wéisa subgroup of G.


Example 4, If ¢ is an identity element ofa group G, then { e } is a subgroup of G.
Solution. Since e is the identity element of group G, therefore e € G.
Let H= { e }, thenH
CG.
Since e e=e EH, therefore closure property holds in H.
Also (e e) =e (e e) =e.
Associatively to holds is H
Since ee=e=ee
e is identity element of H and
e'=eEH.
H itself is a group
H is a subgroup ofG
i Remark. The subgroup G and {e} are called trivial or improper subgroups of G. Any subgroup of groupG
other then G and {e} is called proper subgroup of G.

Scanned with CamScanner


391
oeBRAIC STRUCTURES AND MORPHISM
te

pple §, ({) Can an abelian group have non abelian subgroup ?


(i) Cana non abelian group have an abelian subgroup?
a () No, Let G be an abelian group and H be its subgroup, then the operation on H is commutative.
- js commutative in G and H C G. Hence an abelian group cannot have a non-abelian subgroup.
(i) Yes. A non-abelian group can have an abelian subgroup.
for Example : The {1, — 1, i, — i} is a an abelian subgroup of the non-abelian group of quaternion
y(t 1, + i, +f, + A} under the composition of multiplication.

sample 6. Let G be the group of all 2 x 2 non-singular matrices over the reals. Find the centre of G.

a b
ol Here G = |? ak a,b,c,d ER st. ed-beed|,

Now by definition of C (G),


C(G)={gEG|gx=xg, VWxEG}.

Let|: | € C(G) be any element. Then it should commutate with all elements of G.
c

. . ., }9 1} {| 1 0
In particular it commutes with tori EG.

(eal ol th alle al > Le 2-15 3]


Laks ILE a} = [2 a)-[ete otal
> at+b=a, b=c=0. (using (1))

Hence k b E C(G) is of the form Is “|:


cd 0 a

a 0
texecca)= {[¢ | o#0 eR}.
a

EXERCISE 1.5
1, Show that the set Z of all integers is a subgroup of the set of rational numbers under the operation of
addition.
2. Verify the following statements for being true or false.
(2) The multiplication group {1,— 1} is a sub group of the multiplicative group {1,—1, ,- 4
(b) The additive group of even integers is a subgroup of the additive group of all integers.
(c) The set of odd integers in not a subgroup of < Z, + >

Scanned with CamScanner


392 SPECTRUM DISCRETE MATHEMATIC
al
3. If.x is any element of group G, then show that {x” | © Z} is a subgroup of G.

4. Is Qo, the set of non-zero rational numbers, a subgroup of

G= {a+ 2 b\a,b€Q and a* +b? # 0} a group under multiplication ? Justify.


5. For positive integers m, n, show that n Z is a subgroup of m Z if m|n.
Show that SL(2, R) is a subgroup of the group GL(2, R) under the composition of multiplication of
a

matrices.
7. Show that the set of cube roots of unity H = {1, w, #7} and the set of fourth roots of unity K = {1,~1, /,-4
2h :k =0,1, 2, 3,..., uh und
are subgroups of the group of twelfth roots of unity G = {cs

multiplication of complex numbers.


8. Show that the sets H = {0, 3} and K = {0, 2, 4} are subgroups of the group G = {0, 1, 2, 3, 4, 5}
under the operation addition modulo 6.

1.7, COSETS
Definitions : Let H be a subgroup ofa group G. Ifa € G, then the set Ha= {ha:h €H } is calleda
right coset of H in G determined by a and the set a H = { ah: h € H } is called the left coset of H inG
determined by a.
If the operation is addition, then the right coset becomes H+ a= {h+a:h € H} and the left coset
becomesa+H={ath:h€ H}.
Note
: Ife is the identity element of group G, then H e and e H are right and left cosets of H in G.
Also He={he:hE@H}={h:hEH}
=H.
eH={eh:hEH}={h:hEH}
=H.
If H is a subgroup of a group G, then H itself is a right coset as well as left coset of H inG
determined by e.
Remark : When G is an abelian group then there is no distinction between a left coset and a right coset. ie
left coset = right coset i.e. aH = H a.
Lagrange’s Theorem : The order of each subgroup of a finite group is a divisor of the order of the group.
Proof: Let G bea group of finite order 7.
Let H be a subgroup of G and let O (H) = m.

Suppose A, , Ay, ... Am be m distinct members of H.


Let a &G. Then H ais aright coset of H in G and we have
Ha= {hy a, hy a, ... tm a}

Ha has m distinct members, since h,a=/ya, | Sijsmy,itj

By right cancellation law = h, = h,, a contradiction.

Therefore each right coset of H in G has _m distinct members.

Scanned with CamScanner


sopBRalC STRUCTURES AND Morpiism 393
4a

Any two distinct right cosets of H in G are disjoint Le., they have no element in common. Since G is a
_» soup, the number of distinct right cosets ofH in G will be finite, (say) equal to k.
The union of these & distinct right cosets of H in G is equal to G. Thus if
Ha), Haz,... H ag are the & distinct right cosets of HinG, then G= Ha, UHa, VU... UH ay.
» Number of elements in G = the number of elements in H a, + the number of elements in

of Ha;+...+ the number of elements inH a, = [*"_ two distinct right cosets are mutually disjoint]
>» O(G)=km > n=km

> k= 2 2 misadivisor ofn


m

2 O(H) is a divisor of O(G).


Hence the proof of the theorem.
ote: In the proof, we should prove properties (above) II. [V and V.
Converse of Lagrange’s Theorem. If G is a finite group and m divides O(G), then there exists a
sgoup H ofG such that O(H) = m.
The converse of Lagranges Theorem is not true.
inition : The number of distinct left or right cosets ofa subgroup H in group G is called the index of H
aGand is denoted by ég (H) or [G : H].

\ote. Here A is the index of H in G. We have mr = = Thus & a divisor of ». Therefore the index of every

seroup ofa finite group is a divisor of the order of the group andn=mk =» O(G)= O(H)(G: H]}
Another Method of Theorem.
[8 Normal Subgroups (or Invariant Subgroups or Self Conjugate subgroups)
In general, ifH is a subgroup of a group G, then the left coset a H of H in G may not be equal to the
utesponding right coset H a. In this section, our aim ts to study a particular class of subgroups H for
tich each left coset of H in G is equal to the corresponding right coset of H in G. We call such subgroups
Snormal subgroups.
kfnition : A subgroup H ofa group G is called a normal subgroup ofG if every left coset ofH in G is
Sul to the corresponding right coset ofH in G.
ie. aH=Ha, Va G&G.
Ifthe composition defined on G be addition, then H will be a normal subgroup of G iff.
at+H=H+a, WaeG.
kmark : (i) When G is an abelian group. Then every subgroup H of G is a normal subgroup, for aH = H a,
NEG.
_ (i) The subgroups {e} and G of any group G are always normal subgroups of G. These are called
“Mal normal subgroups.
(ti) If His anormal subgroup of G, then we write itas H AG.

Scanned with CamScanner


x4 SPECTRUM DISCRETE MATHEN iTiy

Theorem ; A sebgroup
H of a group G is a normal eabgroup ofG iffghg ' EH foreveryhEH. g EG
Proof: Firstly. bt H be a normal subgroup of G.
ge He. VeeG
Let 4 E Hand ¢ € G be any clement. Then
gh€egli-He * phEHe
* gh~hy foesame h, EH
® ghy '=% CH
2 ghy ‘GH
Ceaverely. Lct H be a subgroup of G such that
ghe ECW. W AEN, SEG
We shaw that His anormal subgroupie aH Ha, VaEG.
Let a@ @G be any clement. Then by piven hypothesis
oha'GH. WhEN
Let ah Gull beanyelement
Then ahe(aha ')aG@ Ha
* vhkeGHa
a@Heoa AG

Again, Let b= a! be any clement


of G.
Then by piven hypothesis bA Ao! EH.
But baw! =a'h(a'y'a' haGH.
Let hka€Ha beanyelement Then ha=(aa')ha= ala 'ha)€ aH
2* hata!
HaGat.
From (1) and (2), we get
aHeHa, VWaeG.
Hence H is anormal subgroup
of G.
Theorem : Lct H be a subgroup of a group G. Then the following statements are equivalent
(i) ghe'GH WY gEG,AEH G) gHg'=H, VY gEG.
(iz) gH-He VW géG.
Proof, (1) > (is) Since ghg EH. YzEG AEH.
Let ghy'=h, forsome 4,EH
= gHg'=H, YWeEG.
(in) >(iiiletgHg'=H, YezEG
2 (gig')e-Hg

Scanned with CamScanner


ee a — Se
7
i STRUCTURES AND MORPHISM

is 2 gH(se J=Heg
=> vHe=Hg £&
.
(oH es H)
~* gH=Hg,

oy 2 (Let gH-Hg, VeEG


2» gh=-hy,g torsome hh, EH
> ghg'=hGH
> ghe'EH, VeEG AEH.
vegce (1) = Gi) > Ci) > ().

vonce the given statements are equivalent.
agoremt : Let H am nd K be two subgroups ofa group G. Then
!
_) if His anormal subgroup of G, then HK = KH is a subgroup of G.
_,) i Hand K both are normal subgroups, then HK = KH is a normal subgroup of G.
neat i) Given Hisa normal subgroup of G. To show that HK = KH is a subgroup of G.
eth
& K be any element. Then H b= bH [. HA G]
2 Hb-+HE KH, VbEK

2 HK & KEL. (1)


Smilarly DH HbEHK ie, BHEHK, WhEK
° KH G HK. -2-(2)
from (1) and (2), we get HK = KH.
By Theorem, HK (= KH) is a subgroup of G.
(4) Let H and K be both normal! subgroups of G.
By (1) HK = KH is a subgroup of G. To show that H is a normal subgroup of G.
Let ¢ € G be any element. Then
g(HK)g '=gH(g'g)Ke'=(gHg (eK g') CHK.
[: H, K are normal subgroup ». gHg'GH,gKg' & K]
Hence HK is a normal subgroup of G.

ILLUSTRATIVE EXAMPLES}
t b the
pk I. Show that the set H = (° it a,b,c,deERs.t. sd-be=t} is a normal subgroup of
c
fr
ia b
Maen.
2G ile i a,b,c,déeR sad-bes0|,

Scanned with CamScanner


SS
396 ihe

SPECTRUM DISCREy
7

; )
E “ATE

Sol. Since =|} 1 and |I|=1 > IEH. SS

H is a non-empty subset of G.

LetA -| a 1 4 k B | 2| € H be any two elements ,


Cy d, Cz dy

where ay, by, Cl, d, and ap, bp, C2, dy ERS. | A| =a, d\- b c\= | and | B | = a>
4: ~ ba oy =
Now| AB|=|A||BJ=1-l=1 = ABEH.
Atso 1att=| A agja|=lagAl IAL 11,
|A| IAP JAP FAL 1
[or AA'=1 > |AAT|=|1| = JAI[AT|=1
> IAT) =1 > {AT |=11.
> A'EH.
H is a subgroup
of G.
To show that H is a normal subgroup of G.
Let A € H and B € G be any elements, then| A |=1.
Also| B| + 0 > B' exists.

Now BAB] =|Bj [A] {B°'=|B]-1- =I E ati


|B
> BAB'EH,VAEH and BEG.
Hence H_ is anormal subgroup of G. |
Example 2. The centre Z(G) of a group G is a normal subgroup of G.
Sol. We know Z(G) = {g €G;xg=gx VW x€G}.
Clearly Z(G) CG.
Since ex=xe, WxEG => e€& Z(G).
Z(G) is anon-empty subset of G.
Let a, b € Z(G) be any two elements, then

ax=xa, Wx€G and bx=xb, VxEG 2xb'=b'x


Now x (a b~') =(x a) b7'= (ax) b= a(x by! =a(b"' x) =(ab“')x
> x(ab") =(ab"')x WxeEG.
ab'EZ(G) WabEZ(G).
So, Z(G) is a subgroup of G.

Scanned with CamScanner


\GEBRAIC STRUCTURES AND MORPHISM
hot

a“
397
Now, we show that Z (G) is a normal subgroup of G.
Let hEZ(G) and g &G, then
she’ =(gh) @'= (hg) g'=h(gg"')=he=hEZG)
she'EZ(G)WVeEG, hEZ(G)
Hence Z(G) is anormal subgroup of G.
fumple 3. IfH is a subgroup of G of index 2 in G. Then H
is normal subgroup fg
wt Let H be a subgroup ofG such that [G : H] =2.
|
The number of distinct left (or right) cosets ofH
in G is 2.
To show that H is a normal subgroup of
G.
Itis sufficient to prove thatx H=Hx, Wx EG.
Letx & G be arbitrary element of G.
(wel. When x EH,
Sine =x EH So, xH=H=Hx
Hence x H=H x,
Case Il. When x GH.
xH #H and Hx#H.
Also [G: H] =2.
" HUxH =G=HUHx
> xH =Hx.
Combining the two cases, we find that

xH=Hx VxEG.
H is a normal subgroup of G.
tumple 4. A subgroup H of a group G is a normal subgroup of G iff the product of two right cosets of H
2Gis again a right coset ofH in G.
Or
Prove that a subgroup H of a:group G is normal
iff HaHb=HabVa , 6 & G (the composition is denoted multiplicatively).
SL Let H be a normal subgroup of G and
let Ha,H 6 be two right cosets ofH in G. Then
(H a)(H 6) =H (a(H 8))
=H ((a H) 8)
=H (Ha) 4, since H is a normal subgroup of Gso aH=Ha
=H (H (a d))
=(HH)ab
=Hab, since H isa subgroup ofG so HH =H

Scanned with CamScanner


398 SPECTRUM DISCRETE MATHEMATICS

=
(H a) (H 6) =Ha b.
a,bEG > abEG
Hab isaright coset ofH in G.
Thus the product of two right cosets of H in G is again a right coset of H in G.
Conversely, suppose H is a subgroup of a group G such that the product of two right cosets of H ing
is again a right coset of H in G.
To show that H is a normal subgroup of G.
Let g€G be any element.
g 'EG, since G isa group.
Hg, Hg"! be two right cosets ofH in G.
(H g) (H g’'') is again a right coset of H in G.
Since H is a subgroup of G, therefore e € H, where e is the identity element of G.
Since e EH.
(eg)(eg')E(Hg)(Hg"').
= gg'e€(Hs)(Heg"')
> e€(Hg)(Hg"').
Also H isarightcosetofHinG and e€H.
(H g) (Hg ')and H are two right cosets of H, each containing e.

(Hg)(Hg')NH#¢.
Since the two right cosets of H in G are either disjoint or identical.
> (Hg)(Hg')=H.
Let AEH beany element.
(hg) (hg ') € (Hg) (Hg"')
> (hg)(hg ')EH, since (Hg)(Hg ')=H.
h(ghg ')EH.
4

=> ghg' Eh'H.

hEH andHisasubgroup = A'EH => A'H=H


ghg'EH.
Thisistue VgE€G and AEH.
Hence H is a normal subgroup of G,
Example 5. Prove that the intersection of any collection of normal subgroups is itself a normal subgroup.
Sol. Let H,, be normal subgroups of group G for alln © N

and consider H = () H,,


n=]
at i

Scanned with CamScanner


| er = ar
a

399
ofBRAIC STRUCTURES AND MORPHISM

H, is normal subgroup of G for all n E N

> H, is subgroup of G for all n E€ N

ao

> () H,, isa subgroup


of G
n=l

2 Hisasubgroup of G.
Further to show H is normal in G
ao

Let he () H,, be any element


n=1

> h€each H,, foralln EN

» ghg EH, forallgeG, nEN


oO

=H (- n € N)
H, is normal subgroup for each
ghg! € ‘a H_
u

n .

n=]

. hEH > ghg!


EH forallgEG
= Hisnormal inG
“eo

Hence H = ‘a H,, is normal subgroup of G.


n=l

EXERCISE 1.6
1, Let G denotes the group of all non-singular upper triangular 2 x 2 matrices with real entries i.e., the
1 b
matrices of the form ; A a,b,d€@ Rand ac
# 0. Show that H = [3 | be R|
oO
isanormal subgroup of G.
2. Let H be a subgroup ofa group G. Let g & G be a fixed element of G. Then show that gH g7'= {gh
g’' : hE H} is asubgroup ofG and W = 0 gHg" is a normal subgroup of G.
gE

3, Let H be a subgroup of a group G. If x° € H, for all x & G, then prove that H is a normal subgroup
of G.
4. Prove that the intersection of two normal subgroups is a normal subgroup.
5. Let H and K be normal subgroups of a group G such that HM K = {e}. Prove thath k=kh, for all
hEH and KEK.
6. Give an example of a non-abelian group in which all the subgroups are normal.

Scanned with CamScanner


400 SPECTRUM DISCRETE MATHEMATICg
7, IfH is anormal subgroup of G and K is a subgroup of G such that H C K GC G. Show that H is alsog
normal subgroup of K.
8. Give an example of two subgroups H, K which are not normal, but HK is a subgroup.

9. Show that a non empty subset H of a group G is normal subgroup of G iff (g a) (g by! EHVa,
bEH,gEG.

10. Show that the set SL (n, R) of all » x » matrices of determinant 1 over real numbers is a normal
subgroup of GL (n, R), the group of 7 x n invertible matrices over real numbers.
11. A cyclic subgroup T of a group G is normal in G. Prove that every subgroup of T is also normal in G,
12. Prove that a normal subgroup of G. commutes with every complex of G.

1.9. Quotient Groups


The idea of a normal subgroup is a very important concept in the group theory. Now we shall show
that with the help of a normal subgroup of a group, we can construct a new group from the given group,
known as quotient group.
Theorem : Let H be a normal subgroup of a group G. Then the set G/H of all the right cosets of H inG
forms a group under the composition defined by (H a) (H 6) =H ah.
Definition : Quotient group (or Factor group or Residue Class group).

IfH is anormal subgroup ofa group G, then the group G / H of all the right cosets of H in G under the
composition (H a)(H4)=Ha bis called a quotient group or a factor group.
Note. If the composition in G/H is addition, then the composition in G/H is defined by
(H+a)+(H+6)=H+(a +b).

Remark : IfH is anormal subgroup ofa finite group G, then G / H form a group of order OG)
O(H)-
Proof : By Theorem, G/H forms a group.

O(G/H) = The number of distinct right cosets ofH in G.

_= omar
OG)

Theorem : If H is a subgroup of an abelian group G, then the group G/H of all right cosets ofH in G forms
an abelian group under the composition defined by Ha. Hb=H ab.
Proof. IfH is a subgroup of an abelian group G, then H is a normal subgroup of G.
G/H forms a quotient group. (already done)

Let Ha,HbEG/H sothat a,bEG.


(H a)(H 6) =Hab=H ba, sinceGisabelian -. ab=ba
= (H 5) (H a).
Hence G/H is an abelian group.

,
Cg
nr PP SP ee

Scanned with CamScanner


ee. — a SOW ER oT‘Wa

otBRAIC STRUCTURES AND MORPHISM 401

gatke The convese of the above result is not true that is, the quotient group may be abelian even if G
7 got be abelian.

E ample. Give an example of a non-abelian group G and a normal subgroup H of G such that quotient
i G/H is abelian.
up
ol x. (i) Let G= {+1, + é, + j, + k} be the non-abelian group of unit Quaternion under multiplication
gfned by
papak=-l, ij=k=—ji, jk=i= ki, ki=j=—ik.
tet H={1, —-1,4, —i} be asubgroup of G.
O(G) 8
Then [ G:H] ] = —~=-=2
O(H) 4

H is anormal subgroup of G.
G/H is a group of all right (or left) cosets of H in G is of order 2, a prime. Hence G/H is an
pelian group, but G is non-abelian.
Ex. (i) Let Ss = {i, (12), (13), (23),(123), (132)} be the symmetric group on three numbers 1, 2 and 3
sanon-abelian group. Then
Ld

H= {i, (123), (132)} be a normal subgroup of S; such that the quotient group S;/ H = {H, (12) H} is
nabelian group. [-.. every group of order 2 is abelien]
Theorem : LetN be a normal subgroup ofa group G. Show that G / N is abelian iff for all x,y € G.xyx 'y'EN.
Proof. Firstly, let MN be anormal subgroup of G such that G/N is abelian.
Toshow thatxyx7'y"'EN, Vx,y EG.
Now, Nxyx7'yo! =NxNyNx Ny?! =NxNp(Nx)! (N y)7!
=Nx(Nx)' Nyy)! [Since G/N is abelian]
=NN
=N
Thus Nxyx7'p"'=N => xpx tpl EN, VxyEG.
Conversely. Let for all x, y € G, xyx7'y7' EN.
To show that G / N is abelian.
Since xyx7'y 'EN
Nxyx ty! =N > Nx Ny Nx! Ny-' =N
4

Nx Ny(Nx)! (Ny)! =N = NxNy(Nx)'=N(Ny) =Ny


4

NxNy =Ny Nx, VxvpEG


4

=> G/N is abelian.


Theorem : Every quotient group of a cyclic group is cyclic.
Proof. Let G = < a> be a cyclic group generated by a
G is an abelian group
Each subgroup of G is normal subgroup.

Scanned with CamScanner


a,
os

SPECTRUM DISCRETE N
Let H be any subgroup of G. LATHE,
Ry
His a normal subgroup of G. So G/H form a quotient group.

We prove that G / H is a cyclic group generated by H a.

Let Hx €G/H_ be arbitrary element, where x EG.


But G=<a>
x =a" for some integer n.

Hx=Ha" = Ha.a..a

na times

= Ha.Ha....Ha
—$$<_, —__
a times

=(Ha)".
Hx=(Ha), VW HxEG/H.
G/H is acyclic group generated by H a.
So, each quotient group of a cyclic group is cyclic.
Remark. The converse of above result may not be true i.e. quotient group may be cyclic even if the group
may not be cyclic.
Theorem : If G is a group such that G / Z(G) is cyclic, where Z(G) is the centre of G. Then G is abelian.
Proof.
Let N = Z(G) and letG/N=<gN> beacyclic group.
Leta. 6 EG be any two element.
=> aN, bNEGIN.
aN=(gN)” and 6N =(gN)" forsome m,n € 1
> aN=g"N and bN =g'N @ a'g™EN and b'g"EN
> g"aEN, g"bEN. 2 x'eEN
(se xEN
Let ge "a=n, go" b=, forsome m,n EN.
> a="n,, and b=g"n,
ab = ny ny" (mg) m= go (27m) my
= gf" 2" nym me worn my

Similarly, ba 8 (em) ma
("nr (e% m) = (mg) mm
om
ment
al
= gm 1). (cm, ENS Z(G)"

abba.

Hence G is abelian.
Ye

Scanned with CamScanner


x —~_—i—s&
oe
+ gue STRUCTURES AND MORPHISM
vo

(ILLUSTRATIVE EXAMPLES,
~S
se 1. Let Z be the additive group of integers. Let H = 3 Z be the additive grou of ;
wr . = | ~~ of Rte,
=; prove that H is a normal subgroup of Z. Also find the elements of Z/H wee hehetips

La 3 Z forms a subgroup of the additive group Z of all integers. 7


' te gEZ and h EH.
=3n forsome n € Z.

j Thea g+h+(-g)=g +(3n)+(-g)=32EH.


H is a normal subgroup of Z.

Z/H forms a quotient group, where Z/H={H+a:a2€Z}.


Lt @ & Z. By division algorithm, 3 integers g and r such that
@a=3q~-r, where Osrs2.

H+a=H+(3¢+7r)=(H+3¢)er.
=H+vr, since 3q © Hand hence H+ 3 ¢=H.
H-e=H+r where OS-s 2.
H+ aisequaltooneof H+0, H+1, H+2.

. Z4H#*{H,H+1,H+2}.
‘ample 2. If H be a normal subgroup of a group G and (G : H] ~ -, then show that foramy EG, x"EH
wi Snece H ts a normal subgroup of group G such that (G : H] © ar.
O (GH) = m.
VY xHEG/H, where x GG, we have

(rH)" =H [ ifO (G) =n then a = ¢ ¥ a € G}


x"H=H
* x" H.
Thus WrEG, wehave x” GH.
xaitioan. A group G is called a Simple Group if it has no proper normal subgroups.
fample 3, Show that every cyclic group of prime order is a simple group.
WlaG=<a>bea eyelic group of order p, where p is a prime number,
See Landp are the only positive divisor of O(G) = p
By Lagrange’s theorem
G has no normal subgroup other than {¢} and G.
Hence G is a simple group.

Scanned with CamScanner


vA

404 SPECTRUM DISCRETE MATHEMATcg

=
=
Gs

WS
Definition. Let H be a non-empty subset of a group G. Then the set N(H) = {a © G: aH =H a}, is calleg
the normalizer of HinG.

Example 4. Let 1 be a positive integer and N = nZ be a subgroup of the additive group of integers Z, Show
that O(Z /N) =n.
Sol. Let Z={..... —3, —2, —1, 0, 1, 2,3, ......} and

Aa SE
Z/N = { N+ a:a € G} be the quotient group under the composition defined by
(N + a) +(N + 6)=N+(a+
8).

We show that N, N + 1, N+2,......, N+(2— 1) are ndistrict elements of Z/N.


Let a © Z be any element such that
a = 1, 2,3, ......,(1—1).
i (
Then by Division Algorithm Theorem
a=nqt+r, where OSr<n.

Now Nt+a=N+(nqt+r)= (N+nqg)+r=N+r. [ng © Nand


N +a=N fora En}
Therefore O(Z/N)=n.
Example 5. IfN is a normal subgroup of a finite group G such that ({G : NJ, O (N)) = 1. Show that any
element x € G satisfying x°®=e mustbein N.
Sol. Let O(N)=m and [G:N]=/. Then (/,m)=1.
Jda,bEl st. am+b/=l.
Since N is a normal subgroup of G.
G/N isa group of order /.
> (Nx)'=N, forany NxEGMN.

Also (Nx) = (Nx)'=(Nx)’""°'=(Nx)°” (Nx)!


= {(N3)"}" (NX) P= NO N?=N OPEN,
Nx=N . =» x EN.

Example 6. Let (Z, + ) be the group of integers and H = {4 | € Z}. Then find the members of Z/H.

Sol. Let (Z,+)= {....—2,—1, 0, 1,2, ....} be group of integers under addition
and H= {4n|n€ Z} = {....-—8, —4, 0, 4, 8, ....}

For cosets
of H in Z

For 0€Z,0+H={....—8,—4,
0, 4, 8 ....}
For |1E€Z,1+H={....—7,—3,
1, 5,9, «...}
For -1 €Z,-1 +H={....-9,-5,—1,
3, 7, ....}
For 26€Z,2+H= {....—6,—2, 2, 6, 10, ....}

Scanned with CamScanner


|
:| gfBRAIC STRUCTURES AND MORPHISM 405
a
: for -2 € Z,-2+H= {....— 10, -6,—2, 2,6, ....}=2+H
|
| for 3€ Z,3+H= {....—5,—1,3,7, 11, ...}=-1+H
por -3 €Z,-3+H={....—11,-7,-3, 1,5,....} =1+H
| cosets of Z/H are 0+ H, 1 +H,2+H,3+H.
iumple 7. Give an example of a non-abelian group G and a normal subgroup H of G such that quotient
2 GH is abelian.
| Or

Show that if the quotient group G/H is abelian, then G may not be abelian.
(Jammu University 2013, 2014)

(i) Let G = {+1, + 1, + j, + k} be the set of unit quaternion under multiplication defined by
PapokR=-l, ij=k=-ji, fk=i=—ki, ki=j=—ik
form a non-abelian group. Then H = {1, —1, é, —/ is a subgroup of G.

[ ]
O(G) _ 8
O(H) = —4
Also G:H = =

H is a normal subgroup of G.
G/HH is a group of all right (or left) cosets of H in G is of order 2, a prime. Hence G/H is abelian
sup. But G is non-abelian.

(i) Let G = [8 2 |:4.8.6.¢ eR and ad +h. Then G forms a group under matrix

nitiplication. Its identity elements is unit matrix E |

=o
a|=

ra=|6 #]€6G, then At =


d
=
d
o

E
[6
3
3 Jana E
1
3Jes
alle SFO al
o

| lle allo al
llc Slo Silo
oN

G is not an abelian group

Scanned with CamScanner


re

406 SPECTRUM
MATIC
DISCRETE MATHEMATi cg

Let u={[) {|:beR}. thence.


Let A,B, EH

ae! b Opel 1 Slacer.


0 1 0 1
ABs 5) 1 c| | c+b H
=_

“lo allo a} lo 1 |e
Go

- jl -b
A= ' eH WVAeH.
0
H is a subgroup of G. We now prove that H is normal subgroup of G.
Let X&GandA EH.

X= E ' for some a, 6,


d © R such that
ad + 0

A= lo ‘| forsomee ER

H is normal subgroup of G.
G/H forms a quotient group.
Let Hx, Hy € G/H so thatx, yEG.
G/H will be abelian if (H x )( Hy) =(Hy) (Hx).
ie, if Hxy=H yx.
ie, if (xy)(yx)'
EH
ie,if (xy)(<'y')
EH
xyEG
_|a 6 -|c e
x E 4] forsome a6, dERst dw 0 and y=[¢ 1 forsomec, e, fERstcf #0

a ee

Scanned with CamScanner


i GEBRAIC STRUCTURES AND MORPHISM
a
a = 1 =
x =| 7
Then aa yl sl® - ‘
0 — oo —
d f
Then (xy) @'¥"")
1 -~ +
-|% oe | ac acf adf
0 df 0 ]
fd
, ee _.acbh bf
= acf adb fd\euH

(xy)@'y') EH
.. G/H is abelian whereas G is non-abelian.

EXERCISE 1.7
1. Using Lagrange’s theorem prove that if G is a finite group and N a normal subgroup of G, them for
anyx & G, O (x) in G is divisible by O (x)in G/N where X¥=N x.

2. Give the example of a group G and a normal subgroup H such that G/H is cyclic but G may not be
cyclic.
3. If H is a subgroup of a group G, N(H) be the normalizer of H in G. then
(i) N (H) is a subgroup of G.
(if) His anormal subgroup of N(H).

(iit) IfH and K are subgroups of G and H is a normal subgroup of K, then K G N (H) ie., N (H) is
the largest subgroup of G in which H is normal.
(iv) His anormal subgroup of G iff N(H)=G.
4, If H, K are normal subgroups of a group G and H C K, then show that K / H is a normal subgroup of
| G/H.
5. Let N, and N; be two normal subgroups ofa group G. Prove that G/N, = G/N; ifand only ifN;=N2.

Scanned with CamScanner


yA . Ne
ee
ae
oe
ALGEBRAIC STRUCTURES WITH TWO

.
BINARY OPERATIONS, RINGS,
INTEGRAL DOMAIN AND FIELDS
!
3

oss
|e!
$
DHISTROPLOCT IOS i
4
JN seaw we trove sadiod sizetrais yaeros wrth ome by PTY MPT LT PRITHEIY, BF OUD, ETD BOND «ang
pees) Phot dheve¢ney yyy rt pets) wrt'y tucgre freer) ore burvary ppc a any, sd fe$9e oA tor) thetr) rlernely tre 4
'
/ :
of ins port, tne set A tates! purities ety with tee biraery operation whinion and multiplication. 17
i ‘ ‘ * -e=5

«
gl es tute Tageee) « . a, ‘ 3
Hest uty fut yoann wider ade gd ort ander outtiphine sum, yet they du have certam ope ?
'’
Maple wy well, Theve properties ead to the commept of 2 nny whath ee
Sofie oy hs,

es
24, HINA

- en
LAA. Defiettion. Pave:
A semen y vA 4 unter with two Uiaery operstiom, Geetied additively (~) and multiplicatively +.
ip called 2 sieg Nhe cla bh. cE, Ge follwing crices oe ettisfied
S

jj g@+ hE [closed under adéricn}


bij
{7
bu~by+ he 5
o> a4+lb~
ya hepa
cy jadditionis avwxciztrrs)
as

(iii) 2 ancieroet GO waht O- a= a-a+9 {existence of additive idextzy]


fery For every 9%. 2 miclemert -c SK wech thet - o)+ a =0=a+ (-a)
[existence of addxive invert}
toy a+b~boa [addition is commutztiv:]

ee
[closed under multiplication)

et et
Oi 4b EX


boil) (a. bj. c= ab.) {multiplication is associative]
Gila b+ jr a.b+ale md (a4 bjy.e=a.e-boe [distributive laws boid,
A ting % weader
tee binary
ary Operations (+) ed (-)is dereted by the algebraic system <R, Ls-, ->
KMemark: The 2lgetraic hacer in +, > is called 2 ring if R formsan abelian group under addition ae
©
3

senna oup onder meakiplicztion along with datriiative pro


Nel
BA

‘=>

2.1.2. Definition. Commeotative ring A ring <K,+,-> ii which


gh=ba for2#ll 2b ER
holds. is called 2 eommatative ring.
(xherwize Bom called 2 non-tommotative ring

4453

Scanned with CamScanner


RIC STL TURES WHIT HO Brac OPERATIC, RISA ISTIC IAAI SSO FRLIS 98
3 2 Dofus
Defoe. ; with
King 2 BRity.
; ;
A ring .
“H.-~,- > is saidee wo be 2 ving
: wits
ie wey
ie ee
Oo ccc crits
es : , ;
eet i SK tt a.l~e=1lyg fel cer
viem 1 2s celled the anity of the ring
unerwise 1 is called z ring withost unity.
fo werwtons wkny recy 2a ote t4 2 2 “40, le eee
pees ie Geety 0) 2 ring is cleo czlled the identity element of He me

ILLUSTRATIVE EXAMPLES]
* *, 4 foie - o - a 4 eee die ot
sompte J. Prove that the 111 I of ali integers is 2 sing with respect to woe! acoticn anc mukipiication o

si (4) Properties of Addition

ify JV o,bE1 = a+bEl [-- sem of two itegers is an integer]

(9 7 a,b,cEl = la~by~c=a~lb~e} [-- Associative Property holds in imezers]


is) WV a Ei, there exists 0 © I such that 9+ 0=0- ag=a2.
Bere *O ts called additive idermtity.
fa) VW a GI, there exists —o SI such that a-(-aj)=0=(—aj-a
Here — a ts additive inverse of a.
972,561 > a+b=b-a [Commutative property holds in integers}

(By Properties of Multiplication

| i %o,b6E1 = abe) [ -- product of two integers is an integer]


i) J a,b,cEl => fa. bj).c=a.(b_c)

[Associative property holds for integers with respect to multiplication]


(Cy Distributive Laws
5 We Enow that V a, b,cEl
3 a.(b+c)=a.b-a.c
{o+cj)-a=b.a-+c.a

+ <1-,-7isarnge
Seee there exists an integer 1 € I such thar
—e—eEEeEeeEEeEeEEEEEEEeEeEeEeeee

a.}=1)-a=aV¥ aeél

Thos J is a ring with unity.

tise we know Ya,bEl >= a.b=b.a

ae
- Jisalso2 commutative ring.
<4,+.°> is a commutative ring with unity.

Scanned with CamScanner


$10 SPECTRUM DISCRETE MATHEMaT;c, 5
ey

| RING OF GAUSSIAN INTEGERS


Example
P 2. A Gaussian intezer
= is 2 complex
P number a ~ i 6 where a and 6 are integers.
& Show that the-~
1S is _
J (£] of all Gaussian integers is a ring with usual addition and multiplication of complex numbers.
Sol Given J[i]= {a+b} a,b € integers}
Since each element of J [i] is a complex number, so properties of complex numbers are also true fer
the elements of J [i]
(A) Properties of Addition
(it) ¥a-ib, c~id€&J
[i], where a, b,c,
dare integers
(a+ib)+(c+idj=(a+c)+i(b-d)

E J{i] [. ab,c,dEl> at+c,b+deq


Tor

- J [i] is closed under addition.


(ii) V a+ib,c+id,e+ifES
[i], where a,b,c,
d, e, fare integers
a

[(a+ib)+(c+id]+(e+if)

=[(a+c)+i(b+
AD] +(e+ify=[(at+o+e]+i[(b+a+f]
=[a+(c+e)]+i[b+(d+f)] [ ~ Associative Property for integers hold] |
=(a+ib)+[(c+ e)+i(d+ f)]=(at ib)+[(e+ id +(e+is)] i
. addition is associative in J [i]. \y
(iif) VW a+ib€SJ
[i), there exists 0+10€J [i] 5
such that (a+ ib)+(0+i0)=(a+0)+i(6+0)=a+ib
and (0+/0)+(a+ib) =(0+a)+i(0+5)
=at+ib
(a+ ib)+(0+i0) =a+ib=(0+10)+(a+i8)
Thus 0 + /0 is additive identity.
(iv) V a+ibEJ[i], there exists —a—-ib
EJ [i] ( abEl = —-a,-b,ED
such that (a+ib)+(—a—ib)=[a+(—a)]+[b+(-4)]
i =0+0i
and [-a+(-64) i] + [a+ ib]=[(Ca)+a]+i[(—b)+5] =0+/0
“(a+ ib) + [(—a) + (—5) i] =0+10=[C a) + (4) i] + [a+ 15]
Thus (— a) +(—
5) i is the additive inverse
of a + ib.
(B) Properties of Multiplication :
() Vatibct+idEJS
[i]
(a+ib)(c+id)=(actiPbd)+i(adt+bc) =(ac—bd)t+i(ad+bc)
EJ fi] [-- a,b,c,d€l= ac-bd,adt+bcéll
ee 5,7,

“. J [i] is closed under multiplication.

Scanned with CamScanner


sSSRUCSTRUCTURES WITH TWO BINARY OPERATIONS,
RINGS, INTEGRAL DOMAIN ANDFIELDS «= 441] =

iV a-ibe+ide-ifes [iy
4 [ia-fb}(c+ id] (e+ ifp=(a-ib) [(e+id(e-if] [Verify i]
(© Dstributive Law.
mee

lat e-i6,c+ide+ifES
fi]
ee he

Teen (a+). [(c+idj)+(e+if)] =(a+ib).[(e~e-~i(d-f/)]


he |e ee

=[a.(c+e)-b(d+f)]+i[b(c+e)+ald+f)]
cee

=[ac+ae-bd-bf\+il[bc+be+ad-+af]

=[(ac—6 d)+(ae-bf)]~+i[(ad+bc)~(af~
be)]
eee

, And (a+ib).(c+id)+(a+ibj(e+if)
Hee

=[(ac—bd)+i(ad+ boc)]+[(ae-bfp-ilaf-be)]
She

=[(ac—bd)+(ae-bf))+i[(ad+ bc)+(af+be)]
-. Left Distributive Law holds
Similarly Right Distributive Law holds.
Hence J [i] is a ring.
RING OF MATRICES
frample 3. Prove that the set M of all m X n matrices over reals is a non-commutative ring with unity,
=-h zero divisors under addition and multiplication of matrices.
SL Let A, B, C be any members of M.
-. A,B, Care n X nm matrices over reals.

> A=[aij|nxn>B=[bijlaxn,C=[eij|axn

where aij, Diy, Cij ER for | sisn, lsjsn.

(A) Properties of addition


() VA, BEM, we have

A+B=[aisInxnt [bijlnxn
=[aijt+ big Inxn [~- aij, bi; are reals > ait bry is also real]

EM

_ addition is closed.
(i) VW A,B,C EM we have
A+(B+C)= (aij) + (fbi) Leis) = lain nt (bit ciy|nen
=[aijt+(bizt+ ci Wnxn = [laiy+ bij) + (Cu) nxn

[ *- Associative Property holds in reals}


ai
~ s[big yt
lnxnt leislnxn
=(A+B)+C.

Scanned with CamScanner


a
',. 7

SPECTRUM
|}
412 DISCRETE M4 }
tg i+4 y

7
(ci) For A EM. there exists O= [0]...
2 © M such that
A-O ={a,,)- [9]
=[c;,-9]
=[e,j=A
Smmilriy O- A=A
=> A-~O=A=O0-A
. O fs the additive identity.
(fv) Sime A=[ai;Jaxn

= -A=[-ayJaun [- a4,ER =
~ 91, ER
“= A-~(A)=[a,,]-[-a,;] =[21; ~Ca1;))

=[Jnx2=O
and (-A)+A =[-4,,)~{[a,,) =[(-a.,)>+a,,]=[0]n,,=0
$0 that A+ (-A)=O=(-A)-
A.
-. the additive —A ofA © M exits.
(v) For ABEM,
A~B=[aiyjJnen+[bilnan

=[laytbiylaxn =[by-aiy aun

[- aiy*+ by,
=[41,)+[ai,) =B-A
“. commutative Law for addition holds.
(B) Properties of Multiplication
(i) Let A=[aijJnun and B=[bjtInyn EM
nw
Then AB=[cy¢]nyn where c,;4 = dia
=I

ABEM
‘- M is closed under multiplication.
(if) LetA=[aijJnyn; B=[hyi)n, and c= [ckp|n yn be three element of M.
“a
Let AB = [dit Jnyn where dj, = > 4, , bis
f=) hy

and BC=[eyp]n,n where eyp= >


k=]

Scanned with CamScanner


gotSRarc STRUCTURES WITH Two BrvaryY OPERATIONS. RINGS, INTEGRAL DOMAIN AND FIELDS 413

Now (7 pyth element of (AB) C = (ah row of AB) ( pth column of C)


2 oe ) noon
7 2 4 “xp = 2 22s Pak “Kp a 24 ig sk kp

snd (i, p)th element of A (BC) = (ith row of A) (pth column of BC)

j=l j=l \k=


rn rn nm nm

-> da, ye kp de 2 117 Ope kp


y=) k=) k=1 j=
~ A(BC)=(AB)C.
Thus matrix multiplication is associative.
(C) Distributive Laws.

Let A=[aijyInxn, B= [bysnx nand


C= [Gk ]n yn be three elemen
of tM.
Teen B+ C= [byx+
Gk Inxn
ee

. n aT

. (i,k) element
of A (B+ C) = > a,, (bi, + Cy) i >, (a, , by +4; C iy)
k=1
n nm

- de ij Diy aT

= (i, kth element of AB + (i, &)th element of AC


= (i, k)jth element of (AB + AC)
Also A (B + C) and AB
+ AC are of type 1 X n.
M is a ring.
fa) Since the matrix multiplication is not commutative, in general, so M is non-commutative ring.
(6) Also, we know, there is identity matrix I, , , @ Msuchthat AI=~IA=A W AEM.
“. Lis the multiplicative identity.
{c) Also, we know there exists matrices A + O, B + O but AB=O

For ex: , lo 0| 0. B F 3| C ro 0|
J le, 0 O , 0 3 lo 0 ,

Hence M is non-commutative ring, with unity and with zero divisors.


Utample 4. Prove that the set R = {(a, 6) | a, b © R} is a commutative ring under the addition and
| zitiplication of ordered pairs defined as
(a, b)+(c, d)=(at+e,b + d)
a

(a, b)(c, d)=(ac,bd) (c, VER


V (a,b),
i
lien

Scanned with CamScanner


aa SPECTRUM DISCRETE Marites,
wees oe MAT

Nol, Given R= {Qa d)[ abe R}


GA) Properties of Addition,
GW) Let wwe R Then x= (a,b) and y= (ed)
Where a, fe dare reals,
viper add (ad =(atob+d
ER [va bc,dER= 1+ cb+dep

‘Ris closed under addition,


(i) Let wave E& Ry then v= (a,b), y= (Gd and z= (¢/)
Where ay ded, ef are reals,
then WEY) HERD GAD AGN -@tabt Ot @N=latI+eb+a+f]
an xe (vt da@dtlod tes] =(a,b)+(cted+f)=[at(c+e),b+(d+p)]

= [(a te)te (b+ d+ /\


[ a,b,c, d,e,f are reals and Associative Property holds for reals]
(vivytzrexdt(pts) Vay zeER
Gii) Foreach x= (a,b) ER, there is O = (0, 0) € R such that
x + O=(a, 6) + (0, 0) = (a +0, b +0) = (a, 6) =x
similarly 04 x =x
ri QexeOtx,
so that (0, O)is the additive identity.
(iv) For each x = (a, 4) & R there is y= (-a,- 5) ER such that

y+ pe=(a,b)+(- a,b) (' a,bER > —a,~bare reals)


= [(a+ a), b+ (-d)] = [0,0] =0
similarly vt x= 0
veyeOuypry
v= (-a,~b) ER is the additive inverse of x E R.
(vy) Vxv ER, where x= (a,b) and y= (c, d)
vtype(a,b)t(ed=(a+c,b + d)=(e+a,d+ b)
[ For a, b, c, d reals commutative law holds}
=(c,d)+(a,b)=yp+x
xvtpepyty VxyER
(B) Properties of multiplication
() Let x» ER. Then x= (a,b) and y=(c,d)ER
ati xy = (a, b) (c,d) = (ae, b dd)
ER [ abedER & ac, bdare reals)
.
R is closed under multiplication.

Scanned with CamScanner


y

‘ gtRUCTURES WITH TWO


\
Binae
ry e
OPERATIONSINS, Ri
i _ NGS, INTEGRAL DOMAIN AND FIELDS 415.
Fant Re Then £= (a, 6), Y=(¢,d), z=
a a,b, & 4, efare reals
le=aadDEedN =(a "
iM
de “OogDS)=(a(ca(ce)e),,5 b(df))
[~ (ac)e=a(ce), (bd
f= (af)
as

Associative Property in reals under multiplicati


i .

on holds]
e
. . .

apd x z) = (a, b) [(c, d) (e,f)]

= (a, 6) [(ce, df) =(a(ce


), 5 (df))
pene aye“*02) VOyZzER,
(¢ Distributive Laws
Let x02 ER. Then x=(a, b), y=(c, d) and 2=(,f)

wnere 4, 5, ¢, d, e, fare reals,


x.(yt=@ [lcd +(e,f)] = (a,b) (c+e,d+/f)
=lale+e), bd +f) =(act+aebdtbf)
and x.yptx.z=(@, 5) (c, d) + (a, b) (e, f)
=(ac,bd)+(ae,bf)=(ac+ae,bd+bf)
x.(y+z)=x.ytx.z Vxy,zER
Similarly (y+z).x=y.xt+z.x Vx,y,zER
Hence R is a ring with respect to given operations.
Also
V x,y ER, x=(a,5) and y=(c,d)ER

(c, d) =(ac, bd)


6)a,
ry=(
=(c a, db) [.° ac=ca and bd=db]
=(c, d) (a, b)=yx .
“. Risa commutative ring under given operations.
‘umple 5.| RING OF INTEGERS MODULO n
Show the set Z, or J an {0, 1, 2, ...... , n—1} form a finite commutative ring with unity 1, under
ition modulo and multiplication modulo n. (n is prime >
1).
‘Given Z, or Jn= {0, 1,2, 3,...,2-1}, n> 1, n€Z.
The composition defined is addition modulo n.
“ Vabe J,, a*b or at, b= least non negative remainder r when a + 5 is divided by n
© a*b or at,b =r > at+b-—ris divisibl
by en.
= a+b =r(mod n).

be
Scanned with CamScanner
416 SPECTRUM DISCRETE MATHEMATICS

Closure property: Va,bE€ J, Osa,b<n


a+b =r(modn), where 0S r<n.
Now r€&J,,.
the closure property is satisfied.
Associativity: V a,b,c €]J,
The least non-negative remainder remains the same if
(a+ b)+c or at+(b+c)are divided by n.
(a* b)*c =a*(b*c)
Thus associative property holds in J,,.
Commutativity: V a,b EJ,
The least non-negative remainder remains the same if a+ bor b +a is divided by n.
ie. a+b =r(modn) and b+a=r(modn)
a*b=b*a,
Thus commutative property holds in J,,.
Existence of identity: V a €J,, OS a<n.
Here a is the least non-negative remainder when a+ 0 or 0 + aare divided by n
a*x0 =a=0*a
Thus 0 € J, is the identity element.
Existence of inverse : Inverse of0 € J, is 0 itself.
Also forall a €J,, a #0, n—a € J, such that
at+(n-—a) = 0 (mod n)
and (n—a)+a = 0(modn)
ie. a*(n—a) =0=(n-a)*a
Thus n — a is the inverse of a.
Hence <J,, *> is an abelian group of order n.
Closure property under multiplication : The composition multiplication is defined as :
Vabel ne a*boraX, b = least non negative remainder r when a b is divided by n is devisee

ke. a*b or aX, b =r > ab-—-r or ab=r(modn)

Forall a,b€ J, Ilsab<n

ab=r(modn) whereOSr<n
If possible, let r = 0 then a b = 0 (mod n)
> njab-0 > nlab|
But
» is prime so either n|a or n|bwhere |S a,b<n
r#0ierée J, > J, 38 closed under

Scanned with CamScanner


yee?

gTRUCTURES WITH TWO BINARY OPERATIONS, RINGS, INTEGRAL DOMAIN AND FIELDS 417 4
prt I
8 iy vty v abe J, , the least non-negative remainder remains the same if a b or b a is divided
tatl °
ner (mod n)andba =r (mod n)
ie 9
e gtb= b*a
: commutative property holds in J, or
Z,

ative # V bce I, |
" sre least non-negative remainder remains the same if (a 6) c or a (bc) is divided by n.

(atbytc=a*(b*e)
stusassociativity holds in J,
gristence ofidentity Vae J,

a*l=a=1*aasa° 1 and 1 -a leave the remainder a when divided by n


1eJ, works as an identity element for J

aributive: V a,b, cE J,
ax, (b+, c) = ax, (b+c) (+ btie = b+c(modn))

= least non-negative remainder when a (b+ c)=ab+acis divided by n

=ab+_ac
n

= (ax, b)+, (ax, ©) (- ab=a x, 6 (modn),ac= ax, c (mod 7))

Similarly (a+, 5) x, ¢ = (ax, c) +, (6x, ©)


distributive laws holds in J,

Also number of elements in J are i.e. finite

Hence J. or Z, a finite commutative ring with unity 1.

fumple 6. Let R be aring such that x*=x V x ER. Prove


C () Risacommutativering (i) x+x=-O VxER (iit) x+yp=0 > x=y.
wh (i) LetxpER > x+yER [ : Ris closed under addition]
7 (x+yP=x +y => x+xy +yxty=xty

2 + )t(ypxtxpexty [by commutative and associative]


> (x +y) + (yx + xy) = (x +y) + 0 [Property of addition in R]

> yxt+xp=0 [by left Cancellation Law for addition in R}


> yxt+xpHxyptxyp [°. By Part (i), xyt+xy=0 for xy€ R]

> yxraxy [By right Cancellation Law for addition in R]


> Risa commutative ring.

Scanned with CamScanner


418 SPECTRUM DISCRETE MATHEMATi¢g
——S
(iif) Given xER > x+xXER
w (xtxPaxtx > (etx)(xtx)=xtx > (et+x)xt(etxx=xtx

> (Pt )t(C te )axtx > (xtx)t(xtx)=xtx [- Pay


=> (x+x)+(xt+x)=(x+x)+0
> x+x=0 [by left cancellation lay)
Hence the result.
(iii) We have x+y=0
eres [ ~ ByPart (0, x+x=9
= y=x
[by left cancellation law]
> x=y
Hence the result.

Example 7. Show that set of real numbers R is a ring under the composition ® and © defined as
a® b=a+b+landa@b=a+b+abWV abER.
Sol. (A) Properties of Addition
(i) Let a,b€R,thenat+tb+1ER>aGbER

R is closed under the operation ©


(ii) Let a,b,c,ER
Then(a@®b)Gc =(a+b+1l) Ge

=(atb+l)+ect+l=atbt+ct+2
and a ® (b@c)
=a @ (b+ c+ l)=at(btet+l)+l=at+bt+c+2

(a®b)@c
= a@(bGc) V abh,cER

R satisfies associative property. tu

(iii) For each a € R, we want to show there is e € R such that aol


a® e=e®@ a=a

ie at+et+tl=et+at+l=a>e=-l1lER
V aER, there is an identitye=-1ER
(A)
such thata ® e=~e ® a=a
(iv) For each a € R, we want to show there is an element
b € R such that
a® b=e=b@a
=> at+b+1=-l=bt+atl
> =-2-aER (a ER)
VaéER, there is an inverse elementb=—2-—a@Rsuchthat a@® b=—1=5@a

Scanned with CamScanner


we

ic STRUCTURES WITH TWO BINARY OPERATIONS, RINGS, INTEGRAL DOMAIN AND FIELDS = 419
ss 4 ,

6 b=atb+1=b+at+] (- a,bER > a+b=b+a)


.» Now
| 0) =b@a

a® b=b@aVabER
| ° R satisfies commutativity.
ertie of multiplicati
oes '
on
pry abER * a+b+abER
>» a®bER (- a,b
E R(reals) > a+b, abER)
Ris closed under multiplication.
|g (@® b)Oc =(atb+ab)Oc
=atbtabt+ct+(atbt+ab)c=at+bt+abtctactbctabc
| =atbt+ctabtactbct+abe
aO(bOc)=aO(b+c+bc)
: =a+(b+c+be)+a(b+ct+bcj=at+b+ct+bcet+abtactabec
=atb+ct+abtact+bet+abe
(a@b)Oc=aO(bOc) V a,b,cER.
»Rsatisfies associativity under multiplication
0 Distributive Laws : For each a, b,c ER
aO(b@ec)=aO(b+cr
1)
=atbt+ct+lt+a(bt+ct+lh=2at+b+ct+abtactrl
(@@ b)@(a@c)=(atb+ab)S(at+ct+ac)=(atbt+ab)+(at+ct+ac)+1
=2at+bt+ct+abt+act+l
aQ(b@c)=(aOb) S(aOc) V abhcER

Similarly (6 ®@ c)O a=(bO a) @(cOa) V abhcER


Hence (R, ©, ®) is aring.

. 0 x
fumple 8. Prove that set of all matrices of form \ | (x, y © reals) with matrix addition and matrix
y
zltplication is a ring. Is it a commutative ring ?

0 x
wl LetR= fps rereat

\\) Properties of Addition
0
() Let A= . “L| and B= 0%
yy 0 y,
AEE «i 0 x, 1° x5) _ 0 x, +x, ek
O yw} 19 » 0 Wty,
> Ris closed under addition Ce x, +x, reals and y,+y, € reals)

Scanned with CamScanner


ea | ‘

420 SPECTRUM Discreet,


CRETE Mary, \ bg
= + er 4
iy vy,

(ii) LetA
i) Let A=] 0 x] B-
B= *2
, c./°
id x, |
+ where x), x
PhD
Xysd pe a0¥, are (
VU HQ Hy ae reals
ened Lo 32] " %]
fo ex: | fo x 0 (x,+x,)+x |
(A+B)+C=j)
: jo
F Flaj |
y+F2|
Fi = bia ean
¥3| 0 (+ ¥))+9;

7 fo x, ~Y2°°S
+(x, +17.)
0 ¥ +0, +95)] (" Associative pro
P PEMY Holds for eas

=
[9 x, +
fo ed
-
lo Yh , 0 ¥ 7);

=A+(B+C)
=> Associative property holds in R under addition.
0 x . fo 0
(it) For each A = € R, there is
O= ER '
Oy [0 0 |

such thatA+O=|
fo x+0] fo 0 0+
“ -| “laa and O+A= *).|0 * =A
{0 ¥+0 |0 y 0 O+y» 0 y»
A+O=A=O+A

=> o- [9oo;., |...


| is an identity element of R.

(mm) Foreach
0 0 -
A= * ER, there is B = * ER
0 y 0 -y

such that A+ B= . = _- =O
0 y+(-y)} [0 0 9
ond B+ A= QO -x+4+x . 0 0 <0
O -y+y 0 0
=
Le A+B=B+rA=0
=> H=— Ais inverse element of R

fo x QO x,
(vy) Foreach A= jo and B= .
4 OY)
fo x, +3 . 0 t,x,
ArB= = i reals
(«" commutativity holds in re
Lo + ¥2] [0 yy +>,
0 x, O x
a 0 y {tle Mle B+ A
yy [ “
+1,

> A+B HtA


_SSSSSSm,._

‘» matrix addition is commutative.

irene es

Scanned with CamScanner


i

As FIELD 421
INTEGR UL Doster
ps with TWO BINARY OPERATIONS,
Riss,

lication
R is closed under multip!t |
x,
J, ER
‘O
"2 | and C= {5
7 f }
| [0 *]
yyy
p-!° }9 ¥2} { ¥3 |
2 Let A= 0

= 7
> r
VT
- -\r 7 r
xy} 1° (x, a
x. ip x; | \° ¥,¥2|)°
0 x, ||0 : } : ya t
*- 11] =
¥ }i° yz; 4° ¥, ¥z os| ¥s | ” O% ¥2) 3}
sow (ABC = 11g , |\0 - -
“FOL “Jyh

1 Of VW 1) [0 x, Ory % »
r WE 1

vi fe yy {lo xsl} [9 milo ¥2 yy] [Om Oz 93?!


ABO) |g
fk Ory FG HDD)
| ¥, Oy wate, FDI
9 (x, y2) 5
\ wn resis |
as acsociativity hokds
u0 (yy ¥2)¥s =

~ A(BC)=(AB)C
> matrix multiplication is associative.
© Distributive Laws

0 x f Oo x, fo ry ]
let A= I ” 2| ¢- ER
F oc ft y, | e Lo ¥
< L

xX, fo "| . x, |, rye!


0 x, | if
\
New A(BE OCS TH tly oli to yO vs ora}
Om} Uo yay LO Magy Me Ps
0 vy +4)! O x Vy, i oO x, ¥3|

+») O yyy oO», ¥y |


0 yy

7 0 x) fo xy , 0 x, 0 *y

0 Y |o v5 0 yy 0 ¥y
iL 7

=-ABe AC

Scanned with CamScanner


SPECTRUM DISCRETE M
422 ATHEMAT
“MAT
——s
¥

Similarly (B +C) A= BA+CA


:. ° distributive laws holds in R

+ Ris aring under matrix addition and multiplication.


To Check Commutative Property
0 x 0 x,
Let A= and B=
0 yy 0 yy

: Oxy 0 x,y
AB= 172 and BA= a
0 yy, 0 ys;
Clearly AB = BA
so R is a non-commutative ring.

EXERCISE 2.1
(a) Prove that <Q, +,-> where Q is the set of all rationals, is a commutative ring with unity.
(b) Prove that <R, +, > where R is the set of all reals, is a commutative ring with unity.
Prove that the set of matrices of order2 forms a ring under addition and multiplication of matrices.
Prove that the set G= {a+J2 b|a, b€Q} where Q is the set of rationals, is a ring.

Prove that the set R = {(a, b) | a, b © Reals} is a ring under the addition and multiplication of
ordered pairs defined as
(a, b) + (c,d) =(a+c,b +d)
(a, b) (c, d)=(ac-—bd,bc+ad) VW (a,b), (c,d) ER.
. Show that the set of rational numbers Q is a ring under the compositions ® and © defined as
a@b=a+b-1 and a@b=at+b-ab. VabEQ
Let G= {0, I, 2, 3, 4, 5}. Define the compositions @ and © on G as p @ q = The least nom
negative remainder on dividing p + q by 6 for all p,q €G and p © q = The least nom
negative remainder on dividing pq by 6 forall p,g € G.
Show that G is a commutative ring with unity under @ and ©.
7. Show that the set G of all real valued functions of x, defined on [0, 1] is a ring, under the additioa
and multiplication defined as below :
f

(f+ @=f)+2@) Vx [0,1]


| 4

(f2@)=f@gt) VWxE[0,1) where fg EG.


8. Let G be the set of all real valued functions on (— 0, ©). We define addition and multiplicatio8n
below

(f+ a) =f)+gx) VAg EG, xE(-%, ~)


(FX 9W=S[g@] VAgEG rE(-%, 0) . ite
Is G is a ring under these operations ?

Scanned with CamScanner


* a
y¥aoE€ R. Show that < R, +, X > is a commutative ring.
7

_ |
Give an example of the following :
i
it @ A commutative ring without unity. (6) Anon-commutative
resring with unity.
a
(d) A non-commutative ring
() A ring with zero divisors
if Risa system satisfying all the conditions for a ring with unit element with the possible
; exception x +y=ytx VxyvyeER.

prove that this axiom, that x + y= y + x also hold in R and thus R is a ring.
real continuous functions.
R > R: fis a continuous function} ie. F is the set of all
ThenF={f:
[2 ut F forms an infinite commutative ring with unity under the operations of addition and

multiplication defined by (f+ g) (x) =f(x)+e(x) . forall fg & F andx © R( fg) (x) = fix) g(x),
forallfig © FandxER.
Let X be a non-empty set. Let P(X) denotes the power set of X (ie., set of all subsets of X). Then
p(X) forms a commutative ring with unity X, under the operation + and .defined by
A+B=AAB=(AUB)-(ANB)AB=ANB.
Note : If the set X is a finite set, then <P(X),+, -> forms a finite commutative ring with unity
4, Let Ry and Rz be two rings. Define R= R, x R: ie R=R, X R;= {(a,6):a@E Ry ands E Rp}.

Then R forms a ring under the operations + and - defined by (a, 6) + (c, d)= (are, + a) and
(a,b). (c,d) = (ac, 6 d) forall (a, 5), (c,d) € R.
The ring R is called the direct product of rings R, and R2.
Note : (4) Ris commutative ring iff both R, and R; are commutative rings.
(ii) R is with unity iff both R, and R; have unity.
1 1
1 Prove that the set R= (x+y-3° +2-93 5x, 9,26 Q)}is a ring w.r.t. addition and multiplication.

ANSWERS
8 Nota ring
10. (a) Ring of even integers (6) Ring of * m matrices over reals
(c) Ring of 2 x 2 matrices (d) Ring of2 + 2 matrices over reals
12. Properties of Rings
We now prove some basic properties of rings. Before that we point out following conventions which
¥ shall follow henceforth :
Let<R, +, > > be aring and a, 6, c,d€ R.
\. We often drop the symbol for multiplication and write a-b simply as a 6.
2. Multiplication is assumed to be performed before addition [accordingly, a b + ¢ d stands for
(ah) + (cd).

Scanned with CamScanner


424 SPECTRUM DISCRETE MATHEMATIC
— ha,

3. We write a—b instead of a + (— d).

4. We refer <R, +, - > as a ring R under the operations + and - or as a ring R together with the
operations + and - or simply as a ring R.
Theorem : Let R be aring and a, 6, cE R. Then

() a0=0a=0 (i) a(-b)=(a)b=-ab =


(iit) (-a)(—b)ab
(iv) a(b-c)=ab=ac (v) (b-c)a=ba-ca

Proof. (i) a0+a0=a(0+0)=a0=a0+0


> a0=0. [By left cancellation law in <R, +>)
Similarly 0-a=0.
(i) ab+a(-6)=a(b+(-b))=a0=0
=> a(-b)=-ab,. [By definition of additive inverse in <R. +>}
The proof is similar for the other part.
(iii) (- a)(— 6) =-(- a) b= -(- a )), by using (ii).
This completes the proof of the theorem.
Now inthe group<R,+>, -—(—x)=x forallx ER,
where it follows that —(-ab)= ab.
(iv) We have a(b- c)= a(b+ (-c)=ab+a-c)=ab-ae. [By Left Distributive law]
(v) Wehave (b-c)a=(b+(-c))a=ba+(-oc)a
=ba-ca. [By Right Distributive law]
In particular, we notice that (— a)’= @° for all a & R and if the ring R has an identity 1, then
1 (I) =(- D1 =- land 1) (-D= 1,
Considering the property (i), the additive identity 0 is called the zero element of the ring R. Also we note
that if | = 0 inaring R, then a= a-1 = a-0= 0 for all a € R. So in this case R becomes the trivial ring {0}.
The following identities are easy applications of distributive laws :
Theorem : Let R bearing and a, 6, c, dE R. Then
() (at+b)(c+d)=actbet+tadt+bhd (if) (a—b)(ec-d=ac—bec-ad+bd
(ii) (a+by=a+ab+bat+h (iv) (a-bY
=a -ab-bat+h
(v) (a+b)(a—b)=a@-ab+ba-b
Proof. Easy exercise. 7
(Using Distributive Laws)
Theorem : Identity element in a ring R is unique.
Proof. Suppose e, and e, to be two identities of ring R. Then, xe;=e,x=x and xe,=e,x=x VxER

Then from the first equation, replacing x by e, we get e,; e2 = e, and from the second equatics
replacing x by e,, we get e; 2 =e). Thus e; = é7. Hence identity of a ring is unique.
0

q
ipeTakal

Scanned with CamScanner


~ woo 5 & ‘=

1GEBRAIC STRUCTURES WITH TWO BINARY OPERATIONS, RINGS, INTEGRAL DOMAIN AND FIELDS 425
4
sae

qreorem ? Let R be a ring having a unique left identity e; . Then e, is also the right identity.
te | proof: Let R be a ring with a unique left identity e, ie, e.x=x,Vx ER.
We show that e, is also the right identity of R ie, xe,=x,VxER.
Letx© Rand Vy ER, we have (x e,;-—x+e))y=xe,y—xyteyrxy—xyt+yp=y
> xe,-xte,_ is alsoa left identity of R.
But left identity e, is unique.
xey-xte,=e, > xe,=xVxER
ie, e, is also the right identity of R.
pefinition. An element x in a ring R is called idempotent if x* = x.
forexample : In M; (R), the ring of real square matrices of order 3, the following matrices are idempotents.
100 2-2 -4
| A=|0 1 0|],B=|-1 3 4] forA?=A,B?=B.
000 1-2 -3
There are rings in which every element is idempotent.
Definition. A ring R is called a Boolean ring if every
element of R is idempotent, i.e., ~=x for allx ER.
We prove the following interesting properties of a Boolean ring :
Theorem. Let R be a Boolean ring. Then
() 2x=0 forall xER: (ii) xy=yx forall x yER.
Proof.
(i) Let x ER > —x ER.
Now x=x?=(_xy=_x [' Ris Boolean Ring]
x=—-x => 2x=0 forallx ER.
(i) Let x,y ER. Then
X+yHatyPaPtxyptyxtyYaoxtxytypxty

> O=xyt+ypx
. -(1)
(using cancellation under addition)
SOxyEeER xs 2xy=0
fist of (/)]
> xytrxy=O-xytyx
= eye. (using
[By left cancellation (1))
law]
-true* From the abo ve theorem we see that a Boolean ring is a commutative ring but the converse need not

rexample - .
ple : The ring Z is a commutative ring which is not a Boolean ring.
W © Now define the invertible elements of a ring under multiplication.
finit;
: on. Let R be a ring with identity 1 #0. Then an element u ©R is called a unit (or invertible) if
| Reng xiim
© R such that wv =v u= 1. Then v is called the inverse ofu and is. denoted by w7!,
iy TK: Uni ty must be a unit but every unit is not unity.
*Xample : In Z, the ring of integers, the unity is 1. where as units are 1 and — 1.

Scanned with CamScanner


pn i |

426 SPECTRUM DISCRETE MATHEMATICS


es

Theorem. Let R be a ring with unity 1 (= 0). Then the set of units of R forms a group unde
multiplication in R.
Proof. Let U= {u © R:wv=1=vuforsomev GR} ie. U be the set of units of R.
We will show that <U, - > is a subgroup of semigroup <R, - >.
Cleariv U =¢, for 1 © U.
Letusy GU, then dw wo! Rsuchthat ww =l=w'!wanduel=l=v'v.
Consideruv (vbw =u Wwe wl=ulwlt=uw!=1.
Similarly (a ')(uvy=v"! @@wverlv=ul v=.
we(e a )=b=( lw ')uv = uvisaunitand(uvy'=v'w!and souv EU.
Alsol Eu
”. Foreachu @UL3 Ww! GUsuchtha uw!=1l=u'u = uw! EvuU,VuEv.
Hence U forms a subgroup of Semi group <R. - > and so U forms a group under multiplication.
Definition. An element x in a ring R is called nilpotent if x"= 0 for some positive integer m. The smalles
positive integer (forx) with this property is called the degree of nilpotency of the element x.
Remark : The zero element in every ring satisfy the relation 0' = 0.
But there are non-zero elements x in a ring also.
For example:
00 1 ] l 3
In the ring M; (R). if we consider the matrices A=|0 0 0 and B=| 5S 2. 6 |, tha
001 -2 -1.-3
we can easily verify that A* = O and B* = O, though neither A nor B is the zero (null) matrix. Thus ina ring
there may be some non zero elements whose integral power is zero for some positive integer greater than |.
Theorem : The sum of two nilpotent elements of a commutative ring is also nilpotent.
Proof. Let R be a commutative ring and a, b € R be nilpotent elements such that a™ = 0 and 5" = 0 for
soMe positive integers mand n. Now
(a+6)""* = gern. mh a" nip + + mane arts! br +. .+ pute

=q" {a"+ a a’! bt+...... + as B"} - § ae a b+...... + b™} B"

=0. (-- a™ =O and 6"=0)


=> az disalso nilpotent.

ILLUSTRATIVE EXAMPLES)
Example 1. Ifa ring R has no non-zero nilpotent elements then prove that for any idempotent e of R
ex=xeVxrER.
Solution : V x € R,

Consider (re — exe) = (xe — exe) (xe — exe)


= xexve —Xeere — exexe + exeexe

= xere—xe"xe — exexe + exe“xe (But e is idempotent ie. ¢=4


= xexe—xXexre — exere + exexe
=0.

Scanned with CamScanner


_427
1c STRUCTURES WITH TWO BINARY OPERATIONS, RINGS, INTEGRAL DOMAIN AND FIELDS
ot A
g no non-Zero nilpotent
y
Rha
BY xe- exe =0 > xe= exe. ..(1)
2

gjenilarly (* — exey = 0 > ex — exe=0


> x exe. (2)
an d (2 ), w e get ex = xe.
from (1 )
ie 2. If R be an algebraic system satisfying all the axioms of a ring with the possible exception of
per = -aVa,bER. If there exists one element c € R such that
+h
,
gobo a= b¥ a,b ER, prove that R is a ring
€ R.
4 va.be Rand givenc
v
be+ be. (1)
consider (a+ 6) (€ * €) =a(ct+c)+b(c+c)=act+ act
t .-(2)
Also (a+ b)(e+e) =(at b)e+ (a+b) c=act+bet+acbe.
get
from (1) and (2), we
actactbc+be =ac+be+ac+ be.
Using cancellation laws under addition, we get
ac+ be =be+ac
> (a+ b)c =(b+a)c.
iy

By the given property of c, we get


a+b =bta
7 he

> Risaring.
Iumple 3. If R is a ring with identity such that (x yf = 2x’y for all x, y © R, then show that R is
xmmutative, Give an example to show that the above result may be false if R does not have an identity.
ol LetR be a ring with identity such that (x y)* =x" )* for all x,» © R.
Letx,yG@ Rbe any elements -. p+1ER.

[k@+ DP =x (yt IY
(xytxy Hx (7 +2y+1)
(yf +txyxret
ry ary t2xry tx
YY +Ptxpxrivry =e ypt2rytr
xyx =x'y. ---()
Replacing x by x +1 in(1), we get
(+I y@+l =@F)y
(xy+y)(x+ 1) =(°4+142x)¥
Xyxtxytyxt+y =x yptyt2xry
¢)
ytxptpxtyaxrytpt2xy {using (1)]
yx =xy.
R is commutative.

Scanned with CamScanner


sd
=
SPECTRUM DISCRETE MATHEMATICs |


428
example :
ed
Now for the requir

Let R= (eo |:v


Va aser| C M; (R). It is easy to verify that R is a non commutative ring.

(KC)
c

ef cle [se
Let A.B ER. Then,
d
A= a 8 ,B= : for some a,b, c,d ER.
0 0 0 0

Then it is easy to see that


[u
2 oe Ae B: =
atc? a? “4|
(AB) | 0 0
w
1 0
Note. Here we notice that! 0 | is a left identity ofR and R has no right identity.
L
Example 4. If in aring R, x’ = x for all x € R, then show that R is commutative.

Sol Let x, vy € R be real numbers, then

(vy-Pyryeryry-ry ry re yx
ryr-PyPPryt
(
ry rye
wPyry-ryryr-ryrye
=0 [v ste]
e rry- ? yr #0)
> r y ~ xy ¥ x’.
A
Similarly, (yx? =r yx Po y vr yx - yx Cyr ox y v y Vo tx yr’ yx
wy? prey ry Par ye pete ye yee |
=O
7
® yr-ryr=0
- year yr, ee
From (1) and (2), we get

* yeye, moe
Further, (x? - x)= 7 ~ x yur Vr |
= 4-drePey

~* 3x30

Scanned with CamScanner


=

N AND FIELDS 429


pcTt iRES WITH TWO BINARY OPERATIONS, RINGS, INTEGRAL DOMAI

we 12 (x xv) (Px) = 27-2083 0-P -3xtx


pu (?-x) = 6 — wE™ .
a 3yi x Se re

aiecxt
2 ---(4)
ges) 7
Th us from (3); we get

we - = y(t =x)2 [using (4)]


)
(x- eyy=y@-*
2

yr.yx
tyr -yx
°2 xy x
2 [using (3))
pry aye ye
xyp=yx.
2
utative.
Hence, R is comm
Sh ow th at a cobmmutative iff
riangtbR +is2a
mp le 5. (a+b y a
[u for all a,€bR.

mmutative ring
gl Firstly, let R be co
>» agb=ba,foralla,bER.

Now (tb =(atb)(atby=atab+barb


[ ba=abj
agtabtab+h
aa¢tlab+b=a+b+2ab.
Coaversely, Let (a +b) = +t2ab+b’, foralla,heE R.
To show that R is commutative.
Since(at bY =(at b)(at bea tabt barb
i2ahth=etabtbarh
* ahtab=abhiv ba * ab=ba,
Hence Ris commutative ring.
lample 6, (a) Show that any cng of prime order ty commutative,
(hy Show that wring of order p where pis prime, need not be commutative.
bl,
Ma) Let be a fing of prime order p
“Rot is aeyelic group
Take « Ho tae.
a
‘wthat O(a) = O(
R) © p
Let .
eX, ER be any elements
ela, t, ema for lL mee

Scanned with CamScanner


2 a —

SprcraiM DISCRETE MATHEMATICS


430

Now xx, = (a) (ma) =imarm = mia®y ~(may(la)> x54,

2 xx, = Lt, for Ets Ek

Rois commutative nag

a io oy!7} ie fr 17
(6) Let R™ sig oflo afjo 0| \ ||
iL dt \

be set of2 « 2 matrices with second row having zero entries


Here R is a ring under matnx addition and multiplication.
r
Aso oj fo [i ang \ 1] b
3
l(c
r r

0 0}
9 0} jo o| 0 0} |0 0} jo 0
ie Ris not commutative
and it is rmg
of order 4 =2? where
2 is prime

EXERCISE 2.2
1. Prove that a mag K has no non zero nilpotent elements if and only if 0 is the only solution of the
equation x = OinR.
Show that in a ring R. a pon zero idempotent cannot be nilpotent.
sd

3. Which of the following algebraic structures <R, +, - > forms a ring ?


(i) Let X be any set and R ~ the power set of X. Define A+ B= AU Band A-B=AOB for allA
BER.

(ii) Let BR be the set of all real —valued continuous functions defined on R. Define
(f* wbx) < fl) > gtr) and( fog) (x) = f(y (x)) forall fy © R and for all x © R.
4. Let Robe the set of all real numbers and F be the sect of all real-valued continuous functions defined
on R Define (f+ op) Ory = fx) * ¢ dy) and Cf ¢) Or) = f(r) ¢ Ox) for allf, gy © F and for all x € RK
Show that <F, + ,- > i anng under the binary operations defined above.
5. Let Q be the set of all symbols a * a, 1* a;) * a) &, where a, G R,r = O, 1, 2, 3,. Two symbols
Gxt aytta;y tay kand hy + bith, j + bk are considered to be equal if and only if a, @ &, re &
1, 2, 3. Define addition and multiplication as a formal sum and product using the following
relations; P=fek=-lije-slohjhn
hpi kin -ik~j.
Prove that Q is non-commutative ring with identity (this ring, is called the ring of real
quaternions)
6. Give an example ofa ring which contains clements a, 4 such that
() (atby ear 2abeb (i) (ar byla-byaa’. BF
7. Show that if1 — ais invertible in aring Ro with | then so is 1 - 2 a and that
(t-bay'<1+db(l-ab) ‘a.
&. Show that [rx] € Z, is a unit if and only if ped (4, my 1,

+ -

Scanned with CamScanner


| 4
.

. an RES WITH Two Brvagy Oren rom, Bows. INTECeaAL Doster aNp Piet os ax}

« Ua nilpotent in the ring R. Show that |—a@ is a ent,


72 if x.y,
2, fare any elements of ring R, prowe that (x ~ yp ir ~~ iste xO -tele yet
i Let “G. * > be an abelian group and R be the set of all emiomorpieems of G Define
he

(f+ g(r) fr) + ptr) and (fog) (2) > f(g (xy for all fg & & und for six © G Ther
“R. +, > is a ring (which is called the ring of endemorphinm of ©}
eee

ANSWERS
ee
bee

1 ()Notarng (i) Nota ring


he

j
haat agile

ii lategral Domains, Division Rings and Fields


In this section, we develope rings with certam special conditions. To Segm wath, bet ws commader the
ew epee

se reg Z,. the ring of imegers modulo 6 In thes rimg. if we take the product of to mom sere clement,
tae oe os

= Bd [3] (modulo 6),then we have [2}{3] - [6] © [0] Thus thes mng hus ome pom mero gate whose
midact18 zero (additive identity) of the mng. Also im M,(R), the nag of2 * 2 matrices over the set of real
: f Tf 1 To o
uters, we have { 7112 3 lel i |, but nome of the elements on the feft hued sade & aero inafl
Lo oj}(o t) Lo 0}
."Ta)
of the ring. In general, we define the following
hiaitions -

Left zero divisor : Let < R. +. - > ts arom An clement oc © R is called 4 Weft pero diteor if
P20ERst who
\ Right rero divisor : An clement a & & is called a right zero divisor if FA OE Ret band
Zern divisor : An clement a € R is called 2 2erv divisor if be 0 ER
2 ab =O = by.
Nate 22 € R is a zero divisor.
oper rero divisor
, Azclement a # 0 € R is called a proper rere divisor ifdA er OER st ew 0 - de
“te ffuisa Proper ero divisor of RL then 4s alse a proper cero divisor of R.
” : Moths >A rng R ts sant to satisfy left [ght] cancellation property if for all a. Ale €& RL ow # 0 amd
i, PS aefresp Ag = ¢ a) mmphes that 4 = «.
4 The following theorem establishes a relation between cancellation property for sveltaplcation and
4 Qvpors in a ting R
if ; Reorem : 1 et Rbea ring Then the followtg coadinons are equivalent >
| 1) Rha no sero diviswes | (4 R satisties lef cancellation property ;
4 ‘OR satisfies mght cancellation property,
7 rset. (1) * (1) Suppose the ring KR has no sero divinors.
lease
ce Ria #0 antab<ae. Then
{ 4 c-0
j * b-c-0
a5 « # Oand R has no rere divisors.

= bee.

ll

Scanned with CamScanner


432 SPECTRUM DISCRETE MATHEMATICg
ATICS
(ii) > (i): Let the ring R satisfy the left cancellation property.
Let a € R be such that a # 0, Suppose 6 & R such that ab @ 0.
Thena b= 0 a0
=> b=0 by the left cancellation property,
Now if ba=0 then ford #0, ba=0= 60
> a=Q, which is a contradiction.
Thus 4a=0 > b=0,
So a is not a zero divisor. Hence R has no zero divisors.
Similarly we can show that (i) (iii).
Hence the conditions are equivalent.
Now a ring may or may not have zero divisors. In fact, the ring of integers, rationals, reals or
complex numbers do not have zero divisors. Consequently, the rings without zero divisors deserve
particular attention.
Definition : A commutative ring R is called an integral domain (in short, 1.D) if R has no proper zero
divisors.
ie Va bER,
If ab = 0, then either a= 0 or b=0.
or Ifa+0,5
= 0,thenad = 0.

For example : (i) The ring of integers Z, ring of rational numbers Q, ring or real numbers R and the ring
of complex number C are integral domains.
(ii) The ring Z, of integers modulo 6 is not an integral domain.

Here 2 = 0 (mod 6), 3 = 0 (mod 6), but 2-3 = 0 (mod 6).


For the ring Z, of all integers modulo a positive integer nm, we have the following nice
characterization:
Theorem : For any positive integer 7, the ring Z, of all integers modulo 77 is an integral domain if and only
if mis a prime integer.
Proof. Let Z,, be an integral domain. Then [1] # [0] in Z, and hence n> 1.
If 7 is not prime, then 7 =p q for some integers p,q where 1<p,q<n.
So we have [ p} [q] = [n] = [0], but neither [p] nor [q] is zero of the ring Z,, .
This contradiction shows that n must be prime.
Conversely, consider Z, for a prime integer n.

Let {a}, (6] € Z, \{[0}}.


Now if [a] [5] = [0], then [a 5) = (0), ie.. n divides ab.
Since n is prime, we have either n'a or mb.

Scanned with CamScanner


ee = ——— Pa or SS

gate +STRUC TURES WITH TWO BINARY OPERATIONS, RINGS, INTEGRAL DOMALN AND FIELDS 433

yo
. poth the cases are not possible as [a], [b) # [0] de. a & O (mod n), b #0 (mod n) Le. mn does not

gd and 6b.
is an integral domain,
. therefore Ln
an independent |
rough the next theorem follows immediately from Theorem 5.2.3, we provide
fol it. ' 7 . : to os
mA commutative ring R with identity | # 0 is an integral domain if and only if the cancellation
vs id for multiplication.
at LetR be an integral domain. Let a,b,c ER, a # Oandab@ac.
2 ° -c)=90
» b-c=0, as R is an integral domain and a # 0. So we have b = ¢.
law holds. Let
seas let R be a commutative ring with identity, in which the cancellation
ye Rbe such that a Oandadb=0. Then ab=0=a0
t > b=0 (by left cancellation law).
. Thus R has no zero divisors, hence R is an integral domain.
In view of the above theorem, we note that if every non zero element of a commutative ring R with
> J city is a unit, then R satisfies the cancellation law and hence R is an integral domain.
a .

xfinition. A ring R with identity | # 0 is called a division ring or a skew field if every non zero element
éRisaunit (ie. ifforany aE R, a # 0, there exists an element 6 € R such thata b=ba=l),
Note that if R is a division ring, the set of all non zero elements of R forms a group under
rztiplication.
fheorem. Ina ring R with unity 1, units form a subgroup of the semigroup <R,- >
Or The set of all units of'a ring R with unity is a group under the multiplicative operation of R.
tof: Given <R, +, - > is a ring with unity 1.
Let U be the set of all units of R.
Since LS
1 =1 > 1 EU.
U is anon empty sub-set of R.
’ Va,bGU +a, bare units of R
> JAagdeRst.
ac= | = caand bd= 1 = db.
Consider (ab) (dc) = a (bd) c= a (I) c= ac= 1.
Also (de) (ab) = d(ca) b = d(1) b= db=1
(ab) (dc) = 1 = (de) (ab)
> abisaunitie. ab GU.
Also ge= 1 = ca > c=a7'isaunit
= a@' Eu.
Hence U is a subgroup of <R, - >
Le. U itself is a group under the multiplicative operation of R.

a il

Scanned with CamScanner


434 SPECTRUM DISCRETE MATHEMATICS

Another definition of division ring


A ring with unity in which all non-zero elements form a group under multiplication is called a
division ring or a skew field.
Definition : A commutative division ring is called a field.
ie. aring R with identity 1 0 is called a field if R is commutative and every non zero element of
R is a. unit.
For example : The rings Q, R, C are division rings which are fields.
Note that there are division rings which are not fields. We have the following example :

[ILLUSTRATIVE EXAMPLES]
Example 1. Give an example ofa division ring which is not a field.
a _
Sol. Let R = | 7 M,(C) | @, Bdenote the conjugate of a, 4 .

Define addition (+) and multiplication (-) in R by usual matrix addition and matrix multiplication,
We now show that R is a division ring but not a field.
Let a=| avid c+id Be r+it ut+iv =e
-(c-id) a-ib -—(u-iv) r-it

Then,
(atr)+i(b+0) (ctu)t+i(d+v)
A+tB=
—((ctu)-i(d+v)) (atr)-i(b+0
5 ; ; : 0 0 :
Also it is a routine calculation to check that AB & R (verify !). Observe that O = ke | E Ris

the zero element and for any A & R, we have — A € R such that A + (— A) = O. Further, the distributive
1 0
properties hold. Hence R is a ring with identity IJ = E | ER.
. i 1 i i l l i 2i
ER
0
Now, and , ~
a on || hl [ “i
Pioa|f i fifo 2
PE Vpel wt] [-2 of
This clearly shows that R is a non commutative ring with I # O.
ret [ atib ct+id
. | -(c-id) a-ib
be anon zero clement of R. Then cither a+ ib # 0ore+id #0
ie, citherra +b? #00r +d #0.Hencea +h +e +a #0.

|
=

Scanned with CamScanner


. to ee m
ee

ucTURES WITT! — BINARY OPERATIONS, RINGS, INTEGRAL DOMAIN AND FIELDS «435
EE pte + d # 0. Observe that i a-ib c-id
is the inverse of
pet k= a" k —(c+id) a+ib Jer
|
ib aA ER

Ae
ence each non zero element of R has an inverse in R, whence R is a division ring.
en
as Ris non commutative, clearly R is not a field.
put
a b
Take F = {| b | ja,be R|
Je 2. ‘ Let R be set of all reals. ~
gop’

s.
ye F is a field under usual addition and multiplication of matrice
Pro’
5
Ee Fie. A= ab ,B= a 8
a
where a, b,a,BER
:
—_ a -B
a( Let As B

| AtB= ata 5+f where a+a,5+BPER


” —-(b+B) a+a

e F = Addition is closed.
is associative
(i) We know matrix addition
(A +B)+C=A+(B+C).
For A, B, C & F, we have
a b . 0 0
| e& F, there is O= Ee sl EF
wrora=| 5

O a b 0 0 a b =A
h 0 = ab @ >
such thatA + = —' a + _0o

b =A
0 + a b = a
andO+A= 0
-0 O —-b a —b a

ie AtO=O+A=A

wrwr[ 2 Merumena-[¢ “ier


.
-b a
= -b

suchthtA+B=O=BtA

B is additive inverse of A
(v) We know matrix addition is commutative

le for A, B, © F, we have
A+B=BH+A.

Scanned with CamScanner


ay,
=.

P
SPECTRUM
5
DiscrE Tr MATE :
\tic '

& Fand B=
& A -fi . a
er
cS
ey
we have

AB =! aan-dhB opeeal™ aea-bB aBp+ha EF


i »
+1-Oa-
L
2B -dP+aa} |-(a@P+ha) aa-dp

(vet) We know matrix multiplication is associative ie. for A, B,C & F


we have A (BC) = (AB) C

fees 1 0
it) For A= | ; J eremeite|, | EF santaratats=a
Lt @
ere! «to

Here Lis unity of F.

r —b
oiled ° JEP. thereisB- = ik | © Fsuch a
-b a} a-+b-{b a
St>

A+O
AB=BA=I => a#0,orb=0
> a +b? 20

B is inverse
of A.

aa-6bB aB+ba
(x) Here AB = |
-(aB+b5a) aa-bB

_} a a b _ aa-bB ba+aB 7 aa-bB ba+aB


maBa=| 6 Ls ‘]-| 2p 3 ea I eeean teed
oh
~
AB = BA.
a

(xi) We know distributive law holds in matrices


For A, B,C € F, we have A (B+ C)=AB+AC and(B+C)A=BA+CA
Example 3. Show that the set G = {0, 1, 2, 3, 4} forms a field w.r.t. addition and multiplication modulo 5.
Sok. Given G = {0, 1,2, 3,4}
Let x.¥,2EG

core hD > if :
x+y<5
ee Le

Define
roOoif x+y25

where r is remainder on dividing x+y by 5


So Osrs4
OO

x+,y EG

Scanned with CamScanner


OO

AdBRAC STRUCTURES WITH TWO BINARY OPERATIONS, RINGS, INTEGRAL DOStarn AND Fie L Ds 437

The composition table for addition modulo § is

+. |OP LP 23 ]4
mo Toltiatata
l 112/13 ])4 1/0
2>1/2/3laloly
3 1/3 }/4)0]1 42
$74 /0) 1/243

mperties of addition
(i) Clearly all the entries (in each column) belong to G and so G is closed under addition modulo §
closure property holds in G.
(i) VxsyrEG
The least non-negative remainder remains the same if (t+y) += or x + (y+ =) are divided by 5
(x+ 5 y)+5 2 = x+,(v+, 2)

associativity holds for addition modulo 5


(ir) For allx € G, these exists 0 € G
such that x+, 0 =x= 0+, x as x is least non-negative remainder when x+0 or 0+ x15 divided by 5

identity element 0 & G exists


(i) For allx € G, there exists 5—x © Gifx + 0
such that (S—x)+, x =0 x+, (5-~x)

inverse element 5 — x of x exists (x # 0) and inverse of 0 is 0


(¥) the entries in 1°, 2™, 3™, 4™ rows are same with the corresponding elements of 1°, 7“. 3" 4
=Smnns respectively.
addition modulo 5 in G is commutative
hrperties of multiplication
Let G, =G — {0} = 1,2, 3,4}
Tke composition table for multiplication modulo 5 is
X, |1 2 4
tod

2
fod}

4
1
3
(1) Clearty all entries (in each column) belong to G, and so G, is closed under multiplication modulo 5

closure property holds

Scanned with CamScanner


ery
|

438 SPECTRUM DISCRETE MATHEM


AT
(ij) VxpyzE G,

The least positive remainder remains the same if (x y)z or x (yz) are divided by 5
(xX, y)X, z= xX .(yX,:

so associativity holds for multiplication modulo 5


(iif) Forall x € G, , there exists 1 € G,

such that | X; x = xX, l=x

= identity element exists.


(iv) Foreachx € G,, there is inversey ofx is G, such that

xXoy=yXox=l

Since equation xy = | (mod 5) has a solution (5 is Prime)


Inverses of 1, 2, 3, 4 are 1, 3, 2, 4 respectively
=> inverse of each element exists.
(v) Also xX, y = yX, x asxy and yx leave the same least remainder when divided by 5 Positive

=> commutativity holds.


(C) Distributive Laws :
xX, (y+5 z)= xX, Y+uxX, z and O+s z) Xx = yX, x+zX, x

as xX, (y+,z) = xX, (y+z) ( y+, z =y+z(mod


5)
= Least positive remainder when x x (y + z) is divided by 5
= Least positive remainder when x y + x z is divided by 5
= xXyt xz = xXx yrs xxz [*-" xX- y =x y(mod 5)]

Similarly (y+, z)X,x = yXox+zX, x can be proved

G is a field under addition and multiplication modulo 5.

EXERCISE 2.3
1. Prove that the set Q [/2] = {a+ J2 b| a,b e€ Q} where Q is set of rationals, is a field under usual
addition and multiplication of reals.
2. Prove that i
(O(N, +, -)isnotafield (if (Z, +, -) is aring but nota field
(iif) (Q, +, ), (R, +, +) and (C, +, -) are fields. i
3. Show that the set of rationals Q is a field under the compositions @ and © defined as
a@b=at+b-1 and a@b=at+b-ab VabEQ.

Scanned with CamScanner


\ -_
FIELDS 439
TWO BINARY OPERATIONS, RINGS, INTEGRAL DOMAIN AND
Alc STRUCTURES WITH

te set R=as {(a, by a, b & Reals} is a field under the addition and multiplication of
er. Thee

Fe
‘ ordered pairs defined
(a6) * (= +c,b +d)
¥ (a,b), (c,d) ER.
(a,b) (d= (ae~P a, bct+ad)
as

chow that the set F = (0, 1, 2, ....6} forms a field w.rt. addition and multiplication modulo 7.
ee a

re
in .
Every field is an integral domain.
Fe 8

{ tbe R be a field and a, b € R such


that a # Oand ah=0.
” sinceea2 OER a is
a unit and hence a’ ' exists in R. Then
er

1b=0 => a'(ab)=a'0=0


Re

> (a a)b=0
ee

=> 1-b=0
CNet

> 6=0.

R has no zero divisors.


aes i. =.

ae

Thus R is a commutative ring with | # 0 and without zero divisors.


6

Hence R is an integral domain.


2: The converse of above theorem is not true.
For example : The ring Z of integers is an integral domain which is not a field as every non-zero
does not have inverse in Z .
But in the following theorem we show that any finite integral domain is a field.

Then SC R, since R is closed under product.


Ifaa,=a a(1 S i,j sn), then a, =a, (asa #0 .. by left cancellation law)
> — elements of S are distinct. so o(S) = e{R).
ASSCR . S=R.
>
Now satsince R contains
.
1, we have | € S ie.
s
| = aa, for some
isjsn),
2
18 unit and since this happens for every non-zero element a of R
i : . ~

very non-zero element in R is a unit.


Hence Risa field.
“Any finite non- zero ring R without zero-divisors is a division ring.
™. For Any positive integer m, Z,, is a field if and only ifn is a prime integer.

Scanned with CamScanner


>
PA

440 SPECTRUM DISCRETE MATHEAATI¢g

SS
| ILLUSTRATI VEEXAMPLES} :
Example 1. Find the elements in Z,2 which are zero divisors and elements which are not zero divisors, | i
Sol. Since Z12 = {[0], [1], [2],......, [1 1]}, we see that ‘
(0) = (2) [6] = [3] [4] = (8) [3] = [9] [4] = [10] [6] : Hl
Thus, [0], [2], [3], [4], [6], [8], [9] and [10] are zero divisor of Zy2. ipl
Also ged(k,12)=1 > k=1,5,7, 11. («. We know that [x] © Z, is a unit iff ged (x, n)=}) f

{1}, [5], [7] and [11] are units in Z). (


and so these elements are not zero divisors.
Example 2. Find the roots of x*+ 3 x—4 in |

(a) Z (b) Ze (c) Zs. |


Sol. Let f(x) =x +3x—-4=(x-1)(x+4)
(a) Now f(x)=O0inZ => (x-1)(x+4)=0inZ

2 x=1,-4inZ. i
(6b) Now Ze, = {0, 1, 2, 3, 4, 5} under addition modulo 6 and multiplication modulo 6

So f(x)=OinZ, iff (x—1) (x +4) =0 in Ze


x= 1,2,4,5 are the roots in Z,.

For (1-—1)(1+4)= 0= 0(mod6)

(2—1)(2+4)= 6 =0(mod6)

(4-1) (4+
4) =24 = 0 (mod 6)

(5— 1) (5 + 4)
= 36 = 0 (mod6
Example 3. Prove or disprove that there is an integral domain which has six elements. i

Or

Show that an integral domain consisting of six elements does not exists.
Sol. Let R be integral domain of six elements then (R, +) is an abelian group of order 6.
Since
2 | 6 and 3 |6
By Cauchy’s Theorem in group theory

Aelements a and 6 of order 2 and 3 respectively


2a=0 and 345=0.

Scanned with CamScanner


ol sTR NS, RINGs, INTEGRAL Doar ax
se 7b ER, then STAINAND FIELDS — 4.4]

7 (2) =245= (2a) b6=0-56=0.


0 nor 25=0.
| put neither 4 =
| oper zero divisors, which is not possib!
R has Pr € as R isj an int
. .
€gral domain.
|| @ . . .
ts
|, HeHence thLeert e does not exist an integral domain of six elemen
R be a ring with more than one element.
le 4. _ Let for each a E R, there exi .

xista unique 6 in R
Provethat bab=b6b ()) (ii) R is a division ring.
Mpiob a=
then <0.
whenever ax=0O
fisst of all we show that
é
consider 2 #0 CER. Then
=(ab+ax)a =aba+axa
| a(b+x)a
.
=
a +0
a [se ax=Oandaba=a]

| >» a(b+x)a=a.

But b is unique for a such thataba=a.


wehave b+x=b => x=0O.

Similarly, we can show that whenever x a=0 then x =O.


(i) Since aba=a [Pre-multiplying by 4]
> baba=ba

> baba— ba =0

3 (bab—b)a=0
|= bab-—b=0 [- wheneverxa=0 > x=0]

° bab=b.

") Nowaba=a Similarly aba=a


> aba—-a=0 => aba-—a=0

* (@b-1l)a=0 => a(ba—1)=0


* ab-1=0 = ba-1=0
> ab=] > ba=1

Thus ab=l=6@
ts a division ring.
« 86 .

“nce all non-zero elements of R are invertible and so R


.
H

Scanned with CamScanner


442 SPECTRUM DISCRETE Matiry, Ay
of
-MAT]
Example 5. Let a, b be commutative elements of a ring R of characteristic two, show that

(a+b)? = a* +b? = (a-b)*.


4
Sol. R is commutative ring of characteristic 2.

let 2Q FER

(a+b)* =(a+b) (a+b) =ala+b)+Ha+b)=aa+ab+ba+bb


=a*+ab+ba+b =a +2ab+b° (-- R is commutative) —

= q*+bh? (" ch{R)=2) |

(a—b)* = (a—b) (a—b) =a(a—b)+(—b)(a_-b)


d

aa—ab—ba+bb= a ~ab—bat b* = a* -2ab+ b? (-- Ris commutative) —


?
= a? +b? (- ch{R)=2
Example 6. Solve the equation f(x) =x* _ 5x + 6 =O in the ring Z)>.

Sol Since f(x) =x°-5x+6=0


= (x-—2)(x-3)=0.

Clearly x = 2,3, 6, 11 satisfy the given equation for

(2 —2) (2—3)=0 = 0 (mod 12)

(3 —2) (3 —3)=0 = 0 (mod 12)


(6
— 2) (6—3)= 12 = 0 (mod 12)
(11 —2) (11 —3)=72
= 0 (mod 12).
Hence
x = 2, 3, 6 and 11 are the roots of the given equation in Zj2.

Example 7. Find the field of quotients of the integral domain Z [ /2 J.

SoL Let F be the field of quotients ofZ [ /2 ]= {a+ 2 b:a.5€Z}


J { r— ;
Then F=4 [1 meZ[j2].m<o!
is 3

Teke [= a,+J25,,m= a,+4/2 b, where 9,.95,5,,56,EZ

As mez => a, +42 b,- #9

Scanned with CamScanner


f +a 7a
AL DOMAIN AND FIELDs
. agsic STRUCTURES WITH TWO BINARY OPERATIONS, RINGS. INTEGR =>
7 >
=a and 6, cannot be simultaneously
zero
eee
ne ee

ie a+J2b, a,+J26, 27 ¥2by _ (a, 2, -26,6,)+


2 (a, 6, —a, 8.)
father — = = 5 5
mi a. = 2b, a, (25, a, _ 2, as - 25;

_ | 4,4, -26,
5, | 5 ( a,b, —a, 5, )|
~ [ a, 2 —26,; x |tv2)/ { a,3 —26; j|

=x (2 » where x.y €Q

F={x+J2y:x,y€Q}=Q[,2 |.

2) that Q[ /2 Jis the field of quotients of Z[ ./2 ].


}
saaple 8. Show for every prime p. the ring Z’/pZ with the usual modulo operations, is a fieid_

Webaveto show Z/pZ = {0, 1,2, .... >—1} isa field for prime p
We know Z/pZ is a finite ring. so to show it is a field it is sufficient to show ft is an integral domain
{ .
each finite integral domain is a field)

lax @ y=0 for x,» © Z/pZ

=> xyisamultiple ofp

> pixy => pix orpl\y (-: pis prime)

=> x=0 or y=0 (~ x,y


& ZipZ and x, y<p)

Z’pZ is an integral domain.

Hence Z/pZ is a field.

EXERCISE 2.4
(Kapalansky) Prove that if an element of a ring with unity has more than one right inverses, then
the set of its right inverses is infinite.

Let R be a ring with identity. Then show that R is of characteristic n if and only if m is the least
}~

positive integer such that n-1 =0.

ie, ifand only if o(1l)=7n.

Show that the characteristic of a finite ring R divides o (R).


ke

Scanned with CamScanner


did SPECTRUM DISCRETE MATHEMAT:

4 Let R be a commutative ring with characteristic p, where op is a prime number. Prove {


(asd? @ a? ea? where a AER.
& Prove that the ring of quaternions is a division ring which is nota field, |

6 Let R bea finite ting without cero divisors and o (R)> 1, Then show that R is 4 division ring,
Let sG,+ > be a simple abelian group, Prove that the ring of endomorphisms of G is a divis
THA.

SR, and R; are integral domains. Is Ry & R; necessarily an integral domain ?

Q Let Ry and R; be two rings, Show that Ry X R, is an integral domain if and only if any one of th
is an integral domain and the other contains only a zero clement.
10, Suppose R is a ring with unity t such that R has no proper zero divisors. Prove that 0 and | are)
only idempotents in R.
Tt. Show that in an integral domains R (with unity) the only idempotents are zero and unity,
12 Which of the following sets form an integral domain with respect to ordinary addition ¢
multiplication, Ifso state which are fields ?
(a) The set of naturals (6) The set of even integers
ee

(c) The set of numbers of form /2 avaeQ,

ANSWERS
8. No
12. (a) Neither 1D. nora field (6) L.D. but not a field (c) Neither I.D. nora field

Scanned with CamScanner


3
“ISion
BOOLEAN ALGEBRA
POSETS)
, pefinition (
ially ordered set or a
then Anon-empty set P, together with a binary relation R is said to form a part
dition holds:
ait following con
FS the (i) Reflexivity: a Ra forall a & P
bRa > a=b, where a,b EP
) Anti symmetric: Ifa R band
ui Transitive: IfaRb,bRe > aRe forall a,b,cEP
Vand .
A poset is generally denoted by (P, R).

gark: For convenience, we generally use the symbol < in place of R. Thus whenever we say that P is a
_« it would be understood that < is the relation defined on P, unless another symbol is mentioned. We
fas less than or equal to (although it may have nothing to do with usual less than or equal to that we
{ . .
sfyniliar with). Thus (P, S) is a poset.
cumple. The set N of natural numbers form a poset under the usual s

| Similarly, the set of integers, rational numbers also form poset under usual S

‘umple. The set N of natural numbers under divisibility forms a poset.

| Thus here a S 6 mean a/b (a divide b)

Definition : (Least Upper Bound and Greatest Lower Bound)


Let S be a non-empty subset ofa poset P. An element a & P is called an upper bound ofS if x s a.
‘r€S. Further if a is an upper bound of S such that a = 6 for all other upper bounds 6 of S then a is
aed least upper bound (/.u.6) or Supremum of S, We write sup S for supermum S.
; Similarly, an element @ & P will be called a lower bound ofS if a sx V x & S and a will be called
Ft ereatest lower bound (g./.6) or Infimum of S (Inf S) if 6 = a for all other lower bounds 5 of S.

‘mark, (i) Greatest and least element belong to the set whereas / u b and g / b may or may not belong to
Ret.

eg, Let (Z, S) be the poset of integers. Let S= {...,-2,—1, 0, 1, 2}


then Sup S =?

and § has no Inf as there is no element a such that asx Vx ES&S.


an
445
raat
Co tinea

Scanned with CamScanner


446 SPECTRUM DISCRETE MATHEMATIcs

Definition : (LATTICE)
A poset (P, S) is said to be a lattice if every pair of elements have greatest lower bound and lez,
upper bound belongs to P.
Le. VabeP

aVbepP
aANbeSeP

Note : To prove P to be a Jattice we check with the help of operation tables. One for join and other for mee
P is not lattice if at least one a V bora A 6} does not belong to P where a, b € P-

ILLUSTRATIVE EXAMPLES
Example 1. Write down the operation table for V and A for
L= {1, 2,3, 5, 30} under divisibility relation.
Sol. The operation table for VY and A for the lattice L is given below =
where a V b=lub{a,b}=l.c. m {a,b} and aA b=g/1b {a,b} =g.c.d {a, b}

viol 2 3 5S 30
aA|{1l 2 3. 5 30
1 1 2 3 5 30 if}io oa 4 1 4
2 2 2 6 10 30 2/1 2 1 1 2
3 3 6 3 15
=
30
and
3}/1 13 1 3
5 5 10 15 5 30 5/1 1 1 5 5
30 | 30 30 30 30 30 30/1 2 3 5 30
Example 2. The set A= {1, 2, 3, 4, 6, 12} of factors 12 under divisibility form a lattice.
Sol. In divisibility,
a V b=Sup {a, b} = Icm {a, b}
a A b= Inf {a, b} = gcd {a, b}
Operation tables are :
v |} 2 3 4 6 12 a |[1 2 3 4 6 #12
] 1 2 3 4 6 12 l 1 1 1 1 1 1
2 2 2 6 4 6 12 2 1 2 1 2 2 2
3 3 6 3 12 6 12 3 1 1 3 1 3 3
4 4 4 12 4 #12 «12 4 1 2 1 4 2 4
6 6 6 6 12 6 12 6 12 3 2 6 6
12}12 12 12 12 12 «12 I2;1 2 3 4 6 12
From tables
VabEA
aVbEaA
and aAbEA
So set A under divisibility is a lattice.

Scanned with CamScanner


Vv
yarns 447
ssample 3. Prove that poset P = {2, 3, 4, 6} under divisibility is not a lattice.
cal. In divisibility

; Sup {a, 5} =Icem {a, 5}

Now 3,4 © P

Sup {3, 4} =Icm {3, 4}


=12¢P
4.
So P is not a lattice.
13, Definition (Boolean Algebra as Lattices)
A Boolean algebra is a distributive, complemented lattice having at least two element namely the least
cement (generally denoted by 0) and the greatest element (generally deviated by 1).

Since the complement of each element in a Boolean algebra is unique. Thus a complementation is a
yalid unary operation over the set under discussion. Therefore we list it together with other two operations
Goin V and meet A). This will make a distinction between lattices and lattices which are Boolean algebra.
4 Boolean algebra is generally devoted by (B, V, ', 0, 1), where B is a non-empty set having atleast two
sements with three operations (join (V ), meet (A) and complementation ('),
Definition. A Boolean algebra B consists of a set B together with two binary operations A and V on B,a
ciiary operation ' on B and two specific elements 0 and 1 of B such that the following laws hold. We write
tis B = (B, V,A, "3 0, 1).

(a) Associative Laws : For all a,b,c © B.


(aV b)Vc=aVv (bVec) and (aA b)Ac=and(bAc)

(6) Commutative Laws : For all a, 5,EB


(aV 5) =(6V a) and (aA b)=(b A a)
(c) Distributive Laws : For all a,b,cEB

aV(bAc)=(aV b)A(aVec) and aN (6Veo)=(AANBb)V (adc)


(d) Identity Laws : For alla © B

(aV 0)=a and (aA 1l)=a

(e) Complement Laws : For alla €&B

(a V a')=1 and (aA a’')=0


Example 1. Let B = {0, 1} be a set. The operation V, A and ' on B are given by
v{o 1 on ne | ,
0 1
0/0 1 0|0 0
1 0
| 1 |0o0 1
Then (B, V, A, ’, 0, 1) satisfies all the properties listed in 2.1.2. This is one of the simplest example
ofa two-element Boolean algebra.

Scanned with CamScanner


= SPECTRUM DISCRETE MATHEMATICS

set of X. Define two binary operation V and A and the unary


Example 2. Let P(X) denote the Power
operation ' on P(X) by

AVY Be AUB, AA B*ANB and A’ =X —A forall


A,B € P(X)

where U, M and X — A denote the union, intersection and complementation in a set theory, Then
(P(X), U, . . @, X) form a Boolean algebra.

Note, If X has m clements, then P(X) has 2” elements and the diagram of the Boolean algebra is an n-cube
The Partial order relation on P(X) corresponding to the operation U, M is the inclusion C.

The following are the diagram of the Boolean algebra P(X) when X has one element, two elements ang
three elements respectively.
e

Boolean aigebra Boolean algebra for Boolean aigebra for


for singleton set set having twoelements _—_ set having three elements

Example 3. Let 1 be a positive integer and D,, denote the set of all positive divisiors of m then (D,, lcm, god,
‘. 1, 2) form a Boolean algebra if and only if7 is square free, in the sense that it has no factor of the form
7.
p~ , where p is a prime.

Sol. We know that D, is a bounded distributive lattice. Therefore, we need to show that it is complemented
lattice.
Let =p; p>...... p. be the product of distinct primes
Let d be any divisor of n.

Let d= Pi Py ores p, where p & (py, Prs---+-+s Pm}


1 2 r

Take Fp. Pi aneee P, be the product of all the prime divisors of m not dividing d, then
1 “2 4

d. {d,d"}=1
c.a. & =
and Lem
i od
{d,d}
. SS
cal =
n
a ged{d,d}
d isthe complement of d
Hence D, is a complemented lattice and so it is a Boolean Algebra.
Conversely. Let D, be 2 Boolean Algebra. 4
To show that » is square free.

Scanned with CamScanner


Sr" ESS oT]

>

eae h that p's where > 1, the 449


~S peaP
* yetP ot have a complement in D,, because if (p,
d)= 1 for some d€D,, then
¥ ,can rhe
a
oP Aa)

7 fs
ys Ped PA = [p". d| . l
a) [+ ab= (4,6) (a, 6)
<n
, sence P d<pidsn_ i. p.d<n
4 pd _ de
oun UP. dj = ——=
(pd) pa<n and d sod sod cannot be a complement of p .

go p does not have a complement in D,, and so D,, is not a Boolean algebra, a contradiction,
Hence 7 is a square free i.e., nis a product of distinct primes.
“genarke In view of above example, we can easily checked that D3, Deg, Dao, Dex, are Boolean algebra,
"ure a8 Ds, Da, D2, Das, are not Boolean algebra.
jrample 4. Let S denote the set of all statements formula involving a single variable. The algebraic system
+s, v, A, ~ F, T) is a Boolean algebra. Here the binary operation V and A denote the disjunction and
"sajunction respectively and the unary operation ~ denote by negation. The element F and T denote the
smula which are contradiction and tautologies respectively.
+ fxample 5. Let X be any topological space and Let CO(X) be the family of sets that are simultaneously
mm sosed and open (i.e. clopen sets). The family CO(X) is a Boolean algebra w.r.t. the operation intersection
8 meet, union as join, a’ = X'‘\a,0 =m and 1 =X.
4 | Theorem. In a Boolean algebra (B, V, A, ', 0, 1);
if a@Vb=1 and aA b=0, thenb=a’
‘2. the complement of an element in a Boolean algebr
a is unique.
reef. Suppose that aV b=landah
b=0
Now b=h5vQ [Identity law]
=bV(ana’') {Complement law]
| =(bV a) A(bV a’) [Distributive law]

| =(av b)A(bVa') {Commutative law]


=LA(bV a’) [Given]
| =bV a’ [Identity law]

Ts =py a’ )
Setin a! =a yg [Identity law]
3 =a'V (a b) [Given]

~~
Scanned with CamScanner
Pe Se 7H

450 SPECTRUM DISCRETE MATHEMATICS



=(a' V a) A (a’ V Bb) [Distributive Law]

=1A (a’ Vv b) [Complement Law]


=a'Vb [Identity law] .
=bvVa' [Commulative law]
Thus a’ =hVa' ...(2)
from (1) and (2), we get b= a’
Corollary. In a Boolean algebra (B, V, A,', 0, 1), ifa@ V b= 1 anda A 6=0 then b=a’ also a’ = b,
Proof. This follows from above Theorem and commutatively laws in B.
Thus in a Boolean algebra (B, V, A, ', 0, 1), we have .
0’=1 and 1’=0
Proof. Since 0 V |1=1 and OA 1L=0 1
By Corollary (1), we have 0' = 1 and 1'=0 ;
Theorem [DeMorgan Law’s|
For any a and 4 in a Boolean algebra (B, V, A, ’, 0, 1), we have ;
(i) (aV by =a ab’
Gi) (aA bY =a’ VB
Proof. We have
(aV b) V(a' AB’) =[(aV 6) Va'] A [av 6) Vv b']=[(ava')v dj A [av (vb)
=[l Vb)
A [av 1]
=IA1
=]
also (aVb)AAB)=[ANAABV[EAAAB)=[GAAa)AB]V[OV S')Aa]
=[0A 4']) Vv [0A a’]
=O0Vv0
=0
By Theorem, we have (a V b)' =a’ A B'
Similarly, (a A 5) V (a’ V 6') =[a V (a' V Bb‘) A [BA (a’ V B')]
=[(aVa)AB')A[6V b')Va'j=[l Vv BJA [I V a’]
=1Al
=]
Also (aA b)A(a Vb’) =[CAAN hb Aa] V (AA SA B)]=[(aA a) Ab] V [ad (bAB'))
=[0A
4] V [a A 0]
=0V0
=0
By Theorem , we have
(aA b)' =a VB

Scanned with CamScanner


eee
Nee TL ee Le ee EE
‘pia

y
y e oolean
m. Ina B algebra (B, V, A, ’, 0, 1), the followi
n g properties hold

rear idempotent Laws. Foralla & B

(@) aVa=a and aNa=a

alla © B
(b) pound Laws. For
aVl=l and aAN0=0

For all a, 6, EB
(c) Absorption Laws.
aV (aA 6)=a_ and aN(av bj=a

(d) Involution Laws. Foralla EB


(a')'=a
[Identity Law]
proof. (a) a-avVa
=aV(aAa’) [Complement Law]

=(aVa)A(aV a’) [Distributive Law]

=(aVaj)Al [Complement Law]

=ava [Identity Law]

Thus aVa=a
a=aVvVl [Identity Law]
Again
=an(aVa') [Complement Law]

=(aAa)V(adAa’') [Distributive Law]

=(aAa)V 1 [Complement Law]

=aNa [Identity Law]

aNta=a

(6) Now aVli=(aVI)Al [Identity Law]

=(aV 1)A (aV a’) [Complement Law]

=[(avDAa] V [av Aa’) [Distributive Law]

=[(aA a) V(lLAq@] Va ayA(lLAa’)]

=[aV a] V [0 V a’] [Idempotent, identity and Complement law]

=ava' [Complement Law]

=1
Thus aVi=l
Again a@A0=(aA0)V0 [Identity Law]
[Complement Law]
=(aA0)V (aAa’)
[Distributive Law]
=[(a A 0) Val A [aA 9) V a]

Scanned with CamScanner


452
SPECTRUM Discrp |{
-RETE Ma.
MATH sy
“[lavayA(OvaAflavay ny i
a’)) ia
=fahala{lna’] [Idempotent, identit
}
Y
and c
and Compten. 7"
"aha
. ew}
c fj

Thus ahQ=G
(c) Now avian by) (an IV (an by
~
“a hl) (lv Vb)
[Distri
ibutiy J|
~ah(bVly !;
[Commutative
= a A J
.
= d
[Boundeg | au}
Thas aVlanbj=a
Again aN(aV by =(avOjyA
lay by
ae

=a (On b) [Identity 1 44]

[Distributive L,.,
"av (b AO)
[Commulatiye 7
La

“avo
[Bounded | au
=@
Thus ahlaVvhbyora

(dj Since eV xexVx'= |] and x Ax=x Ax =0 |
Thus EVx~"l and xAx=]
Therefore by Complement law, we have

(x')' =x
3,4, Principle of Duality for Boolean Algebra
Let (ts, ¥, A,‘,0, 1) be a Boolean algebra (under <) and $ be a true
statement for B.If S* is obtained
from S by replacing < by z, ¥ by A, A by V, 0 by I and J by 0, then S*
js also a true statement. We 33)
that the statement S* ts dual of the statement § and
Vice Versa.
For example : If (8, ¥, A, ', 9, 1) be a Boolean algebra, then following
, .
.
statement I‘ ’ to 10’ ’ are * dual ¢
; af

the statement } to 10 and vice versa,


(i) avVvhebva (1)' ahhwbana
(2) aVibVecywlavhyve (2) uhlbnce(anbynre f
(3) ahk(hVcjymlanbyvilah By aVvi(brcye(av bya (ave)
(4) aV¥0°OVaea 4)" aAl@tAa@wa
(S$) aVa'~» | (SY aha

Scanned with CamScanner


a
nics 453
a
; ) a2¥a=a (6) ahe=e
i @ aviel (Ty aAG=9
(3) aVilan bj)=a (By acAflav bj=e

a fav by=an Bb (Sy (aA by za'v &


(a’)'=a (oy (ay =e
(10)

yeark (1) The above statement I to 10 and (1) to (19)' are known az Boolean Identities (or Basic
zokan algebra laws).
paark. (2) The notation for operations in Boolean algebra is derived from the algebra of logic. However,
i gor notations are used. These are summarized in the following
le chart
week -

Test Set | Computer me


(Mathematician’s Notation Designer's
Notation) | Notation
Vv U - | Jom or Sum
A n | meet or and or Product
. Cc | Complement
| < c = |
Mathemetician most frequency use the notation of text and om occasion use the set notation for
Soolean algebra. Computer designers use the notation (+) and (-} as most of the computers did not have
embol YA.
Thus in the letter notation BOOLEAN IDENTITIES looks like 2s :

(1) a+b=b+a (1) @-b=6-a


(2) a+(b+c)=(a+hjtec @y a-(é-o)=(a-b)-e
GB) a-(b+c)=a-bra-c (3) at+té-e= (a+b): (a+)
(4) @+0=0+a=a (47 e-l=l-e=a
G) a+a'=] (Y a-a'=0
(6) at+ana (6)) a-a=a
M atts] (TY a@-0=0
(8) a@ta-boa (8 a-(a+b)=a
O) (a+ by =a bs (9! (a: by = a'r bh
(10) (a'y' = 4 10)’ (a) =e
t
‘ample 1. Prove the following BOOLEAN IDENTITIES :
9) at(a'- bye ged (b) a-(at+hy=a-6
{e) (gq. b)+(a-b'y=a (a) (a: h-oht(a-d)=a-b

_
Scanned with CamScanner
454 SPECTRUM Discp
ATi,

Ney
Sol (a) atta’b) 7fara’)-(a*)
=}-{a->
6)
ag-

Thus a+fa -by=arb


(sy a-(a'-by-a-ara-b
=O-+a0-b
=g-b
fc} (a-b)+(a-
b') =a- (b+)
mo- i
=@
Thass (a-b)+(a-Sj}
=a
(dy {a-bh-cy-(a-
bp =ta- b)-c+la- by:
=(a- Byle+ 1)
=fa-&p-}

=a-é
Example 2. In any Boolean algebra, show that
fr) @=5 @ gh’ +n’ =0 (ii) a@=0 @ ah’+a'b=b
Sel. (7) Now at’
+ ah &@ G
@ af’ + bb’ +o'a+ab=G [- a'a=0= 834
(a> bp b'* a'fa~hy=0
o Oo

fa~byb +la+bjya' =o
’ee

fa+ bla
+h y=0
{a + 53 {ahj’
= 9
a+ h= a
{ >" xx’ = 0 always}
a=
fii)
[- a+arazed
Now ab" + obo &
= ab’ + bb’ + ca+'b=ab
~e fa+ bb + alae
by~h
eo {a+ bh’ *(a+bya'=b
~~ (a+ bjla' +b y= h-4
e s+tbh~b and [ b-1-8
ai +h}
. _ and gin
[0+ illst
j21+8* :
> @G
,

Scanned with CamScanner


EINES ETL SS 22

3 simplifyelon
the following Boolean ex
=
“en
pressj

" (arb) + (a+ bY


(0) (arc)+ C+ [0 +8") +c] (0) (a'-b'-¢y’ + (ab! +c) + (arb'+c’)
(d) (1-a)+(0-a")
8) =[a-b(a+ ayy
arb) + (a+
!
qh ) (a'-b'*¢) + (a:b' ‘c) + (a:b! “c') [ x! $y" = (x ‘y)']

=[(a':(b'-¢)) + (a-(b' ‘C))) +


(a-p' c')=[(a' +a) b'c '
= cb’ + ac'b' = (c+ ac')b' + (ab c')=(1+b'c) + (a-b'c’)
la’ +a) be}
“(C+
a) (C+e')b =(o4 yo.)
(c+a)1+5
@ ret et [b+b)-=actett.g =(c+a)b'=e-b'+ ab!

earctete=acte=(at]).exf.gn,
i (1d) + Ora’) =a4a'
=|
sample 4, Prove by using Boolean algebra that

(J A*B.CH(A*B).(A+C) (i) A+ K.C=A+C


where A isthe complement of A, forall A, B, C ina Boolean algebra
ca () LHS.=A+B.C=A,1+B.C
=A(1+C)+B.C [v 1+C=]]
=A.1+A.C+B.C=A.(1+B)+A.C+B.C=A+A.B+A.C+B.C
=A.A+tA.B+A.C+B.C
[v A.A=A]
=A .(A+B)+(A+B).C=(A+B).A+(A+B).C
=(A+B).(A+C)=R.HS.
(i) LHS. =A+ AC=(A+ A).(A+C) [Using ()]
=1.(A+C) [es At A=]
=A+C=RHS.
Trample 5. Reduce the following using rules of Boolean algebra.
() A.B+ABC+A(B+AB) (i) AB+ AC +A BC(AB+C)
Ml() Now A(B+AB) =A(B+A).(B+B)
=A(B+A).1 [v B+ B=1]
=AB+A.A=ABt+A
=A(B+1) =A.1 [. B+l=]]
=A
Also AB + ABC =A(B +BC)=A(B +B)(B +0)
=A.1( B+O)
=A(B +C)=AB + AC

Scanned with CamScanner


.

SPECTRUM DI
SCRETE

AB+ABC = AB+AC
= AB .AC
= (K+(B)). (A
= (A+B)(A+C)
=A+BC
Thus AB+ABC+A(B+AB) =A+A+BC =1+BC
=B+B+BC=B+B(1+C)=B+B.
Hence A.B+ABC+A(B+AB) = 1=0.
(ii) AB+AC +A BC(AB+C)=AB+AC+A
BCAB+A BCC
=AB+AC+AABBC+A BC
aks

=AB+AC+0.0.C+ABC
[x x’ = 0)
=AB+AC+0+ABC=AB+AC+A
BC.
Example 6. Using Boolean algebra, show that
abctabc'+ab'cta bc=abt+berea.,
Sol. LH.S.=abct+abe'+ab'ct+a bc=ab(ctec')t+ab'ct+a' be
=ab.lt+ab'ct+a' bc=abtab'ct+a bc=al(b+b'c)t+a' be
=a(b+b')(b+c)+a' bc=a.1.(b+e)+a'
be
=abtacta bc=abt+(ata' b)c=abt(ata')(at+
bc
=ab+1.(at+b)c=abtactbe=RHs.
Remark: We represent a@®b=ab'+a' bin XOR-gate
Example 7. Using Boolean algebra, show that
() xytxztyz=xy+@y)z (iit) x' py’ zt+x'
yz’ +xy' z'+xyz=x@By
Bz.
Sol. (() RHS.=xy+(e@y)z=xyt(x'
ptxy')z
=xyptx' yztxy'z =xp(lt2+x'prtxy'z
=xytxyztx’pztxyz=xytx' yztxz(yty’')
HSxyptx'yztxz
=y(xetx'z)t+xz=yetx)(x+z2+xz=y(xt+z2)t+xz
=xytyzt+xz=L.Hs.
(i) RHS.=x
Oy Oz
=(xp'+x'y)@z=(xy'+x'y)z't+(xy'
+x' yz

=xylizitx'yz'+(@y)'.@'y)')z

Scanned with CamScanner


=xyiz'tx'’y2'+(x'+y).(x +y')z [-- (a. b)'=a'.b']
axy2'+x'yz't('xtx'y'+pxtyy')z
=xy'z't+x'yz'+(x'y'+xy)z [.- aa'=0]
=axy'z'tx'yz't+x'y'ztxyz
=L.H.S.

EXERCISE 3.1
1, IfD, denotes the set of all positive divisions ofn (n & N). Show that Dg, Dio, Djs, Do are Boolean
algebra where as Dy, Dj2, Dig, are not Boolean algebra.

1. Let (X, J) be a topological space such that A, Ae J for every A © Z Show that (9,1, U, ‘, , X) is
Boolean algebra.
3, Prove that in a Boolean algebra B, the following conditions are equivalent
(jx Sy (ix Ay’ =O (ii)x' V y=1 (iv)x A y=xand(y)x V y=y forallx,y € B.
4, Show that a V (a' A b)=aV banda A (a’ V b)=a A b ina boolean algebra.
§, Reduce the following in boolean algebra.
(Px AQ’ Vy) (DX Ay AZ)VG' AY AZVOAY’')
(iii) Ay) V &' Az) VW Az)
6. Prove by using Boolean algebra
() xtx'z=xt2 (ii) xptxy'=x (iii) @+y)- ty) =x
(iv) xyz t+ x'y + xyz’ =y (vy) xztay=2(xt+y) (vi) @+y)' + @' +p) =0
7, Using Boolean identities, show that
(a) [a(6' + c)]' [b+ (ac')'}’ = abc’ = (b)_ a’ [(b' + c)' + (bc)] + [(a + B’)' c] = a'b
8. In any Boolean algebra, show that
(i) (a+ b') (b+ c') (c+ a')=(a' +b) (b' +c) (c’ + a)
(ii) (a+b) (a'+c)=acta'b =acta'b+be
(iii) as bat be=batc)

ANSWERS
}_5 (i) xay (ii) (AZ V (Ay) (iid) (Ay)V A 2)
155. Boolean Expressions or Boolean forms
Definition. Boolean Polynomial (Boolean expression, Boolean form or Boolean formula)

Let My Xa5 ssseee x, be a set of n variables (or symbols). A Boolean polynomial (Boolean expression,
Boolean form or Boolean formula) F(X, ,%9, deere ; x,,) in the variables x,, X,,......,%,, is defined

"cursively as follows :
(1) The symbols 0 and 1 are Boolean polynomials.

Scanned with CamScanner


458 SPECTRUM DISCRETE MATHEMAy
: 1
(2) po Ags cevey X, are all Boolean polynomials.

(3) If F(x, Xing covers ;x,) and 8 (x5, sani ; x.) are two Boolean polynomials, then so are

Cf (+ Xa seecenes x)

(5) There are no Boolean polynomial in the variable x,, x,, ...... , x, other than those obtained in
accordance with rule | through 4.
Thus Boolean expression is an expression formed from the given variables using Boolean expressi
' long
V,Aand’.
For example : For the variable x, y, z the expressions
fy, 2z) =@Vyp Az
fy. 2) =@VP)V OAL
f3(%¥,2) = RV OAD VEAGA I)
Fy (xyz) = QV yA AY)
are Boolean expressions.
Note that a Boolean expression in variables may or may not contain all the n variables.
Definition (Equivalent Boolean Expressions)
Two Boolean expression Sf, (44X55 cee »x,) and Sa (x, eo ; x, )are said to be equivalent if they
assume the same value for every assignment of values to the n variables.
For example. The expression fj (x;,%2,x3) = (4) 4 x3) v G “4 x4)

and fy (%,x2,x3) = x, A(x, v x3) are equivalent.


’ a ’
x, x4 x; x; X) AX | XP AXZ | XA; I b

0 0 0 ] 0 0 1 0 0
0 0 ] 0 0 0 0 0 0
0 l 0 ] 0 0 1 0 0
0 ] 1 0 0 0 1 0 0
1 0 0 I 0 ] 1 1 ]
] 0 ] 0 0 0 0 0 0
l ] 0 ] ] ] ] ] 1]
! | I 0 | 0 ] 1 1
Since f, (x,,X2,x3) and f3 (x,,x»,x3) assume the same value for every assignment of the values of
the variables x,, x, and x;. So fi = fo.

Scanned with CamScanner


yrnicks 459
“gfaition (Boolean function)
rita

Let (B, V, A, ', 0, 1) be a boolean algebra. A function from B” to B called boolean function it can be
init

ifed by a boolean expression of n variables.

| efition (Minterm or complete product or a Fundamental product)


;

+ A Boolean expression ofn variables Xy2 Xqy vv X, 1S Said to be minterm or complete product or a
‘ ugdamental product of » variables if it is of the forms Xp AX Neve AX ys where x; denotes either
i « * * . ~ ~ ~— ~~ .

oy
is ort).
}

' Observe that each minterm is completely determined by a sequence of 0’s and 1’s of length n, and any
, = 5
! sch sequence determines a number between 0 and 2” — 1 in binary representation.

A particular minterm will be denoted by min , or m, if the associated sequence of its exponent gives
J

\4¢ number j in binary representation (Here 0 < j <= 2” -1). Thus, we have 2” minterms in n variables
. For example, in three variables ms, =X, A x5 A Xs, become 5 in the
2” -1
jszary representation | 0 1.
Also, these minterms satisfy the following fundamental properties

2” -1
(i) mam, =Oifi #j (ii) 4 = My VM V ones vm, _ =)

i Definition (Maxterm)
2

A Boolean expression of n variables Xp Xs covers , xis said to be maxterm if it is of the form

i, v x, V sees Vv x, , when z, denotes either x; or x .

| Similarly the maxterm satisfy the following fundamentals properties :


(i) M,;vM,=1 ifi*j

2" -1
(ii) rey M; = M,~M, 4 saute AM, _,=9:

There are 2” maxterm in n variables denoted by M 0? M,, sebean ,M

definition. (Disjunctive Normal form or Sum of Product or SOP) or DNP


A boolean expression over two-valued Boolean algebra ({0, 1}, V, A, ‘', 0, 1) is said to be in

For example: (x) A x, 4X3) v (x) AX, A%3) Vv r


Ax, AX;) oF

s of Xf XQ x3 + x] x x4 +x] x2 x3 = LZ m(1,0, 7)
5a boolean expression in disjunctive normal form of three minterms.

Scanned with CamScanner


460 SPECTRUM DISCRETE MATHEMATICg
es
4
.
3
Definition (Conjunctive normal form or Product of Sum or POS) CNP
A boolean expression over two-value Boolean algebra ({0. 1}, V. A, |, 0. 1) is said to be ip
conjunctive normal form (or Product of sum) if it is meet of maxterms.
For example. (x) v x} V x4) A (4) VX) V XS) AQ v x5 v x5) Or

(x, +X, +x) (x, + X4 +35) (x, +X, + x3) = TIM (4, 6,0)

is a boolean expression in conjunctive normal form of three variables.


Obtaining Boolean expression in Disjunctive Normal form and conjunctive normal form
(1) A Boolean expression can be obtained in disjunctive normal form corresponding to this
function by having a minterm corresponding to each ordered n-table of 0 and | for which the value of
the function is 1.
(2) A Boolean expression can be obtained in conjunctive normal form corresponding to this
function by having a maxterm corresponding to 0 and | at which the value of function is 0.
Remark : Boolean function represented as a sum of minterms or product of maxterms are said to be in
canonical form.
Example 1. Simplify the Boolean expression
f(y y,z) =@ AZV(AZV(VAz')

and write in minterm normal form.


Sol. S(xyz) = AZVWAIDVWAZ')

=(x' Az) V[pA(zV 2’)] (Using distributive law)


=(x'Az)¥ (Al) [ee 2V2=]]
=(x' Az)V y

Xx y z c x Az f= (x' AzV yy min


0 0 0 ] 0 0 my

0 0 ] I ] l m

0) J 0 | 0 l m,

0 l l ] | | m,

1 0) 0 0 0 0 my

| 0 l 0 0 0 Mt.

| 1 0 0 0) l m,

I l | () 0 | me

Since minterms corresponds to each ordered triple of 0 and | for which the value of the functios v |
fo

Scanned with CamScanner


SP 0

46
§ arTiCS
oo
minterm are m= xX Ay Azmi =x AyAd,-—s mer A PAS

tae
wer hyA zm," xrNyaArz

Hence Minterm Normal form


=
m4 Vom.
m,V \ m,V \ mv x m,
a(x AY ADVI Ay AD) Vie A vAZVURAYATIVY AAS
prample 2- Simplify the Boolean expression
f(x, yz) (CA ¥ AZVEEA ¥ A 3)

and find its conjective norma! forms.


Sil. L(Y KAY ADVE Ky As) HATA) VAZAY)
=[(e Az) A ty Vy] =[r AZAD]
=x Az
x y : | fexaz Max

0 0 0 6 M
0 0 I o M,
' t

0 1 | Q 0 } 0M,
0 1 | 1 0 | Mf.
:
| y
\ 0 0 0 | M,
0 | { 1 | Mf
i 0 0 | My
i a Se
| |
Since Maxterm corresponds to each ordered triple of 0 and | for which the value of the fanction is 0.
Maxterm are My= xo Vw Wr. M = rv vs Mie x Ver, Mier vrve
MpaxVy
.
V2, M,oexV pve
Hence disjunctive normal fonn
Mi A MA M.A M.A M.A My
WV PV SDA VY VDA VE V TDA VE VDA VY VDA GV V2)
Algorithm for obtaining complete Sum-of- Product Expression
Let the given boolean expression be f(y)...
-
0)rt

Step 1. Find a product Pin (Xp ye ces x, which does not contain the variable x, and then multiply P

° (vy +4), deleting any repeated products (as rx’ © Land P+ P= P)

Step 2, Repeat Step ft until every product inf (4,04), .....4),)i8 a minterm ie. every product P contains
ul the m-variables.

Scanned with CamScanner


462 SPECTRUM DISCRETE MATHEMATICS

Algorithm for obtaining Product of sum canonical form


Let the given boolean expression be f (x,,

Step 1. Find a sum S in f (x,,%5, eeaeee ; x) which does not contains the variable x; and then add S by

(x, x}), deleting any repeated sum (as xx’ = 0 and SS = S)

Step 2. Repeat Step | till every sum in f (x) X4 4 dais ; x,,)is a maxterm i.e. every sum S contains all the |
n-variables.
Example 3. Using Boolean algebra, construct the DNF of the boolean function
f@,y¥, 2) =x +z)
Sol. Here Sy, Zz) =x(ytz) =xyt+xz=xy-14+xz°1
= xyz + 2") + xz(y + y') = aye + xy2't xyz + xy'z
= (xyz + xyz) + xp'z + xyz’ = xyz + xy'z + xyz’
which is in the DNF of the boolean function f(x, », z).
Example 4. Express x, , ++ x ¥, and x,x, in its complete Sum-of-Product term in three variables x,, x, x,

Sol. (7) Now xytxX, = x,-1+ Xy


= . .
1 =
x (%_ + x5) + xy (x, > x) . .

XjpXq + XX, + XX_ + XYX,


= t t

— ’ i
i X,X5°1 + XX5 +1 + X}X, -1

X|X5 (x, +3) + x4x5 (x3 +3) +XX5 (x3 + x3)


= ’ ’ r f

' ' ’ r r r f
*1*9%3 + X)%9%3 + %4%X5%3 + XyX4X + Xy%Xo%3 + Xy%o%X4

—_
XXqXq. + XyXqXZ , + XXX
'
+ XY ’ AQXz + AjXQXz
’ '
+ XyXZ’ x.

Which is the complete Sum-of-Product form.


(if) Also, X\X5 = x)xq-1L=x,x, (x3 + 24) = xx5%3 + XX5X5

which is the complete Sum-of-Product form.


Example 5. Obtain Product of Sum Canonical form of boolean expression x,x, in three variables
Xy1 Xa X53.
Sol. Here = x,x, = (% +0) (x) + 0)= [xy + Ge x5 Ix, + Oy x1)

= (x, + x5)(%, + 44) G, + x) Gy + x) xx' =0

= (x, + x) (% +5) @, + >) [cs xx =a]


= [(x, + x5) + (x3 x5) e+ x4) + @ *4)I [(xy + x}) + C3 %5)]
= (x, +X) + X,)() + X5 + 5) (+ x4 +45)
(x, + x5 + x5)(%, + x) + 3) GQ + x, + x3)

which is the required product of Sum Canonical form.

7 ‘

Scanned with CamScanner


~

JATTICES
a
gsample 6. Show that (x; X% x3 xi) EG (x xi xy x4) ie (x! x, x, x,) a (x! x, x, x4) _ x eS

gh LS. = (xj xh xh x) + Caf xh xh ay) + Rf 25 x5 x) + Cha a x)


[xf 5 205.4) + (xf xh xh xy] + [Gf 2h 23 24) + C4 25 4)
XX
X35 (x4 +4) + x" x4 x5 (4 +24) = XH AZ-1 4+ AP XhQ-1S x) x) (x; +x.)
* 3
= x,x5-1= x, XS

=R.H.S.
Example 7. Show that the following Boolean expressions are equivalent to one another. Obtain their g
ofProduct Canonical form um.

() fy, 2) =@+y@'+2)04+2) GD LG. ¥, 2) =@2)+G'y)+0+2)


i) fy. 2) =@+NG't+D (iv) f(y 2) =z tx'y
Sol. The binary valuation of the given boolean expression are
x ¥ z xt+y x'+z |ytz] f, | | x'y | yz f, Lh]

0 0 0 0 1 0 0 0 0 0 0 0
0 0 1 0 ] ] 0 0 0 0 0 0
0 1 0 1 1 l ] 1 0 ] 0 I 1
0 1 1 1 1 1 l I 0 1 1 1 1
] 0 0 1 0 0 0 0 0 0 0 0 |0
] 0 1 1 1 1 l ] ] 0 0 1 ]
1 1 0 1 0 1 0 0 0 0 0 0 0
1 1 1 l l 1 ] ] 0 l ] 1
Since the values of the boolean expressions for /,. f,, J; and f, are equal over every triple of the two-
value element Boolean algebra. So these are equivalent.
To write them in Sum-of-Product cannonical form.
We have SQ. y, 2) Sxztx'yqxz-l + x'yrl=xz(yt+yp')+x'p(zt+2’)

= xyz t+ xp'z + x'ypz + x'yz’


which is in the Sum-of-Product canonical form.

} Example 8. Find the Boolean Expression that defines the function / by

f(0, 0,0) =0 f(0, 0,1) =0

Sd, 0,0) = 1 fd, 1, 0) =0


f(0,1,0) =1 F(0, 1,1) =90

f0,0,) =1 f0,1,1) =1
-_

Scanned with CamScanner


~~

_
464 SPECTRUM DISCRETE Matnp
Sol. The minterms are f(0,1,0), (10,0), f(1,0,1), f(,1,) tty
Le. (Ay Az), (CAV Az), @AY A2D,AAYA2Z)
D.NFis f(x,y,2) =O AYAZ)VOVY VZ)V RAY ADVEAyA 2
can be simplified as
=(xX’ Ay Az’) VxA[(y Az) V(y' Az) V(yAz2)] (Distrib j

=(x’ AyAZz')VxXA[Q' AG V2a}V(yAD] [Distributive .


=(x' AyAz') VxAC(y VQ A2z2)) [2 Vesty ay,
=X AyAZ')VXA(yY VY)AC(y AZ) [Distributive 4
=(x' ApAz')VxA(y' Vz) oe

=(x' AyAz')V (ZA) VO&AZ)] [Distributive a j

Example 9. Find the Boolean Expression in CN-form.


x y z f
0 0 0 1
0 0 1 0
0 1 0 ]
0 1 1 0
1 0 0 0
] 0 1 0
] 1 0 0
—_

—_

CN form is
<XV¥YVZ)ACXVPAZ)ACR’ VyVZARVypVZ)ACG' Vy’ Vz)
Example 10. Using Boolean laws show
(XAPVV(XAPAZ)VXACYV
(AY) andx Ay
are equivalent. |
(XAVYV(iXAPAZVXAWV(XAY))]

HK AY) V@AVAZV[GAYVEAY)]
=H(XAVY)V(XAPAZ)V(XAY)
=((KAY)V(XAP)VKAYAZD=AKAPWVKAVAZ)
=(xAy)V(1 Az)
=(xAy)VI=xAy
ee

Scanned with CamScanner


———/
EE EE

ATTICES 465

gsample 11. Simplify the Boolean expression f(x, y,z) = (x Az) V Qy Az) V (vy A Z) and write in min.
ym normal form.
st f =(EAZDVOYADV(YA Z)=H(EADVIVACGY Z)] [Distributive law]
=(XAZ)V(VAI) [-e zV Z2=)]

=(xAz)Vy
x y Zz x x AZ
| 0 0 0 l 0 0 my

| 0 0 1 1 i l m
0 1 0 I 0 m,
| 0 1 1 1 \ 1 m4
1 0 0 0 0 0 m.
1 0 1 0 0 0 m6
1 1 0 0 0 1 m4
1 1 I 0 0 | mg
Min. termare my =x Ay AZ =

m,=X AYAZ, my=X VYAz

Mg =x AVAZ

Min. term Normal form is


m, Vm, V mg V m7 V mg

(XA PA Z)VCXAVA ZIVCXAPAZVAAPVA Z)V(QRAVAZ

Max. term is

’ (XV PVZ)ACX VYV ZIACEVPVZAAAVYV ZAKVYV2)

3.6 Representation of Boolean Functions


The existence of one-one correspondence between every Boolean function f{: B” ~ B and a Boolean
expression in m variables. We can represent a Boolean function by any one of the Boolean expression to
Which the function corresponds. Such a representation of a Boolean function will be found convenient for
4 One purpose.
This way of representing a Boolean function is simply to give the Boolean equation for the values of
the symbol (s) in the output combination in terms of some subset of the elements of the input combinations.

Scanned with CamScanner


~

466 SPECTRUM DISCRETEy AT


An,
; the output intermsTN
For example : The function p=f(xy,z0=x.[yt. t')] gives Oty ’b
variables x, »,z and t. * ty

Another method of representation of Boolean function is to form a table. (truth table) 7


exhaustively all possible combination of the input variables and which for each such input conf a
record a functional value. Ra
For example : Consider the function
A%y.2z)=xyz' then
x y Ff fF xy xyz
0 0 0 l 0 0 tl

0 0 1 0 0 0 6
| 0 | 0 0 |
0

; 9 «OO 1 0 0
9 | 2? 0 0
re ee
| J
1 0 1 0
. method for representing Boolean function js circuit diagra
m.
0
The seg of AND. OR, NOT GATES.
compe AND A
OR NOT

B
()
B
®) Oy
sord Gates are NAND, NOR, XOR, XNOR GATE

tan
-
*
AY
te w

D NOT
7 NAND
OS

Bo “OR NOT 8_) >,


NOR '
A
B
oor

Scanned with CamScanner


467

AB+AB’ A

- .) me AB’
XNOR =AQ@B
(©)

This represe
ntation does seem appropriate since Boolean functions can express the functioning of
to be connected to which other circuits, It
circuit. Beca circuit diagram actually shows which circuit are s and thus
asi ona lly possib le to make use of a circuit diagram to eliminate unnecessary connective
is occ
rcuit.
yield a simpler ci
ram of the following Boolean functions
for exampl e: The circuit diag
Git) Ap y,z)=xtyz
() fi Ys 2) =XVZ
(iv) fa(x.y,z)=xyp't+x'z are
(iii) AW Yl ztx' yz +xy!
aX A
() fi.» z=xyz

fi

(i) A@yz)axt+y'z

yz
A

Gi) AG, y,z=x' y'ztx'yztxy'


x
x

ts

Scanned with CamScanner


“ay |
es
oJ
SPECTRUM DISCRETE |

(iv) AQ y ax sexy" SY
ysl
i!
f

xy gl!
yoo fh
Also A (x, y, z) =’ y' rt yet xy" =y'z (y’ +y) + x}"

=x'z.ltxy'=x'zt+xy' =fy (x,y, 2).


Example 1. Write the circuit (gate) diagram of the following Boolean function.

@) fi 1 .%2.%3,%.) =x. (x, (3 .x4))

(it) fa Or X25)
= (x2 +3). HHS) HH5.
Sol. (i) The circuit diagram of the function Si (1. X25%3,X4) =X. (x, (X3 Xy ))

is as shown below

Ro _ X,.X> .X.
>o- | ene edie:
Sf;

or

30S] ye b=
%4o——WWwWwWw"_ id

(ii) The circuit diagram of the function

Si 1 X23) = (X) kn t3) (Ot ty


is as shown below:

1, -——————

(Xy Np + X35) (x, + X3)

Scanned with CamScanner


9
7 simplify the following Boolean functi
ction and d realise the logic diagram of the reduced function
yer e pelp of N AND gate only
ateh
3.CD)= ABCD+ABCD+ABCD+ABCD+ABCD +ABCD+ABCD + ABCD
BCD+AB
F(A. B CD)= ABCD+A
al

_-«;,
ol

al
wl
>|

bal
oO
a


+

+
V

oI
al
ol
wl
>|

QO

oy
+
>
wo
ll

>

+ABC+AC(B+B) = ABD+ABC+AC
wl
ol
>|
ll

ABD+A(BC+C)= BD+A(C+B)(C+C)

=ABD+A(C+B).1=ABD+AC+AB

= AC+B(AD+A)=AC+B(A+A)(A+D)
| =AC+B.1(A+D)=AC+BA+BD.
| F with the help of NAND GATE

c
o|

B
D
F with the help of NAND GATE

F=(AC+BA + BD) = AC.BA.BD

=(A+C).(B +A).(B+D) Lv (@. by =a' +b). @’y' =a]


(A +C).(BB+
BD +AB+AD) = (A+C).(B+BD+AB+AD)
i

AB+ ABD +A AB+A AD + CB+ BCD + ABC+ ADG

AB + ABD + AB
+ AD + CB+ BCD + ABC + ADC
AB+AD(1+C)+BC(1+A)+ABD+BCD [- a+a=al
it

De
Scanned with CamScanner
~ —

SPECTRUM DISCRET
470

= AB+AD+BC + ABD + CBD Ltons
a=] Vqj

- AB (1+ D)+BC(1+D)+AD
= AB+BC+AD
>|
oO
ol
>|

BOOLEAN RING

Theorem : Prove that a Boolean algebra forms a ring under the binary operations + and - defined as above,
Proof: Let B be a Boolean algebra.

Then for all a, 6 € B,

at+h=(an b')V (a' A 5b)

ahbh=anb
i
(/) Addition is closed

For a4bE€B, = a',b' EB

Therefore a A b',a’ AbEB


~~ (aAb')V(a' Ab) EB
n> a+b EB
(Mf) Addition is Commutative : Fora,
b EB,

ath=(an b')V(a' A by@(a' an b)V (a A b')=(b Aa’) Vv (b'


Aa)
= b+ a
(4!) Addition is associative :
For a, bcEB

(a+ b) +e [Cat b) A c') V [(a+ by)!


A c]
“[{(aAb')v(a'n b)} Ac']
Vv [{(a A b') Vv (a' a b)}'
Ac]

Scanned with CamScanner


ATA
qricEs
— =[(a NBA SWAN BACSYVTKCAAB'Y ACa’ AbYY A cl

=a nN BAYV C@’ REN 'TYNNV LK’ Vv by A CaN BY Nel


=[(aN BAc'yVInAbAcekyvtt’ Vv DA a vita VAD, AK cl

=[CaABAcCMNAA EBA CYN LC’ ADV CbhADMNAADIJN GOABYA cy

=(a AVA) VASA CNV LS Aayv Ca’ ABYA cl

H(ARVUARCIWWVAR EAR GVSKRAROVAAH AD\


=S(A AWA VW ATABAC)VSAKRaAAR QHNVAT'AD' Rey

Similarly

(b+ Q+a=(bANRAAWM*VNE'ACARA)V(EAKBARAVE'AC Aad


=(b' Ac’ANavebArac AagWyVCcAbBNR aN} AcHKRaA’Y

=(anN BVAc')VIABAcC')VSKR aR QVO'ADB' Ac)

Thus

(a+ b)t+c=(b+0)+a=a+(b+o0

(iv) Additive Identity : For alla © B,


a+0 =(aAO0')V
(a AD=(AAVYVO=avo
=a

and 0O+a=OAaAd)vO Aa=O0VA Aay=ONa

=a

Thusa+O0=a=O+a forall aGB

Hence 0 is additive identity in B.

(\) Additive Inverse : For any a © B, we have

ata=(anAa')NV (a’ Aa)

=Ov 0

=Q
Thus a itself is additive inverse of a in B.

(vi) Multiplication is closed : For all a,b EB,

ab=aAbeEeB

Scanned with CamScanner


472 SPECTRUM DISCRETE MATHEMATICS

(vif) Multiplication is associative ; For a, b,c © B,

a-(b-c) =a-(bAc)=aA(bAc)=(ab b)Ac=(a-b)Ac

=(a-b)-c

(viii) Multiplication is distributive over addition

For a, b, c € B, we have

a-(b+c) =aN(b+c)=ad [(6Ac')V(b' Ac] =(aNbAc')V(AaAB Ac)

Also abt+a:c=(aA b)+(andc)

=[(aA b)A(adc)'] Vv [(aA by A(aAc)]

=[(@A b)A(a'Vec')] V [(a Vv B') A (aA c)]

=(aNbaAa)V(artbaAc')V(a AadAc)V(b' Aadc)

=((a@Aa)AbBV(ANbAc)VOAcCV(b'
Aadc)

=OV(aNbAc’)VOV(aAB
Ac)

=(aNbaAc')V(aAb'
Ac)

Thus a(b+c) =abt+ac

Similarly we can show that

(a+b)c =act+be

Thus all the eight properties of a ring are satisfied.

Hence <B, +, -> forms a ring.

Corollary 1. If B is a Boolean algebra, then the ring <B, +, -> is commutative ring with unit element.

Proof: For all a, 5 € B, we have

ab =aNb=bNa=ba

Therefore B is commutative

Now for all a € B,

a-l=aAl=a

and l-a=lAa=a

Thus | is unit element of B.

Scanned with CamScanner


icéS 473
ry? gach element of B is of order < 2 in its additive group
olla
( OR
that 2a = 0
ror any element a © B, prove
0

B,
pot: FOr any a €
2a=ata

=(a Na’) V(a' Aa)


=0vV0

=0

Thus under addition order ofa is < 2.

corollary 3. All elements of B are idempotent.


OR
€ B.
inthe ring <B, +, ‘>, we have a’ = a forall a

Proof : For all a © B, we have

x =—a-a

=aNa

=a

Thus all elements of B are idempotent.


Definition : A ring is said to be Boolean ring if all its elements are idempotent.

Note : From above definition and result, it is clear that “Every Boolean algebra is a Boolean ring with
nity”,

Lemma : Ina Boolean ring <B, +, ->, prove that


() x+x=0 (i) x+y=0 ox=y (iii) xy=yx (Boolean ring is commutative)

Proof: We know that cancellation law holds in any ring.

() For anyx EB,x+xEB


Since B is a Boolean ring

(xtxyP =x+x
> (x+x)(x+x) =xtx

Scanned with CamScanner


474 SPECTRUM DISCRETE Matinyg
> (xtx)xt(xtx)x arte

> wax tere? arte

> xtxtxtx Hxtx

> (x +x) +(x +x) =(x tx) +0

> x+x =0

(ii) We have x+y =0

” id Using(i)
° yrx

(iii) Forx,y © B, x+yEB

Since B is a Boolean ring,

(x+y) =xty
< (x+y)(@@+y) =xty
> (xt y)xttyy =xty

> x tyxtxyty? =xty

> xtyxtayty=xty

: > (x+y) + Qx+2xy) =(x+y)+0


=> xpt+xp =0

-_ yx =xy using (ii)

Theorem ;: Any Boolean ring with unity defines a Boolean algebra.

Proof : Let <B, +, -> be a Boolean ring with unity.

Define two binary operations A and V on B by

aNb=arb

aVb=a+b+ab foralla,bEB

(i) Laws of idempotency : For a € B,

aNa=a‘a=a=a

aVa=atataa=0+a@=a

Scanned with CamScanner


a
AA eT a er ae

+
ee
TICES
Comutative laws : For a, b © B
us
“qj
aNhb=a-b=b-a=bnara

aVb=a+b+ab=b+at+ba=bVa

(iii) Associative laws: For a, b, cE B

aN(bAc) =a (bc)=a(be)=(ab)c=(aN by Ac

aV(bV c) =a+(bVc)t+a(bVv c)=atbt+ct+bera(b+cr


be)

=at+b+c+bet+ab+ac+abc=(a+bt+ab)+ce+(a+bt+abje

=(aV b)+e+(aVv b)c=(aVejVe

(iv) Absorption laws : For a,b&€ B

aN(aVv b) =a(aVv b)=a(a+6+ab)=a


+ ab+a(ab)

=at+ab+(aa)b=a+ab+ab=a+t(ab+
ab)

=at+0

=a

av (a A b) =a
V (ab)

=a+ab+a(ahb) =a+ab+(aa)b=at+ab+ad¢b=a+(ab+
ab)
=a+0

=a

Thus
< B, +, -> forms
a lattice.

Now for a, b, c € B, we have

aAN(6Vc)=a(bVc)

=a(b+c+bc)=ab+act+ abce=ab+ac+dbe asa =a

=ab+ac+a(ab)c=ab+act+a(ba)c

= ab+ ac+ (ab) (ac) = (ab) V (ac)

=(aANb)V (aAc)

Thus a A (b V c)=(a A 5)V (adc) foralla,b,cEB

Therefore B is a distributive lattice.

Scanned with CamScanner


SPECTRUM DISCRETE MATHEMAT, ;
476
AS
If 0 and 1 denotes zero and unit elements of the ring B, then
0-a =0 forall aE B

> 0Aa=0 forall aE B

Therefore 0 is the zero element of the lattice B


Also l-a =a for all a & B (as ring)

- lAa=a for alla EB

Therefore | is the unit element of the lattice B.

Lastly we show that the lattice B is complemented.


For any a € B, we have

a A(a+1) =a(at l)=a ta=ata=0

and aV (a+1) I)
=at+(atl)t+a(at

=(at+a)+1 +a +a

=04+1+(a+a)

=1+0
=]

Therefore a’ =a+ |, Thus B is a distributive and complemented lattice with 0 and 1.


Hence B is a Boolean algebra.
Note : Boolean algebra and Boolean ring with unity are equivalent systems.

EXERCISE 3.2
1. Show that the Boolean functions /) (x,y. 2) = (4) ¥ X4) VX; and f(x,y, 2) = x, v (x) Vv x,) are
equivalent.

2. Construct the truth table for the following expressions

(a) Sf (*).*5) (b) Sy (x,.%5) = Xp AX, (c) ta (x)=

3. Find the truth value of f(x,,¥5,43)= (4 Va )ACK VX ACK V5)

4. Write the following Boolean expressions in an equivalent sum of Product cannonical form in thre:
variables Xys%qXy

(a) x, Ax, (5) x, vx; (c) (xp Vx3) Vv Cy Ax)

Scanned with CamScanner


|| 47)
i ckgs ve - Fa . —_
form in three variables VsX>.¥, for expressions.
\y ql in the product of sums cannonical 2.
/ 4 ob
(b) YVAN

(ay %2 v3 a9
|
below
| ad te yalue of the Boolean expression given

| ead “Ge TIM TIV GS) foryv lye bands 0

1 po ree OP LEH
dye

ms
wi the value of the Boolean for
; on ACK vx5) (b) x, AN, (c) x V4 049)

the sum of Products and Product-of-Sums canonical forms of the following :


obta
5.
(a)a) ¥\"x42 +3 (6) Loy +.) 8 )T

other tea Teo


. 7 6X) (Xt +N, XN
481%)
ANSWERS
, a ny | ARV) | eam | GoM
| > | 0 0 0
0 l 0
| 0 | 0 0

\ \ | | 0
EE

» Pape | | ave | ive | av] Sept)


ro | o | 0 0 1 °
0} oft 0 0 °
0} 1 | 0 ° °
0) 1] ? °
1 | 0 | 0 |
1} of 1 | ° °
1 | 1| 0 0 ° °
pe) tik 1 |9 | ° .

Scanned with CamScanner


44
torienratcgry
BVT DI tit y, by hb
ESM MSL eer ep tees SER Rete epee TST EAT PSOE
: : , / _ a” de
4, (eH) (\, AW “ yy fy My A ry)

CPV TN ay AN IYO
ae

AyOa DC
:
ty OIG
. f
4g OAT
'

oT A /‘

4 on
: an ial) oe . _ , . + ty al . qh,
(v) (4 AN ANGI OK) AND A vivian / ry A4y) HW fkg 4,)

A (a) (Np Vy Vy ACN V¥y Vay)

(t) (Np Vy VAC Vay YG) AUN Vy Vea) (x, Vy 4 hy )1 (Z, thy thy) (x iif
- "4 2/ES
“ “ay

O(a) | (b) 0)
‘ Ny NAN, x XV iy Ay fs (x4 Vegh hy Vhs 4)

oe ecm rete fl pm ererreeeeee d ene eren

0 | 9 0 | 4 | 0 Q
() | () | | ()
Y aeky
ate teeny ff eon ssc oe
(j

=. TRE IET | SIRT foe eNREEe Reiner

memes ard 9 aw ere cues

Scanned with CamScanner


Scanned by
Scanned by TapScanner
TapScanner
ctt”
jotrohrtheo ry was born in 1736 with Euler’s paper in which he solved the Konigsberg bridge
tn 1947, G.R. Kirchoff developed the theory of trees for their application in electrical network. A
5 discovered trees while he was trying to enumerate the isomers of saturated hydrocarbon C,, Hy, « >.
aie y do wn four color conjective, which states that four colour are sufficient for colouring any atlas
He) 8 in intries with common boundaries have different colour.
ee, Theory is employed in many areas, such as Communications, Engineering,
escs Soci
~ Sciences etc. On account of diversity of its application, it is useful to develop and
ie a c ot form and then import its results. In general areas of computer science such as
cal design artificial intelligence, formal languages, computer graphics, operating
useful.
, weShall define the various components of the graph theory along with suitable
as been made to show that graphs can be useful to represent any problem involving
A
attshaemptt 4 Pelee where concern is not with the internal properties of these objects but with
1 ‘atte a e

‘an gem ents 0!

ar ener ee ees chilement of (G) is assigned an ordered pair of vertices

“ ia
| kiting
|.
le d the initial vertex and b is called the terminal
Peis a

~ Scanned
Scanned by
by TapScanner
TapScanner
482
ee es oe *

gad 2 2 ehelegns “atid HE ie axa


we sao oo eenah zs a ; eae

(DIRECTED GRAPH) (UN =


Lm

al

REMARK :
&
tt it the v
and edges are represented by line segments joining its end vertices.
. (ii) It does not matter whether the joining of the two
Be. bs longer or shorter.

a[s 5h Adjacent vertices | it ae Agi


ry ith Me wo jon Yo
| oe a | Two vertices uw and v- of a graph G = WB) as i,
P ke 1
4 connecting wu and v. Also the edge e is said to be ineic
ee For example :~ In the above diagram a and b are a v
ol

joining a and b. Also the vertices ¢ and d arasiot ivi Boa :


ee

Loop (or self loop) a |


fo

ee a
es
=
aN
Se oi
eS
ss
1
wr’
9
ane
a
aE c=
i — rt
ee

aie
<7 adel
Paar
2

Scanned by
Scanned by TapScanner
TapoScanner
=
SPECTRUM Discpg-
(iv) Weighted graph : Let
G = (V, E) be any graph andw:E>R beg
of the edge, is known as weighted graph. ri i: c
i d “i

The above graph isa weighted graph a each dg i asianed wit


ha nb
(v) Finite graph : A graphG = Ce grapt
(vi) Infinite graph : Aaah Geils Bae ed an in

Scanned by
Scanned by TapScanner
TapoScanner
486 es ea SPECTRUM DISCRETE - _ , the pe of vertices of odd eee iseven,
In directed graph pee
In undirected graph 7 ~ivy . oe be n-vertices and 1, é2, ......, & be e-edges. in the graph G. Then
deg G(x) = deg G’(x) + deg G(x) Thad wn
by ;
“. deg G (a)=2+2=4 so tae i
deg G(a) = 4 a eas? pots Ste i . a al phe e's ig aires ra
degG(c) =2+3=5 deg G(e)= 5 | me" i
deg G(d) =3+1=4 deg G(d) = 4
Pendent vertex (End vertex)
ee
A vertex whose degree in a graph is one is called pendent vertex.
fs | ome
For example :- In the following graph
. | | ei
:

'b c
Sao,

The vertices a and ¢ are pendent vertices.


geet
ann

since deg G (a) = 1 and deg G(c)=1 Lee Ce


Pires

Definition Regular Graph: A graph


in which all the vertice al ae
Definition k-Regular Graph: A graph in which all he. Nextaes
mM
:

k-Regular graph.
For example:

Scanned by
Scanned by TapScanner
TapoScanner
s Chey

Theorem 5. Prove that the number of edges in a complete graph with» Vertices ig (n

other vetten a 4 a .
Proof. Since every vertex in a complete graph is joined with every
. The degree of every vertex ina complete graph of 7 vertices is »)_ ‘Bh one
ae
“. If e be the total number of edges in G. Then by first theorem on graph
n ' a
> d (v) =2e tie og ea
err 2Sfo
i=l

n(n-l) =2e
n(n-1) te
ss ‘a= = oe 7 aed

: ; 4 pe ith y A, x ol Padme: mi .
nab) ae Tl ee 7k
-. Total number of edges in G = om ace 1.

Scanned by
Scanned by TapScanner
TapoScanner
SPECTRUM px 2
Pr:

Example 5. Is there a simple graph G with six vertices of degree 1, 1,3,4,6,79


ae Ei) be wo pale. The


6 si al ‘0 -
Sol. Here number of vertices in the graph, n = 6
we know that es Ore) Ge mtomete

Maximum degree of any vertices in a simple graph=m—1=6-1=5 |

But G has a vertices of degree 7, which is not possible ina simple graph,
Hence there = ipeiepinard Set inverters as fr ena
edges.
has 12 es
Example 6. (a) Find k, if a k-regular graph with 8 vertic
there be a graph with 8 vertics and 29 edges ? Justify.
Sas
ee
eS SS
»
en oe
iiea ce
ote
aes

Scanned by
Scanned by TapScanner
TapoScanner
asinine ECTRUM I
Ee Asse = Ps

o Lae ae econ, apes

Scanned by
Scanned by TapScanner
TapoScanner
494
7 SPECTRUM Disc: ATS I inte chine tice eae
Example 7. Which of the following pair of graphs are isomorphic? z gy OOOares eae aaa?
i,
c
a 8

Scanned by
Scanned by TapScanner
TapoScanner
ae His) ee LOW et a
. Us gmax. number ofedge =
vertices
7 in 2 4. Dr
eds 7
A
etsy fs rae

he a :
“ ee te El nia Hy a ‘ So ae coum .
as cieepen ier =21. ‘ore wt ay ws Ug es y-
f ans So sam

. which means number of edges in G # number of edges in iG

So G andG cannot
be isomorphic. Pe}.
| Example 10. Is there a graph with 8 vertices of degree 2, 2,3,6,5,’
Babine Sol. Total degree of all the vertices=2+2+3+6+5+7+8+4
ry aa We know total degree = 2 | E ine | = number ee
Se } i JF aie 2 eG a7 } gece 3 oe
A { Aur 37 =2 |E| sl ah aes a3 oF yer dk ae

which isnot possible. eee 2 oa


Hence given graph does not exists.
pe

Scanned by
Scanned by TapScanner
TapoScanner
498 SPECTRUM DISCRETE
M,
(ii) |
Vi V2

(G)

Vy Vs

16. Let G be a complete graph of n vertices. Find the compliment of G.


17. Let G = (V, E) be a loop free undirected graph, where V = {01, U2, U3... Vig}, If deg
deg (02) =3, deg (05) =3, deg (04) = 5, deg (05) = 1, deg (U4) =2, deg (01) = 5, deg (ng 2
(vo) = 3, deg (v9) = 2. Determine the deg (v,) in the compliment G, for all lsi = 10. ae

ANSWERS

vi Vi Ve JX . Sa?
One Vertix Two Vertices Three Vertices

% peas (6 it PG ela oaae rin


a at

Scanned by
Scanned by TapScanner
TapoScanner
500

sree Me aig Wa ye inancinetag ote


12. with nine vertices because the sum of degree of wy, | - : ,

13. i 1a | walk!" ugh G is finite sequence adenine tape inten

Scanned by
Scanned by TapScanner
TapoScanner
502 — ~—
sia SMO
(a) This path is a cycle since it is closed and has distinct vertices Cs = eifloop is also a circuit, but converse isnot true
(6) This path is simple since it vertices are distinct. It 1S not cycle since it is nop a: ee A Be ea ne nae
(c) This not even a path since {X, Y} is not an edge. 4 slop, 2 le 2 pir of parle edgs, 3. ces wig. te isa
Length of path Tee
: EY tt oo rege
For example. Consider the following graph

Fig. (i)

v= Vi, el, V2, ©, V3, €3, V3, ay V4, €5,Vs, C6, V2.

Then W is a walk of length 6 as shown by the bold line in fig. (j). The above a
vertices V; and V; appear twice in the walk W. However
the walk fe
W'=Vi, e1, V2, €2, V3, 4, Va, 5 Vs
is a path of length 4 as shown by the bold line in fig (i). Moreover, the a
open walk as their terminus vertices are different. . he
+ But the walk
| W'’= Vi, e1, V2, €3, Vs, 3, V3, €4, Was @5, Vs» 6. V25 €1 Vi
is a closed walk as the terminus vertices are same. — ae ae
Remark. (i) An edge which is not a self loop is a path of length1.
(ii) A self loop can be included in a walk but not ina path.
as (iii) The terminus vertices of a path are of degree | and the i lernal alk areof
ae | CIRCUIT : A circuit is a closed walk in which no vertex(except the d terminal ye

Hao

Scanned by
Scanned by TapScanner
TapoScanner
SPEC
= ==

Example 1. Which of following are connected, graphs.

| eas he )
(a) Only II1 is connected.
Sia of
(6) All are graphs.
the fol low ing gra phs as : Str ongly connected §
Exa mpl e 2. Cla ssi fy
weakly connected graph and disconnected graph. ee ae

)
Sol. ;
Py 3c
ad (a) The graph (i) is weakly connected

Scanned by
Scanned by TapScanner
TapoScanner
oo —
—-oso

Lyte So M=

Adjacency mat of r
muli
ti-x
graph (undirected)
Let G be undirected graph of m vertices that may contain parallel edges
n X n matrix defined by M = [4y; ]nxn
|

Scanned by
Scanned by TapScanner
TapoScanner
ILLUSTRATIVE EXAMPT we,
Example 1. Find the adjacency matrix A of the multigraph, a

€, ) 1
—-) STy
rary ‘
19
was a
ASIA ates :
Set a,, =n, where n is the no. of edges between
V; and V , and
ee ee

Yo" Ngee Vane


| 0
“oa

Scanned by
Scanned by TapScanner
TapoScanner
Scanned by
Scanned by TapScanner
TapoScanner
$12
Example 7, Consider the graph G. Find
I. All simple paths fromA to F r
I. d(A, F) ,
HII. diam(G)
TV. All cycles which include vertex A. |[
V. All cycles in G. D
Sol. (1) A-—B-C-F,A-B-E-F, ADEF, A~B-EF etc. a ;
(2) d(A, F) =3 (no. of edges) RO One :
(3) diam(G) = size ofG = no. of edges = 8 ay BY:
| (4) A-~B-E-D-A,A-B-C-E-D-Aetc.
sj Example 8. Show that the graphs G, G' are isomorphic. =

Scanned by
Scanned by TapScanner
TapoScanner
Oe AE ee ge we bie A
6. A graph has adjacency matrix

0
&
&&

0
=

aete
M= < e wf , are oe e- - f . ee “— 4 nib Atres eee 7

0
oo
-

0
=-
o

Is the graph connected ?


7. Write adjacency
matrix of multigraph

Scanned by
Scanned by TapScanner
TapoScanner
516
‘ SPECTRUM Digg.
a (i) A complete graph of five vertices is non-planar i.e., Ks is non-planar,

¥ SeeVietugaley

Fig (a) Fig (6)


Since the graph shown in fig. (a) cannot be drawn in. plane without erosia
Ks is non-planar graph. .
Region: A plane graph partitions the Pe into. sever
Lag region is depicted by the set of edges. on eee
: # ray Cycle: The boundary of the region R of graph G is cycle if the
é q i, | of G. i.e., contain no edge such that on removing ee inR it)
re Wt
t * La

mi Degrer of face: IPG be graph apa ¢ Cele ce, the

Scanned by
Scanned by TapScanner
TapoScanner
Sol. (i) In fig. () |V| =6, |E| = 10, R=6
“By Euler’s formula, R=|E|—|V|*? ‘a
ie, 6 = 10-6 +2, which is true a
(iIni)fig. (if) [V|=6, |E|=8, R= 4 a iver ood ay ae fe ‘ae Sie: os tie 5a

By Euler’s formula, R = |E|—[|V| +2 ors gaye ‘ come


So

“. Ee] Fare
ie, 5=+29 which6
, - is true
Example 2. Determine the number of regions defined bya con
Bae a

Sol. Here {V| =4, JE|=8
a -
fs By Euler’s formula.

Scanned by
Scanned by TapScanner
TapoScanner
= a
520 et fs
SPEC SUM Disc ‘ f - for the graph (c)
1.10. Coloring leas 4 Cp rupaehth ve ee
Suppose G be a simple graph with 7 ver
tic es,
toe a
ymber

‘= Vértices have the same colour. we are to pai nt all its Wertices such that ng,
- Chromatic Number : co
The minimum number of colours needed to paint all the vertices of
adjacent vertices have the same colour is called chromatic number the “ee
of G and denoteg setGta
For example.

Na
Blue Red

Scanned by
Scanned by TapScanner
TapoScanner
in | 3 sneral_
Theorem : Fiye Colour Theorem : If G is planar graph then ne chromatic number of the complete graphs kes kyo and

C(G) <5. to colour a ke graph since every vertex is adjacent to every sii ain a
z _ ola the graph Ay
=e
and n
Nie
Simila rly it takes ten colou r to colou r
Proof : Basis: A graph with one vertex has chromatic number of one. aSoe every one.

Induction: Let us assume that all planar graphs with n ~ | vertices have a chromatic
less. Let G bea planar graph with 7 vertices.
Blue
tn

a
“. Ha vertex V with degree (V) < 5.
Let G-V be the planar obtained by deleting V and all edges that co
Now by the Induction Hypothesis G-V has a 5-coole g. Let us
white, blue, green and yellow. SNEbend i ae

Scanned by
Scanned by TapScanner
TapoScanner
ike Ee _ SPEC)
Example 1. Find the shortest path between a and z. iiyas seh Tugel Sein

wz ; ; -

" i bel ; ir Gone, 2F


RE Step
I : Include the vertex ain and determine all the dire
pe going through any other vertices. aa Ties

a a b e

0 2) 1a) 4
Step II: Include vertex in S, neatorer
a and di
vertex. The nearest vertex is c.

a,c, >
:

0. 24a) Lhe ag
ets

tee
a
7

my
Mi
ee
dala

a
etn
4

Bee ol
an...

i
“ami,
‘i 7

Scanned by
Scanned by TapScanner
TapoScanner
is

Example 3. Find shortest path using, Dijkstra’s algorithm.

Sol. Step 1. Initially there is no vertex in S.


Take source s in S and find all paths from Source vert
to allex
other
other vertex,
vertices with out
i.
s a b c d if fa
s 0 22 (s) 00 19 (s) 0 0 aa
Step 2 : Take nearest vertex
to s. Include vertex C. Sat
a b c d ff ds inh a
Ss
5,c 0 22%) 3930) 9G 4163) oe
Step 3 : Include next nearest vertex e in set S. :
5 a b c d aay a ~

S,C,a 0 22 (s) 37 (s, a) 19 (s) 40 (s, a) rie


Step 4 : Include next nearest vertex b in set S. 2, SA
ae
s a b 3
ahi a scab 0 -22(s) «37 (a) 19)
Rte Step 5 : Include next nearest vertex dinsetS.
s a b ote
S,c, a,b, d 37(s,a)

Scanned by
Scanned by TapScanner
TapoScanner
328 —_—<——

1.11. Euler Paths and Circuits


In this section, we discuss an important application of graph th
Scometrica l figure. We want to traverse all the edges of graph by tray
“aC
may or may not be possible. The solution of such type of problems was
Euler
ms (1707-1783), First we discuss the terms Euler Circuit and Euler Path, then.nw
Euler Path : A simple path in a graph G is called Euler Path if it tr
once, |
fies Ths ae
Euler Circuit : Euler Circuit is a circuit in graph G Which traverses every
Euler Circuit is simply a closed Euler path, It is also called Euler line, os
Eulerian Graph roe . ‘ 3 ;

A graph which contain either Euler Path or Euler Circuit is called Bul eria
Example : ek
A

Fig. I Fig. I ae
| “ae 6 a =a

Fig. 1 has Euler Circuit ABCDA


Fig.Il has Euler Path

Scanned by
Scanned by TapScanner
TapoScanner
530 | SPECTRUM Discreyy, 4 w
€ventually comes to an end, we must have reached the other vertex of odd degree. al _
raph were traced, we would get an Euler Path other wise remove those edges that
hac.
obtain a Subgraph formed by the remaining edges.
The degree of
Again by theorem 2, this subgraph has an Euler Circuit. Since the vertic es of this sub sa
original graph ig raph
‘must a path between the two subgraph and thus we obtain a path that contain
once. Hence, we get an Euler Path.
all the eg 5
hig
Remarks : From above theorems, we can
check wheater a given graph contains Euler
hone. Summary is given below: Circuit, rat k
ES
(2) If all the vertices are of even degree then graph has Euler Circuit.
(6) If two vertices are of odd degree, the graph has Euler path
but no circuit,
(c) If more than two vertices are of odd degree then graph has neither
Euler Path nor
Definition

Bridge : Let G be a connected graph. Then an edge e is called bridge if by de! ctin 3
disconnected or we can say G—e is disconnected. Bridge isalso called cut edge. eog
Example : = Tees va

: a ‘
Vis ai
i ly
ite poet A, cca
*

Scanned by
Scanned by TapScanner
TapoScanner
532
liao a SPECTRU\ Petey 2 )
Example 2, Apply Fleury’s algorithm
to construct an Euler circuit for following
graph,

Sol. deg(A)=2 deg (B)=4 deg (C)=4 ice


deg(D)=2 deg (E) =4 deg (F)=2
Since every vertex of Graph has = af
begin fro even degree and ae is conne
from anywhere. Let us begin fro ted
m A. Detailed steps are
Sr.No. Current Path (P) Next Edge |)
JA che {A,B}

Scanned by
Scanned by TapScanner
TapoScanner
SPECTR *
Example 5. Which of the following graphs are traversable ;

f ; een

ee 'h

;
Sol. Graph I is traversable as it contain | circuit is shown below
Euler circuit. Euler aed»
a+f>ecrdrerfrebreszc>b>a
Graph II is not traversable because it is not connected. It contain two cor cen

EXERCISE 1.4
In Questions from I to 5, tell wheter graph has Euler Circuit, Eule
Also find path or circuit (if exists) by showing all steps.

a3

Scanned by
Scanned by TapScanner
TapoScanner
also deg(a)= 2, dea(b) ~3, deg(c) = 2, desta)= 3 at
So all vertices have degrees = 2
which is 2
2
“+ theorem II also true.
Similarly for theorem III we have m= 5 andn=4

I
18) 5" 2 -3n+2) = =t 1é-— 12+2)=3 ee atUi sffee

me(at Sa
“+ graph is Hamiltonian
and Hamiltonian circuit is
Note : The converse a,b,c, da
of these ore ms is ne - ea
but still they are Hamil wi Is not true. We have some graphs
For example in graph
=
wl

ek ts
a
PS

nets
eg
=)

er ee

cet
ret
o
ee

es
hrs
Pome
a eS

bp
ae

eae

c
eet)

Meg)
+ deg(Cy= 2424)» pect se
ed
ee

pe
ot
eS
>

a
=
2
ee

at
ee

Scanned by
Scanned by TapScanner
TapoScanner
Scanned by
Scanned by TapScanner
TapoScanner
es |

oe iy you answer. Als give Hamitonipatanh a ie

i2 d
? ‘
= 10g
wei +t
+ 10h 8 + 12

} (Neither Euler circuit nor Hamilton

1 iltoniar c ‘ a |
“ana J (GA .

: as
«a
"

me tS

Scanned by
Scanned by TapScanner
TapoScanner
cy tyes tring souancme bck wis acy

Scanned by
Scanned by TapScanner
TapoScanner
344
SPECTRUM Dy sere
Step 5 To mark minimum straight lines to coall
ve zeros
rin order to find req
If vase
bai a matrix obtained in step 3 by using following procedure. h
(‘) Mark (Vv) all rows that do nothave assignments, = ai
(i?) Mark (V ) all columns that have zeros in marked row taine from ste
(iii) Mark (Vv ) all rows (not already marked) she have assignment in rmatked rae
step 5 (ii),
ee ae
ae | (iv) eon
0) Rt (v) Draw straight lines through all unmarked rows and marked limp. ne
Uae If the number of lines drawn (m) is equal to number
be optimal otherwise go to step 6.
of rows
(or column), the ¢
aaa ,
Se iad) ere i
ss nee ays=

7 Subtract this asian ofa


at
each value located inter
(iii) ee
Step 7, Next Iteration Cas

Scanned by
Scanned by TapScanner
TapoScanner
‘5 a< TF a ee
i. he
=eees —

Scanned by
Scanned by TapScanner
TapoScanner
Scanned by
Scanned by TapScanner
TapoScanner
Scanned by
Scanned by TapScanner
TapoScanner
ane bis TIO) 98 SHOWN:

a=

Scanned by
Scanned by TapScanner
TapoScanner
Scanned by
Scanned by TapScanner
TapoScanner
(5) Since graph co
(Cc) Number of vertices = 6
ESE ak PeaiPats tae

| (d) Number contains


Graph of edgesa cycle
= 5 as soedges
it is
(e) Graph is a tree “* i
) Graph is not tree as it anne
aSe

Example ae LetA ual {a b, . contains


the root.
a

) Sol, We are given five verti ices an


d four e ges. R wi
() €dges connect all y oriGee
(ii) edges will not form
_ From R, it is clea‘B*r has ing
alledgesasbelow = =
i

Scanned by
Scanned by TapScanner
TapoScanner
Scanned by
Scanned by TapScanner
TapoScanner
Scanned by
Scanned by TapScanner
TapoScanner
Bedtenpedsh tc) sods crit}
ca Ure)
- , : Coal
RES =
- ogee
7 =z i tah
POP as ygnte eae
Sol. The step ete KO er ao
Eames

mila [2g Ape : a ie so Sto OO RE Se

Rin
"be 1S: In theira binaryeee
tren cae
tree with height
eee
6 and c<1... 1TTTh ay
RE
lon
mite
ie

tay ae
|

Sn al
*

Scanned by
Scanned by TapScanner
TapoScanner
as: eet!

ze containsh all eS ae qe
: . whic means we have to remove 2 edges.

G is connected.
Remark. (Cayley's Th
eorem) The
For example : There ar
e 16 spannii

Scanned by
Scanned by TapScanner
TapoScanner
ies 2
ct pier
<i.

fe ic +
- es ms sie a
; eet aa

ae Pa Me ;
F “ree +2, Ae
se se | ae
(i) . = co 7

Select next edge BD

od

(iii)

A a a
:
Select next edge AB
; ‘ _ ’ rh es
ee
ee

Scanned by
Scanned by TapScanner
TapoScanner
i
!
t
ee

Scanned by
Scanned by TapScanner
TapoScanner
a

AS number of vertices are g


Sum of weightsin=2 4342494444

Scanned by
Scanned by TapScanner
TapoScanner
Scanned by
Scanned by TapScanner
TapoScanner
Scanned by
Scanned by TapScanner
TapoScanner
=e
a
SPECTRUM DISCRETE MATHEMA Ty, .

eq

ie

Scanned by
Scanned by TapScanner
TapScanner

You might also like